Download as pdf or txt
Download as pdf or txt
You are on page 1of 497

American Board of Family Medicine

IN-TRAINING EXAMINATION

TIME–4 HOURS

Publication or reproduction in whole or in part is strictly prohibited.


Copyright © 2015 The American Board of Family Medicine. All rights reserved.
1. A 4-week-old white male is brought to your office with a 2-week history of increasing dyspnea,
cough, and poor feeding. The child appears nontoxic and is afebrile. On examination you note
conjunctivitis, and a chest examination reveals tachypnea and crackles. A chest film shows
hyperinflation and diffuse interstitial infiltrates and a WBC count reveals eosinophilia.

What is the most likely etiologic agent?

A) Staphylococcus species
B) Chlamydia trachomatis
C) Respiratory syncytial virus
D) Parainfluenza virus

2. A 36-year-old obese female presents to your office with a chief complaint of amenorrhea. On
examination you note hirsutism and body acne. She is on no medications and a pregnancy test
is negative. Serum testosterone is at the upper limits of normal and TSH is within normal limits.

In addition to weight loss and exercise, which one of the following would be the most
appropriate initial management?

A) High-dose combined oral contraceptives


B) Progestin-only contraceptives
C) Metformin (Glucophage)
D) Levothyroxine (Synthroid)

3. A factory worker sustains a forced flexion injury of the distal interphalangeal (DIP) joint,
resulting in a small bone fragment at the dorsal surface of the proximal distal phalanx (mallet
fracture). Which one of the following is the most appropriate management strategy?

A) Buddy taping and early range of motion


B) Splinting the DIP joint in extension
C) Splinting the DIP joint in flexion
D) Referral for surgical repair

4. Which one of the following drugs is NOT effective for maintenance therapy in bipolar disorders?

A) Haloperidol
B) Lamotrigine (Lamictal)
C) Lithium
D) Quetiapine (Seroquel)
E) Valproate sodium (Depacon)

1
5. A 30-year-old ill-appearing male presents with right hand and arm pain and a rapidly expanding
area of redness. On examination he has a temperature of 38.9°C (102.0°F), a pulse rate of 120
beats/min, and a blood pressure of 116/74 mm Hg. He also has erythema from the dorsal hand
to the elbow, violaceous bullae on the dorsal hand and wrist, and severe pain with dorsiflexion
of the wrist or fingers.

Which one of the following is the most appropriate initial step in the management of this patient?

A) Oral dicloxacillin and outpatient follow-up within the next 24 hours


B) Intravenous metronidazole
C) Consultation with an infectious disease specialist for antibiotic management
D) Immediate surgical consultation for operative debridement
E) Incision and drainage with wound cultures in the emergency department

6. Patients being treated with amiodarone (Cordarone) should be monitored periodically with serum
levels of

A) cortisol
B) creatine phosphokinase
C) creatinine
D) LDH
E) TSH

7. A mother brings her 2-year-old daughter to your office because the child is not using her left
arm. Earlier in the day the mother left the toddler under the supervision of her 12-year-old sister
while she went to the store. When she returned the toddler was playing with toys using only her
right arm, and was holding the left arm slightly pronated, flexed, and close to her body. The
older daughter was unaware of any injury to the girl’s arm, and the child does not seem
distressed or traumatized.

Physical examination of the child’s clavicle, shoulder, wrist, and hand do not elicit any signs of
pain or change in function. She does seem to have some tenderness near the lateral elbow and
resists your attempts to examine that area. There is no ecchymosis, swelling, or deformity of the
elbow.

Which one of the following would be most appropriate at this point?

A) Plain radiographs of the affected elbow


B) Ultrasonography of the affected elbow
C) Evaluation by an orthopedic surgeon within 24 hours
D) Attempted reduction of the subluxed radial head
E) Placement in a splint and follow-up in the office if there is no improvement in the next
1–2 weeks

2
8. A 12-year-old male uses a short-acting bronchodilator three times per week to control his
asthma. Lately he has been waking up about twice a week because of his symptoms.

Which one of the following medications would be most appropriate?

A) Inhaled medium-dose corticosteroids


B) A scheduled short-acting bronchodilator
C) A scheduled long-acting bronchodilator
D) A leukotriene inhibitor

9. Which one of the following is the most appropriate first-line therapy for primary dysmenorrhea?

A) Combined monophasic oral contraceptives


B) Combined multiphasic oral contraceptives
C) Subdermal etonogestrel (Nexplanon)
D) Intramuscular medroxyprogesterone (Depo-Provera)
E) NSAIDs

10. While performing a routine physical examination on a 42-year-old female you discover an
apparent nodule in the left lobe of the thyroid measuring approximately 1 cm in diameter, which
is confirmed on ultrasonography. The most appropriate next step in the evaluation of this finding
is a

A) serum calcitonin level


B) serum free T3 level
C) serum TSH level
D) serum thyroglobulin level
E) radionuclide thyroid scan

11. Which one of the following medications should be started at a low dosage and titrated slowly to
minimize the risk of Stevens-Johnson syndrome?

A) Carbamazepine (Tegretol)
B) Divalproex (Depakote)
C) Lamotrigine (Lamictal)
D) Lithium
E) Ziprasidone (Geodon)

3
12. You are the medical director of a long-term-care facility that has 60 residents. Several patients
experience fever, cough, and upper respiratory symptoms. Two of these patients test positive
for influenza A (H1N1) virus.

Which one of the following is recommended by the Centers for Disease Control and Prevention
(CDC) for this situation?

A) Chemoprophylaxis with appropriate medications for all residents


B) Treatment initiated on an individual basis once testing confirms that a resident has
influenza
C) Prophylaxis only for staff who have had direct patient contact with a resident with
laboratory-confirmed infection
D) No chemoprophylaxis for staff or residents who have been appropriately vaccinated

13. Information derived from which one of the following provides the best evidence when selecting
a specific treatment plan for a patient?

A) Meta-analyses
B) Prospective cohort studies
C) Expert opinion
D) Consensus guidelines

14. Examination of a 2-day-old infant reveals flesh-colored papules with an erythematous base
located on the face and trunk, containing eosinophils. Which one of the following would be most
appropriate at this time?

A) An allergy evaluation
B) Low-dose antihistamines
C) Hydrocortisone cream 0.5%
D) A sepsis workup
E) Observation only

15. American Urological Association guidelines define asymptomatic microscopic hematuria as


which one of the following in the absence of an obvious benign cause?

A) ³1 RBCs/hpf
B) ³3 RBCs/hpf
C) ³10 RBCs/hpf
D) A positive dipstick reading for blood

4
16. A 70-year-old male with widespread metastatic prostate cancer is being cared for through a local
hospice. Surgery, radiation, and hormonal therapy have failed to stop the cancer, and the goal
of his care is now symptom relief. Over the past few days he has been experiencing respiratory
distress. His oxygen saturation is 94% on room air and his lungs are clear to auscultation. His
respiratory rate is 16/min.

Which one of the following would be best at this point?

A) Morphine
B) Oxygen
C) Albuterol (Proventil, Ventolin)
D) Haloperidol

17. A 30-year-old female with a history of prolonged QT syndrome presents with severe acute
bacterial sinusitis. Which one of the following antibiotics should be avoided?

A) Amoxicillin
B) Clarithromycin (Biaxin)
C) Amoxicillin/clavulanate (Augmentin)
D) Moxifloxacin (Avelox)
E) Cefuroxime (Ceftin)

18. Which one of the following is associated with treatment of COPD with inhaled corticosteroids?

A) An increased risk of monilial vaginitis


B) An increased risk of bruising
C) Consistent improvement in FEV1
D) A decreased risk of pneumonia
E) Decreased mortality

19. A 56-year-old male complains of daily early awakening and low energy for the past 3 weeks.
Six weeks ago he had a myocardial infarction treated with a coronary artery stent. During that
hospitalization his CBC, fasting glucose level, and thyroid function were normal. A recent phone
note from the cardiac rehabilitation nurse indicates that he became apathetic and stopped
attending his rehabilitation sessions. He admits to a feeling of hopelessness. He denies chest
pain, dyspnea, orthopnea, and palpitations. His vital signs and physical examination are
remarkable for a healing radial artery catheterization wound.

In addition to resumption of cardiac rehabilitation, which one of the following would be most
appropriate at this point?

A) Reassurance and a follow-up appointment in 6 weeks


B) A Patient Health Questionnaire 9 (PHQ-9)
C) Polysomnography
D) A BNP level
E) An exercise thallium stress test

5
20. A 5-year-old white male is brought to your office with a chief complaint of chronic nocturnal
limb pain. His mother states that his pain is often severe enough that it awakens him at night and
she often gives him ibuprofen to help alleviate his calf pain, but she has never seen him limp or
heard him complain of pain during the day. She also has not noticed any grossly swollen joints,
fever, rash, or weight change. She is concerned because of a family history of juvenile
rheumatoid arthritis in a distant cousin. The physical examination is within normal limits, as are
a CBC and an erythrocyte sedimentation rate.

Which one of the following would be most appropriate at this point?

A) Bilateral plain radiographs of the lower extremities


B) Testing for antinuclear antibody
C) Testing for rheumatoid factor
D) Referral to orthopedic surgery
E) No further workup

21. According to the guidelines developed by the JNC 8 panel, which one of the following should
NOT be used as a first-line treatment for hypertension?

A) ACE inhibitors
B) Angiotensin receptor blockers
C) Calcium channel blockers
D) $-Blockers
E) Thiazide-type diuretics

22. A 67-year-old male presents with a 10-day history of bilateral shoulder pain and stiffness
accompanied by upper arm tenderness. On examination there is soreness about both shoulders
and the patient has great difficulty raising his arms above his shoulders. There is no visual
disturbance, and no tenderness over the temporal arteries. C-reactive protein is elevated and the
erythrocyte sedimentation rate is 65 mm/hr (N 0–17).

Which one of the following would help to confirm the most likely diagnosis?

A) The use of published validated diagnostic criteria


B) Synovitis of the glenohumeral joint on ultrasonography
C) A response to treatment with prednisone
D) A response to NSAIDs
E) A lack of systemic symptoms

6
23. A 70-year-old male with hypertension, benign prostatic hyperplasia, depression, and
well-controlled diabetes mellitus sees you because of increasing fatigue. His medical history also
includes stent placement for coronary artery disease. A physical examination is unremarkable
except for decreased peripheral pulses. A CBC, basic metabolic profile, hemoglobin A1c level,
free T4 level, and TSH level are all normal, except for a serum sodium level of 125 mEq/L (N
135–145). His serum osmolality is 268 mOsm/kg (N 275–290). His urine sodium level is 50
mEq/L (N <20) and his urine osmolality is 300 mOsm/kg.

Which one of the patient’s medications is most likely to cause this problem?

A) Losartan (Cozaar)
B) Tamsulosin (Flomax)
C) Metformin (Glucophage)
D) Atorvastatin (Lipitor)
E) Sertraline (Zoloft)

24. A 21-year old female comes to her family physician’s office with an unintended pregnancy and
states that she wishes to have a medical abortion. Elective abortion is against the physician’s
personally held moral principles.

According to the American Academy of Family Physicians, which one of the following would
be the most appropriate course of action for the physician in this situation?

A) Explaining the rationale for morally opposing medical abortions


B) Providing no further assistance at this visit
C) Offering to match the patient with prospective adoptive parents
D) Advising the patient that it would be safer for her to continue the pregnancy
E) Providing resources that explain how to access a safe and legal medical abortion

25. A 65-year-old male presents to an urgent care center with a foot ulcer. His past medical history
is significant for hypertension, COPD, and diabetes mellitus. He has been hospitalized several
times in the past year for COPD exacerbations and a hip fracture. He does not have any other
current problems.

On examination he has a temperature of 37.3°C (99.1°F), a pulse rate of 105 beats/min, a


respiratory rate of 16/min, and a blood pressure of 142/83 mm Hg. His examination is
unremarkable except for a 2-cm ulcer on the ball of his left foot that has 3 cm of surrounding
erythema and some purulent drainage. His CBC is normal except for a WBC count of
14,300/mm3 (N 4300–10,800).

Which one of the following would be the most appropriate choice for initial treatment?

A) Amoxicillin/clavulanate (Augmentin)
B) Linezolid (Zyvox)
C) Ciprofloxacin (Cipro)
D) Ceftriaxone (Rocephin) and levofloxacin (Levaquin)
E) Piperacillin/tazobactam (Zosyn) and vancomycin (Vancocin)

7
26. Which one of the following is the most common cause of unintentional deaths in children?

A) Motor vehicle accidents


B) Drowning
C) Poisoning
D) Fires
E) Falls

27. A 45-year-old male presents with shortness of breath and a cough. On pulmonary function
testing his FVC is <80% of predicted, his FEV1/FVC is 90% of predicted, and there is no
improvement with bronchodilator use. The diffusing capacity of the lung for carbon monoxide
(DLCO) is also low.

Based on these results, which one of the following is most likely to be the cause of this patient’s
problem?

A) Asthma
B) Bronchiectasis
C) COPD
D) Cystic fibrosis
E) Idiopathic pulmonary fibrosis

28. A 45-year-old male presents to the emergency department with a complaint of acute, sharp chest
pain relieved only by leaning forward. On examination you hear a pericardial friction rub. An
EKG shows diffuse ST elevations. Echocardiography reveals a small pericardial effusion.

Which one of the following is the most appropriate initial treatment?

A) $-Blockers
B) Nitrates
C) Glucocorticoids
D) NSAIDs

29. A 4-year-old male has a BMI of 17.5 kg/m2, which places him between the 90th and 95th
percentiles for BMI. According to the CDC, he should be classified as being

A) at a healthy weight
B) overweight
C) obese
D) morbidly obese

8
30. A 13-year-old female is being evaluated for primary amenorrhea. On examination she has short
stature, a webbed neck, and a low hairline. A physical examination reveals no signs of pubertal
development.

Which one of the following is most likely to provide a diagnosis?

A) MRI of the pituitary


B) FSH and LH levels
C) A prolactin level
D) Pelvic ultrasonography
E) Karyotyping

31. A 71-year-old female comes in for follow-up of hypertension. She is worried about her heart and
says that some of her friends have had stress tests and she would like to get one as well just to
be on the safe side. She has no chest pain, shortness of breath, or exercise intolerance, and a
complete review of systems is negative.

The patient’s current medications include lisinopril (Prinivil, Zestril), 20 mg daily; metoprolol
succinate (Toprol-XL), 25 mg daily; and omeprazole (Prilosec), 20 mg daily. Her past medical
history includes hypertension, obesity, and gastroesophageal reflux disease. A physical
examination reveals a blood pressure of 130/70 mm Hg, a heart rate of 90/min, and a BMI of
31.2 kg/m2. An EKG 2 years ago was normal.

Which one of the following should be ordered to assess this patient’s cardiovascular risk?

A) A lipid profile
B) A coronary artery calcification score
C) A C-reactive protein level
D) An EKG
E) An exercise stress test

32. Which one of the following is true regarding respiratory syncytial virus (RSV) infection?

A) Most infections in the United States occur between August and December
B) Corticosteroids should be a routine part of treatment
C) The diagnosis is usually based on positive serology
D) It is rarely associated with bacterial co-infection

9
33. An 80-year-old female is seen for progressive weakness over the past 8 weeks. She says she now
has difficulty with normal activities such as getting out of a chair and brushing her teeth. Her
medical problems include hypertension, diabetes mellitus, and hyperlipidemia. Her medications
include glipizide (Glucotrol), simvastatin (Zocor), and lisinopril (Prinivil, Zestril). Findings on
examination are within normal limits except for diffuse proximal muscle weakness and normal
deep tendon reflexes. A CBC, urinalysis, erythrocyte sedimentation rate, TSH level, and serum
electrolyte levels are normal. Her blood glucose level is 155 mg/dL and her creatine kinase level
is 1200 U/L (N 40–150).

Which one of the following is the most likely diagnosis?

A) Statin-induced myopathy
B) Polymyalgia rheumatica
C) Guillain-Barré syndrome
D) Diabetic ketoacidosis

34. A 3-year-old female is brought to your office with coughing and a tactile fever. Her only other
symptom is mild rhinorrhea. She has a temperature of 38.2°C (100.8°F) and is mildly
tachypneic. Her vital signs are otherwise normal and she appears to be well and in no respiratory
distress. Her examination is unremarkable except for decreased breath sounds and crackles in
the right lower lung field. She has no allergies to medications.

Which one of the following would be the most appropriate treatment?

A) Amoxicillin
B) Azithromycin (Zithromax)
C) Cefdinir
D) Moxifloxacin (Avelox)
E) Ceftriaxone (Rocephin)

35. When compared to a figure-of-eight dressing, which one of the following modalities of treatment
has been shown to have similar fracture-healing outcomes and increased patient satisfaction for
nondisplaced mid-shaft clavicular fractures?

A) A shoulder sling
B) A short arm cast
C) A long arm cast
D) Operative fixation

10
36. The mother of a 6-year-old male is concerned about his snoring, and she recently observed him
stop breathing for a few seconds while he was sleeping. He has also been more sleepy during
the day recently. His height and weight are normal. Polysomnography confirms obstructive sleep
apnea.

Which one of the following would be the most appropriate primary treatment?

A) Methylphenidate (Ritalin)
B) Lorazepam (Ativan)
C) Fluoxetine (Prozac) on a daily basis
D) A mouthguard
E) Adenotonsillectomy

37. A 70-year-old male sees you for a routine annual evaluation. He complains of fatigue but has
no other symptoms. He has a history of hypertension but has not fully adhered to his drug
regimen, which includes hydrochlorothiazide, amlodipine (Norvasc), and lisinopril (Prinivil,
Zestril).

Laboratory Findings

Hemoglobin. . . . . . . . . . . . . . . . . . . . . . . . . . . 9.0 g/dL (N 13.5–17.2)


Serum creatinine. . . . . . . . . . . . . . . . . . . . . . . 2.2 mg/dL (N 0.6–1.2)
Glomerular filtration rate. . . . . . . . . . . . . . . . 26 mL/min/1.73 m2
Serum iron. . . . . . . . . . . . . . . . . . . . . . . . . . . . 30 :g/dL (N 60–170)
Total iron binding capacity. . . . . . . . . . . . . . . 300 :g/dL (N 240–450)
Ferritin. . . . . . . . . . . . . . . . . . . . . . . . . . . . . . . 55 ng/mL (N 46–100)
Mean corpuscular volume.. . . . . . . . . . . . . . . 77 :m3 (N 80–100)

One year ago the patient had a serum creatinine level of 2.0 mg/dL. A colonoscopy 6 months
ago was unremarkable and a stool test for occult blood is negative.

Which one of the following would be most appropriate at this point?

A) An erythropoietin level
B) Transfusion of packed RBCs
C) Epoetin alfa (Procrit)
D) Ferrous sulfate orally
E) Intravenous iron therapy

38. Which one of the following is most likely to be seen with diastolic dysfunction?

A) A dilated left ventricle


B) A preserved ejection fraction
C) Aortic insufficiency
D) Pericardial effusion

11
39. Slipped capital femoral epiphysis is most likely in which one of the following patients with no
history of trauma?

A) A 3-day-old male with a subluxable hip


B) A 7-year-old male with groin pain and a limp
C) A 13-year-old male with knee pain
D) A 16-year-old female with lateral thigh numbness

40. A 43-year-old female smoker has type 2 diabetes mellitus, morbid obesity, and a recent
diagnosis of symptomatic peripheral arterial disease. You have started her on atorvastatin
(Lipitor), offered a supervised exercise program, and discussed smoking cessation and
interventions.

Which one of the following should be recommended to prevent cardiovascular events in this
patient?

A) Aspirin
B) Cilostazol (Pletal)
C) Enoxaparin (Lovenox)
D) Pentoxifylline
E) Warfarin (Coumadin)

41. A 56-year-old female comes in for evaluation of gradually worsening right hip pain. She
describes her pain as located in the groin and dull in nature, and with activity often notes a
clicking sensation associated with sharp pain. On examination her hip range of motion is intact
but pain is elicited with extremes of internal and external rotation and her groin pain is
exacerbated with the FABER test (knee flexion, abduction and external rotation of the leg until
the ankle rests proximal to the contralateral knee) and FADIR test (knee flexion, adduction, and
internal rotation of the leg).

Which one of the following is the most likely diagnosis?

A) Femoral neck fracture


B) Femoral hernia
C) Trochanteric bursitis
D) Hip labral tear

42. A doctor and patient are discussing using a particular drug to treat the patient’s uncontrolled
hypertension. Which one of the following potential effects of the drug is a patient-oriented
outcome that should be discussed during shared decision-making?

A) A decrease in diastolic blood pressure


B) A decrease in hemoglobin A1c
C) A decrease in carotid intimal thickness
D) A decrease in all-cause mortality
E) Improvement in the Framingham cardiac risk score

12
43. A 12-month-old male is brought to your office for a routine well child visit. His father has
epilepsy and takes seizure medication.

Which one of the following vaccines will slightly increase the child’s risk of a febrile seizure for
up to 2 weeks after administration?

A) Hepatitis B
B) MMR
C) HiB
D) Pneumococcal
E) Polio

44. A right-hand–dominant 38-year-old male comes to your office because of right elbow pain. He
recently began participating in a highly competitive adult volleyball league, and 2 weeks after
he first began playing he developed mild pain in the medial elbow of his right arm. While
completing an overhead serve last night he felt an acute worsening of the elbow pain. After the
match he noted bruising over his medial elbow.

When you examine him you find bruising and pain to palpation around the medial elbow. With
his shoulder in 90° of abduction and external rotation you rapidly flex and extend the elbow
while maintaining valgus torque on the elbow (the moving valgus stress test). The patient reports
pain between 70° and 120° of flexion.

This clinical presentation is most consistent with which one of the following causes of elbow
pain?

A) Medial epicondylitis
B) Biceps tendinopathy
C) Cubital tunnel syndrome
D) Ulnar collateral ligament injury
E) Triceps tendinopathy

45. Which one of the following is true regarding the live attenuated intranasal influenza vaccine?

A) It is preferred in all children >6 months of age


B) It is more effective in children age 2–6 years than the inactivated vaccine
C) It is more effective in children >6 years of age than in younger children
D) It is the vaccine of choice for pregnant women
E) It is less effective in adults age 18–49 than the inactivated vaccine

13
46. A 63-year-old female with corticosteroid-dependent COPD has developed pneumonia. Which
one of the following pathogens should the antibiotic regimen cover in this patient that would be
unlikely in someone with pneumonia and otherwise healthy lungs?

A) Streptococcus pneumoniae
B) Mycoplasma pneumoniae
C) Haemophilus influenzae
D) Staphylococcus aureus
E) Pseudomonas aeruginosa

47. A 30-year-old male presents to your office because he thinks he may be suffering from alcohol
withdrawal. He was dependent on alcohol for at least 10 years and has completed treatment
programs twice. He had been abstinent for over a year until he began drinking heavily after his
wife filed for divorce 2 weeks ago. A friend found him in a bar last night and has kept him from
consuming alcohol for the past 12 hours.

The patient is now nauseated, miserable, restless, shaky, and sweating, and says he can feel his
heart pounding. He has not had any seizures or episodes of delirium tremens. His temperature
is 37.5°C (99.6°F), pulse rate 100 beats/min, and blood pressure 150/92 mm Hg. His palms are
moist and he has a mild tremor on arm extension. He is oriented but cannot perform serial
additions. A CBC, basic metabolic panel, and urine drug screen are normal.

You decide that outpatient treatment would be appropriate. Which one of the following alcohol
withdrawal management options is supported by the best evidence?

A) Thiamine and magnesium


B) Carbamazepine (Tegretol)
C) Phenytoin (Dilantin)
D) Chlordiazepoxide
E) Clonidine (Catapres)

48. In the United States, cow’s milk is not recommended for children until the age of

A) 4 months
B) 6 months
C) 9 months
D) 12 months
E) 15 months

49. A 15-year-old male presents to the emergency department after suffering a lateral dislocation of
his patella. Which one of the following would be the best method for reducing this dislocation?

A) Medially directed pressure on the patella while extending the leg


B) Medially directed pressure on the patella while flexing the leg
C) Rapid leg extension
D) Lateral retinacular release

14
50. In a 15-year-old female with no known chronic medical conditions, which one of the following
is essential before initiating oral combined hormonal contraception?

A) Bimanual pelvic examination


B) Clinical breast examination
C) Cervical cytology and HPV screening
D) Blood pressure measurement
E) Weight measurement

51. A 68-year-old male with end-stage lung cancer is being treated for pain secondary to multiple
visceral and skeletal metastases. He has been on oral ibuprofen and parenteral morphine.
However, over the past few weeks he reports progressive worsening of his pain. In order to
achieve better pain control his morphine dosage has been continuously titrated up. In spite of this
increase he continues to report severe pain that is now diffuse and occurs even when his
caregivers touch him.

Which one of the following would be most appropriate at this time?

A) Increase the morphine dosage until continuous sedation is obtained


B) Attempt a reduction in the morphine dosage
C) Add an anxiolytic to help relieve anxiety
D) Advise the family that nothing more can be done for his pain

52. A 42-year-old male with a 4-year history of multiple sclerosis (MS) presents with an acute attack
manifested by ataxia, incoordination, and dysarthria. Which one of the following is indicated for
managing this flare-up of his MS?

A) Fingolimod (Gilenya)
B) Glatiramer (Copaxone)
C) Interferon-$ (Avonex, Betaseron)
D) Methylprednisolone (Medrol)
E) Pramipexole (Mirapex)

53. A 24-year-old female presents to the emergency department because she thinks she is having an
allergic reaction to her medication for depression. About 3 hours after taking her first dose of
citalopram (Celexa) she noted extreme anxiety, agitation, palpitations, and a dry mouth. On
examination she has a blood pressure of 180/110 mm Hg, a pulse rate of 120 beats/min, a
respiratory rate of 24/min, and a temperature of 37.2°C (99.0°F). Her pupils are dilated and
she has slow, continuous horizontal eye movements. Marked hyperreflexia is noted in the lower
extremities.

In addition to supportive care, the patient should be given intravenous

A) propranolol
B) diphenhydramine
C) haloperidol lactate (Haldol Lactate)
D) flumazenil (Romazicon)
E) diazepam

15
54. In a patient with sepsis, which one of the following would confirm a diagnosis of septic shock?

A) A 1.0 mg/dL increase in the creatinine level


B) A platelet count of 20,000/mm3 (N 150,000–350,000)
C) A WBC count of 25,000/mm3 (N 4300–10,800)
D) A serum bilirubin level of 7.0 mg/dL (N <1.0)
E) A serum lactate level of 2.0 mmol/L (N 0.5–1.0)

55. A study finds that the positive predictive value of a new test for breast cancer is 75%, which
means that

A) among patients with known breast cancer who had the test, 75% had a positive test
B) among patients with no breast cancer who had the test, 75% had a negative test
C) 75% of patients who tested positive actually had breast cancer
D) 75% of patients who tested negative did not have breast cancer

56. A 49-year-old male brings you a copy of his laboratory results obtained during an insurance
examination. The patient says he feels fine, but his bilirubin level was 2.5 mg/dL (N <1.0). He
says he averages 5 alcoholic beverages per week and takes no medications other than occasional
ibuprofen. On examination he is not jaundiced and has no scleral icterus, and the remainder of
the examination is within normal limits, including palpation of the liver and spleen. Laboratory
testing reveals a normal CBC, normal liver enzyme levels, and normal serum haptoglobin.
Bilirubin fractionation reveals an indirect level of 2.0 mg/dL and a direct level of 0.5 mg/dL (N
<0.4).

The most likely diagnosis is

A) asymptomatic cholecystitis
B) alcoholic liver disease
C) Gilbert’s syndrome
D) hemolytic anemia

57. A healthy 18-year-old female sees you for a preparticipation evaluation and well care visit prior
to soccer season. She has no significant previous medical history and no current problems. She
says she is not sexually active. She has completed the HPV vaccine series.

Which one of the following would be most appropriate for cervical cancer screening for this
patient?

A) No screening at this visit


B) Annual Papanicolaou tests
C) Papanicolaou testing alone every 3 years
D) Papanicolaou testing and HPV testing every 3 years

16
58. Which one of the following can help to minimize the pain of lidocaine (Xylocaine) injection?

A) Slowly inserting the needle through the skin


B) Avoiding injection into the subcutaneous tissue
C) Injection of the solution only after fully inserting the needle at the target site
D) Cooling the solution to refrigerator temperature prior to injecting it
E) Buffering the solution with sodium bicarbonate

59. Which one of the following is most appropriate for patients with asplenia?

A) Lifelong daily antibiotic prophylaxis


B) Antibiotics for any episode of fever
C) An additional dose of Hib vaccine
D) Avoiding live attenuated influenza vaccine
E) Withholding pneumococcal vaccine

60. A 37-year-old graphic designer presents to your office with a history of several months of radial
wrist pain. She does not recall any specific trauma but notes that it hurts to hold a coffee cup.
Finkelstein’s test is positive and a grind test is negative, and there is tenderness to palpation over
the radial tubercle.

Which one of the following would be most appropriate at this point?

A) Plain radiography focusing on the scaphoid


B) Rest and a thumb spica wrist splint
C) MRI of the wrist
D) A short arm cast

61. A 19-year-old college wrestler presents with cellulitis of his left arm extending from a small
pustule on his hand to the axilla. He appears acutely ill and has a temperature of 38.9°C
(102.0°F). His WBC count is 22,000/mm3 (N 4300–10,800). He is admitted to the hospital.

The initial drug of choice for this patient would be

A) ciprofloxacin (Cipro)
B) clindamycin (Cleocin)
C) doxycycline
D) trimethoprim/sulfamethoxazole
E) vancomycin

17
62. Which one of the following is an indication for a second dose of pneumococcal polysaccharide
vaccine in children?

A) A cerebrospinal fluid leak


B) Cyanotic congenital heart disease
C) Type 1 diabetes mellitus
D) Sickle cell disease
E) Chronic bronchopulmonary dysplasia

63. A 66-year-old male who was hospitalized 2 months ago for an episode of heart failure sees you
for follow-up. He complains of pain in his chest and on examination you note tenderness and a
slight fullness deep to his nipple bilaterally.

Which one of the following drugs on his medication reconciliation list is most likely to cause this
type of discomfort?

A) Digoxin (Lanoxin)
B) Enalapril (Vasotec)
C) Eplerenone (Inspra)
D) Hydralazine
E) Spironolactone (Aldactone)

64. A 30-year-old female reports that she and her husband have not been able to conceive after
trying for 15 months. She takes no medications, has regular menses, and has no history of
headaches, pelvic infections, or heat/cold intolerance. Her physical examination is
unremarkable. Her husband recently had a normal semen analysis.

Which one of the following would be the most appropriate next step?

A) Observation for 1 year


B) TSH, free T4, and prolactin levels
C) Hysterosalpingography
D) An estradiol level
E) A luteal-phase progesterone level

18
65. A 48-year-old male sees you for a routine health maintenance examination. His blood pressure
is 142/90 mm Hg and you recommend that he return for a repeat blood pressure measurement.
Eight weeks later his blood pressure is 138/88 mm Hg. He denies any symptoms on a review
of systems. He tells you that on his 40th birthday he abruptly stopped smoking after smoking a
pack of cigarettes a day since his early twenties. He is adopted and cannot provide a family
history.

According to U.S. Preventive Services Task Force guidelines, which one of the following
conditions should this patient be screened for now?

A) Abdominal aortic aneurysm


B) Peripheral arterial disease
C) Colon cancer
D) Type 2 diabetes mellitus
E) Hemochromatosis

66. Which one of the following conditions is the leading cause of death for patients with rheumatoid
arthritis?

A) Infections
B) Coronary artery disease
C) Thromboembolic disease
D) Lymphoma
E) Lung cancer

67. A 67-year-old male presents with a persistent, intermittent cough. He says that his exercise
tolerance has decreased, noting that he becomes short of breath more easily while playing tennis.
He smoked briefly while in college but has not smoked for over 45 years, and reports no history
of known pulmonary disease.

You obtain pulmonary function testing in the office to help you diagnose and manage his
respiratory symptoms. His FVC and FEV1/FVC are both less than the lower limit of normal as
defined by the Third National Health and Nutrition Examination Survey. Repeat testing
following administration of a bronchodilator does not correct these values.

Which one of the following would be most appropriate at this time?

A) A methacholine challenge test


B) A mannitol inhalation challenge test
C) Exercise pulmonary function testing
D) Testing for diffusing capacity of the lung for carbon monoxide (DLCO)

19
68. You see a 5-year-old white female with in-toeing due to excessive femoral anteversion. She is
otherwise normal and healthy, and her mobility is unimpaired. Her parents are greatly concerned
with her appearance and possible future disability, and request that she be treated.

You recommend which one of the following?

A) Observation
B) Medial shoe wedges
C) Torque heels
D) Sleeping in a Denis Browne splint for 6 months
E) Derotational osteotomy of the femur

69. Effective treatments for obsessive-compulsive disorder include

A) Freudian analysis
B) benzodiazepines
C) amphetamine salts
D) atypical antipsychotics
E) repetitive exposure to fearful stimuli

70. A 77-year-old male presents with significant postherpetic neuralgia in a chest wall distribution.
Which one of the following is most likely to be effective in diminishing his discomfort?

A) Oral valacyclovir (Valtrex)


B) Topical lidocaine (Xylocaine) patches
C) Thoracic epidural corticosteroid injections
D) Herpes zoster vaccine
E) Acupuncture

71. A 50-year-old male presents to your office with a 1-hour history of an intense retro-orbital
headache. This started while he was jogging and eased somewhat when he stopped, but has
persisted along with some pain in his neck. Other than a blood pressure of 165/100 mm Hg, his
examination is unremarkable. Noncontrast CT of the head is also unremarkable. His pain has
persisted after 2 hours in the emergency department.

Which one of the following would be most appropriate at this time?

A) MRI of the head


B) Angiography
C) Nifedipine (Procardia) sublingually
D) Sumatriptan (Imitrex) subcutaneously
E) A lumbar puncture

20
72. A healthy 68-year-old male is seen in December for a routine examination. A review of his
immunizations indicates that he received a standard dose of inactivated influenza vaccine at the
health clinic in September. He received 23-valent pneumococcal vaccine (Pneumovax 23) at age
65.

He should now receive which one of the following?

A) High-dose influenza vaccine


B) 13-valent pneumococcal conjugate vaccine (Prevnar 13)
C) 23-valent pneumococcal vaccine
D) No vaccines at this time

73. A 68-year-old female with diabetes mellitus, coronary artery disease, fibromyalgia, and
dyspepsia presents for follow-up. She has been taking omeprazole (Prilosec) for 10 years. It was
started during a hospitalization, and her symptoms have returned with previous trials of
discontinuation.

Which one of the following adverse events is this patient at risk for as a result of her omeprazole
use?

A) Hypermagnesemia
B) Urinary tract infections
C) Nephrolithiasis
D) Hip fractures

74. A 14-year-old female bumped heads with another player in a soccer game. She was knocked
down, appeared briefly dazed, and now has a headache and mild dizziness while seated on the
sidelines.

Which one of the following would be most appropriate at this point?

A) Return to play after symptoms have resolved for at least 30 minutes


B) Immediate neuroimaging to rule out intracranial injury
C) Complete cognitive and physical rest for 24 hours before returning to normal activities
D) Initial complete cognitive and physical rest followed by an individualized graded return
to activity
E) No sports participation until symptoms have been absent for 1 week

21
75. A 62-year-old male comes to your office as a new patient. He has a past history of a myocardial
infarction and is currently in stage C heart failure according to the American Heart Association
classification. His ejection fraction is 30%.

Which one of the following medications that the patient is currently taking is potentially harmful
and should be discontinued if possible?

A) Diltiazem (Cardizem)
B) Lisinopril (Prinivil, Zestril)
C) Carvedilol (Coreg)
D) Spironolactone (Aldactone)
E) Atorvastatin (Lipitor)

76. You evaluate an 18-month-old male with fecal impaction and determine that disimpaction is
indicated. Which one of the following would be most appropriate initially?

A) An oral stimulant such as sennosides (Senokot)


B) An oral osmotic agent such as polyethylene glycol 3350 (MiraLax)
C) An enema using saline, mineral oil, or phosphate soda
D) A bisacodyl (Dulcolax) rectal suppository
E) Manual disimpaction

77. A school nurse discovers head lice on a fourth-grade student. When should the student be
permitted to return to class?

A) Immediately
B) When there are no visible nits
C) After a single treatment with a topical agent
D) After two treatments with a topical agent, 7 days apart

78. A 42-year-old female presents with a cough productive of blood-streaked sputum for the past 3
days. Her hemoptysis was preceded by several days of rhinorrhea, congestion, and subjective
fever. She estimates the total amount of blood loss to be approximately 1 tablespoon. She is a
nonsmoker and her past medical history is unremarkable. Vital signs are within normal limits,
and other than an intermittent cough there are no abnormal findings on the physical examination.

Which one of the following would be the most appropriate next step?

A) Observation
B) A chest radiograph
C) Chest CT
D) Bronchoscopy
E) Antibiotics

22
79. A 33-year-old female presents with highly pruritic raised wheals on her extremities and torso.
They only last for a few hours but have recurred over the last several days. There has been no
oral swelling or respiratory symptoms.

Which one of the following is the best first-line treatment for this condition?

A) Topical corticosteroids
B) H1-histamine blockers
C) H2-histamine blockers
D) Leukotriene-receptor antagonists
E) Injectable epinephrine

80. A 30-year-old female is referred to you by a local optometrist after she was treated several times
for anterior uveitis. You are concerned about an associated systemic disease. She feels well
otherwise, and denies back or joint pain, rash, cough, or fever. A chest radiograph reveals
enlarged mediastinal lymph nodes.

Which one of the following is most likely to be associated with her recurrent uveitis?

A) Cat-scratch disease
B) Lyme disease
C) Sarcoidosis
D) Syphilis
E) Tuberculosis

81. In addition to exercise, which one of the following vitamin supplements is recommended by the
U.S. Preventive Services Task Force to help prevent falls in elderly patients living at home?

A) A
B) B complex
C) C
D) D
E) E

82. Which one of the following is a significant risk factor for esophageal adenocarcinoma?

A) Aspirin therapy
B) Ibuprofen therapy
C) Helicobacter pylori infection
D) Obesity
E) Crohn’s disease

23
83. In older patients with aortic stenosis and a systolic murmur, which one of the following would
be most concerning?

A) Weight loss
B) Frequent urination
C) Jaundice
D) Worsening headache
E) Exertional dyspnea

84. A 30-year-old female presents with dysuria and flank pain. She reports a fever of 102°F
yesterday morning. She has not taken any antipyretics since that time, and today her temperature
is 36.7°C (98.1°F). She has a pulse rate of 93 beats/min, a respiratory rate of 16/min, and a
blood pressure of 116/58 mm Hg. The remainder of her physical examination is unremarkable,
except for marked costovertebral angle tenderness.

A CBC reveals a WBC count of 14,590/mm3 (N 4300–10,800) with 85% neutrophils, 12%
lymphocytes, and 3% basophils, but is otherwise normal. A urine $-hCG is negative. A urine
dipstick is positive for leukocyte esterase, and urine microscopic analysis is notable for <1 RBC
and >50 WBCs/hpf. Urine culture results are pending.

You confirm she has no medication allergies. Which one of the following oral antibiotics would
be most appropriate for empiric therapy?

A) Amoxicillin
B) Ciprofloxacin (Cipro)
C) Erythromycin
D) Metronidazole (Flagyl)
E) Nitrofurantoin (Furadantin)

85. A 40-year-old male respiratory therapist presents for a health examination prior to hospital
employment. His history indicates that as a child he lived on a farm in Iowa. His examination
is unremarkable, but a chest radiograph shows that both lung fields have BB-sized calcifications
in a miliary pattern. No other findings are noted. A PPD skin test is negative.

The findings in this patient are most likely a result of

A) HIV infection
B) histoplasmosis
C) coccidioidomycosis
D) tuberculosis
E) cryptococcosis

24
86. A 43-year-old female complains of easy bruising. She is otherwise asymptomatic. A CBC
reveals a platelet count of 23,000/mm3 (N 150,000–450,000). A peripheral smear reveals giant
platelets. A workup is negative for autoimmune causes, including Graves disease, HIV,
Epstein-Barr virus, cytomegalovirus, varicella zoster, hepatitis C, and Helicobacter pylori. She
is on no prescription or over-the-counter medications and denies alcohol or drug use.

Which one of the following would be the most appropriate initial management?

A) Platelet transfusion
B) Corticosteroids
C) Thrombopoietin-receptor agonists
D) A bone marrow biopsy
E) Splenectomy

87. A 22-year-old female with a 2-week history of paroxysmal cough is found to have pertussis
confirmed by a polymerase chain reaction test and a nasal swab culture. Which one of the
following is the antibiotic of choice for this patient?

A) Amoxicillin
B) Azithromycin (Zithromax)
C) Ciprofloxacin (Cipro)
D) Clindamycin (Cleocin)
E) Doxycycline

88. While performing a digital rectal examination of the prostate on a 67-year-old patient with
diabetes mellitus, you note the findings shown below. The patient confirms that the area has been
itchy for some time but he has been reluctant to seek care. He has tried a variety of
over-the-counter moisturizing lotions with limited success.

Of the following topical treatments, which one is most likely to provide significant improvement?

A) Antibacterial ointment
B) Antifungal cream
C) Antiviral ointment
D) Corticosteroid cream
E) Rubbing alcohol

25
89. A 73-year-old male is seen for follow-up of elevated blood pressure. He has no comorbidities.
His blood pressure after several months of lifestyle modifications is 160/102 mm Hg. He is
started on lisinopril (Prinivil, Zestril), 10 mg daily.

According to the JNC 8 panel, the blood pressure goal for this patient is which one of the
following?

A) <160/100 mm Hg
B) <150/90 mm Hg
C) <140/90 mm Hg
D) <130/85 mm Hg
E) 120/80 mm Hg

90. Terminally ill cancer patients who receive palliative chemotherapy

A) survive longer
B) are less likely to die at home
C) are less likely to undergo CPR
D) are less likely to undergo mechanical ventilation
E) are referred to hospice earlier in their disease course

91. A 14-year-old male presents to your office with a high fever that began suddenly. He has a
diffuse petechial rash and some nuchal rigidity on examination. A lumbar puncture is performed,
and gram-negative diplococci are found. You admit him to the hospital for treatment.

Which one of the following would be most appropriate for prevention of secondary disease at
this time?

A) Immediate chemoprophylaxis for his entire school


B) Immediate vaccination of all contacts
C) Chemoprophylaxis for family members and very close contacts only
D) Isolation of all family members for 1 week
E) No preventive measures until culture results are available

92. A 58-year-old male with COPD presents with a 5-day history of increased dyspnea and purulent
sputum production. He is afebrile. His respiratory rate is 24/min, heart rate 90 beats/min, blood
pressure 140/80 mm Hg, and oxygen saturation 90% on room air. Breath sounds are equal, and
diffuse bilateral rhonchi are noted. He is currently using albuterol/ipratropium by metered-dose
inhaler three times daily.

In addition to antibiotics, which one of the following would be most appropriate for treating this
exacerbation?

A) A single dose of intramuscular dexamethasone


B) Oral prednisone for 5 days
C) Daily inhaled fluticasone (Flovent)
D) Hospital admission for intravenous methylprednisolone sodium succinate (Solu-Medrol)
E) No corticosteroids at this time

26
93. A 25-year-old male daycare worker presents with a 3-week history of bloating and foul-smelling
stools. On examination the patient has mild, diffuse abdominal tenderness and increased bowel
sounds.

Which one of the following is the most likely cause of this patient's problem?

A) Hepatitis A
B) Clostridium difficile
C) Enterotoxigenic Escherichia coli
D) Giardia lamblia
E) Campylobacter

94. A 60-year-old female has a strong family history of breast cancer and is considering tamoxifen
(Soltamox) to reduce her risk. Which one of the following is an effect associated with this
treatment that should be included in the shared decision-making discussion with the patient?

A) An increased risk of bone fractures


B) An increased risk of endometrial cancer
C) A reduction in leg cramps
D) A decreased risk of thromboembolic events
E) A reduction in vasomotor symptoms

95. A 57-year-old male presents to the emergency department complaining of dyspnea, cough, and
pleuritic chest pain. A chest radiograph shows a large left-sided pleural effusion. Thoracentesis
shows a pleural fluid protein to serum protein ratio of 0.7 and a pleural fluid LDH to serum
LDH ratio of 0.8.

Which one of the following causes of pleural effusion would be most consistent with these
findings?

A) Cirrhosis
B) Heart failure
C) Nephrotic syndrome
D) Pulmonary embolism
E) Superior vena cava obstruction

96. A copper T 380A intrauterine device (ParaGard) would be preferred over a


levonorgestrel-releasing intrauterine device (Mirena) in a patient with a history of which one of
the following?

A) Nulliparity
B) Current smoking
C) Acute deep vein thrombosis
D) Severe cirrhosis
E) Heart failure

27
97. The mother of a 2-year-old calls you for advice because her child has an acute cough that is
keeping him awake at night. Which one of the following has been shown in a double-blind,
randomized, placebo-controlled study to decrease nighttime cough and improve sleep in children
with this problem?

A) Sugar water
B) Cinnamon
C) Turmeric
D) Ginger
E) Honey

98. Which one of the following is an effect of long-term treatment for narcotic addiction with
methadone and buprenorphine?

A) Greater success at producing minimal opiate use than detoxification programs


B) Significant teratogenic effects
C) Frequent diversion of opiates
D) Decreased associated cocaine abuse

99. How many arteries and veins are normally found in the umbilical cord on a newborn
examination?

A) 1 artery, 1 vein
B) 1 artery, 2 veins
C) 2 arteries, 1 vein
D) 2 arteries, 2 veins
E) 2 arteries, 3 veins

100. A 12-month-old male is brought to your office by his mother because of concerns about his
eating. She states that he throws tantrums while sitting in his high chair, dumps food on the
floor, and refuses to eat. She has resorted to feeding him cookies, crackers, and juice, which
are “all he will eat.” A complete physical examination, including a growth chart of weight,
length, and head circumference, is normal.

Which one of the following would be the most appropriate recommendation?

A) Use disciplinary measures to force the child to eat a healthy breakfast, lunch, and dinner
B) Leave the child in the high chair until he has eaten all of the healthy food provided
C) Play feeding games to encourage consumption of healthy meals or snacks
D) Skip the next meal if the child refuses to eat
E) Provide healthy foods for all meals and snacks, and end the meal if the child refuses to
eat

28
101. A 55-year-old female presents with lateral hip pain over the outer thigh. She has no history of
injury, although she has just begun a walking program to lose weight. She has increased pain
when she lies on that side at night. Her examination is unremarkable except that she is
overweight and has tenderness over the greater trochanter. There is no pain with internal and
external rotation of the hip. A radiograph reveals minimal osteoarthritic changes.

Which one of the following would be most appropriate at this point?

A) Serum protein electrophoresis


B) A bone scan
C) A bone density study
D) MRI
E) A corticosteroid injection

102. A 70-year-old male who recently moved to your area sees you for the first time. He has a
previous history of myocardial infarction, has a pacemaker, and has hypertension that had been
well controlled on hydrochlorothiazide and atenolol (Tenormin) for several years. About 6
months ago his previous physician had to add amlodipine (Norvasc) to his regimen. On
examination he has mild arteriolar narrowing in his fundi and there is a systolic bruit just to the
right of his umbilicus. He has a log of home blood pressure readings that average 138/88 mm
Hg for the past 2 months. His serum creatinine level has gone from 1.2 mg/dL to 1.4 mg/dL (N
0.6–1.2) in the past 2 months.

Which one of the following would be most appropriate at this time?

A) Referral for stent placement


B) Scheduling an arteriogram
C) A captopril renal scan
D) Adding losartan (Cozaar) to his regimen
E) Continued monitoring of serum creatinine

103. A 25-year-old male presents with a 3-day history of cough, chills, and fever. The patient was
previously healthy and has no chronic medical problems. He has no known drug allergies. On
examination he is alert and oriented, and has a temperature of 38.4°C (101.1°F), a pulse rate
of 88 beats/min, a blood pressure of 120/70 mm Hg, a respiratory rate of 16/min, and an
oxygen saturation of 98%.

Auscultation of the lungs reveals no wheezing and the presence of right basilar crackles. A chest
radiograph shows a right lower lobe infiltrate.

There is a low rate of macrolide-resistant pneumococcus in the community. Which one of the
following is the most appropriate initial management of this patient?

A) Outpatient treatment with azithromycin (Zithromax)


B) Outpatient treatment with cefuroxime (Ceftin)
C) Inpatient treatment on the medical floor with ceftriaxone (Rocephin) and azithromycin
D) Inpatient treatment on the medical floor with piperacillin/tazobactam (Zosyn) and
levofloxacin
E) Inpatient treatment in the intensive-care unit with ceftriaxone, levofloxacin, and
vancomycin (Vancocin)

29
104. A mother brings in her 10-year-old son because of a swollen area in his neck that she first
noticed yesterday. He has also had symptoms of an upper respiratory infection. On examination
the child has a runny nose but otherwise appears well. Palpation reveals a soft, 1.5-cm, slightly
tender mass, inferior to the angle of the mandible and anterior to the sternocleidomastoid muscle.

The most likely diagnosis is

A) thyroglossal duct cyst


B) dermoid cyst
C) branchial cleft cyst
D) thyroid tumor

105. Treatment of rhabdomyolysis should routinely include which one of the following?

A) Bicarbonate-containing fluids
B) Loop diuretics
C) Mannitol
D) Parenteral corticosteroids
E) Isotonic saline

106. A 26-year-old pet groomer sustained a dog bite to her left hand 2 hours ago. On examination a
4-cm × 2.5-cm laceration is noted on the thenar eminence of her palm. Although the wound
shows some gaping there is minimal active bleeding. No neurovascular injury is noted.

Which one of the following is an indication for antibiotics in this patient?

A) A wound size >2 cm


B) The presence of wound gaping
C) A bite involving the hand
D) The patient’s occupation

107. You see a 27-year-old male with autosomal dominant polycystic kidney disease. He has no other
medical problems and his renal function has always been normal on annual testing. Today the
patient reports his blood pressure at home has been 142–150/84–90 mm Hg. His blood pressure
at this visit is 145/88 mm Hg.

Which one of the following medications is preferred for the initial management of hypertension
in this patient?

A) Amlodipine (Norvasc)
B) Chlorthalidone
C) Furosemide (Lasix)
D) Lisinopril (Prinivil, Zestril)

30
108. A 30-year-old female is being evaluated for chronic pain, fatigue, muscle aches, and sleep
disturbance. Which one of the following would be best for making a diagnosis of fibromyalgia?

A) A structured symptom history


B) Examination for tender points
C) Laboratory testing
D) A muscle biopsy
E) Electromyography

109. Sympathomimetic decongestants such as pseudoephedrine and phenylephrine can be problematic


in elderly patients because they can

A) decrease blood pressure


B) cause bradycardia
C) worsen existing urinary obstruction
D) enhance the anticholinergic effects of other medications
E) enhance the sedative effects of other medications

110. You have prescribed oral iron replacement for a 46-year-old female with iron deficiency anemia
related to heavy menses. She wants to be sure that the iron she takes will be absorbed well.

Which one of the following would you suggest for improving iron absorption?

A) Calcium
B) Vitamin C
C) Coffee
D) Tea

111. Which one of the following conditions can affect hemoglobin A1c levels?

A) Heart failure
B) Chronic hemolytic anemia
C) COPD
D) Hypothyroidism

112. Many of the changes that occur as part of aging affect pharmacokinetics. Which one of the
following is INCREASED in geriatric patients?

A) Drug absorption
B) The glomerular filtration rate
C) Lean body mass
D) The volume of distribution of water-soluble compounds such as digoxin
E) The percentage of body fat

31
113. A 60-year-old male with a long-standing history of hypertension seeks your advice about pain
relief from his osteoarthritis. He has tried acetaminophen and topical capsaicin cream without
much benefit. He is concerned about media reports of NSAIDs causing heart problems and is
unsure which ones would be safest for him to use.

Based on current evidence, which one of the following NSAIDs would you recommend as being
LEAST likely to be associated with an increased risk of myocardial infarction?

A) Celecoxib (Celebrex)
B) Diclofenac (Zorvolex)
C) Ibuprofen
D) Meloxicam (Mobic)
E) Naproxen (Naprosyn)

114. A 26-year-old male presents with a sore throat and a temperature of 38.3°C (101.0°F). On
examination you note muffling of the voice and unilateral tonsillar swelling with a shift of the
uvula away from the affected tonsil. A rapid test for Streptococcus pyogenes is negative.

Which one of the following would be most appropriate at this point?

A) Laboratory testing for infectious mononucleosis


B) Immediate tonsillectomy
C) Initiation of antibiotics with close clinical follow-up
D) Culture of the throat and delayed initiation of antibiotics pending results

115. A 7-year-old male is brought to your office with a 2-day history of rash. He developed two itchy
spots on his legs yesterday and today he has multiple purple, slightly painful lesions on his legs.
A few days ago he was ill with cold-like symptoms, stomach pain, and a fever up to 101.2°F.
He complained of leg pain at the time and his left ankle is now swollen. His fever resolved 2
days ago and he now feels fine but limps when he walks.

On examination he is afebrile with a normal blood pressure and pulse rate. He is active in the
examination room. His physical examination is normal except for purpuric lesions on his legs
and buttocks and edema and mild pain of the left ankle. A urinalysis is negative.

Which one of the following would be most appropriate in the management of this patient?

A) Acetaminophen
B) Amlodipine (Norvasc)
C) Amoxicillin
D) Cyclophosphamide
E) Prednisone

32
116. A 32-year-old male smoker presents with a 4-day history of progressive hoarseness. He is almost
unable to speak, and associated symptoms include a cough slightly productive of yellow sputum,
as well as tenderness over the ethmoid sinuses. He is afebrile and has normal ear and lung
examinations. His oropharynx is slightly red with no exudate, and examination of his nasal
passages reveals mucosal congestion.

Which one of the following would be the most appropriate treatment?

A) Amoxicillin for 10 days


B) Omeprazole (Prilosec), 40 mg daily
C) Azithromycin (Zithromax) for 5 days
D) Symptomatic treatment only

117. A 26-year-old female presents with acute low back pain. She says it started a week ago after she
lifted a sofa when helping a friend move. The patient's medical history is otherwise negative.
The patient says the pain is limited to the lower back. The physical examination is normal,
including the neurologic examination.

Which one of the following would be the most appropriate choice for imaging at this time?

A) No imaging
B) A plain film of the lumbar spine
C) MRI of the lumbar spine
D) A DXA scan
E) A PET scan

118. A 50-year-old female sees you for follow-up of uncontrolled hypertension. Her recent blood
pressure measurements average >175/105 mm Hg. The patient has diabetes mellitus and a BMI
of 32.3 kg/m2. Physical findings are otherwise noncontributory. Recent laboratory studies
include three different potassium levels <3.5 mEq/L (N 3.5–5.0) despite increasing dosages of
oral potassium supplements, with the dosage now at 100 mEq daily.

Which one of the following would be most appropriate at this point?

A) Measurement of peripheral aldosterone concentration and peripheral renin activity


B) CT of the abdomen
C) Renal CT angiography
D) An aldosterone suppression test

33
119. You are asked to medically manage a 66-year-old patient who is scheduled for an elective
cholecystectomy. He is also being treated for panhypopituitarism secondary to a pituitary
macroadenoma resection many years ago. His medications include levothyroxine (Synthroid),
125 :g/day; prednisone, 10 mg in the morning and 5 mg in the evening; and fludrocortisone,
10 mg/day.

Preoperative orders for this patient should include which one of the following?

A) Normal saline intravenously as a bolus


B) ACTH daily while on intravenous fluids
C) Hydrocortisone, 25 mg intravenously every 8 hr
D) Levothyroxine, 250 :g intravenously daily

120. In which one of the following patients should a creatine kinase level be obtained to detect
Duchenne muscular dystrophy?

A) A 2-month-old male who is unable to roll over from prone to supine


B) A 7-month-old male who is unable to get into a sitting position unassisted
C) A 15-month-old male who is walking but is unable to stand up from a supine position
without support
D) A 16-month-old male who is not walking unassisted
E) A 6-month-old with high neuromuscular tone on physical examination

121. You see a 4-year-old male in your office for evaluation of persisting fever, rash, and red eyes.
In a discussion with his father you learn that the child has had temperatures in the 99°F–102°F
range for 6 days, along with what the father describes as “pink eye.” Today the child broke out
in a rash on his chest and back and also has cracked red lips. On examination you confirm that
he has bilateral nonpurulent conjunctival injection and a generalized maculopapular rash, as well
as erythema of his hands and feet.

Which one of the following is recommended at this time to evaluate for cardiac complications?

A) An EKG
B) Transthoracic echocardiography
C) Cardiac CT
D) Magnetic resonance (MR) coronary angiography
E) A radionuclide myocardial perfusion scan

122. Which one of the following is true regarding electronic cigarettes?

A) They release lower concentrations of particulate matter than tobacco cigarettes


B) They are a nicotine-free alternative to tobacco
C) They are not regulated by the FDA
D) They have been proven to be effective for smoking cessation
E) They have been shown to be safe in pregnancy

34
123. Which one of the following is the leading cause of human death in the world as a whole?

A) Ischemic heart disease


B) Premature birth
C) Diarrheal diseases
D) HIV/AIDS
E) Cancers of the lungs, bronchi, and trachea

124. A 25-year-old female who is 3 months post partum presents with multiple complaints, including
increasing weakness and fatigue, intolerance to warm environments, a weight loss of 30 lb
despite an increased appetite, difficulty sleeping, awareness that her heart is beating faster and
“pounding” in her chest, increasing restlessness and difficulty concentrating, increased
tremulousness, and a significant swelling in her neck. She takes no medication, has experienced
no recent trauma, and has not ingested large amounts of iodine.

When you examine her you find no exophthalmos or lid lag and no pretibial edema, but her skin
is warm, smooth, and moist. You also find a smooth, non-nodular, nontender, enlarged thyroid
gland, clear lungs, a resting tremor, and hyperactive reflexes.

Laboratory testing reveals a low TSH level, elevated free T3 and free T4, and high uptake on a
radioactive iodine uptake scan.

Which one of the following is the most likely diagnosis?

A) Postpartum thyroiditis
B) Silent thyroiditis
C) Subacute thyroiditis
D) Graves disease
E) Exogenous thyroid ingestion

125. A previously healthy 59-year-old male is brought to the emergency department by his wife, who
describes symptoms of confusion and ataxia. She also says that he has had a fever and cough for
the past 2 weeks. On examination he has a temperature of 39.0°C (102.2°F), a heart rate of 125
beats/min, a respiratory rate of 25/min, a blood pressure of 85/46 mm Hg, and an O2 saturation
of 88%. Laboratory findings include a WBC count of 15,500/mm3 (N 4300–10,800), a
glomerular filtration rate of 45%, and a hemoglobin level of 9.1 g/dL (N 13.0–18.0). A chest
radiograph reveals a large left lower lobe infiltrate.

You start the patient on an appropriate antibiotic regimen. Which one of the following is the
most appropriate initial treatment of this patient’s hypotension?

A) Dobutamine
B) Dopamine
C) Norepinephrine
D) Aggressive fluid resuscitation
E) Packed red blood cell transfusion

35
126. A 17-year-old male high school football running back is hit on the lower leg by an opposing
player’s helmet when the other player dives for a fumble. The running back presents to the
emergency department after the game with significant swelling and bruising of the lower leg.
Symptoms include exceptionally severe pain that is worse with stretching the calf muscles. There
is no weakness of the extremity and sensation is intact. You examine the leg and can palpate
pulses. Plain radiographs do not show a fracture.

Which one of the following should be ordered next?

A) Noninvasive arterial ultrasonography of the leg


B) Noninvasive venous ultrasonography of the leg
C) CT of the calf region
D) MRI of the calf region
E) Tissue pressure studies

127. A 23-year-old healthy male is sexually active with other men and does not use condoms. He is
interested in reducing his risk of contracting HIV by using a daily oral antiretroviral medication.

Which one of the following laboratory tests should be done no more than 7 days before initially
prescribing pre-exposure prophylaxis with emtricitabine/tenofovir disoproxil (Truvada)?

A) A CD4 cell count


B) Antibody testing for HIV
C) Hemoglobin concentration
D) A platelet count
E) An ALT level

128. A patient is admitted to the hospital for acute deep vein thrombosis of the lower extremity and
started on anticoagulation therapy. The nursing staff asks for an activity order.

Which one of the following should be ordered?

A) Activity as tolerated
B) Bed rest until the patient has been hospitalized for 24 hours
C) Bed rest with bathroom privileges until the patient has been hospitalized for 24 hours
D) Bed rest until discharged
E) Bed rest with bathroom privileges until discharged

129. Which one of the following potential bioterrorism agents requires treatment with 60 days of
continuous antibiotics?

A) Anthrax
B) Botulism
C) Pneumonic plague
D) Smallpox
E) Tularemia

36
130. Which one of the following is recommended in all patients with croup, including those with mild
disease?

A) Humidification therapy
B) Oral dexamethasone as a single dose
C) Oral diphenhydramine (Benadryl) every 6 hours until improvement
D) Subcutaneous epinephrine as a single dose
E) Intramuscular ceftriaxone (Rocephin) as a single dose

131. A 17-year-old male presents to the urgent care clinic 15 minutes after being stung by a wasp.
He feels weak, his voice is hoarse, and he is beginning to have trouble breathing.

Which one of the following should be administered first?

A) Intramuscular epinephrine
B) Intravenous diphenhydramine (Benadryl)
C) Intravenous famotidine (Pepcid)
D) Intravenous methylprednisolone sodium succinate (Solu-Medrol)
E) An intravenous bolus of normal saline

132. A pet reptile is most likely to transmit which one of the following to human contacts?

A) Hantavirus
B) Psittacosis (Chlamydophila psittaci)
C) Plague (Yersinia pestis)
D) Pasteurella multocida
E) Salmonella

133. A mother brings in her 2-month-old infant for a routine checkup. The baby is exclusively
breastfed, and the mother has no concerns or questions.

In addition to continued breastfeeding, which one of the following would you recommend
continuing or adding at this time?

A) Iron supplementation
B) Vitamin D supplementation
C) A multivitamin
D) 8 oz of water daily
E) 4 oz of cereal daily

37
134. A 78-year-old white male presents to your office with his daughter for a follow-up visit for his
diabetes. He has a history of peripheral neuropathy and mild Alzheimer’s dementia. He
continues to be socially active in his community. He is on several medications, including insulin
glargine (Lantus), amitriptyline, donepezil (Aricept), and clonazepam (Klonopin). His daughter
asks whether he should continue to drive his car.

Which one of the following would be most appropriate in the context of this office visit with
regard to evaluating his driving safety?

A) A thorough history focused on the patient’s driving, from both him and his daughter
B) A written driving test
C) A road test to observe his driving
D) A letter to the local agency in charge of drivers’ licenses advising license removal

135. Which one of the following is more typical of a keloid rather than a hypertrophic scar?

A) Location on an extensor surface


B) Expansion beyond the margins of the inciting injury
C) Development soon after the inciting trauma
D) Regression over time

136. Which one of the following should be monitored during testosterone replacement therapy?

A) Patient Health Questionnaire 9 (PHQ-9) scores


B) Fasting glucose levels
C) Fasting lipid profiles
D) Hematocrit
E) Overnight polysomnography

137. A 67-year-old male presents with thoracic spine pain and is found to have two thoracic vertebral
compression fractures. He has no history of recent trauma. His general health has been
satisfactory except for a seizure disorder controlled with levetiracetam (Keppra). He does not
smoke and uses alcohol rarely. A CBC, comprehensive metabolic panel, and erythrocyte
sedimentation rate are within normal limits. A DXA scan shows a T-score of –2.8.

Which one of the following would be most appropriate at this time?

A) Protein electrophoresis
B) HIV screening
C) A testosterone level
D) A parathyroid hormone level

38
138. A 45-year-old male was admitted to the hospital for nausea resulting from chemotherapy for
colon cancer. He has no other chronic diseases and takes no routine medications. He was mildly
dehydrated on admission and has been receiving intravenous fluids (D5 ½-normal saline with
potassium chloride) at slightly higher than maintenance rates through an indwelling port for the
last 24 hours. The nausea is being controlled by antiemetics, and his condition is improving.
Results of routine blood work at the time of admission and from the following morning are
shown below.

Test Admission Following Morning

Glucose 109 mg/dL (N 65–110) 371 mg/dL


BUN 13 mg/dL (N 7–21) 9 mg/dL
Creatinine 0.9 mg/dL (N 0.6–1.6) 0.9 mg/dL
Sodium 143 mEq/L (N 136–144) 129 mEq/L
Potassium 3.7 mEq/L (N 3.6–5.1) 6.6 mEq/L
Chloride 110 mEq/L (N 101–111) 108 mEq/L
Total CO2 20 mEq/L (N 22–32) 22 mEq/L

Which one of the following would be the most appropriate next step?

A) Start an intravenous insulin drip


B) Order blood work taken from a peripheral vein
C) Restrict the patient’s free water intake
D) Switch from normal saline to hypertonic saline
E) Treat with diuretics

139. Which one of the following is most likely to be associated with resistant hypertension in adults?

A) Obstructive sleep apnea


B) Primary aldosteronism
C) Renal artery stenosis
D) Renal parenchymal disease
E) Thyroid disease

140. A 78-year-old female presents with a red eye. She reports drainage and pain in her left eye since
she woke up today, but no photophobia. Examination of the eye shows conjunctival erythema
and a mucopurulent discharge. The pupil is normal in size and reactive to light.

Which one of the following should prompt immediate referral to an ophthalmologist?

A) Bilateral eye redness


B) A corneal abrasion noted on fluorescein staining
C) Copious mucopurulent drainage from the eye
D) Bright red blood noted under the conjunctiva
E) Reduction of visual acuity

39
141. A long-term care resident is admitted to the hospital. The patient has a living will which specifies
that “treatment be withheld or withdrawn and that I be permitted to die naturally with only the
administration of medication or the performance of any medical treatment deemed necessary to
alleviate pain.” The patient has appointed his wife as his health care surrogate. He has mild
Alzheimer’s disease and scored 26 out of 30 on a Mini-Mental State Examination performed
within the last month. He is alert and pleasant and responds appropriately to questions but cannot
remember the current date. His wife is with him.

Which one of the following would be most appropriate with regard to decision making and
ordering related to the patient’s code status?

A) Determine the patient’s competence


B) Assess the patient’s decision-making capacity
C) Confirm the code status with the patient’s wife
D) Write a Do Not Resuscitate (DNR) order
E) Order comfort measures only

142. A 42-year-old female presents with a 2-month history of right-sided shoulder pain. A history
reveals that her job requires repetitive motion, including abduction of the shoulder. Ibuprofen
has not been helpful and the pain interferes with her sleep. The physical examination suggests
rotator cuff tendinitis. A radiograph of the shoulder is normal.

You discuss treatment options and the patient decides to proceed with a corticosteroid injection.
Which one of the following is the appropriate anatomic location for the injection?

A) The acromioclavicular joint


B) The subacromial space
C) The intra-articular shoulder joint under fluoroscopy
D) The area of insertion of the deltoid muscle
E) The area of insertion of the long head of the biceps

143. A 35-year-old female asks you about options for weight loss. She weighs 104 kg (229 lb) and
has a BMI of 34 kg/m2. Her health problems include hypertension and depression.

According to the U.S. Preventive Services Task Force, which one of the following is the most
appropriate initial recommendation for weight-loss management in this patient?

A) A high-protein diet
B) A low-carbohydrate diet
C) Behavioral counseling
D) Bariatric surgery

40
144. A 7-year-old male is brought to your office with a 10-day history of cough and fever. A chest
radiograph shows no acute air-space process but four posterior healing rib fractures. The child’s
past medical history is unremarkable.

Which one of the following would be most appropriate at this point?

A) A skeletal survey
B) Studies to evaluate for osteogenesis imperfecta
C) Studies to evaluate for rickets
D) An immediate referral to initiate a child abuse investigation

145. A 50-year-old male sees you for a health maintenance visit. He has not been to a physician for
5 years because he feels very healthy and believed he was up-to-date on all preventive
screenings. You review his medical record and notice he has never had an HIV screening test.
On further questioning you confirm that he is at very low risk for contracting HIV.

Based on recommendations from the U.S. Preventive Services Task Force, you tell him that you
routinely conduct opt-out HIV screening for

A) all patients age 5 to 75


B) all patients age 15 to 65
C) all patients younger than 50, and patients 50 or older who are at high risk
D) only patients at high risk for HIV, regardless of age

146. A 28-year-old gravida 1 para 0 at 39 weeks gestation presents for routine outpatient obstetric
care and is found to have a blood pressure of 145/95 mm Hg. A complete review of systems is
notable only for chronic low back pain causing poor sleep. The physical examination is normal,
including a nontender, gravid uterus and a fetal heart rate of 150 beats/min. The cervical
examination reveals firm consistency, 1 cm dilation, 50% effacement, and –3 station. The
patient’s blood pressure is checked 5 hours later and is 142/94 mm Hg.

Based on the 2013 ACOG guidelines for management of hypertension in pregnancy, which one
of the following should be the next step in management?

A) Admit the patient for induction of labor


B) Measure 24-hour urine protein, with induction of labor if the level exceeds 300 mg
C) Begin oral nifedipine (Procardia) and recheck her blood pressure in 24–48 hours
D) Place the patient on strict bed rest and check her blood pressure twice weekly
E) Begin twice-weekly office visits with assessment for preeclampsia

41
147. A 40-year-old obese African-American male presents with a history of excessive daytime
drowsiness. At home he falls asleep shortly after starting to read or watch television. He admits
to nearly crashing his car twice in the past month because he briefly fell asleep behind the wheel.
Most frightening to the patient have been episodes characterized by sudden loss of muscle tone,
lasting about 1 minute, associated with laughing. An overnight sleep study shows decreased
sleep latency and no evidence of obstructive sleep apnea.

Appropriate treatment includes which one of the following?

A) Methylphenidate (Ritalin)
B) Zolpidem (Ambien) at bedtime
C) Carbidopa/levodopa (Sinemet)
D) Weight reduction
E) Avoidance of daytime napping

148. A 68-year-old female presents with a 2-month history of painful, swollen wrists and knees. The
pain is always present and is accompanied by stiffness in these joints for 2–3 hours every
morning. Her past medical history, family history, and social history are unremarkable. She
takes a daily multivitamin.

A complete physical examination is notable only for symmetric, moderately swollen, slightly
erythematous, and very tender wrists and knees. Range of motion is intact but increases her
pain. Plain radiographs of these joints show erosions at the ulnar styloids. Lyme disease
serologies are negative. Anti–cyclic citrullinated peptide (CCP) antibody testing is positive.

Which one of the following would be appropriate for this patient as a sole therapy for her joint
condition?

A) Aspirin
B) Doxycycline
C) Methotrexate
D) Naproxen
E) Prednisone

149. A 50-year-old female presents to your office for evaluation of a 2-month history of dyspnea on
exertion and a nonproductive cough. She has a previous history of hypertension, overactive
bladder, gastroesophageal reflux disease, and recurrent urinary tract infections. Vital signs are
unremarkable and she has an oxygen saturation of 94%. She has inspiratory crackles in the
posterior lung bases that do not clear with coughing. Office spirometry shows that the FVC is
only 80% of normal, but the FEV1/FVC ratio is 0.85.

Which one of the patient’s current medications is most likely to be the cause of her problem?

A) Lisinopril (Prinivil, Zestril)


B) Conjugated estrogens (Premarin)
C) Omeprazole (Prilosec)
D) Solifenacin (Vesicare)
E) Nitrofurantoin (Macrodantin)

42
150. A 25-year-old gravida 1 para 0 sees you for a routine prenatal visit. This is a planned pregnancy
and you calculate her to be at approximately 14 weeks gestation based on the dates of her last
menstrual period. She is healthy without any medical problems, takes no medication, and does
not use tobacco products. She is adopted and does not know her family history. She feels well
today and has no specific concerns. Her vital signs are stable, her weight is normal, and fetal
heart tones are auscultated with a Doppler stethoscope at approximately 140 beats/min.

Which one of the following should be completed today?

A) A 1-hour glucose tolerance test


B) A group B Streptococcus screen
C) A TSH level
D) A urinalysis and urine culture
E) Evaluation for bacterial vaginosis

151. A 2-year-old male is brought to your office for a well child examination. Developmental
screening reveals that he has about a 10-word vocabulary. His mother attributes this to their
bilingual home but admits she is concerned about autism.

Which one of the following behaviors would provide additional evidence that the child may have
autism?

A) Use of gestures rather than words to communicate ideas


B) Frequently being engrossed in pretend play with dolls
C) Becoming upset by normal noises
D) Seemingly excessive attempts to attract attention with his behavior
E) Repeated copying of parental facial expressions

152. A 16-year-old female presents for follow-up after a tibial stress fracture. The fracture was
diagnosed 5 weeks ago by characteristic physical examination findings and radiographs showing
a transverse fracture of the tibial diaphysis. She was placed on non–weight-bearing status for 2
weeks and after that was advised to limit activities that caused discomfort. In addition, she was
placed on appropriate calcium and vitamin D supplementation based on results of her laboratory
workup. The patient is a basketball player and would like to begin practicing with the team in
1 week. She says she is now able to walk without discomfort but has not tried running or
jumping.

Which one of the following is necessary for this patient to be able to return to basketball
participation next week?

A) Consultation with a sports medicine physician


B) A normal physical examination of the affected area
C) Normal radiographs of the tibia
D) A normal hydroxyvitamin D level

43
153. A 28-year-old male has had bright red blood in his semen with his last three ejaculations. He is
sexually active. He considers himself in good health, takes no medications, has no other
symptoms to suggest a coagulopathy, and has no other genitourinary symptoms. Examination
of the testes shows no masses or tenderness. Findings on a digital rectal examination are normal.

Which one of the following would be appropriate at this time?

A) Coagulation studies including a platelet count and a prothrombin time


B) A serum PSA level
C) A urine probe for Neisseria gonorrhoeae and Chlamydia trachomatis
D) CT of the pelvis
E) Referral to a urologist

154. A 34-year-old female with newly diagnosed diarrhea-predominant irritable bowel syndrome
(IBS) presents with worsening abdominal discomfort. Her abdominal discomfort is not severe
but it is constant. She has tried dicyclomine (Bentyl) without relief and is interested in trying a
different approach.

The patient has had negative testing for inflammatory bowel disease and celiac disease, along
with normal blood tests. She asks about specific dietary modifications or medications that may
be helpful for her abdominal discomfort.

Which one of the following interventions would you recommend?

A) Amitriptyline
B) Clarithromycin (Biaxin)
C) Loperamide (Imodium)
D) Increased intake of insoluble dietary fiber

155. A 22-year-old male presents to your office for evaluation of fatigue, poor appetite, and nausea.
He states that when he stands too long he often gets dizzy but this is relieved by sitting. His
symptoms have been gradually getting worse over the past year. His vital signs are normal but
he is found to be orthostatic. A physical examination is unremarkable except for
hyperpigmentation in his palmar creases and around his nipples. A basic metabolic panel is
notable for a sodium level of 131 mEq/L (N 135–145) and a potassium level of 5.1 mEq/L (N
3.5–5.0).

Which one of the following is the most likely cause of this patient’s symptoms?

A) Addison’s disease
B) Cushing syndrome
C) Neurally mediated hypotension
D) Postural orthostatic hypotension and tachycardia syndrome
E) Hypothyroidism

44
156. A 26-year-old female has had a severe anaphylactic reaction to eggs in the past. Which one of
the following influenza vaccines would be safest for her?

A) Live attenuated trivalent influenza vaccine


B) Recombinant trivalent influenza vaccine
C) Inactivated trivalent influenza vaccine
D) Inactivated quadrivalent influenza vaccine

157. A 35-year-old male with a 4-month history of pain in the medial aspect of his right knee sees you
for follow-up. He has been doing physical therapy for the past month with minimal benefit. A
plain radiograph is negative and MRI shows a tear in the medial meniscus.

Which one of the following is most likely to yield the best long-term result?

A) Referral for meniscectomy


B) Corticosteroid injection
C) Hylan GF 20 (Synvisc) injection
D) Continued physical therapy
E) A knee brace

158. Which one of the following effects of antioxidant supplementation has been demonstrated in
randomized clinical trials?

A) Decreased mortality with vitamin A supplementation


B) Decreased mortality with $-carotene supplementation
C) Decreased mortality with vitamin E supplementation
D) Increased mortality with some antioxidant supplements

159. A 42-year-old male has symptoms of hypogonadism. Which one of the following should be
ordered first?

A) Early morning total serum testosterone


B) Early morning total and free serum testosterone
C) Early morning total and late afternoon total serum testosterone
D) Early morning and late afternoon free serum testosterone
E) Early morning and late afternoon total and free serum testosterone

160. Which one of the following is the recommended first-line test for investigating suspected hyper-
or hypothyroidism?

A) Free T3
B) Free T4
C) TSH
D) Antithyroglobulin
E) Antithyroid peroxidase

45
161. A 34-year-old G2P0101 at 11 weeks gestation comes to your office to establish care for her
pregnancy. In reviewing her history you find that her first pregnancy was complicated by
preeclampsia and she required induction of labor at 33 weeks. She also has chronic hypertension
treated with chlorthalidone. Her blood pressure today is 128/78 mm Hg.

Which one of the following medications, if started today, will lower her risk of preeclampsia in
this pregnancy?

A) Aspirin
B) Calcium
C) Labetalol
D) Nifedipine (Procardia)
E) Vitamin E

162. A 45-year-old female has ultrasonography of her kidneys as part of an evaluation for
uncontrolled hypertension. The report notes an incidental finding of stones in the gallbladder,
confirmed on right upper quadrant ultrasonography. She has no symptoms you can relate to the
gallstones. Other than hypertension she has no chronic medical problems.

Which one of the following should you recommend to her at this time regarding the gallstones?

A) Expectant management
B) Oral dissolution therapy
C) Extracorporeal lithotripsy
D) Endoscopic retrograde cholangiopancreatography (ERCP)
E) Laparoscopic cholecystectomy

163. A mother brings her 7-year-old son in for a well child check and you find that their main
concern is bedwetting. He has never achieved consistent nighttime continence. He currently wets
the bed about 4 nights per week but has no difficulty maintaining continence during the day and
reports no symptoms such as dysuria or urinary frequency. The parents have tried limiting his
evening fluid intake but this has not helped. He is otherwise healthy. The patient wants to stop
wearing nighttime diapers.

Which one of the following interventions has the best evidence of long-term success in
addressing this condition?

A) A reward system for achieving dry nights


B) Use of a bed alarm
C) Desmopressin (DDAVP)
D) Imipramine (Tofranil)
E) Oxybutynin

46
164. A 55-year-old nonsmoking African-American female with diabetes mellitus sees you for a
routine visit. She has no other cardiac risk factors. Her blood pressure is 120/74 mm Hg and she
has a fasting total cholesterol level of 180 mg/dL, an HDL-cholesterol level of 52 mg/dL, and
an LDL-cholesterol level of 100 mg/dL. Her calculated 10-year risk of atherosclerotic
cardiovascular disease is 5.8%.

According to the 2013 American College of Cardiology/American Heart Association cholesterol


guidelines, which one of the following is recommended for this patient?

A) No statin therapy
B) Low-intensity statin therapy
C) Moderate-intensity statin therapy
D) High-intensity statin therapy

165. A 65-year-old male with end-stage renal disease requires postoperative pain management. Which
one of the following medications would be safest to use?

A) Fentanyl
B) Hydrocodone
C) Hydromorphone (Dilaudid)
D) Meperidine (Demerol)
E) Morphine

166. A 30-year-old male presents with a 2-week history of swelling of the right posterior elbow. He
recalls bumping his elbow against a door, but his pain quickly subsided. He began to notice the
swelling over the next 2 days. On examination he has normal range of motion with a boggy,
nontender mass over the olecranon.

Which one of the following would be most appropriate at this point?

A) A posterior splint
B) Aspiration
C) A corticosteroid injection
D) A uric acid level and erythrocyte sedimentation rate
E) A compression dressing

47
167. A 23-year-old female sees you for the first time for a routine health maintenance evaluation. She
tells you that her father just had a “heart valve replacement” at age 47. On examination you note
a harsh 3/6 systolic murmur at the right upper sternal border. She feels well and her exercise
tolerance is normal. Her history indicates that she has been well throughout her life and received
appropriate childhood vaccinations and care for routine illnesses. She denies tobacco, alcohol,
and drug use now and in the past. Her blood pressure today is 132/84 mm Hg. You are
concerned about aortic valve disease and order an echocardiogram for further evaluation.

Which one of the following is the most likely cause of aortic valve disease in this patient?

A) Hypertension
B) Endocarditis
C) Bicuspid aortic valve
D) Rheumatic heart disease
E) Coronary atherosclerosis

168. A resting ankle-brachial index of 1.50 indicates which one of the following?

A) Normal circulation to a lower extremity


B) Borderline normal circulation which may not be problematic in an asymptomatic patient
C) Mild peripheral artery disease in a lower extremity
D) Severe peripheral artery disease in a lower extremity
E) Incompressible vessels in a lower extremity

169. A 69-year-old male sees you for a routine evaluation. He has been in good health and takes no
medication other than tamsulosin (Flomax) for symptoms of benign prostatic hyperplasia. He has
never smoked. His blood pressure is 121/78 mm Hg, pulse rate 72 beats/min, and respiratory
rate 18/min. His general physical examination is unremarkable, including cardiac and abdominal
examinations. A digital rectal examination reveals mild enlargement of the prostate, without
nodules.

According to the U.S. Preventive Services Task Force, this patient should be screened for

A) elder abuse
B) aortic aneurysm
C) multifactorial fall risk
D) dementia
E) hepatitis C

170. Which one of the following has been shown to be effective for Lyme disease prophylaxis after
removal of an engorged deer tick?

A) Amoxicillin
B) Ceftriaxone (Rocephin)
C) Cefuroxime axetil (Ceftin)
D) Doxycycline
E) Clarithromycin (Biaxin)

48
171. A 58-year-old female sees you 3 days after she was clearing her sinuses with steam and burned
her face. She developed small patches of dry, painful erythema without blisters on her chin, the
left side of her mouth, and her left cheek. She had no difficulty breathing. She applied cold
water to the burn and decided to self-treat initially but came in because she was experiencing
some pain. Her injury is shown below.

She received Td vaccine last year. In addition to analgesics for pain control, which one of the
following would be appropriate?

A) Cleaning the wound with povidone iodine (Betadine)


B) Covering the wound with an occlusive dressing
C) Applying aloe
D) Applying hydrocortisone 1% cream
E) Starting broad-spectrum antibiotics

172. A 61-year-old female tells you that her brother was recently diagnosed with hereditary
hemochromatosis and his physician suggested that she get tested. She feels well and has no
significant health problems.

Which one of the following would be most appropriate for initial screening?

A) Serum transaminases
B) A CBC and a serum iron level
C) Testing for the HFE gene
D) Ferritin and transferrin saturation
E) Total iron binding capacity

173. Complications of hypoparathyroidism include

A) somnolence
B) low vitamin D
C) muscle flaccidity
D) hyperkalemia
E) refractory heart failure

174. A 42-year-old female presents to the emergency department with a 2-hour history of palpitations.
Her physical examination is normal except for what seems to be a regular rhythm tachycardia
and a blood pressure of 84/54 mm Hg. An EKG reveals a regular narrow-complex tachycardia
at a rate of 180 beats/min without clear atrial activity.

The optimal treatment for this patient is

A) intravenous adenosine (Adenocard)


B) intravenous amiodarone (Cordarone)
C) intravenous diltiazem
D) intravenous verapamil
E) electrical cardioversion

49
175. You see a 2-year-old African-American male for a well child check. He is a new patient and his
examination is within normal limits except for an approximately 0.75-cm umbilical hernia that
is easily reducible. The father states that the hernia has been present since birth, although he
thinks it has grown slightly over the last year. The child does not seem to be bothered by the
hernia and the father does not think it has ever become incarcerated.

Which one of the following should you do now?

A) Reassure and observe


B) Advise daily application of pressure dressings
C) Order an ultrasound examination
D) Refer for surgical repair

176. Which one of the following medications is associated with a higher risk of death due to stroke
or sudden cardiac death in patients with dementia?

A) Diazepam (Valium)
B) Fluoxetine (Prozac)
C) Paroxetine (Paxil)
D) Quetiapine (Seroquel)
E) Venlafaxine

177. A 65-year-old female is admitted to the hospital for a carotid endarterectomy and you are asked
to make preoperative recommendations in advance of her surgery scheduled for tomorrow. She
takes only low-dose aspirin. The physical examination is normal, including her blood pressure,
as is an EKG. She has good exercise capacity and denies any symptoms of angina. You judge
her to be stable for surgery.

Which one of the following should you recommend that the patient start today?

A) An ACE inhibitor
B) A $-blocker
C) A statin
D) A diuretic

178. The intensely pruritic rash shown below is typical of

A) contact dermatitis
B) herpes simplex
C) pityriasis rosea
D) tinea corporis

50
179. A 76-year-old male completed chemotherapy for carcinoma of the pancreas 3 months ago. He
was seen 6 weeks ago by his oncologist and thought to be in remission. He presents to your
office today with a 2-week history of malaise and epigastric pain. You perform an examination
that indicates a possible epigastric mass.

Elevation of which one of the following laboratory studies would suggest recurrent pancreatic
cancer?

A) "1-Antitrypsin
B) "-Fetoprotein
C) Serum amylase
D) CA 19-9
E) CA-125

180. The most common source of chest pain in children is

A) pulmonary
B) cardiac
C) musculoskeletal
D) gastroesophageal
E) psychogenic

181. A 59-year-old male is being evaluated for a 2-day history of intermittent “heart fluttering.” He
reports that he has had high blood pressure for several years and has been reluctant to start
treatment for it. His EKG is shown below.

Which one of the following laboratory tests is most likely to be abnormal in this patient?

A) D-dimer
B) Potassium
C) Troponin
D) TSH

51
182. An asymptomatic 60-year-old male sees you for a health maintenance visit. His past medical
history is significant for hypertension and hyperlipidemia. His medications include
chlorthalidone, 25 mg daily, and atorvastatin (Lipitor), 20 mg daily. He smoked 2 packs of
cigarettes a day for 20 years but quit 5 years ago. The physical examination is normal.
Laboratory findings include a normal basic metabolic panel, a cholesterol level of 210 mg/dL,
an HDL-cholesterol level of 34 mg/dL, an LDL-cholesterol level of 150 mg/dL, and a
triglyceride level of 200 mg/dL.

Which one of the following screening tests is recommended by the U.S. Preventive Services
Task Force for this patient?

A) Prostate-specific antigen (PSA)


B) A bone density test
C) Abdominal ultrasonography
D) Low-dose chest CT
E) Carotid ultrasonography

183. A 63-year-old female with community-acquired pneumonia is being treated with appropriate
antibiotics. The only abnormality on a basic metabolic panel is a serum sodium level of 121
mEq/L (N 135–145). She reports that her shortness of breath and cough are improving. She has
no other complaints on a review of systems.

On examination the patient is noted to have normal vital signs and mucous membranes are moist.
She has crackles in her right lower lobe. Skin turgor is normal. The remainder of the physical
examination is normal. Further testing reveals the following:

Urine sodium. . . . . . . . . . . . . . . . . . . . . . . . . . 50 mEq/L


Serum osmolality. . . . . . . . . . . . . . . . . . . . . . . 276 mOsm/kg (N 280–285)
Urine osmolality. . . . . . . . . . . . . . . . . . . . . . . 300 mOsm/kg

Which one of the following would be most appropriate at this point?

A) Intravenous diuretics
B) Intravenous hypertonic saline
C) Intravenous isotonic saline
D) Fluid restriction
E) No further interventions

52
184. A 53-year-old female without risk factors for colorectal cancer undergoes a screening
colonoscopy. A high-quality examination reveals five 3- to 7-mm sessile polyps in the sigmoid
and rectal areas. Biopsy results show that they are hyperplastic polyps. No other abnormalities
are noted.

When should this patient have her next colonoscopy?

A) 1 year
B) 3 years
C) 5 years
D) 10 years
E) No further colonoscopies are needed

185. Screening for chronic hepatitis B infection is NOT recommended for which one of the following?

A) Patients on chronic immunosuppressive therapy


B) Patients with end-stage renal disease who are on hemodialysis
C) Household contacts of individuals with chronic hepatitis B
D) Pregnant women with no risk factors for hepatitis B
E) All newborns

186. A 67-year-old male with moderate macrocytosis complains of paresthesias of his feet. If the
patient has a borderline low vitamin B12 level, elevated levels of which one of the following
would suggest vitamin B12 deficiency?

A) Serum gastrin
B) Reticulocytes
C) Methylmalonic acid
D) Serum ferritin
E) Serum folate

187. In the United States the most common form of child abuse is

A) physical abuse
B) emotional abuse
C) sexual abuse
D) neglect

53
188. A 65-year-old female presents to the emergency department as directed by her primary care
physician because of “high potassium” that was found today during routine laboratory
monitoring. The patient has a past medical history significant for diet-controlled diabetes
mellitus, hypertension, and asthma. She feels well and specifically denies palpitations, fatigue,
changes in urine output, and muscle cramps. You do not have access to the patient’s outpatient
medical records and order a chemistry panel in the emergency department with the following
results:

Sodium. . . . . . . . . . . . . . . . . . . . . . . . . . . . . . . 143 mEq/L (N 135–145)


Potassium. . . . . . . . . . . . . . . . . . . . . . . . . . . . . 6.3 mEq/L (N 3.5–5.0)
CO2.. . . . . . . . . . . . . . . . . . . . . . . . . . . . . . . . . 27 mEq/L (N 22–30)
Creatinine.. . . . . . . . . . . . . . . . . . . . . . . . . . . . 1.6 mg/dL (N 0.6–1.0)
BUN. . . . . . . . . . . . . . . . . . . . . . . . . . . . . . . . . 30 mg/dL (N 7–21)

Which one of the following is the first additional test that should be obtained in the diagnostic
evaluation of this patient?

A) A urinalysis
B) A CBC
C) Arterial blood gases
D) An EKG
E) Renal ultrasonography

189. A 58-year-old male delivery truck driver is diagnosed with type 2 diabetes mellitus and after
several months of working on lifestyle modification his hemoglobin A1c is 8.0%. You suggest
it is time to start a medication to help control his condition but he is very worried about having
a “low sugar reaction” that would prevent him from driving. He is on no other medications at
this time. His only other health problem is long-standing controlled hypertension. His BMI is
33.1 kg/m2 and his serum creatinine level is 1.2 mg/dL (N 0.6–1.5).

Which one of the following medications would be least likely to cause hypoglycemia in this
patient?

A) Canagliflozin (Invokana)
B) Glimepiride (Amaryl)
C) Glipizide (Glucotrol)
D) Insulin glargine (Lantus)
E) Metformin (Glucophage)

190. The most common carcinoma diagnosed in the United States is

A) colon adenocarcinoma
B) prostate carcinoma
C) breast carcinoma
D) basal cell carcinoma
E) malignant melanoma

54
191. The parents of a 5-year-old male bring him in for evaluation of likely attention
deficit/hyperactivity disorder. You have suspected this diagnosis for some time, and the parents
have brought in surveys filled out by themselves and the child’s kindergarten teacher which
confirm your suspicions.

The most appropriate initial treatment is

A) behavioral therapy
B) "2-receptor agonists such as guanfacine (Intuniv, Tenex)
C) psychostimulants such as methylphenidate (Ritalin)
D) atomoxetine (Strattera)

192. Which one of the following is most characteristic of hoarding disorder?

A) Collecting eccentric or bizarre items


B) Collecting only seemingly worthless items
C) Deriving pleasure from collected items
D) Anxiety and emotional distress if collected items are disposed of

193. A 24-year-old female presents with a painless ulcer on her labia, which has been present for a
week. You suspect primary syphilis, but a rapid plasma reagin (RPR) test is negative.

Which one of the following is the best strategy for confirming or ruling out syphilis in this
situation?

A) A spinal fluid analysis


B) A serum fluorescent treponemal antibody absorption (FTA-ABS) test now
C) A Treponema pallidum particle agglutination (TPPA) test now
D) A Venereal Disease Research Laboratory (VDRL) test now
E) Repeating the RPR test in 2 weeks

194. A 53-year-old female with a past medical history of hypertension and high cholesterol presents
to discuss options for tobacco cessation. She has a 30-pack-year history of smoking and currently
smokes between 1 and 1½ packs per day. She tried varenicline (Chantix) but had nightmares
while she was using it and does not want to try it again. Many of her family members have
seizure disorders and she is therefore hesitant to try bupropion (Wellbutrin). She has used
nicotine patches with minimal success.

Which one of the following pharmacotherapies would be most likely to help in her effort to stop
smoking?

A) Fluoxetine (Prozac)
B) Naltrexone (ReVia)
C) Nortriptyline (Pamelor)
D) Selegiline (Eldepryl)
E) St. John’s wort

55
195. The specificity of a screening test is best described as the proportion of persons

A) with the condition who test positive


B) with the condition who test negative
C) with the condition who test positive, compared to the total number screened
D) without the condition who test positive
E) without the condition who test negative

196. An 18-month-old previously healthy infant is admitted to the hospital with bronchiolitis. Pulse
oximetry on admission is 92% on room air.

Which one of the following should be included in the management of this patient?

A) Tracheal suction to clear the lower airways


B) Nasal suction to clear the upper airway
C) Chest physiotherapy
D) Corticosteroids
E) Azithromycin (Zithromax)

197. Which one of the following is associated with bisphosphonate use for the treatment of
osteoporosis?

A) Hypercalcemia
B) Hyperphosphatemia
C) Vitamin D deficiency
D) Atypical femoral shaft fractures
E) Renal failure

198. A 20-year-old male college student comes to the emergency department in January acutely short
of breath and looking very ill, with tachypnea, tachycardia, nausea, and a headache. Pulse
oximetry shows an oxygen saturation of 100% on room air, and arterial blood gas measurement
shows a PaO2 of 95 mm Hg.

Of the following, which one is the most likely diagnosis?

A) Carbon monoxide poisoning


B) Adult respiratory distress syndrome
C) Methemoglobinemia
D) Lobar pneumonia
E) Viral pneumonia

56
199. A 25-year-old gravida 1 para 1 presents for insertion of a levonorgestrel-releasing intrauterine
device (Mirena). She is on the last day of her menses, which began 5 days ago. A urine
pregnancy test in the office is negative. You insert the device without complications and she asks
how long she needs to use backup contraception.

Which one of the following would be the most appropriate advice?

A) Backup contraception is not necessary


B) She should use backup contraception for the next 48 hours
C) She should use backup contraception for the next 7 days
D) She should use backup contraception for the next 14 days
E) She should use backup contraception for the next month

200. Which one of the following classes of diabetes medications increases the risk of genitourinary
infections by blocking glucose reabsorption by the kidneys?

A) SGLT2 inhibitors such as canagliflozin (Invokana)


B) GLP-1 receptor agonists such as exenatide (Byetta)
C) DPP-4 inhibitors such as sitagliptin (Januvia)
D) Meglitinides such as repaglinide (Prandin)
E) "-Glucosidase inhibitors such as acarbose (Precose)

201. Which one of the following is first-line treatment for chronic Achilles tendinopathy?

A) NSAIDs
B) Bracing
C) Eccentric strengthening exercises
D) Corticosteroid injection
E) Therapeutic ultrasonography

202. A 69-year-old female presents with her first episode of Clostridium difficile colitis, which is
characterized as severe. Which one of the following is the most appropriate initial therapy?

A) Oral metronidazole (Flagyl)


B) Intravenous metronidazole
C) Oral vancomycin (Vancocin)
D) Intravenous vancomycin
E) Rifaximin (Xifaxan)

203. Which one of the following is the most common cause of sudden cardiac death in young athletes?

A) Coronary artery abnormalities


B) Myocarditis
C) Hypertrophic cardiomyopathy
D) Brugada syndrome
E) Idiopathic left ventricular hypertrophy

57
204. A 44-year-old male is being evaluated for a 3-month history of cough. His chest radiograph is
shown below.

Which one of the following abnormalities is seen on the radiograph?

A) Bronchiectasis
B) A pulmonary cavitary lesion
C) A hiatal hernia
D) A thoracic aortic aneurysm
E) Pericardial effusion

205. A 15-year-old male presents to the emergency department at 10 p.m. with a 2-hour history of
severe, acute scrotal pain associated with vomiting. On examination the right testicle is swollen.
Ultrasonography is inconclusive.

Which one of the following would be most appropriate at this point?

A) Repeat ultrasonography in the morning


B) Antibiotics
C) Corticosteroids
D) Scrotal support
E) Immediate surgical consultation

206. A 66-year-old female sees you for the first time. She has a history of iron deficiency anemia and
chronic diarrhea associated with a diagnosis of celiac disease.

This history increases her risk for which one of the following?

A) Diverticulitis
B) Ulcerative colitis
C) Crohn’s disease
D) Colon cancer
E) Osteoporosis

58
207. A 44-year-old female is brought to your office by her mother. The patient was in a severe car
accident 2 weeks ago. Her husband was killed instantly and she was extracted by emergency
responders almost an hour later. She received a full examination at a local emergency
department and was discharged home with only minor contusions and abrasions and no evidence
of a closed head injury.

The patient has been panicked and unable to sleep. She has recurrent flashbacks of the event and
dreams repeatedly about her husband’s death. She says that sometimes, even while awake, she
can almost sense her husband’s lifeless body near her. She has refused to get into a car since the
accident, which is the reason she has not sought care sooner. She has not been able to focus on
daily tasks but has been able to eat and drink adequate amounts.

Which one of the following diagnoses best describes her condition?

A) Acute stress disorder


B) Major depressive disorder
C) Obsessive-compulsive disorder
D) Panic disorder
E) Generalized anxiety disorder

208. You are covering the inpatient service and following up on a 67-year-old female admitted 3 days
ago for severe pancreatitis. CT on admission showed edema and mild inflammation. Currently
the patient is receiving intravenous fluids, daily laboratory evaluations, and pain medications.
She is NPO and afebrile, with a blood pressure of 130/78 mm Hg and a pulse rate of 88
beats/min.

Which one of the following therapies should be initiated to lower complication rates and shorten
the patient’s hospital stay?

A) Enteral nutrition
B) Parenteral nutrition
C) Surgical debridement
D) Prophylactic antibiotics

209. A 30-year-old female complains of dysmenorrhea, pelvic pain, and dyspareunia. Which one of
the following would be appropriate to detect endometriosis?

A) A CA-125 assay
B) Transvaginal ultrasonography
C) CT of the pelvis
D) MRI of the pelvis
E) Colonoscopy

59
210. The CDC has designated several diseases as neglected parasitic infections in the United States.
Which one of these, if untreated, has potential consequences that include cardiomyopathy, heart
failure, and fatal cardiac arrhythmias?

A) Trichomoniasis
B) American trypanosomiasis (Chagas disease)
C) Toxoplasmosis
D) Cysticercosis
E) Toxocariasis

211. A 30-year-old female stepped off a curb earlier today and twisted her left ankle. She was able
to bear weight immediately following the injury and tried to continue her normal routine, but the
pain in her ankle and foot increased over the next few hours.

She comes to your office and your examination reveals swelling of the ankle and bruising of the
lateral foot. Tenderness to palpation is present over the distal aspect of the fibula and lateral
malleolus and to a lesser degree over the proximal fifth metatarsal. No bony tenderness is
present along the medial aspect of the ankle or foot.

According to the Ottawa Ankle Rules, which one of the following would be most appropriate
at this point?

A) Radiographs of the ankle and foot


B) Radiographs of the foot only
C) Radiographs of the ankle only
D) No radiographs

212. A U.S. Preventive Services Task Force “D” recommendation indicates

A) high certainty that the net benefit is substantial


B) high certainty that the net benefit is moderate
C) moderate or high certainty that the service has no net benefit or that the harms outweigh
the benefits
D) that the decision to provide the service should be based on professional judgment and
patient preferences
E) that current evidence is insufficient to assess the balance of benefits and harms of the
service

213. According to the Diagnostic and Statistical Manual of Mental Disorders, 5th edition (DSM-5),
the severity of anorexia nervosa is based on which one of the following?

A) Refusal to eat
B) The frequency of episodes of binge eating or purging behavior
C) Body mass index (BMI)
D) The presence or absence of amenorrhea
E) Orthostatic changes in pulse or blood pressure

60
214. A 29-year-old previously healthy male presents with a 1-hour history of the sudden onset of
progressively worsening shortness of breath. On examination he has a blood pressure of 126/96
mm Hg, a heart rate of 110 beats/min, an oxygen saturation of 90%, and a respiratory rate of
24/min. A chest radiograph is shown below.

Which one of the following is the recommended treatment?

A) Observation
B) The Valsalva maneuver
C) Needle aspiration
D) Intravenous heparin
E) Intravenous methylprednisolone sodium succinate (Solu-Medrol)

215. Which one of the following comorbid conditions increases the risk that latent tuberculosis
infection will progress to active disease?

A) Hypertension
B) Lung cancer
C) Obesity
D) Coronary artery disease
E) Hyperlipidemia

216. Which one of the following immunizations is indicated for all pregnant women at any stage of
pregnancy?

A) MMR
B) Varicella
C) Influenza
D) HPV

217. A 24-year-old gravida 2 para 1 at 9 weeks gestation sees you for a routine prenatal check. She
complains of significant nausea, and recommended dietary modifications have not helped. She
drives a school bus so she would like to avoid sedating medications. She appears well-hydrated
and her examination is otherwise normal.

Which one of the following would be best for relieving this patient’s nausea?

A) Auricular acupressure
B) A scopolamine patch (Transderm Scop)
C) Vitamin B6 (pyridoxine)
D) Methylprednisolone (Medrol)

61
218. A local dentist contacts you for a prescription for the appropriate antibiotic dosage for one of
your patients who has an appointment for dental cleaning to eliminate a significant plaque
buildup. The patient is a 55-year-old male who has controlled hypertension and mitral valve
prolapse with mitral regurgitation. He is allergic to sulfonamides.

Which one of the following would be the most appropriate prophylaxis for this patient?

A) Amoxicillin, 2 g orally 1 hour prior to the procedure


B) Amoxicillin, 3 g orally 1 hour prior to the procedure and 1.5 g orally 6 hours after the
procedure
C) Ceftriaxone (Rocephin), 1 g intramuscularly 1 hour prior to the procedure
D) Clindamycin (Cleocin), 600 mg orally 1 hour prior to the procedure
E) No antibiotic prophylaxis

219. A 40-year-old female sees you for a health maintenance visit. She has no complaints and other
than being overweight she has an unremarkable examination. Laboratory results are also
unremarkable except for her lipid profile. She has a total cholesterol level of 251 mg/dL, an
HDL-cholesterol level of 31 mg/dL, and a triglyceride level of 1250 mg/dL. The
LDL-cholesterol level could not be calculated and measured 145 mg/dL.

In addition to lifestyle changes, this patient would most likely benefit from

A) niacin
B) omega-3 fatty acid supplementation
C) atorvastatin (Lipitor)
D) ezetimibe (Zetia)
E) fenofibrate (Tricor)

62
220. A 57-year-old female with a past medical history significant for well-controlled type 2 diabetes
mellitus, hypertension, and hyperthyroidism presents to your office with a chief complaint of a
sore throat and a fever to 101.5°F at home. She has had chills and night sweats but has not had
a cough, chest pain, or abdominal pain.

Physical Findings

General.. . . . . . . . . . . . . . . . . . . . . . . . . . . . . . ill appearing


HEENT. . . . . . . . . . . . . . . . . . . . . . . . . . . . . . diffuse tender anterior cervical
adenopathy; thyroid nontender;
oropharynx erythematous with some
purulence on her tonsils
Cardiovascular.. . . . . . . . . . . . . . . . . . . . . . . . tachycardia without murmur
Lungs. . . . . . . . . . . . . . . . . . . . . . . . . . . . . . . . clear to auscultation bilaterally
Skin. . . . . . . . . . . . . . . . . . . . . . . . . . . . . . . . . mild jaundice

Laboratory Findings

Rapid strep test. . . . . . . . . . . . . . . . . . . . . . . . negative


Total WBC count. . . . . . . . . . . . . . . . . . . . . . . 3000/mm3 (N 4500–11,000) and absolute
neutrophil count 0
Total bilirubin. . . . . . . . . . . . . . . . . . . . . . . . . 5 mg/dL (N 0–1.0)
Alkaline phosphatase. . . . . . . . . . . . . . . . . . . . 151 U/L (N 38–126)

Which one of the following medications is most likely to cause these laboratory abnormalities?

A) Amlodipine (Norvasc)
B) Aspirin
C) Metformin (Glucophage)
D) Methimazole (Tapazole)

221. A 31-year-old gravida 1 para 0 presents for a routine visit at 32 weeks gestation. She has
gestational diabetes mellitus (GDM) and has been following the dietary guidelines from her
dietitian. However, her blood glucose is still elevated and you discuss starting medications for
management of her GDM. She is adamant about not starting insulin but is willing to consider
taking metformin (Glucophage). Before making a decision she would like to know the specific
benefits to her and her baby.

You would tell her that one benefit of treatment of GDM is a decreased risk for

A) maternal type 2 diabetes mellitus after delivery


B) maternal preeclampsia
C) perinatal death
D) a small-for-gestational-age infant

63
222. A 34-year-old white female sees you for a routine follow-up visit. She takes haloperidol, 2 mg
after each meal, for schizophrenia, and you notice that she seems unable to sit still and is
extremely anxious.

The most likely cause of her restlessness is

A) drug-induced parkinsonism
B) akathisia
C) tardive dyskinesia
D) hysteria
E) dystonia

223. A 45-year-old male is seen for a well-demarcated, nonpruritic rash in the right axilla. It is
fine-scaled with a cigarette-paper appearance. The rash has a coral-red fluorescence under a
Wood’s light.

Which one of the following is the most likely diagnosis?

A) Candidiasis
B) Tinea cruris
C) Erythrasma
D) Inverse psoriasis

224. A 28-year-old female just delivered a male infant over an intact perineum. She has had
polyhydramnios during this pregnancy, but her prenatal course has otherwise been normal. Her
only significant chronic medical problem is asthma, treated with a long-acting
$-agonist/corticosteroid combination inhaler. Vital signs were stable throughout her labor. After
delivery of the placenta, bleeding becomes brisk and you note a soft, boggy, uterus.

Which one of the following medications is contraindicated in this patient?

A) Carboprost (Hemabate)
B) Methylergonovine
C) Misoprostol (Cytotec)
D) Oxytocin (Pitocin)

64
225. A 52-year-old male presents with a swollen and tender area anterior to the left ear and extending
to below the left angle of the mandible. One week ago he had a Nissen fundoplication for
intractable GERD. This was complicated by difficulty swallowing and drinking. On examination
his tympanic temperature is 37.7°C (99.9°F), his blood pressure is 110/70 mm Hg, and his
pulse rate is 95 beats/min and regular. His left parotid gland is diffusely enlarged and tender.
Purulent material is noted coming from the left parotid duct orifice.

Which one of the following would be most appropriate at this point?

A) Amoxicillin/clavulanate (Augmentin)
B) Penicillin
C) CT of the parotid gland
D) Incision and drainage of the parotid gland
E) Excision of the parotid gland

226. A 3-week-old infant is brought to your office with a fever. He has a rectal temperature of
38.3°C (101.0°F), but does not appear toxic. The remainder of the examination is within normal
limits.

Which one of the following would be the most appropriate management for this patient?

A) Admit to the hospital and obtain urine, blood, and CSF cultures, then start intravenous
antibiotics
B) Admit to the hospital and treat for herpes simplex virus infection
C) Follow up in the office in 24 hours and admit to the hospital if not improved
D) Order a CBC and a urinalysis with culture, and send the patient home if the results are
normal

227. A 70-year-old female presents to your office as a new patient. She is healthy and has no
complaints. She walks for exercise 30–45 minutes daily and takes no prescription medications.
Her blood pressure is 125/75 mm Hg, heart rate 72 beats/min, and respiratory rate 14/min.

On examination she has a systolic crescendo-decrescendo murmur heard loudest at the right
upper sternal border. An EKG in the office is within normal limits. Echocardiography shows
mild aortic stenosis based on peak aortic jet velocity, aortic valve area, and mean pressure
gradient. Her ejection fraction is 55%. At a follow-up visit she states that she continues to be
symptom free.

Which one of the following should be the next step in the evaluation and management of her
aortic stenosis?

A) Exercise treadmill testing


B) Right heart catheterization
C) Repeat echocardiography in 3 years
D) Cardiothoracic surgery consultation
E) Initiation of statin therapy

65
228. A 79-year-old female had a total knee replacement yesterday. She has mild dementia as a result
of a stroke 10 years ago, but her dementia has been stable since then. Last night she became
confused and agitated, striking out at nurses, and could not be consoled.

Which one of the following would be most appropriate at this time?

A) Soft restraints
B) CT of the head
C) Adequate pain control
D) A sedating SSRI such as paroxetine (Paxil)
E) Lorazepam (Ativan) intravenously as needed

229. A 58-year-old male sees you for a physical examination so he can receive a commercial driver’s
license. On examination you note a 2-cm hard, nodular protuberance on his hard palate, shown
below. He believes that this has been there for some time, but says it seems to be enlarging.

The most likely diagnosis is

A) osteoid osteoma
B) torus palatinus
C) mucocele
D) osteosarcoma
E) calcinosis cutis

230. Which one of the following is the best diagnostic test for vitamin D deficiency?

A) Ionized calcium
B) Serum phosphorus
C) 24-hour urine for calcium
D) 1,25-hydroxyvitamin D
E) 25-hydroxyvitamin D

231. A 31-year-old male has experienced multiple outbreaks of the rash shown below. He was
initially told that the rash was due to an allergy to an antibiotic prescribed for a suspected dental
abscess, but avoiding all medications has not prevented the recurrences.

Which one of the following oral medications has been shown to reduce the severity, duration,
and recurrences of this type of rash?

A) Acyclovir
B) Cetirizine (Zyrtec)
C) Prednisone
D) Ranitidine (Zantac)
E) Terbinafine (Lamisil)

66
232. Which one of the following is the only medication that has consistent evidence for decreasing
depressive symptoms in children and adolescents?

A) Fluoxetine (Prozac)
B) Venlafaxine (Effexor XR)
C) Nortriptyline (Pamelor)
D) Aripiprazole (Abilify)
E) Paroxetine (Paxil)

233. Which one of the following, especially in homeless people, is a vector for Bartonella quintana,
which causes trench fever, an influenza-like syndrome with relapsing fever?

A) Fleas
B) Maggots
C) Bedbugs
D) Scabies
E) Lice

234. A 72-year-old previously healthy female comes in for evaluation of recent headaches. She
describes the pain as generalized all over her head and persisting over the past several months.
She reports feeling more achy and fatigued in the past several weeks, with a decreased appetite
and unintentional weight loss of 4 lb in the past 2 months. She denies any other symptoms
including sinus congestion, nausea, vomiting, numbness, tingling, weakness, or vision changes.
Acetaminophen has been minimally helpful for the pain.

On examination you note a temperature of 37.9°C (100.2°F), normal cranial nerves, a normal
eye examination, and no tenderness to palpation of the head. She is mildly tender to palpation
of the shoulders and upper arms. Laboratory testing reveals an erythrocyte sedimentation rate
of 88 mm/hr (N 1–25).

Which one of the following is necessary to confirm the most likely diagnosis?

A) EEG
B) CT of the head
C) MRI of the head
D) A temporal artery biopsy
E) A lumbar puncture

67
235. A 3-year-old male is brought to the urgent-care clinic on a Monday morning by his mother with
a 1-day history of complaining of ear pain. The child’s mother says she has not noticed any fever
during this time. He is up to date on all immunizations and has no previous history of ear
infections or history of recent illness. The history is negative for medication allergies.

On examination the child has a temperature of 38.2°C (100.8°F) and seems to be


uncomfortable. When you examine his ears you note moderate bulging of the tympanic
membrane in both ears. All other findings are normal.

According to the guidelines published by the American Academy of Pediatrics, which one of the
following would be the most appropriate initial management?

A) Amoxicillin, 40–50 mg/kg, for 10 days


B) Amoxicillin, 80–90 mg/kg, for 10 days
C) Amoxicillin/clavulanate (Augmentin), 90 mg/kg/day of amoxicillin and 6.4 mg/kg/day
of clavulanate, divided into two doses, for 7 days
D) Cefdinir, 14 mg/kg/day for 10 days
E) Ciprofloxacin (Cipro), 10–20 mg/kg for 7 days

236. A 20-year-old male with a history of exercise-induced bronchoconstriction presents to your


office with a complaint of cough and decreasing performance when he runs. He is training for
a marathon and is currently running 30 miles/week, but has noted that his times have been
worsening and that he is using his albuterol inhaler (Proventil, Ventolin) as needed for symptom
relief 5 days a week.

Which one of the following is the best regimen for treatment of this condition?

A) Inhaled albuterol before he runs


B) A daily low-dose inhaled corticosteroid
C) A daily inhaled long-acting $2-agonist
D) A daily low-dose oral corticosteroid
E) Immunotherapy

237. A 44-year-old obese female complains of intermittent right upper quadrant pain that is worse
after fatty meals. Which one of the following is the preferred initial imaging modality for
evaluating her complaint and confirming the diagnosis?

A) A plain radiograph
B) Ultrasonography
C) Cholescintigraphy
D) Contrast-enhanced CT
E) Contrast-enhanced MRI

68
238. A 52-year-old healthy male presents with a 2½-week history of diarrhea, consisting of 4–6
watery stools daily. He is afebrile and his examination is normal. You recommend symptomatic
care. Two days later the laboratory notifies you that Salmonella is growing in his stool culture.
You call the patient and he remains free of fever but with ongoing diarrhea.

Which one of the following would you recommend?

A) Azithromycin (Zithromax)
B) Ciprofloxacin (Cipro)
C) Clindamycin (Cleocin)
D) Doxycycline
E) No treatment

239. A mother brings her 5-year-old daughter to see you because she found a mass in the child’s
neck. The mass appeared over the past week and was preceded by a sore throat. Her pharyngitis
is now resolved but she still has a fever, although it is not as high. The mother is most concerned
because the mass developed over a short span of time, and it is warm, red, and tender. When
asked, she says that her daughter has had no recent exposure to cats.

When you examine the child you note that her temperature is 38.0°C (100.4°F). You also find
shotty adenopathy in both anterior cervical lymph node chains, and a 2.5-cm warm, firm,
moderately tender lymph node in the right anterior cervical chain. The overlying skin is also
erythematous.

Which one of the following would be the most appropriate management at this time?

A) Ultrasonography of the neck mass


B) CT with intravenous contrast of the neck mass
C) Ultrasound-guided fine-needle aspiration of the mass
D) Immediate referral to a head and neck surgeon
E) Empiric antibiotic therapy with observation for 4 weeks

240. A 45-year-old male with diabetes mellitus returns to your office for follow-up. He is on
metformin (Glucophage), 1000 mg/day, as well as atorvastatin (Lipitor), 40 mg daily for
hyperlipidemia. There is no diagnosis of hypertension, and his blood pressure at today’s visit
is 120/70 mm Hg. Laboratory results include a hemoglobin A1c of 6.4% and an LDL-cholesterol
level of 105 mg/dL. His urine albumin/creatinine ratio is in the microalbuminuric range for the
first time.

Which one of the following would be most appropriate at this point?

NOOOOOOOOO
A) Renal ultrasonography
B) A repeat urine albumin/creatinine ratio
C) 24-hour urine for microalbumin
D) Increasing the atorvastatin dosage
E) Stopping metformin

69
‫‪American Board of Family Medicine‬‬

‫‪2016 IN-TRAINING EXAMINATION‬‬


‫‪Questions & answers‬‬

‫مت مجع األسئلة مع األجوبة يف ملف واحد لتسهيل مذاكرتها‬

‫مع حتيات متدربي دبلوم طب األسرة مبكة املكرمة‬

‫د‪/‬عبداجمليد فريق‬

‫د‪/‬رجا شافعي‬

‫د‪/‬أحالم صفدر‬

‫د‪/‬االء فران‬

‫د‪/‬انعام العقاد‬

‫د‪/‬البتول اجليالني‬

‫‪1‬‬
1. Which one of the following classes of medications for the treatment of diabetes mellitus is
most likely to lead to weight loss?
A) GLP-1 receptor agonists such as exenatide (Byetta)
B) Meglitinides such as repaglinide (Prandin)
C) Sulfonylureas such as glipizide (Glucotrol)
D) Thiazolidinediones such as pioglitazone (Actos)
E) Insulin

ANSWER: A
GLP-1 receptor agonists are the most likely of these medications to lead to weight loss. They
work by activating the GLP-1 receptors, which increases insulin secretion, decreases
glucagon secretion, slows gastric emptying, and increases satiety. Other diabetes medications
that may promote weight loss include metformin, amylin mimetics, and SGLT-2 inhibitors.
The other medications listed have all been shown to cause weight gain. DPP-4 inhibitors are
felt to be weight neutral.

Ref: American Diabetes Association: Standards of medical care in diabetes—2015:7. Approaches to glycemic treatment. Diabetes
Care 2015;38(Suppl 1):S41-S48.

2. The strategy that will identify the highest percentage of previously unrecognized cases of
chronic hepatitis C is to screen which one of the following?
A) All sexually active men and women between the ages of 18 and 29
B) Everyone born between 1945 and 1965
C) Everyone who had a blood transfusion or organ transplant before 1992
D) Everyone who has ever used intravenous drugs
E) All men who have sex with men

ANSWER: B
The hepatitis C virus is spread through contact with infected blood (usually unscreened blood
transfusions, which were the norm before 1992), intravenous drug use, or high-risk sexual
activity. However, studies have shown that screening only patients with high-risk medical or
clinical histories will identify just 20%–33% of infected patients. As a different strategy, both
the CDC and the U.S. Preventive Services Task Force have recommended one-time screening
of patients born between 1945 and 1965. It is estimated that this cohort includes 75% of all
patients who have chronic hepatitis C.
Ref: Chou R, Cottrell EB, Wasson N, et al: Screening for hepatitis C virus infection in adults: A systematic review for the US Preventive
Services Task Force. Ann Intern Med 2013;158(2):101-108. 2) Smith BD, Yartel AK, Krauskopf K, et al: Hepatitis C virus antibody
positivity and predictors among previously undiagnosed adult primary care outpatients: Cross-sectional analysis of a multisite
retrospective cohort study. Clin Infect Dis 2015;60(8):1145-1152.

2
3. A school nurse calls you for recommendations about a healthy 6-year-old female who was
found to have head lice yesterday and was treated with permethrin 1% shampoo last night.
The nurse reports that on examination today the child appeared well, weighed 30 kg (66 lb),
and had 6 nits on her scalp.
In addition to home eradication measures and regular nit combing, you recommend which
one of the following for this child?
A) Oral ivermectin (Stromectol) and return to class today
B) Topical lindane and return to class today
C) Topical benzyl alcohol (Ulesfia) and return to class as soon as she is nit free
D) Return to class today and retreatment with permethrin 1% shampoo in 6 days
E) Retreatment with permethrin 1% shampoo in 6 days and return to class as soon as she is
nit free

ANSWER: D
Family physicians are frequently consulted by patients and institutions regarding outbreaks
of head lice. Optimal treatment involves home eradication measures and medication.
Permethrin 1% shampoo remains the first-line treatment. Ivermectin appears effective for
treatment but is not FDA approved and is not considered first-line treatment. Lindane carries
an increased risk of toxicity and should not be used in children weighing <50 kg. Topical
benzyl alcohol is FDA-approved for treatment but is expensive. The current recommendation
from the American Academy of Pediatrics Council on School Health and Committee on
Infectious Diseases is to return children to class despite the presence of nits.

Ref: Frankowski BL, Bocchini JA Jr; Council on School Health and Committee on Infectious Diseases: Head lice. Pediatrics
2010;126(2):392-403. 2) Gunning K, Pippitt K, Kiraly B, Sayler M: Pediculosis and scabies: Treatment update. Am Fam Physician
2012;86(6):535-541.

4. An 82-year-old male nursing home resident is sent to the emergency department with
crampy lower abdominal pain and bloody diarrhea. On examination he has mild abdominal
tenderness and slightly decreased bowel sounds. He has a previous history of vascular
dementia, peripheral artery disease, hypertension, and hyperlipidemia. On examination he is
afebrile, and a nasogastric aspirate is negative for evidence of bleeding.
Which one of the following is the most likely cause of this patient’s bleeding?
A) Peptic ulcer disease
B) Ischemic colitis
C) Diverticular bleeding
D) Angiodysplasia
E) Infectious colitis

ANSWER B
This patient most likely has ischemic colitis, given his abdominal pain, bloody diarrhea, and
cardiovascular risk factors. Peptic ulcer disease is unlikely because the nasogastric aspirate

3
was negative. Diverticular bleeding and angiodysplasia are painless. Infectious colitis is
associated with fever.

Ref: Wilkins T, Baird C, Pearson AN, Schade RR: Diverticular bleeding. Am Fam Physician 2009;80(9):977-983. 2) Kasper DL,
Fauci AS, Hauser SL, et al (eds): Harrison’s Principles of Internal Medicine, ed 19. McGraw-Hill, 2015, p 1957.

5. Which one of the following screening tests has the greatest potential for overdiagnosis?
A) Fecal occult blood testing
B) Papanicolaou testing
C) Prostate-specific antigen
D) Mammography
E) Low-dose CT to detect lung cancer

ANSWER: C
Overdiagnosis is the diagnosis of a disease that will not produce symptoms during a patient’s
lifetime. It tends to occur with cancers that have very slow rates of growth. Prostate cancer is
most often a slow-growing cancer and is often present without symptoms in older men. The
introduction of prostate-specific antigen (PSA) screening was accompanied by a marked rise
in the rate of diagnosis of prostate cancer while mortality decreased much less significantly,
and this decrease was probably largely attributable to improved treatment. The problem of
overdiagnosis remains a significant problem with PSA screening and was a factor in the U.S.
Preventive Services Task Force’s recommendation against routine PSA-based screening for
prostate cancer (D recommendation).

Ref:US Preventive Services Task Force: Screening for cervical cancer: Recommendation statement. Am Fam Physician
2012;86(6):555-559. 2) Tirona MT: Breast cancer screening update. Am Fam Physician 2013;87(4):274-278. 3) Gates TJ: Screening
for cancer: Concepts and controversies. Am Fam Physician 2014;90(9):625-631. 4) Mulhem E, Fulbright N, Duncan N: Prostate
cancer screening. Am Fam Physician 2015;92(8):683-688.

6. A 32-year-old female has recently started a regular running program to try to lose weight.
Her BMI is 29.3 kg/m2. She presents to your office with a complaint of anterior knee pain
that is worse on the right, with no history of an acute injury to her knee. She is not sure exactly
when the pain began. On examination the patient has normal range of motion, no pain with
palpation, and no effusion. Crepitus and lateral patellar tracking are noted in both knees,
somewhat greater on the right.
Which one of the following is the most appropriate treatment for her condition?
A) Patellar taping
B) Reducing her running distance, and physical therapy
C) Glucosamine and chondroitin
D) Intra-articular corticosteroid injection
E) Knee arthroscopy

ANSWER:B

4
This patient most likely has patellofemoral pain syndrome, as evidenced by her recent
increased activity and anterior knee pain with increased lateral patellar tracking. The most
effective treatment for patellofemoral pain syndrome is physical therapy and activity
modification. Glucosamine and chondroitin have been used to treat osteoarthritis in the past
but should not be used to treat symptomatic osteoarthritis of the knee, according to the
American Academy of Orthopedic Surgeons. Intra-articular joint injections can be used for
knee arthritis but are not indicated for patellofemoral pain syndrome. Knee arthroscopy would
not be indicated for patellofemoral syndrome without evidence of internal knee derangement.
The evidence for patellar taping is inconsistent, but it may be helpful as an adjunct treatment.
Ref: Dixit S, DiFiori JP, Burton M, Mines B: Management of patellofemoral pain syndrome. Am Fam Physician 2007;75(2):194-202.
2) Jones BQ, Covey CJ, Sineath MH Jr: Nonsurgical management of knee pain in adults. Am Fam Physician 2015;92(10):875-883.

7. A healthy 83-year-old female presents for an annual Medicare visit. Her blood pressure is
165/95 mm Hg on initial evaluation today and 160/92 mm Hg on repeat measurement. She
has no health complaints.
The JNC 8 panel recommends which one of the following as the goal blood pressure for this
patient?
A) <135/85 mm Hg
B) <140/90 mm Hg
C) <150/90 mm Hg
D) <160/90 mm Hg
E) <160/100 mm Hg

ANSWER: C
In general, all patients with hypertension, regardless of age, benefit from treatment. Patients
≥80 years of age who are healthy and functionally independent should be treated according
to current recommendations for patients >65 years old. Available guidelines recommend a
target blood pressure of <150/90 mm Hg unless the patient is frail or has numerous
comorbidities. In these cases, the target blood pressure should be determined on a case-by-
case basis (SOR B).
Ref: Benetos A, Rossignol P, Cherubini A, et al: Polypharmacy in the aging patient: Management of hypertension in octogenarians.
JAMA 2015;314(2):170-180.

8. A 55-year-old male is found to be in cardiac arrest. When the EMTs arrive they note
ventricular fibrillation on the monitor. The patient began an antibiotic 3 days ago for a mild
case of community-acquired pneumonia.
Which one of the following antibiotics is most commonly associated with this scenario?
A) Doxycycline
B) Amoxicillin
C) Cefuroxime (Ceftin)
D) Azithromycin (Zithromax)

5
ANSWER: D
The likelihood of azithromycin inducing an arrhythmia is small, but given its widespread use
this possibility must be kept in mind. The arrhythmia results from prolongation of the QT
interval and is also more common in patients with a prior cardiac history. The other three
antibiotics have not been implicated in an increase in cardiac deaths.
Ref: Ray WA, Murray KT, Hall K, et al: Azithromycin and the risk of cardiovascular death. N Engl J Med 2012;366(20):1881-1890.

9. A 6-year-old male is brought to an urgent care center with a 48-hour history of left hip pain.
He is noted to have a significant limp, but will walk. His father reports that the child prefers
to lie on his back with his bent left knee out to the side. He is afebrile and well appearing,
with an erythrocyte sedimentation rate of 10 mm/hr (N 3–13), a peripheral WBC count of
7000/mm3 (N 5000–10,000), and a C-reactive protein level of 6 mg/L (N <10). Radiographs
of the left hip and knee are unremarkable.
Which one of the following would be most appropriate at this time?
A) Ibuprofen
B) Acetaminophen/codeine
C) Vancomycin
D) Physical therapy referral
E) Urgent orthopedic consultation

ANSWER: A
This child most likely has transient synovitis, based on the fact that he is well appearing and
afebrile, and his WBC count, erythrocyte sedimentation rate, C-reactive protein level, and
radiographs are all normal. This is a self-limited inflammatory condition, and he will likely
respond to NSAIDs such as ibuprofen and relative rest. Physical therapy and orthopedic
consultation are not required, as this condition lasts less than a week. Septic arthritis would
require treatment with intravenous antibiotics, but there is no evidence that he has this
condition.
Ref:Naranje S, Kelly DM, Sawyer JR: A systematic approach to the evaluation of a limping child. Am Fam Physician 2015;92(10):908-
916.

10. Supraventricular tachycardia may require treatment both acutely and over the long term.
Which one of the following medications is useful for both of these purposes?
A) Adenosine (Adenocard)
B) Amiodarone (Cordarone)
C) Esmolol (Brevibloc)
D) Procainamide
E) Verapamil (Calan, Verelan)

ANSWER: E

6
Calcium channel blockers such as verapamil or diltiazem can be used acutely to decrease
heart rate and terminate supraventricular tachycardia (SVT) and chronically to prevent SVT
recurrence. Adenosine, amiodarone, esmolol, and procainamide all can be used to treat SVT
acutely, but they are not suitable for long-term therapy.
Ref: Helton MR: Diagnosis and management of common types of supraventricular tachycardia. Am Fam Physician 2015;92(9):793-
800.

11. A 16-year-old female is brought to your office by her mother, who is concerned that her
daughter has seemed tired lately. The patient denies any specific health concerns or recent
illnesses. She is taking an oral contraceptive and reports that her menstrual bleeding is light
in flow. Recent laboratory findings include a TSH level of 1.44 :U/mL (N 0.5–5.0), a
hematocrit level of 38% (N 36–46), a mean corpuscular volume of 71 :m3 (N 78–102), an
RBC count of 5.7 million/mm3 (N 4.10–5.10), and ovalocytes on a peripheral smear.
Which one of the following is most likely to explain this patient’s initial laboratory
abnormalities and lead to a diagnosis?
A) A vitamin B12 level
B) A ferritin level
C) A free T4 level
D) Hemoglobin electrophoresis
E) A fluorescent spot test

ANSWER: D
This presentation is consistent with $-thalassemia minor trait in a generally asymptomatic
patient. Hemoglobin electrophoresis will be abnormal, with HbA2 increased and HbA
decreased. The free T4 level is likely to be normal in a patient with a normal TSH level. A
ferritin level is also likely to be normal given the normal levels of hemoglobin and hematocrit.
A fluorescent spot test is used to screen for G-6-PD deficiency, which would be associated
with bite cells and Heinz bodies. A vitamin B12 level would be useful for evaluating
macrocytosis, which is not present in this patient.
Ref: Wang M: Iron deficiency and other types of anemia in infants and children. Am Fam Physician 2016;93(4):270-278.

7
12. An 80-year-old male is admitted to the hospital for pneumonia. He develops what the
nurses describe to you as “sundowning” behavior that includes nighttime disorientation and
some mild agitation. His wife says he is not like this at home. During morning rounds he is
pleasant and answers questions appropriately except he forgets why he is in the hospital. His
examination, including a neurologic examination, is normal except for crackles on chest
auscultation consistent with the pneumonia. He is not able to say the days of the week
backwards.
Which one of the following is most likely in this patient?
A) Alzheimer’s disease
B) Delirium
C) Vascular dementia
D) Encephalitis
E) Stroke

ANSWER: B
A diagnosis of delirium based on the Confusion Assessment Method (CAM) algorithm
requires the presence of an acute onset and a fluctuating course, inattention, and either an
altered level of consciousness or disorganized thinking. The patient described in this question
exhibits an acute onset, fluctuation, inattention, and an altered level of consciousness.
This patient’s presentation is more consistent with delirium than encephalitis, as patients with
encephalitis frequently have signs of systemic illness such as fever, lethargy, seizures, and
neurologic deficits, as well as a nonspecific rash in some cases. Furthermore, the fluctuations
in the level of consciousness seen in delirium do not occur with encephalitis. Vascular
dementia and Alzheimer’s disease develop over years, not acutely as in this case. Stroke,
while a consideration and a potential cause of delirium, would not be the most likely diagnosis
in an older patient hospitalized with pneumonia.
Ref: Inouye SK, van Dyck CH, Alessi CA, et al: Clarifying confusion: The confusion assessment method. A new method for detection
of delirium. Ann Intern Med 1990;113(12):941-948. 2) Barr J, Fraser GL, Puntillo K, et al: Clinical practice guidelines for the
management of pain, agitation, and delirium in adult patients in the intensive care unit. Crit Care Med 2013;41(1):263-306. 3) Inouye
SK, Westendorp RG, Saczynski JS: Delirium in elderly people. Lancet 2014;383(9920): 911-922.

13. A 40-year-old female is concerned that she may have amyotrophic lateral sclerosis (ALS).
Her paternal uncle died in his forties of this disease and she has visited multiple Internet sites
that have exacerbated her fears.
Which one of the following findings on examination would be considered a clinical
manifestation of ALS?
A) A bull’s-eye rash
B) Paresthesias
C) Pseudoclaudication
D) Asymmetric leg weakness
E) Charcot joint

8
ANSWER: D
Amyotrophic lateral sclerosis is a pure motor disorder involving anterior horn cells in the
spinal cord and their brainstem homologues. It may begin as lower motor or bulbar palsy, and
the median survival is 3–5 years. Pseudoclaudication is associated with spinal stenosis, and
paresthesias are caused by sensory problems. A bull’s-eye rash is associated with Lyme
disease. Charcot joint is seen with conditions that involve sensory loss, such as diabetes
mellitus or syphilis.
Ref: Kasper DL, Fauci AS, Hauser SL, et al (eds): Harrison’s Principles of Internal Medicine, ed 19. McGraw-Hill, 2015, pp 2631-
2635.

14. A 25-year-old male presents with pleuritic chest pain. He has not had a fever or symptoms
of a respiratory infection. He has no risk factors for thromboembolism, and no past medical
or family history of thromboembolism. His vital signs and examination are normal, including
clear lungs on auscultation and no chest wall tenderness. Laboratory findings include a normal
CBC and a normal D-dimer level. A chest radiograph is also normal.
Which one of the following is the most appropriate next step in this patient’s management?
A) An NSAID
B) An anticoagulant pending further imaging
C) C-reactive protein and antinuclear antibody levels, and corticosteroids
D) A rib belt

ANSWER: A
Causes of pleuritic chest pain include pneumonia, chest wall trauma, pulmonary embolus, and
vasculitis. If these conditions are deemed unlikely based on the history, physical examination,
and limited laboratory studies, a chest radiograph is obtained. If this is within normal limits
then viral pleuritic pain is most likely, and can be treated with an NSAID. Given that the
history and physical findings are not suspicious for thromboembolism and a D-dimer is
negative, anticoagulation is inappropriate. With no other systemic symptoms or findings of
collagen-vascular disease, corticosteroids are not indicated. Since there is no rib tenderness
and no radiographic findings of an acute rib fracture, a rib belt is not indicated.
Ref: Kass SM, Williams PM, Reamy BV: Pleurisy. Am Fam Physician 2007;75(9):1357-1364. 2) Marx JA, Hockberger RS, Walls
RM (eds): Rosen’s Emergency Medicine: Concepts and Clinical Practice, ed 8. Elsevier Saunders, 2014, pp 992-996.

15. A 65-year-old white female with terminal cancer and severe hepatic dysfunction has
musculoskeletal and neuropathic pain at the end of life. Which one of the following
medications should not be used in this patient because of her hepatic dysfunction?
A) Fentanyl
B) Hydromorphone (Dilaudid)
C) Methadone
D) Morphine
E) Oxycodone (OxyContin)

9
ANSWER: C
Methadone has a widely variable half-life (7–72 hours) and bioavailability, as well as an
inactive sedating metabolite. It is metabolized and cleared by the liver, and should therefore
not be used in patients with severe hepatic impairment. It is, however, a reasonable option for
patients with severe renal impairment (SOR C). Morphine, oxycodone, fentanyl, and
hydromorphone can be used cautiously in patients with hepatic impairment, with reductions
in the initial dosage and close monitoring (SOR C).
Ref: Nicholson AB: Methadone for cancer pain. Cochrane Database Syst Rev 2007;(4):CD003971. 2) Groninger H, Vijayan J:
Pharmacologic management of pain at the end of life. Am Fam Physician 2014;90(1):26-32.

16. A 55-year-old female with hypertension sees you for a routine follow-up visit. A physical
examination is normal except for a BMI of 34.0 kg/m2. Laboratory findings are notable for
an ALT (SGPT) of 53 U/L (N 7–35), and an AST (SGOT) of 28 U/L (N 10–34). She had a
similar elevation a month ago when she was seen in the emergency department. Her
comprehensive metabolic panel is otherwise normal, and a CBC is also normal. She has had
negative hepatitis B surface antigen and hepatitis C antibody testing in the past. She does not
drink alcohol or take medications that affect liver function.
Which one of the following is the most appropriate test for evaluating this patient?
A) Anti–smooth muscle antibody
B) Ceruloplasmin
C) Hepatitis D antibody
D) Iron studies
E) A liver biopsy

ANSWER: D
This patient is obese and has an isolated elevation of ALT. The initial history and workup,
including testing for chronic hepatitis, a medication review, and alcohol screening, have all
been conducted. Current guidelines suggest iron studies to screen for hemochromatosis as
part of the initial evaluation (SOR C). Anti–smooth-muscle antibody and ceruloplasmin levels
can detect more rare causes of hepatocyte inflammation (autoimmune hepatitis and copper
deposition from Wilson’s disease). These tests can be considered in persistent transaminitis
based on clinical judgment (SOR C). Hepatitis D is a superinfection seen in patients with
hepatitis B, and is not a consideration in this patient. A liver biopsy may also be considered
in persistent transaminitis, especially in the setting of clinical or serologic evidence of
advanced liver fibrosis, but is not recommended at this stage (SOR C). Patients with fatty
liver disease are at increased risk for developing liver cirrhosis.
Ref: Chalasani N, Younossi Z, Lavine JE, et al: The diagnosis and management of non-alcoholic fatty liver disease: Practice guideline
by the American Association for the Study of Liver Diseases, American College of Gastroenterology, and the American
Gastroenterological Association. Hepatology 2012;55(6):2005-2023. 2) Grover M, Rutkowski R, Nashelsky J: FPIN’s Clinical
Inquiries: Evaluation of elevated serum transaminase levels. Am Fam Physician 2012;86(8):1-2.

10
17. Pregnant women who empty cat litter boxes are at increased risk for which one of the
following?
A) Toxoplasmosis
B) Toxocara mystax (Toxocara cati) infection
C) Chagas disease
D) Cysticercosis
E) Aspergillosis

ANSWER: A
Cats that are infected with Toxoplasma gondii can shed the virus for weeks when newly
infected. Cleaning the cat’s litter box is a high-risk activity, and transmission to the fetus can
occur. Toxocara cati infection is less common and usually results from ingesting
contaminated cat meat or soil containing cat feces. Chagas disease is spread by insect bites.
Cysticercosis is generally acquired by eating contaminated pork. Aspergillosis usually occurs
in immunocompromised people who inhale the spores found in soil (SOR A).
Ref: Woodhall D, Jones JL, Cantey PT, et al: Neglected parasitic infections: What every family physician needs to know. Am Fam
Physician 2014;89(10):803-811.

18. Which one of the following has been shown to result from the use of continuous
prophylactic antibiotics in patients with moderate COPD?
A) A clinically significant reduction in COPD exacerbations
B) A clinically significant improvement in quality-of-life scores
C) A demonstrable decrease in the number of resistant organisms in the sputum
D) The development of conduction defects, including prolonged QT intervals
E) An increased incidence of tinnitus

ANSWER: A
A review of multiple studies has revealed that continuous daily macrolide antibiotic use for 1
year decreased the number of exacerbations in older patients with COPD (SOR A). Quality
of life did not show improvement, however, and resistance to macrolide and quinolone
antibiotics rose. There was no increase in conduction abnormalities or tinnitus, but the
incidence of hearing loss increased with the use of azithromycin.
Ref:Herath SC, Poole P: Prophylactic antibiotic therapy in chronic obstructive pulmonary disease. JAMA 2014;311(21):2225-2226.

19. Which one of the following services must be covered by any insurance purchased through
the insurance marketplace, as long as the patient sees an in-network provider?
A) Preventive dental care
B) Annual eye examinations by an optometrist or ophthalmologist
C) Colorectal cancer screening
D) Travel immunizations for work-related travel
E) Bariatric surgery for morbidly obese patients

11
ANSWER: C
Patients with new health insurance plans or policies purchased under the Patient Protection
and Affordable Care Act are expected to be covered for certain services. Preventive services
with a U.S. Preventive Services Task Force grade of A or B, such as colorectal cancer
screening, are covered without cost-sharing when performed by an in-network provider.
Although coverage for vaccines such as diphtheria, tetanus, and influenza is required with
these plans, those that are needed only for travel are not covered. In addition, although
consumers can purchase vision and dental coverage in addition to health coverage through
the marketplace, they are not a required part of coverage. While coverage for obesity
counseling is required, coverage for bariatric surgery still varies by state.
Ref: US Preventive Services Task Force: Guide to Clinical Preventive Services 2014. Agency for Healthcare Research and Quality,
2014. 2) National Conference of State Legislatures. Health reform and health mandates for obesity. 2016. 3) Health benefits and
coverage: What marketplace health insurance plans cover. US Centers for Medicare and Medicaid Services, HealthCare.gov.

20. A 65-year-old male has severe liver cirrhosis from a combination of hepatitis C infection
and alcohol abuse. He previously underwent a transjugular intrahepatic portosystemic shunt
(TIPS) procedure.
While the TIPS procedure reduces the likelihood of most complications of cirrhosis, it may
increase the risk for which one of the following?
A) Anasarca
B) Ascites
C) Hepatic encephalopathy
D) Upper gastrointestinal bleeding from esophageal varices
E) Upper gastrointestinal bleeding from portal hypertensive gastropathy

ANSWER: C
The transjugular intrahepatic portosystemic shunt (TIPS) procedure, by shunting blood
destined for the liver into the systemic circulation, lowers pressure in the portal veins, thereby
decreasing portal system hypertension and making variceal bleeding and portal hypertensive
gastropathy less likely. TIPS may decrease the likelihood of variceal bleeding by as much as
90%. TIPS also reduces the pressure that leads to ascites and lower extremity edema, or the
massive edema of anasarca. The diversion of blood from the liver circulation compromises
the liver’s role in removing toxins, including the serum marker ammonia. This may make
patients more vulnerable to episodes of hepatic encephalopathy.
Ref: Fidelman N, Kwan SW, LaBerge JM, et al: The transjugular intrahepatic portosystemic shunt: An update. AJR Am J Roentgenol
2012;199(4):746-755. 2) Goldman L, Schafer AI (eds): Goldman’s Cecil Medicine, ed 25. Elsevier Saunders, 2016, pp 1023-1031.

12
21. A 77-year-old white male tells you he has had urinary incontinence for more than a year.
The incontinence occurs with sudden urgency. No association with coughing or positional
change has been noted, and there is no history of fever or dysuria. He underwent transurethral
resection of the prostate (TURP) for benign prostatic hyperplasia a year ago, and he says his
urinary stream has improved. A rectal examination reveals a smoothly enlarged prostate
without nodularity, and normal sphincter tone. No residual urine is found with post-void
catheterization.
Which one of the following is the most likely cause of this patient’s incontinence?
A) Detrusor instability
B) Urinary tract infection
C) Overflow
D) Fecal impaction
E) Recurrent bladder outlet obstruction

ANSWER: A
In elderly patients, detrusor instability is the most common cause of urinary incontinence in
both men and women. Incontinence may actually become worse after surgical relief of
obstructive prostatic hypertrophy.
Infection is unlikely to be the cause of persistent incontinence in this patient in the absence of
fever or symptoms of urinary tract infection. Overflow is unlikely in the absence of residual
urine. Impaction is a relatively rare cause of urinary incontinence, and associated findings
would be present on the rectal examination. Normalization of the urinary stream and the
absence of residual urine reduce the likelihood of recurrent obstruction. The prostate would
be expected to remain enlarged on rectal examination after transurethral resection of the
prostate (TURP).
Ref: Gibbs CF, Johnson TM II, Ouslander JG: Office management of geriatric urinary incontinence. Am J Med 2007;120(3):211-220.
2) Khandelwal C, Kistler C: Diagnosis of urinary incontinence. Am Fam Physician 2013;87(8):543-50.

22. A 45-year-old female with a BMI of 50.0 kg/m2 comes to your office to discuss treatment
of her chronically elevated fasting glucose levels. Recent test results include a hemoglobin
A1c of 6.0%. In the past she was not able to tolerate metformin (Glucophage) and would like
to try another medication to reduce her risk of developing diabetes mellitus.
Which one of the following medications would be most appropriate for this patient?
A) Acarbose (Precose)
B) Bromocriptine (Parlodel)
C) Canagliflozin (Invokana)
D) Glipizide (Glucotrol)
E) Sitagliptin (Januvia)

ANSWER: A

13
Individuals with impaired fasting glucose or impaired glucose tolerance have a higher risk of
developing diabetes mellitus in the future. Randomized, controlled trials have shown that
taking metformin, "-glucosidase inhibitors (acarbose), orlistat, or thiazolidinediones
significantly reduces the risk of developing diabetes mellitus. The U.S. Diabetes Prevention
Program Outcomes Study demonstrated a 34% reduction in the development of diabetes
mellitus at 10 years. In addition to medications, it is also recommended that patients be
counseled about weight loss and engaging in moderate physical activity for at least 150
minutes per week. The other medications listed are indicated for the treatment of diabetes but
have not been shown to be effective for prevention.
Ref: Chiasson JL, Josse RG, Gomis R, et al; STOP-NIDDM Trial Research Group: Acarbose treatment and the risk of cardiovascular
disease and hypertension in patients with impaired glucose tolerance: The STOP-NIDDM Trial. JAMA 2003;290(4):486-494. 2)
American Diabetes Association: Standards of Medical Care in Diabetes—2016. 4. Prevention or delay of type 2 diabetes. Diabetes
Care 2016;39(Suppl 1):S36-S38.

23. A 33-year-old patient with allergic rhinitis tells you he feels drowsy when taking
diphenhydramine (Benadryl) and asks if there is an alternative that will cause less sedation.
You tell him that most second-generation antihistamines have a better adverse effect profile
and cause less sedation than first-generation antihistamines.
Which one of the following second-generation antihistamines would you advise the patient to
avoid, however, because it is known to cause sedation?
A) Cetirizine (Zyrtec)
B) Desloratadine (Clarinex)
C) Fexofenadine (Allegra)
D) Levocetirizine (Xyzal)
E) Loratadine (Claritin)

ANSWER: A
Desloratadine, fexofenadine, levocetirizine, and loratadine are among the second-generation
antihistamines that have a better adverse effect profile and cause less sedation than first-
generation antihistamines (SOR A). Second-generation antihistamines have more complex
chemical structures that decrease their movement across the blood-brain barrier, reducing
central nervous system adverse effects such as sedation. Although cetirizine is generally
classified as a second-generation antihistamine and a more potent histamine antagonist, it is
known to cause sedation (SOR A).
Ref: Bender BG, Berning S, Dudden R, et al: Sedation and performance impairment of diphenhydramine and second-generation
antihistamines: A meta-analysis. J Allergy Clin Immunol 2003;111(4):770-776. 2) Sur DK, Plesa ML: Treatment of allergic rhinitis.
Am Fam Physician 2015;92(11):985-992.

14
24. The mother of a 16-year-old male calls to report that her son has a severe sore throat and
has been running a fever of 102°F. As you question the mother further, which one of the
following would be most specific for peritonsillar abscess?
A) A 1-day duration of illness
B) Ear pain
C) Difficulty opening his mouth
D) Hoarseness
E) Pain with swallowing

ANSWER: C
Trismus is almost universally present with peritonsillar abscess, while voice changes, otalgia,
and odynophagia may or may not be present. Pharyngotonsillitis and peritonsillar cellulitis
may also be associated with these complaints. Otalgia is common with peritonsillar abscess,
otitis media, temporomandibular joint disorders, and a variety of other conditions.
Peritonsillar abscess is rarely found in patients who do not have at least a 3-day history of
progressive sore throat.
Ref: Galioto NJ: Peritonsillar abscess. Am Fam Physician 2008;77(2):199-202. 2) Kasper DL, Fauci AS, Hauser SL, et al (eds):
Harrison’s Principles of Internal Medicine, ed 19. McGraw-Hill, 2015, p 235.

25. A 14-year-old goalkeeper dives for a save during a soccer game and strikes her head on
the goalpost. She does not lose consciousness but develops a headache and is removed by the
coach for the remainder of the game. She becomes nauseated and has balance problems over
the next several hours but can clearly recall the event. She comes to your clinic the following
day. Her school requires baseline neurocognitive testing prior to all sports participation each
year and you have a copy of this year’s results.
Which one of the following would be the most appropriate recommendation at this time?
A) CT of the head
B) Return to practice when her symptoms are relieved by medication
C) Neurocognitive testing at this visit
D) Physical and cognitive rest
E) The use of protective headgear for the remainder of the season

ANSWER: D
There are estimated to be 1.6–3.8 million sports-related concussions each year in the United
States, and appropriate management can prevent many of the long-term complications. In this
scenario, the patient meets the criteria for a concussion, with headache, nausea, and imbalance
following an injury. She was appropriately removed from the game. The next step in
management is complete physical and cognitive rest until her symptoms resolve without
medication (SOR C). Complete physical rest includes avoidance of any physical activity that
exacerbates symptoms. Cognitive rest means avoidance of activities that require
concentration or attention, such as schoolwork and electronics use.

15
Once the patient’s symptoms have resolved without medication, she should be reevaluated
and can start a graded return-to-play protocol (SOR C). Reevaluation may include repeat
neurocognitive testing and postural stability testing (SOR C). Both types of testing can be
important in the evaluation but there is insufficient evidence that either affects outcomes.
In the initial evaluation of concussion, imaging is not indicated in the absence of focal
neurologic findings, loss of consciousness for more than 60 seconds, or evidence of skull
fracture (SOR C). If imaging criteria are met, CT of the head is indicated. Plain films are not
indicated in the evaluation.
There is no evidence that protective gear reduces the incidence of concussion (SOR C). Even
though some protective gear can prevent other types of injuries, it is not indicated for
concussion prevention. Eliminating dangerous behaviors such as heading the ball in soccer
can help, however.
Ref: Scorza KA, Raleigh MF, O’Connor FG: Current concepts in concussion: Evaluation and management. Am Fam Physician
2012;85(2):123-132. 2) Armstrong C: Evaluation and management of concussion in athletes: Recommendations from the AAN. Am
Fam Physician 2014;89(7):585-587.

26. A 70-year-old male has a smoking history of >30 pack-years. He quit 10 years ago.
Which one of the following is recommended by the U.S. Preventive Services Task Force with
regard to lung cancer screening for this patient?
A) Low-dose noncontrast chest CT one time only
B) A standard chest radiograph and sputum cytology one time only
C) Low-dose noncontrast chest CT annually
D) Low-dose noncontrast chest CT every 3 years
E) No screening

ANSWER: C
The 2013 U.S. Preventive Services Task Force guidelines recommend annual lung cancer
screening with low-dose CT for asymptomatic adults 55–80 years of age who have a 30-pack-
year smoking history and currently smoke or have quit smoking within the past 15 years
(grade B recommendation). Screening should be discontinued when the patient has not
smoked for 15 years or develops a health problem that substantially limits life expectancy or
the ability or willingness to have curative lung surgery.
Ref: Final Recommendation Statement: Lung Cancer Screening. US Preventive Services Task Force, 2014.

16
27. A 25-year-old primigravida at 33 weeks gestation presents to the labor suite with a
complaint of painful uterine contractions every 3 minutes for the last 2 hours. A fetal monitor
confirms the contraction pattern, and the fetal heart rate pattern is category 1. A fetal
fibronectin test is ordered. Thirty minutes later the contractions have stopped, but results of
the fetal fibronectin test are positive. A speculum examination shows no indication of ruptured
membranes. The cervix is closed and thick to palpation.
The positive result on this patient’s fetal fibronectin test
A) has a low positive predictive value for preterm delivery
B) indicates a high likelihood of delivery before term
C) indicates a high likelihood of delivery within the next week
D) indicates occult rupture of the membranes
E) is diagnostic for preterm labor

ANSWER: A
The fetal fibronectin test is used to help determine the risk of early delivery. Unfortunately,
the positive predictive value for delivery within the next week is <20% in patients who have
clinical indications of preterm labor. In a patient who does not have any features of preterm
labor, it does not affect management. A negative result is reassuring, on the other hand,
because the negative predictive value is >95%. In studies that tested gravidas every week,
positive results had a mild correlation with preterm delivery, but not enough to guide
management. False-positive test results can be caused by maternal blood or amniotic fluid,
but fetal fibronectin does not indicate rupture of the membranes when other tests are negative.
Ref: Lowe MP, Zimmerman B, Hansen W: Prospective randomized controlled trial of fetal fibronectin on preterm labor management
in a tertiary care center. Am J Obstet Gynecol 2004;190(2):358-362. 2) Fetal fibronectin does not affect outcomes of preterm labor. J
Fam Pract 2004;53(6):442. 3) Sayres WG: Preterm labor. Am Fam Physician 2010;81(4):477-84. 4) Abbott DS, Hezelgrave NL, Seed
PT, et al: Quantitative fetal fibronectin to predict preterm birth in asymptomatic women at high risk. Obstet Gynecol 2015;125(5):1168-
1176.

28. The rash associated with erythema infectiosum (fifth disease) is characterized by which
one of the following?
A) Small red papules with central umbilication
B) Annular patches with raised borders and central clearing
C) Pink pustules that form a thick yellow crust
D) An erythematous “slapped cheek” facial rash
E) A symmetrical rash in a Christmas tree pattern

ANSWER: D
Fifth disease, caused by parvovirus B19, is a common childhood infection that typically
presents with a prodrome of low-grade fever, malaise, sore throat, nausea, and headache,
followed a few days later by an erythematous facial rash on the cheeks. The rash fades in a
few days but a lacy, reticular rash develops on the extremities. Once the rash appears, patients

17
are no longer considered contagious. NSAIDs and antihistamines are often used for symptom
relief (SOR A).
Ref: Allmon A, Deane K, Martin KL: Common skin rashes in children. Am Fam Physician 2015;92(3):211-216.

29. A 64-year-old male with nonvalvular atrial fibrillation has received anticoagulation
therapy with warfarin (Coumadin) for the past 3 years but it has been difficult to maintain his
INR in the therapeutic range. He has been afraid to transition to one of the newer oral
anticoagulants because of the lack of a reversal medication should he develop life-threatening
bleeding.
Which one of the following now has a reversal agent?
A) Apixaban (Eliquis)
B) Dabigatran (Pradaxa)
C) Edoxaban (Savaysa)
D) Rivaroxaban (Xarelto)

ANSWER: B
Idarucizumab has been approved for the urgent reversal of the anticoagulant effect of
dabigatran. The dose is two consecutive 2.5-g infusions and the cost is $3500. Apixaban,
edoxaban, and rivaroxaban—all factor Xa inhibitors—are not affected by this medication and
they do not currently have a reversal agent.
Ref: Idarucizumab (Praxbind)—An antidote for dabigatran. Med Lett Drugs Ther 2015;57(1482):157-158.

30. Both the American Academy of Family Physicians and the American College of
Obstetricians and Gynecologists recommend that non–medically indicated (elective)
inductions of labor or cesarean deliveries should be scheduled after what gestational age?
A) 37 weeks, 0 days
B) 39 weeks, 0 days
C) 40 weeks, 0 days
D) 41 weeks, 0 days
E) 42 weeks, 0 days

ANSWER: B
The American Academy of Family Physicians and the American College of Obstetricians and
Gynecologists collaborated on the Choosing Wisely campaign recommendation regarding
scheduled inductions of labor or cesarean deliveries. These organizations noted that delivery
prior to 39 weeks, 0 days is associated with an increased risk of learning disabilities and may
increase morbidity and mortality. While delivery prior to 39 weeks and 0 days may be
indicated in certain situations, a mature fetal lung test by itself is not an indication for delivery
(SOR C).

18
Ref: Main E, Oshiro B, Chagolla B, et al; (California Maternal Quality Care Collaborative): Elimination of non-medically indicated
(elective) deliveries before 39 weeks gestational age. California: March of Dimes; 2010. 2) American Academy of Family Physicians:
Fifteen things physicians and patients should question. ABIM Foundation, Choosing Wisely campaign, 2013. 3) Quinlan JD, Murphy
NJ: Cesarean delivery: Counseling issues and complication management. Am Fam Physician 2015;91(3):178-184. 4) American
College of Obstetricians and Gynecologists: Ten things physicians and patients should question. ABIM Foundation, Choosing Wisely
campaign, 2016.

31. Which one of the following is effective in preventing seizures associated with alcohol
withdrawal syndrome?
A) Carbamazepine (Tegretol)
B) Chlordiazepoxide
C) Clonidine (Catapres)
D) Gabapentin (Neurontin)
E) Phenytoin

ANSWER: B
Benzodiazepines, such as chlordiazepoxide, can prevent alcohol withdrawal seizures.
Anticonvulsants such as carbamazepine, gabapentin, and phenytoin have less abuse potential
than benzodiazepines but do not prevent seizures. Clonidine, an "-adrenergic agonist, reduces
the adrenergic symptoms associated with withdrawal but does not prevent seizures.
Ref: Muncie HL Jr, Yasinian Y, Oge L: Outpatient management of alcohol withdrawal syndrome. Am Fam Physician
2013;88(9):589-595.

32. A 66-year-old female with a history of shortness of breath returns to your office to review
her pulmonary function test results. Her FEV1/FVC ratio is 76%. You calculate that her FVC
is below the normal range for an adult. Her DLCO is also low.
Which one of the following is most consistent with her pulmonary function test results?
A) COPD
B) Asthma
C) Idiopathic pulmonary fibrosis
D) Chronic pulmonary emboli
E) Morbid obesity

ANSWER: C
This patient’s pulmonary function test (PFT) findings are consistent with a restrictive defect
with a low DLCO. Idiopathic pulmonary fibrosis, asbestosis, hypersensitivity pneumonitis,
and sarcoidosis are restrictive lung diseases with a low diffusion capacity due to alveolar
damage. COPD and asthma are both obstructive lung diseases, but can be associated with an
abnormal DLCO. Morbid obesity causes a restrictive lung disease associated with a normal
DLCO. The presence of chronic pulmonary emboli is associated with a low DLCO due to
pulmonary vascular disease, but PFTs are normal.
Ref: Johnson JD, Theurer WM: A stepwise approach to the interpretation of pulmonary function tests. Am Fam Physician
2014;89(5):359-366.

19
33. Which one of the following is the recommended initial treatment for allergic rhinitis in a
patient whose symptoms are affecting his quality of life?
A) An oral corticosteroid
B) An intranasal corticosteroid
C) An intranasal antihistamine
D) Subcutaneous immunotherapy injection
E) Sublingual immunotherapy

ANSWER: B
An intranasal corticosteroid alone should be the initial treatment for allergic rhinitis with
symptoms affecting quality of life (SOR A). Intranasal corticosteroids act by decreasing the
influx of inflammatory cells and inhibiting the release of cytokines, thereby reducing
inflammation of the nasal mucosa. Intranasal corticosteroids are more effective than oral and
intranasal antihistamines in the treatment of persistent or more severe allergic rhinitis (SOR
A). Intranasal antihistamines also have more adverse effects than intranasal corticosteroids
(SOR C). Subcutaneous and sublingual immunotherapy are not considered first-line
treatments but should be considered for moderate or severe persistent allergic rhinitis that is
not responsive to usual treatments (SOR A).
Ref: Price D, Bond C, Bouchard J, et al: International Primary Care Respiratory Group (IPCRG) guidelines: Management of allergic
rhinitis. Prim Care Respir J 2006;15(1):58-70. 2) Sur DK, Plesa ML: Treatment of allergic rhinitis. Am Fam Physician
2015;92(11):985-992.

20
34. A 32-year-old male presents with an acute onset of chest pain. His EKG is

Which one of the following agents would be most appropriate to relieve this patient’s chest
pain?
A) Hydrocodone (Lortab)
B) Indomethacin (Indocin)
C) Nitroglycerin
D) Omeprazole (Prilosec)
E) Oxygen

ANSWER: B
Acute pericarditis is a common cause of chest pain in young adults. It presents with an acute
to subacute onset of severe retrosternal and left precordial chest pain that is characteristically
alleviated by leaning forward or sitting up and worsened with lying supine. Viral and
idiopathic acute pericarditis are most common and occur at all ages, but are more common in
young adults. The pain, which is inflammatory in origin, typically responds to an NSAID such

21
as indomethacin. High-dose aspirin (2–4 g/day) has also been shown to be effective. Patients
with pain resulting from myocardial ischemia would benefit from nitroglycerin, while those
with gastroesophageal reflux disease would benefit from a proton pump inhibitor such as
omeprazole. Hydrocodone can be used to relieve moderate to severe pain but lacks anti-
inflammatory properties (SOR C).
Ref: Snyder MJ, Bepko J, White M: Acute pericarditis: Diagnosis and management. Am Fam Physician 2014;89(7):553-560. 2) Kasper
DL, Fauci AS, Hauser SL, et al (eds): Harrison’s Principles of Internal Medicine, ed 19. McGraw-Hill, 2015, pp 1571-1573.

35. A 40-year-old male presents to the emergency department with a fever and acute, severe
abdominal pain. A complete history and careful physical examination fail to localize the
source of his pain, and basic blood and urine tests also prove to be nondiagnostic. His pain is
diffuse, and you are concerned about a potentially serious pathology.
Which one of the following would be most appropriate to order next?
A) Plain radiographs of the abdomen
B) An upper gastrointestinal series with small-bowel follow-through
C) Ultrasonography of the abdomen
D) CT of the abdomen and pelvis
E) MRI of the abdomen

ANSWER: D
The differential diagnosis of acute nonlocalized abdominal pain is broad. CT is typically the
imaging modality of choice if there is significant concern about serious pathology or if the
diagnosis is unclear from the history, physical examination, and laboratory testing. In this
instance, the American College of Radiology recommends CT of the abdomen and pelvis,
preferably with contrast. One prospective study of patients with nontraumatic abdominal pain
in an emergency department setting found that CT results changed the leading diagnosis in
49% of patients and the management plan in 42% of patients.
Ref: Cartwright SL, Knudson MP: Diagnostic imaging of acute abdominal pain in adults. Am Fam Physician 2015;91(7):452-459.

36. A 46-year-old male consults you because of chronic, moderate, nonradiating low back
pain. The pain had a gradual onset and is partially relieved by rest; ibuprofen, 600 mg as
needed; duloxetine (Cymbalta), 60 mg daily; and gabapentin (Neurontin), 300 mg 3
times/day. He denies weakness, changes in urination, or changes in sensation. His physical
examination is notable for mild tenderness over the lumbar paraspinal muscles. A recent MRI
of his lumbosacral spine showed mild degenerative disc disease.
Which one of the following would you recommend as most likely to have long-term benefit?
A) A walking program
B) A trial of chiropractic care
C) A series of epidural corticosteroid injections
D) A transcutaneous electrical nerve stimulation (TENS) unit
E) A spinal cord stimulator

22
ANSWER: A
Family physicians are faced with many treatment options for managing chronic low back
pain. Both supervised walking and physical therapy resulted in decreased disability and pain
scores at 6 months in a 2015 randomized, controlled trial. A walking program is the best
choice because it is much more affordable than physical therapy. Cochrane reviews found
that neither epidural corticosteroid injections nor transcutaneous electrical nerve stimulation
were better than placebo, and spinal manipulation showed no benefit after 6 months. Spinal
cord stimulators may help patients with severe pain after failed back surgery, but lack
moderate or better evidence to support their widespread use.
Ref: Khadilkar A, Odebiyi DO, Brosseau L, Wells GA: Transcutaneous electrical nerve stimulation (TENS) versus placebo for chronic
low-back pain. Cochrane Database Syst Rev 2008;(4):CD003008. 2) Kwan G, Balady GJ: Cardiac rehabilitation 2012: Advancing the
field through emerging science. Circulation 2012;125(7):e369-e373. 3) Hurley DA, Tully MA, Lonsdale C, et al: Supervised walking
in comparison with fitness training for chronic back pain in physiotherapy: Results of the SWIFT single-blinded randomized controlled
trial (ISRCTN17592092). Pain 2015;156(1):131-147. 4) Herndon CM, Zoberi KS, Gardner BJ: Common questions about chronic low
back pain. Am Fam Physician 2015;91(10):708-714

37. A 30-year-old female with hyperthyroidism is treated with methimazole (Tapazole).


Which one of the following is a potential adverse reaction?
A) Agranulocytosis
B) Aplastic anemia
C) Thrombocytopenia
D) Ototoxicity
E) Renal toxicity

ANSWER: A
Agranulocytosis occurs in about 1 in 500 patients who are taking methimazole. Prior to
starting therapy the patient should be advised to be alert for a fever and severe sore throat. A
recent survey showed that 61% of patients were unaware of potential adverse reactions.
Another serious reaction is hepatotoxicity.
Ref:Burch HB, Cooper DS: Management of Graves disease: A review. JAMA 2015;314(23):2544-2554.

38. A 42-year-old female visits your office. She has alcohol use disorder and wants to quit
drinking. She recently went through inpatient detoxification and is attending Alcoholics
Anonymous meetings and counseling. She continues to have strong cravings and is fearful of
a relapse. Her medical history is notable for renal disease and osteopenia.
Which one of the following would be most appropriate for this patient?
A) Acamprosate
B) Disulfiram (Antabuse)
C) Naltrexone (ReVia)
D) Bupropion (Wellbutrin)

ANSWER:C

23
Medications are effective for alcohol use disorder and should be offered in conjunction with
psychosocial interventions. Oral naltrexone is the most effective medication to prevent
relapse. Acamprosate is moderately effective but is contraindicated in patients with renal
disease. A recent meta-analysis indicated that disulfiram is not effective for preventing
relapse. Bupropion is used for smoking cessation.
Ref: Jonas DE, Amick HR, Feltner C, et al: Pharmacotherapy for adults with alcohol use disorders in outpatient settings: A systematic
review and meta-analysis. JAMA 2014;311(18):1889-1900. 2) Centers for Disease Control and Prevention. Fact sheets—Alcohol use
and your health. 3) Hendry S, Mounsey A: PURLS: Consider these medications to help patients stay sober. J Fam Pract
2015;64(4):238-240.

39. Which one of the following antidepressants should be avoided in the elderly?
A) Bupropion (Wellbutrin)
B) Sertraline (Zoloft)
C) Mirtazapine (Remeron)
D) Paroxetine (Paxil)
E) Venlafaxine

ANSWER: D
Geriatric patients are at greater risk of adverse drug reactions compared to their younger
counterparts. Guidelines recommend several antidepressant agents as good first-line options,
including venlafaxine, bupropion, and mirtazapine. Among the SSRIs, citalopram,
escitalopram, and sertraline are all good choices, but paroxetine is associated with more
anticholinergic effects and should be avoided (SOR C).
Ref: Kovich H, DeJong A: Common questions about the pharmacologic management of depression in adults. Am Fam Physician
2015;92(2):94-100.

40. The American Academy of Pediatrics recommends administration of an autism-specific


screening tool at which well child visits?
A) 18 and 24 months
B) 2 and 3 years
C) 3 and 5 years
D) 5 and 10 years
E) 10 and 12 years

ANSWER: A
Since early intensive behavioral therapy can improve functional outcomes for children with
autism and autism spectrum disorders, early diagnosis is critical. The American Academy of
Pediatrics recommends screening all children with an autism-specific screening instrument at
18 and 24 months in addition to surveillance of developmental issues at all well child visits.
Ref: Carbone PS, Farley M, Davis T: Primary care for children with autism. Am Fam Physician 2010;81(4):453-460. 2) Zwaigenbaum
L, Bauman ML, Fein D, et al: Early screening of autism spectrum disorder: Recommendations for practice and research. Pediatrics
2015;136(Suppl 1):S41-59.

24
41. A 25-year-old female is concerned about contracting human papillomavirus (HPV) and
cervical cancer because she has a new sexual partner who was recently treated for genital
warts. She has not received any medical care in over 5 years. She reports a lifetime total of
four sexual partners and denies any chronic medical problems, but is a smoker. Her maternal
grandmother died of cervical cancer. Her chart indicates that she completed the HPV
vaccination series. A physical examination is normal.
Which one of the following is the most appropriate screening test for this patient?
A) Colposcopy
B) HPV DNA typing
C) HPV antibodies
D) A Papanicolaou (Pap) test and HPV cotesting
E) A Pap test with reflex high-risk HPV testing

ANSWER: E
Women between the ages of 21 and 29 at average risk for cervical cancer should be screened
with cytology every 3 years (USPSTF A recommendation). Because of the high prevalence
of HPV infection in this age group and because there are no clear benefits to HPV testing,
testing is not recommended (USPSTF grade D). However, most clinicians will order reflex
testing for high-risk HPV types if the Papanicolaou smear shows atypical squamous cells of
uncertain significance (ASCUS), based on a recommendation by the American Society for
Colposcopy and Cervical Pathology.
Women 30–65 years of age at average risk for cervical cancer may be screened with cytology
with HPV cotesting every 5 years or with cytology alone every 3 years (USPSTF grade A).
HPV DNA typing and colposcopy are not screening tests and are used for further evaluation
of cytologic abnormalities. HPV antibody results have no role in screening. Women with a
history of HPV immunization should continue to be screened according to usual guidelines.

25
42. A 56-year-old male is hospitalized for workup of a syncopal episode. He has a history of
type 2 diabetes mellitus and dyslipidemia, which are well controlled with metformin
(Glucophage) and atorvastatin (Lipitor). In addition, he takes a daily low-dose aspirin and a
multivitamin.
The cardiac examination reveals occasional skipped beats but the remainder of the physical
examination is normal. While hospitalized he has another syncopal episode. His clinical
examination and EKG post syncope are normal. A telemetry rhythm tracing captured during
this syncopal episode is

Which one of the following is the most likely cause of his syncope?
A) Atrial fibrillation
B) Mobitz type I second degree atrioventricular block
C) Mobitz type II second degree atrioventricular block
D) Third degree atrioventricular block

ANSWER: D
Third degree atrioventricular (AV) block is characterized by the inability of atrial impulses to
reach the ventricles. Hence, the EKG will show both atrial activity (P waves) and ventricular
“escape” activity (QRS complexes) with no correlation between them. Hemodynamically
unstable patients with third degree AV block should be emergently treated with atropine and
temporary pacing. With Mobitz type I AV block there is progressive PR interval prolongation
preceding a nonconducted P wave. With Mobitz type II AV block the PR interval remains the
same but there is a sudden dropped P wave. Second and third degree AV blocks are seen in
patients with advanced heart disease. The EKG in atrial fibrillation lacks distinct P waves.
Additionally, fibrillary waves and irregularly irregular intervals between QRS complexes are
characteristic (SOR C).
Ref: Link MS: Evaluation and initial treatment of supraventricular tachycardia. N Engl J Med 2012;367(15):1438-1448. 2) Goldberger
AL, Goldberger ZD, Shvilkin A: Goldberger’s Clinical Electrocardiography: A Simplified Approach, ed 8. Elsevier Saunders, 2012.

26
43. Which one of the following is the most common neurologic complication of Paget’s
disease of the bone?
A) Headache
B) Facial palsy
C) Hearing loss
D) Spinal stenosis
E) Hydrocephalus

ANSWER: C
Neurologic complications related to Paget’s disease of the bone can result from bony
compression of the cranial nerves, spinal cord, and/or nerve roots, resulting in pain or loss of
function, or from the creation of a vascular steal of blood supplying CNS structures, which
causes paralysis that is often reversible. In rare cases it is caused by basilar invagination of
the C2 vertebra compressing the brain stem and leading to hydrocephalus. Hearing loss, the
most common neurologic complication of Paget’s disease, was initially believed to be caused
by compression of the vestibulocochlear nerve but is now thought to be the result of cochlear
damage.
Ref: Singer FR, Bone HG 3rd, Hosking DJ, et al: Paget’s disease of bone: An Endocrine Society clinical practice guideline. J Clin
Endocrinol Metab 2014;99(12):4408–4422.

44. A 4-year-old female slipped and fell off the edge of a dock but did not go underwater
because her mother caught her by the hand and pulled her back. Since that time she has been
upset and has not wanted to play, keeping her arm slightly bent and close to her side.
The physical examination supports your initial diagnosis. Which one of the following would
be the best management?
A) No immobilization, with follow-up in 24 hours
B) Placing the arm in a sling
C) Attempting reduction by hyperpronation
D) A long arm posterior splint
E) A figure-of-eight harness

ANSWER: C
This patient’s presentation and mechanism of injury suggest radial head subluxation, or
“nursemaid’s elbow.” This is a clinical diagnosis that usually does not require imaging prior
to attempted reduction, either by hyperpronation or the supination and flexion method. There
is no evidence of a fracture requiring immobilization of the joint or the arm, but the injury
does require treatment.
Ref: Fleisher GR, Ludwig S (eds): Textbook of Pediatric Emergency Medicine, ed 6. Lippincott Williams & Wilkins, 2010, pp 1832-
1833. 2) Tintinalli JE, Kelen GD, Stapczynski JS (eds): Emergency Medicine: A Comprehensive Study Guide, ed 7. McGraw-Hill,
2011, p 983.

27
45. A 30-year-old male is diagnosed with Lyme disease and is concerned because he read that
this can lead to heart disease. Which one of the following is the most common cardiac problem
associated with this disease?
A) Endocarditis
B) Heart block
C) Pericardial effusion
D) Pulmonary edema
E) Valvular heart disease

ANSWER: B
The most common cardiac abnormality associated with Lyme disease is fluctuating degrees
of atrioventricular (AV) block. This may be first degree, second degree, or complete heart
block. The condition is usually temporary, lasting from 3 to 42 days, with more advanced
block lasting longer than first degree AV block. The degree of AV block can fluctuate within
minutes. The highest risk of AV block is in patients with a PR interval >300 msec. Other
cardiac abnormalities such as myocarditis, left ventricular failure, and pericarditis are less
common.
Ref: Kasper DL, Fauci AS, Hauser SL, et al (eds): Harrison’s Principles of Internal Medicine, ed 19. McGraw-Hill, 2015, pp 1150-
1151.

46. An orthopedic surgeon contacts you about abnormal preoperative laboratory results for
one
of your patients. The patient is a 58-year-old male who takes acetaminophen and
over-the-counter naproxen for pain but is otherwise healthy and has no symptoms.
Laboratory Findings
WBCs …………………………3000/mm3 (N 3000–11,600)
Hemoglobin ……………………8.9 g/dL (N 13.0–18.0)
Hematocrit …………………… 26.9% (N 38.5–52.0)
Platelets ………………………118,000/mm3 (N 130,000–140,000)
Mean corpuscular volume ……. 89 :m3 (N 80–98)
Red cell distribution width ……. 12.6% (N 11.0–14.5)
Serum vitamin B12 ……………… 457 pg/mL (N 220–960)
Ferritin …………………………258 :g/L (N 24–336)
Reticulocytes …………………… 1.5% (N 0.8–2.1)
Which one of the following is most consistent with these results?
A) Folate deficiency
B) Iron deficiency
C) Hemolysis
D) Acute blood loss
E) Bone marrow suppression

28
ANSWER: E
This patient’s test results suggest a moderate anemia that is associated with a mild reduction
of platelets. The mean corpuscular volume is solidly in the normal range, so this is a
normocytic anemia and is unlikely to be due to iron or folate deficiency, which usually result
in microcytic and macrocytic anemias, respectively. A normal ferritin level also suggests a
diagnosis other than iron deficiency.
Acute blood loss and hemolysis are two common causes of normocytic anemia. However,
with both of these conditions a high reticulocyte count would be expected. The fact that the
reticulocyte count is in the normal range despite significant anemia suggests that this patient
has decreased bone marrow production of red blood cells, which is at least contributing to his
anemia. Bone marrow response to anemia is often evaluated by using the reticulocyte index,
which is calculated by multiplying the measured reticulocyte percentage (1.5 in this patient)
by the ratio of the patient’s hematocrit to a normal hemotocrit based on the person’s age and
sex (26.9 divided by 45 in this case). If the patient has a normal bone marrow, the reticulocyte
index should be 2%–3% or higher. In this case the result is 0.9%, suggesting a profound
suppression of bone marrow.
Ref: Hoffman R, Benz EJ, Silberstein LE, et al: Hematology: Basic Principles and Practice, ed 6. Elsevier Inc, 2013, pp 418-426.

47. A 35-year-old female has had a foul-smelling vaginal discharge for the past week since
having sexual intercourse with a new male partner. It does not seem thicker than her usual
discharge. She does not have any itching or pain with intercourse.
Which one of the following are you most likely to find on examination?
A) Evidence of excoriation of the vulva
B) Vulvar erythema
C) Vaginal atrophy
D) A vaginal pH of 4.0–4.5
E) A vaginal pH >4.5

ANSWER: E
This patient is more likely to have bacterial vaginosis than other causes of vaginitis, based on
the presence of a foul-smelling discharge in the absence of pruritus or dyspareunia. The odor
of bacterial vaginitis is more likely to appear in the presence of semen because of the increase
in vaginal alkalinity. Bacterial vaginosis is associated with a vaginal pH >4.5. Vulvovaginal
candidiasis is associated with vulvar erythema, excoriation of the vulva, and a normal vaginal
pH (4.0–4.5). Vaginal atrophy is seen in women with estrogen deficiency, and atrophic
vaginitis is unlikely in this patient due to her age and lack of pruritus and dyspareunia.
Ref: ACOG Practice Bulletin. Clinical management guidelines for obstetrician-gynecologists, number 72, May 2006: Vaginitis. Obstet
Gynecol 2006;107(5):1195-1206. 2) Hainer BL, Gibson MV: Vaginitis: Diagnosis and treatment. Am Fam Physician 2011;83(7):807-
815.

29
48. A 29-year-old female presents to your office for a routine prenatal visit at 30 weeks
gestation. She received Tdap vaccine during her last pregnancy 3 years ago.
Which one of the following should she receive today?
A) Live attenuated influenza vaccine
B) MMR
C) Tdap
D) Varicella vaccine
E) No vaccine

ANSWER: C
Pregnant women should receive a dose of Tdap, optimally between 27 and 36 weeks of
gestation, regardless of when they last received it. MMR vaccine and varicella vaccine are
both live attenuated vaccines and should not be given to women known to be pregnant.
Influenza vaccine is recommended for all women who are or will be pregnant during influenza
season, but pregnant women should receive the inactivated influenza vaccine.
Ref: National Center for Immunization and Respiratory Diseases: General recommendations on immunization—
Recommendations of the Advisory Committee on Immunization Practices (ACIP). MMWR Recomm Rep 2011;60(2):1-64. 2) Centers
for Disease Control and Prevention: Guidelines for Vaccinating Pregnant Women. 2013.

49. A 72-year-old female sees you for a preoperative evaluation prior to cataract surgery. Her
history and physical examination are unremarkable, and she has no medical problems other
than bilateral cataracts.
Which one of the following is recommended prior to surgery in this patient?
A) An EKG only
B) An EKG and chest radiography
C) A CBC only
D) A CBC and serum electrolytes
E) No testing

ANSWER: E
According to a Cochrane review, routine preoperative testing prior to cataract surgery does
not decrease intraoperative or postoperative complications (SOR A). The American Heart
Association recommends against routine preoperative testing in asymptomatic patients
undergoing low-risk procedures, since the cardiac risk associated with such procedures is less
than 1%.
Ref: Keay L, Lindsley K, Teisch J, et al: Routine preoperative medical testing for cataract surgery. Cochrane Database Syst Rev
2012;(3):CD007293.

30
50. A 74-year-old male who recently underwent lumbar laminectomy for spinal stenosis
complains of pain in his right great toe. He is unable to ambulate due to the pain. His other
chronic medical problems include hypertension and stage 4 chronic kidney disease. On
examination he has erythema, edema, warmth, and tenderness over the right first metatarsal
joint.
Which one of the following is the best initial treatment for his condition?
A) Allopurinol (Zyloprim)
B) Colchicine (Colcrys)
C) Indomethacin
D) Prednisone
E) Probenecid

ANSWER: D
This patient has gout, based on his history and examination. NSAIDs, colchicine, or
corticosteroids may be used as first-line treatment of gout (SOR B). The best initial treatment
in this patient would be prednisone, due to his advanced chronic kidney disease. Allopurinol
may be a good choice to prevent future episodes if his uric acid level is elevated, but should
not be started as an acute treatment. Probenecid would also not be an acute treatment but can
be used in some patients for prevention of future attacks.
Ref: Hainer BL, Matheson E, Wilkes RT: Diagnosis, treatment, and prevention of gout. Am Fam Physician 2014;90(12):831-836.

51. A 40-year-old male is brought to the emergency department with seizures and a change
in his mental status. He is found to be severely hyponatremic with a serum sodium level of
112 mEq/L (N 135–145).
Initial management while evaluating this patient further should include which one of the
following?
A) Infusion of 0.45% saline
B) Infusion of 3% saline
C) Infusion of normal saline
D) Fluid restriction
E) Hemodialysis

ANSWER: B
Severe hyponatremia with symptoms of confusion and seizures requires raising the serum
sodium level until symptoms improve. Symptomatic hyponatremia occurs when sodium
levels decrease over less than 24 hours. Once symptoms resolve, the cause should be
determined. The rate of sodium correction should be 6–12 mEq/L in the first 24 hours and 18
mEq/L or less in the first 48 hours. An increase of 4–6 mEq/L is usually sufficient to reduce
symptoms of acute hyponatremia. Rapid correction of sodium levels can result in osmotic
demyelination (previously called central pontine myelinolysis).

31
Infusion of normal saline or 0.45% saline will not correct the sodium as rapidly as 3% saline
in acute, severe hyponatremia. Desmopressin, 1–2 :g every 4–6 hours, can be used
concurrently with 3% saline.
Ref: Braun MM, Barstow CH, Pyzocha NJ: Diagnosis and management of sodium disorders: Hyponatremia and hypernatremia. Am
Fam Physician 2015;91(5):299-307.

52. An 82-year-old bedridden male develops a partial thickness skin loss ulcer (stage 2) over
his right heel. He is noted to have a dry, intact eschar without erythema or fluctuance. After
gentle cleansing with saline, which one of the following is the most appropriate management
of this wound?
A) Additional cleansing with a chlorine-based (Dakin’s) solution
B) Additional cleansing with a povidone-iodine solution (Betadine)
C) Application of a moist, nonadhesive bandage
D) Application of a wet-to-dry dressing
E) Sharp debridement of the dry eschar

ANSWER: C
After cleansing with saline or tap water, application of a moist, nonadhesive bandage is the
preferred way to manage a pressure ulcer. A moist wound environment assists in healing and
aids in autolytic debridement. Wet-to-dry dressings may impede healing by causing pain and
unnecessary debridement when a fully dry dressing is removed. Chlorine-based and
povidone-iodine solutions should be avoided because they may impede granulation tissue
formation. While it is important to debride necrotic tissue and slough, a dry, intact eschar over
the heels without any sign of infection should be left in place as a natural biologic cover.
Ref: Raetz JG, Wick KH: Common questions about pressure ulcers. Am Fam Physician 2015;92(10):888-894.

53. Which one of the following has been consistently identified as the most common cause of
medication-related adverse events across health care settings in the United States?
A) Antibiotics
B) Anticoagulants
C) Chemotherapeutic agents
D) Diabetic agents
E) Opioids

ANSWER: B
Anticoagulant medications have been consistently identified as the most common cause of
adverse drug events across health care settings in the United States. The top three categories
responsible for adverse drug events are anticoagulants, opioids, and diabetic agents.
Ref: US Department of Health and Human Services Office of Disease Prevention and Health Promotion: National Action Plan for
Adverse Drug Event Prevention, 2014. 2) Harris Y, Hu DJ, Lee C, et al: Advancing medication safety: Establishing a national action
plan for adverse drug event prevention. Jt Comm J Qual Patient Saf 2015;41(8):351-360.

32
54. A 63-year-old female complains of left shoulder pain and stiffness that have been
increasing in severity for the last year. She works as a hotel housekeeper. On examination you
note decreased passive and active range of motion of her left shoulder compared to the right.
Both active and passive motion produce pain.
Which one of the following is the most likely diagnosis?
A) Glenohumeral osteoarthritis
B) Subdeltoid bursitis
C) Adhesive capsulitis
D) Partial rotator cuff tear
E) Teres minor myositis

ANSWER: C
Shoulder pain is the third most common musculoskeletal reason patients consult primary care
physicians, and rotator cuff disease is the most common cause of shoulder pain. Pain and
restricted active and passive range of motion, accompanied by pain and joint stiffness, are
diagnostic of adhesive capsulitis. Range of motion would not likely be affected with a partial
rotator cuff tear, subdeltoid bursitis, active myositis, or osteoarthritis. Limitations that occur
only with active motion suggest impairment of rotator cuff muscles.
Ref: Hermans J, Luime JL, Meuffels DE, et al: Does this patient with shoulder pain have rotator cuff disease?: The Rational
Clinical Examination Systematic Review. JAMA 2013;310(8):837-847.

55. A 34-year-old female with no significant past medical history is being evaluated for acute
abdominal pain. CT of the abdomen reveals a 3-cm right adrenal mass.
This lesion is most likely to be which one of the following?
A) A benign adenoma
B) An adrenocortical carcinoma
C) A pheochromocytoma
D) A neuroblastoma
E) A metastatic lesion

ANSWER: A
An incidentally discovered adrenal mass is a common finding on abdominal CT and MRI,
occurring in approximately 3%–4% of scans. However, only about 1% of these are malignant,
and malignancies rarely occur in lesions <5 cm in size. Metastatic lesions are rare in patients
without a history of cancer. An incidentally discovered adrenal mass 1–4 cm in size is most
likely to be a benign adenoma, although follow-up is often indicated to ensure stability of the
lesion.
Ref: Willatt JM, Francis IR: Radiologic evaluation of incidentally discovered adrenal masses. Am Fam Physician
2010;81(11):1361-1366. 2) Hitzeman N, Cotton E: Incidentalomas: Initial management. Am Fam Physician 2014;90(11):784-789.

33
56. Which one of the following is a risk factor for uncomplicated cystitis?
A) Obesity
B) Low fluid intake
C) Frequent sexual intercourse
D) Use of hot tubs
E) Wearing synthetic underwear

ANSWER: C
There are multiple risk factors for uncomplicated cases of cystitis. Sexual intercourse is the
most common. Others include spermicide use, previous urinary tract infection, a new sex
partner, and a family history of urinary tract infections in a first degree female relative.
Various studies have shown no relationship between cystitis and water consumption,
urinating after intercourse, patterns of wiping after urination, use of hot tubs, type of
underwear, or obesity.
Ref: Hooton TM: Uncomplicated urinary tract infection. N Engl J Med 2012;366(11):1028-1037.

57. A 65-year-old female sees you for a routine health maintenance visit. She has a newborn
granddaughter that she is planning to watch a few days a week and asks if there are any
vaccines that she can get that will help protect the health of her granddaughter. She has not
received any vaccines other than yearly influenza vaccine in the past 20 years.
According to the Centers for Disease Control and Prevention, which one of the following
would be recommended for her to protect her infant grandchild from illness?
A) Meningococcal vaccine (Menactra)
B) 13-valent pneumococcal conjugate vaccine (Prevnar 13)
C) 23-valent pneumococcal polysaccharide vaccine (Pneumovax 23)
D) Tdap
E) Varicella vaccine

ANSWER: D
Infants less than 12 months of age have higher rates of pertussis infection and have the largest
proportion of pertussis-related deaths. The majority of pertussis cases, admissions, and deaths
occur in children under 2 months of age before they receive their first vaccines. The Advisory
Committee on Immunization Practices recommends that all unvaccinated family members get
a dose of Tdap to help protect infants from pertussis. Both 13-valent and 23-valent
pneumococcal vaccine are indicated for someone over 65 years of age, but there is not a
demonstrated benefit for the health of the infant in this case. Meningococcal and varicella
vaccines are not routinely given to adults over age 65 and have not been shown to help protect
the infants they have contact with.
Ref: Centers for Disease Control and Prevention (CDC): Updated recommendations for use of tetanus toxoid, reduced diphtheria toxoid
and acellular pertussis vaccine (Tdap) in pregnant women and persons who have or anticipate having close contact with an infant aged

34
<12 months—Advisory Committee on Immunization Practices (ACIP), 2011. MMWR Morb Mortal Wkly Rep 2011;60(41):1424-
1426.

58. A 64-year-old female with a history of controlled hypertension reports that her older sister
was recently hospitalized for a stroke. The patient feels well and has never had stroke
symptoms. She requests your advice regarding carotid artery screening for stroke risk
reduction.
Which one of the following would you recommend?
A) Carotid artery ultrasonography only if she has two or more cardiovascular risk factors
B) Carotid artery ultrasonography once between the ages of 65 and 75 if she is a former
smoker
C) Carotid artery ultrasonography once at age 65
D) Carotid artery auscultation annually after age 65
E) No screening for carotid artery disease

ANSWER: E
The U.S. Preventive Services Task Force recommends against screening for asymptomatic
carotid artery stenosis (grade D recommendation), citing with moderate certainty that risks
outweigh benefits. Although carotid artery stenosis is a risk factor for stroke, which is a major
cause of death and disability, screening tests were not found to improve patient outcomes.
Asymptomatic carotid artery stenosis has a low prevalence (0.5%–1%) and carotid
ultrasonography has a high rate of false-positives, exposing patients to harm from unnecessary
treatment. Surgical treatments for carotid artery stenosis have a 30-day risk of stroke and
mortality of 2.2%–3.8%. Carotid auscultation has not been found to be accurate or beneficial,
and screening has not been shown to help optimize medical therapy.
Ref: Final Recommendation Statement: Carotid Artery Stenosis Screening. US Preventive Services Task Force, 2014.

59. A 52-year-old female presents with a 5-day history of nasal congestion, facial pressure,
heavy nasal discharge, and decreased sense of smell. She has not had a fever and says her
symptoms have not started to improve. She is mildly tender over both maxillary sinuses. Even
though you have reassured her that this is most likely a viral illness, she would like antibiotics
because she is going on vacation in 2 days and she wants to be better for her trip. Which one
of the following strategies has been shown to improve the acceptance of symptomatic care
only and reduce the use of antibiotics in this situation?
A) Using medical terminology for the condition, such as acute bronchitis or acute tracheitis
B) Providing a “pocket” prescription with advice to fill it after a defined period without
improvement
C) Ordering sinus radiographs
D) Referral to a specialist

ANSWER: B

35
In spite of good evidence that antibiotics are ineffective for the treatment of acute bronchitis,
and that 90% of cases are caused by viruses, rates of antibiotic prescription for acute
bronchitis remain in the 60%–80% range. Several strategies have been shown to reduce the
rate of antibiotic prescribing for this condition. These include careful use of nonmedical
terminology such as referring to the problem as a “chest cold,” providing “pocket”
prescriptions with advice to fill the prescription only if the patient does not improve in a
defined period of time, and educating patients about the natural history of bronchitis,
informing them that symptoms may persist for 3 weeks. Specialists are not less likely than
primary care physicians to prescribe antibiotics. Sinus films would not provide evidence to
confirm that the infection is viral.
Ref: Zoorob R, Sidani MA, Fremont RD, Kihlberg C: Antibiotic use in acute upper respiratory tract infections. Am Fam Physician
2012;86(9):817-822. 2) Aring AM, Chan MM: Current concepts in adult acute rhinosinusitis. Am Fam Physician 2016;94(2):97-105.

60. A 19-year-old female sees you for evaluation of severe dysmenorrhea. She tells you she
is not sexually active and has never had intercourse. A physical examination is unremarkable
and you determine that a pelvic examination is not necessary.
Which one of the following is the treatment of choice for this patient?
A) Acetaminophen
B) Hydrocodone
C) Medroxyprogesterone acetate (Depo-Provera)
D) An NSAID
E) An oral contraceptive

ANSWER: D
NSAIDs should be used as first-line treatment for primary dysmenorrhea (SOR A). A
Cochrane review that included 73 randomized, controlled trials demonstrated strong evidence
to support NSAIDs as the first-line treatment for primary dysmenorrhea. Since no NSAID has
been proven more effective than others, the choice of NSAID should be based on
effectiveness and tolerability for each patient. The medication should be taken 1–2 days
before the expected onset of the menstrual period and continued on a fixed schedule for 2–3
days. Oral contraceptives may be effective for relieving symptoms of primary dysmenorrhea
but the evidence is limited. Hydrocodone, acetaminophen, and medroxyprogesterone acetate
are not appropriate choices.
Ref: Osayande AS, Mehulic S: Diagnosis and initial management of dysmenorrhea. Am Fam Physician 2014;89(5):341-346.

36
61. A 44-year-old male presents to the emergency department in the evening with facial, lip,
and tongue edema that has been gradually progressing over the last few hours. He was started
on lisinopril (Prinivil, Zestril) earlier today for mild hypertension. He does not have a rash or
pruritus. He reports tightness in his throat and, although he is moving air well at this time,
you do note some mild stridor.
Which one of the following is most likely to prevent the need for intubation?
A) Epinephrine
B) An antihistamine such as diphenhydramine (Benadryl)
C) An angiotensin receptor blocker such as losartan (Cozaar)
D) A bradykinin receptor antagonist such as icatibant (Firazyr)
E) A corticosteroid such as methylprednisolone (Medrol)

ANSWER: D
Less than 1% of patients started on an ACE inhibitor develop angioedema, but some studies
have reported that up to 10% of these patients require intubation. This type of angioedema is
due to increased bradykinin rather than histamine, and antihistamines, anticholinergics,
corticosteroids, and epinephrine would not be effective. Icatibant is a bradykinin receptor type
2 blocker and is recommended in patients with laryngeal angioedema compromising airway
function (level 2 evidence). Angiotensin receptor blockers, although probably not harmful,
would not be helpful.
Ref: Bezalel S, Mahlab-Guri K, Asher I, et al: Angiotensin-converting enzyme inhibitor-induced angioedema. Am J Med
2015;128(2):120-125.

62. A 62-year-old male is admitted to the hospital for urosepsis. His past medical history is
significant only for hypertension. On examination he has a temperature of 36.5°C (97.7°F), a
TSH level of 0.2 :U/mL (N 0.4–5.0), and a free T4 level of 0.4 ng/dL (N 0.6–1.5).
Which one of the following is the most likely explanation for these findings?
A) Pituitary adenoma
B) Graves disease
C) Subacute thyroiditis
D) Subclinical hypothyroidism
E) Euthyroid sick syndrome

ANSWER: E
The euthyroid sick syndrome refers to alterations in thyroid function tests seen frequently in
hospitalized patients, and abnormal thyroid function tests may be seen early in sepsis. These
changes are statistically much more likely to be secondary to the euthyroid sick syndrome
than to unrecognized pituitary or hypothalamic disease (SOR C). Graves disease generally is
a hyperthyroid condition associated with low TSH and elevated free T4. Subclinical
hypothyroidism is diagnosed by high TSH and normal free T4 levels. Subacute thyroiditis
most often is a hyperthyroid condition.

37
Ref: Sakharova OV, Inzucchi SE: Endocrine assessments during critical illness. Crit Care Clin 2007;23(3):467-490. 2) Adler SM,
Wartofsky L: The nonthyroidal illness syndrome. Endocrinol Metab Clin North Am 2007;36(3):657-672. 3) Melmed S, Polonsky KS,
Larsen PR, Kronenberg HM (eds): Williams Textbook of Endocrinology, ed 13. Elsevier Saunders, 2016, pp 354-355.

63. A 56-year-old male with daily heartburn symptoms is found to have Barrett’s esophagus
on endoscopy. Biopsies do not show any evidence of dysplasia.
Which one of the following should be recommended for surveillance of this condition?
A) Endoscopy every year
B) Endoscopy every 3 years
C) Endoscopy every 10 years
D) A PET scan every 2 years
E) No routine surveillance, with endoscopy only if symptoms worsen

ANSWER: B
Endoscopic screening results in the detection of Barrett’s esophagus in 6%–12% of patients
with prolonged gastroesophageal reflux disease symptoms. Barrett’s esophagus, in which
specialized intestinal columnar epithelium replaces the normal esophageal lining in response
to chronic inflammation, is a precursor of esophageal adenocarcinoma. The annual cancer
risk for patients with nondysplastic Barrett’s esophagus is 0.12%–0.4%, with a significant
increase in risk if dysplasia is present. Surveillance with endoscopy every 3 years is
recommended for patients with Barrett’s esophagus without dysplasia. Patients with
adenocarcinoma of the esophagus found during surveillance endoscopy are more likely to
have early-stage, curable cancer than those whose cancer is found during a diagnostic
endoscopy for evaluation of symptoms.
Ref: Rustgi AK, El-Serag HB: Esophageal carcinoma. N Engl J Med 2014;371(26):2499-2509.

64. A healthy 65-year-old female is noted to have a 1.5-cm thyroid nodule during a routine
health maintenance visit. She has no history of radiation exposure or cancer, and no family
history of endocrine cancers.
Which one of the following would be most appropriate at this point?
A) A TSH level
B) Antithyroid antibody
C) A fine-needle aspiration biopsy
D) Thyroid ultrasonography
E) A thyroid radionuclide scan

ANSWER: A
If TSH is suppressed in this patient it indicates that the nodule is producing thyroid hormone
and further evaluation with a radionuclide scan is indicated. If the TSH is normal or elevated
the next step is to determine whether the nodule needs to be biopsied. Thyroid
ultrasonography can determine the size and characteristics of the nodule to help determine
whether to refer the patient for a fine-needle aspiration biopsy. With a large, firm lesion that

38
is highly suspicious for malignancy, it may be appropriate to refer directly for a fine-needle
aspiration biopsy. However, for this patient the lesion did not appear suspicious. If the patient
were hyperthyroid it might be appropriate to check antithyroid antibodies to look for Graves
disease.
Ref: Knox MA: Thyroid nodules. Am Fam Physician 2013;88(3):193-196.

65. A 40-year-old male develops a keloid 6 months after a laceration repair. Which one of the
following is the most appropriate initial treatment to decrease the size of the keloid?
A) Topical retinoids
B) Topical corticosteroids
C) Intralesional corticosteroid injections
D) Surgical excision
E) Mohs surgery

ANSWER: C
Keloids are overgrowths of scar tissue seen more commonly in individuals with dark skin.
The best initial treatment is intralesional corticosteroid injections. If this does not produce
acceptable results, other treatment modalities include surgery, laser therapy, and bleomycin
injection.
Ref: Kundu RV, Patterson S: Dermatologic conditions in skin of color: Part II. Disorders occurring predominately in skin of color. Am
Fam Physician 2013;87(12):859-865.

66. A 45-year-old male reports being held up at gunpoint while on vacation 3 months ago.
Since that time he has had intrusive memories of the event, as well as nightmares. Further
questioning reveals that he has been having dissociative reaction flashbacks and meets the
criteria for posttraumatic stress disorder.
Which one of the following is the most appropriate pharmacotherapy for this patient?
A) Clonazepam (Klonopin)
B) Clonidine (Catapres)
C) Mirtazapine (Remeron)
D) Sertraline (Zoloft)
E) Risperidone (Risperdal)

ANSWER: D
The dissociative reactions (flashbacks) in this patient are consistent with the diagnosis of
posttraumatic stress disorder (PTSD). The first-line medications for this disorder are SSRIs
and SNRIs. Paroxetine and sertraline have FDA approval for PTSD. Other antidepressants
such as mirtazapine would be second-line therapy. The effectiveness of central "2-agonists
such as clonidine are unknown, and even though benzodiazepines might help with
hyperarousal symptoms, they can worsen other symptoms. Atypical antipsychotics such as
risperidone are not recommended.

39
Ref: Warner CH, Warner CM, Appenzeller GN, Hoge CW: Identifying and managing posttraumatic stress disorder. Am Fam Physician
2013;88(12):827-834.

67. Which one of the following is more characteristic of Crohn’s disease, as opposed to
ulcerative colitis?
A) Bloody diarrhea
B) Perianal manifestations
C) Involvement of the rectum
D) Proximal progression
E) Associated inflammatory arthropathies

ANSWER: B
Crohn’s disease typically spares the rectum. Ulcerative colitis usually has rectal involvement,
progresses proximally, and rarely has perianal or systemic manifestations (SOR A). Both
conditions may cause bloody diarrhea as well as inflammatory arthropathies, eye
inflammation such as uveitis, and skin findings such as erythema nodosum.
Ref: Cheifetz AS: Management of active Crohn disease. JAMA 2013;309(20):2150-2158.

68. A 75-year-old male has a past medical history significant for atrial fibrillation, ischemic
cardiomyopathy, diabetes mellitus, and hyperlipidemia. He is admitted to the hospital with
bronchiolitis obliterans organizing pneumonia (cryptogenic organizing pneumonia).
Which one of the medications he takes is the most likely cause of this problem?
A) Amiodarone (Cordarone)
B) Carvedilol (Coreg)
C) Digoxin (Lanoxin)
D) Lisinopril (Prinivil, Zestril)
E) Pioglitazone (Actos)

ANSWER: A
Many drugs can cause lung disease. Amiodarone has been known to cause both bronchiolitis
obliterans organizing pneumonia (BOOP) and interstitial pneumonitis. BOOP, also known as
cryptogenic organizing pneumonia, is characterized by interstitial inflammation
superimposed on the dominant background of alveolar and ductal fibrosis. This is a very
distinctive pattern of lung response to exposure to several drugs, including amiodarone,
bleomycin, gold, penicillamine, sulfasalazine, radiation, interferons, methotrexate,
mitomycin C, cyclophosphamide, and cocaine.
Interstitial pneumonitis is the most common manifestation of drug-induced lung disease.
Drugs that can cause this include amiodarone, azathioprine, bleomycin, chlorambucil,
methotrexate, phenytoin, statins, and sulfasalazine.
Ref: Broaddus VC, Mason RJ, Ernst JD, et al (eds): Murray & Nadel’s Textbook of Respiratory Medicine, ed 6. Elsevier Saunders,
2016, pp 71, 1275-1294.

40
69. A 4-year-old female is brought to your office with a limp that was first noted 2 days ago.
There has been no known injury or recent illness. Her vital signs are normal. Observation of
her gait reveals shortening of the stance phase to take weight off her left leg. The physical
examination reveals an otherwise healthy female with no focal tenderness or pathology of her
lower legs or abdomen.
Which one of the following would be most appropriate at this point?
A) A CBC, erythrocyte sedimentation rate, and C-reactive protein level
B) Plain radiographs of both lower extremities
C) Ultrasonography of both hips
D) MRI of the left lower extremity, including the hip
E) A bone scan

ANSWER: B
This patient demonstrates an antalgic gait without a clear etiology. The evaluation of a
limping child begins with a thorough history, observation of the child’s gait, and a physical
examination. If the history is not contributory and the physical examination demonstrates no
focal source of pain, radiographs of both lower extremities should be the first step in the
workup (SOR C). If there is a focal source of pain, radiographs of the affected joint would be
appropriate (SOR C). If the patient demonstrates systemic signs of illness such as fever or
anorexia, a laboratory evaluation (CBC, erythrocyte sedimentation rate, and C-reactive
protein) should be performed in addition to radiographs. Ultrasonography is useful
subsequently if there is concern about joint effusion. If no source of the problem is found,
additional testing should include a bone scan (SOR C) and MRI.
Ref: Naranje S, Kelly DM, Sawyer JR: A systematic approach to the evaluation of a limping child. Am Fam Physician
2015;92(10):908-916.

70. A 26-year-old male who is in a monogamous sexual relationship with another male sees
you for a routine health maintenance visit. He is employed in a grocery store, is in good health,
and reports having the usual immunizations as a child.
Based on expert consensus, which one of the following would be most appropriate at this
visit?
A) Pre-exposure HIV prophylaxis
B) Screening for STIs now and every 3 months
C) Meningococcal vaccine
D) Hepatitis B surface antigen (HBsAG) testing

ANSWER: D
Men who have sex with men but are in a monogamous relationship need not be offered
preexposure or postexposure HIV prophylaxis, unlike men with multiple or anonymous
sexual partners. Meningococcal vaccine is not indicated unless there are other risk factors.
Since this patient is in a monogamous relationship, screening for sexually transmitted

41
infections once a year is considered adequate. Screening for hepatitis C at this visit is
recommended, as well as testing for hepatitis B infection.
Ref: Workowski KA, Bolan GA: Sexually transmitted diseases treatment guidelines, 2015. MMWR Recomm Rep 2015;64 (RR-3):13-
16. 2) Final Recommendation Statement: Hepatitis B Virus Infection: Screening, 2014. US Preventive Services Task Force, October
2014. 3) Knight DA, Jarrett D: Preventive health care for men who have sex with men. Am Fam Physician 2015;91(12):844-851.

71. A 65-year-old white female requests bone density screening because her mother had
premenopausal osteoporosis. The patient has no previous history of fracture, and her past
medical and family histories are otherwise unremarkable. She has not had a hysterectomy and
is not taking any medications or supplements. Her dietary calcium intake is low. DXA shows
T-scores of –1.8 at the hip and –2.1 at the spine. Her estimated 10-year risk of hip fracture is
1.3%. Serum TSH, calcium, and 25-hydroxyvitamin D levels are normal.
Which one of the following would be most appropriate at this time?
A) An increase in dietary calcium to 1200 mg daily and starting vitamin D3 (cholecalciferol)
supplementation, 800 IU daily
B) Alendronate (Fosamax), 70 mg orally once weekly for 5 years
C) Conjugated estrogen/medroxyprogesterone (Prempro), 0.625 mg/5 mg orally, once daily
D) Calcitonin-salmon (Miacalcin) in alternating nostrils daily
E) A repeat DXA scan in 2 years and initiation of osteoporosis treatment if her FRAX score
shows her 10-year risk of major osteoporotic fracture is >10%

ANSWER: A
Based on the patient’s age it is appropriate for her to be screened. Her bone density is
consistent with osteopenia and she has no identified secondary causes of osteopenia. Because
her estimated 10-year risk of hip fracture is <3% and she has not had any fractures to date,
prescription medications such as bisphosphonates or calcitonin are not indicated. Estrogen
does increase bone density but it is not indicated for osteoporosis prevention or treatment, due
to associated cardiovascular risks. Professional organizations vary on the daily calcium and
vitamin D intake recommended for postmenopausal women, but 1200 mg of dietary calcium
and supplementation with 800 IU of vitamin D3 are reasonable recommendations. The
National Osteoporosis Foundation suggests treatment of osteoporosis if the 10-year risk of
major osteoporotic fracture is >20%.
Ref: Final Recommendation Statement: Osteoporosis Screening. US Preventive Services Task Force, 2011. 2) Bauer DC: Calcium
supplements and fracture prevention. N Engl J Med 2013;369(16):1537-1543. 3) Jeremiah MP, Unwin BK, Greenawald MH, Casiano
VE: Diagnosis and management of osteoporosis. Am Fam Physician 2015;92(4):261-268. 4) Black DM, Rosen CJ: Postmenopausal
osteoporosis. N Engl J Med 2016;374(3):254-262.

42
72. A 32-year-old male presents to your office with a right knee injury that occurred while he
was playing tennis. His description of the injury indicates that he twisted the knee while his
foot was planted. Given the mechanism of injury, you suspect a meniscal tear.
Which one of the following is the most useful single maneuver to evaluate for a meniscal
tear?
A) The anterior drawer test
B) The pivot-shift test
C) The valgus stress test
D) Lachman’s maneuver
E) Thessaly’s maneuver

ANSWER: E
Meniscal tears are a common source of knee pain in acute knee injuries, occurring in
approximately 10% of cases presenting with acute pain after an injury. Although the
McMurray test (passive extension of the knee while applying valgus and varus stresses to the
knee) has historically been used to detect meniscus injuries, the Thessaly test has superior
positive and negative predictive value for meniscus injuries compared with the McMurray
test. The Thessaly test is performed by having the patient stand on the affected leg while it is
flexed 20° and internally and externally rotate the knee three times while holding the
examiner’s hands for support. Locking, catching, or joint-line pain constitutes a positive test.
The pivot-shift, Lachman, and anterior drawer tests are used to detect injuries to the anterior
cruciate ligament, not meniscal injuries. The valgus stress test detects injuries to the medial
collateral ligament.
Ref: Grover M: Evaluating acutely injured patients for internal derangement of the knee. Am Fam Physician 2012;85(3):247-252.

73. An 18-year-old female is seen for a preparticipation physical examination for soccer. She
exercises 2–3 hours/day but states that she has low energy levels. Her past medical history
includes a stress fracture of the third metatarsal that healed properly. She has intermittent
amenorrhea. A physical examination is normal except for a BMI of 18.0 kg/m2.
Which one of the following would be most appropriate for this patient?
A) No further evaluation
B) An exercise stress test
C) Echocardiography
D) Radiographs of both feet
E) A bone mineral density test

ANSWER: E
This patient has low energy, menstrual irregularities, and a history of stress fractures, which
is consistent with the female athlete triad. She is at risk for altered bone density and needs a
bone mineral density test. This condition is also associated with disordered eating and low

43
body mass. Echocardiography may be indicated if there is a personal or family history of
cardiac problems. Radiographs of the feet are not sensitive for osteoporosis evaluation. A
stress test is not indicated.
Ref: Payne JM, Kirchner JT: Should you suspect the female athlete triad? J Fam Pract 2014;63(4):187-192. 2) Weiss Kelly AK, Hecht
S; Council on Sports Medicine and Fitness: The female athlete triad. Pediatrics 2016;138(2):pii:e20160922

74. A 27-year-old male presents to the emergency department with a 1-week history of
fatigue, dyspnea, chest pain, and fever. He is a heroin addict but has no other significant
previous history. On examination his lungs are clear but he has a systolic heart murmur. A
chest radiograph shows multiple lesions consistent with emboli.
Pending blood culture results, initial antibiotic coverage should include which one of the
following?
A) Clindamycin (Cleocin)
B) Levofloxacin (Levaquin)
C) Rifampin (Rifadin)
D) Trimethoprim/sulfamethoxazole (Bactrim)
E) Vancomycin (Vancocin)

ANSWER: E
Over 85% of cases of infectious endocarditis are caused by gram-positive cocci. In patients
suspected of having acute infectious endocarditis, empiric antibiotic treatment should be
started immediately after obtaining initial blood cultures and should include coverage against
gram-positive cocci with vancomycin. For patients with prosthetic heart valves, initial
coverage should include vancomycin plus rifampin. Clindamycin, levofloxacin, and
trimethoprim/sulfamethoxazole have no role in the initial treatment of infectious endocarditis.
Ref: Pierce D, Calkins BC, Thornton K: Infectious endocarditis: Diagnosis and treatment. Am Fam Physician 2012;85(10):981-986.

75. You see a 12-year-old male in October for a preparticipation physical examination for
basketball. He has no complaints. You are able to document that he was up-to-date with
immunizations at 6 years of age, but has not been vaccinated since. His vital signs and
examination are normal.
In addition to Tdap, which one of the following sets of vaccines should be given at this visit?
A) HPV and varicella
B) HPV and influenza
C) HPV, meningococcal, and influenza
D) Meningococcal, varicella, and influenza

ANSWER: C
Vaccination rates for younger children (4–6 years of age) generally surpass 90%, but rates are
much lower in older children. Only 34% of boys receive HPV vaccine at age 11 or 12, and

44
only 40% receive Tdap. Family physicians need to be familiar with routine immunizations in
this age group so they can be recommended at the appropriate time.
This patient was up to date with immunizations at age 6 years, so the only catch-up vaccines
needed are those in the routine 11- to 12-year-old set, which includes influenza,
meningococcal, HPV, and Tdap vaccines. DTap and varicella vaccine are not routinely
recommended after the age of 6 years.
Ref: Ackerman LK, Serrano JL: Update on routine childhood and adolescent immunizations. Am Fam Physician 2015;92(6):460-468.
2) Centers for Disease Control and Prevention. Birth–18 Years & “Catch-up” Immunization Schedules. 2016.

76. A 52-year-old healthy female nonsmoker who has a family history of coronary artery
disease presents with episodes of left-sided chest pain that last 10–15 minutes but are
unrelated to activity. A resting EKG is normal. She is on medication for hypertension and is
in good physical condition.
Which one of the following is the best study to order at this time?
A) Exercise treadmill testing
B) Stress echocardiography
C) Coronary CT angiography
D) Stress myocardial perfusion imaging
E) Dobutamine echocardiography

ANSWER: A
Early studies of ischemic heart disease included mostly male subjects. More recently there
has been a determined effort to understand the special considerations associated with this
problem and its management in women. In 2014 the American Heart Association published
a consensus statement summarizing the research on how to best evaluate women with
suspected ischemic heart disease. Its recommendations focused on the level of pretest risk for
ischemic heart disease (low, intermediate, and high), a normal or abnormal resting EKG, the
ability of the subject to exercise, and potential risks of radiation exposure. The patient in this
scenario would be considered low to intermediate risk for ischemia due to her age and risk
factors, along with a history of atypical chest pain. Since her resting EKG is normal and she
is physically fit, she should undergo an exercise treadmill test without imaging.
Ref: Mieres JH, Gulati M, Bairey Merz N; American Heart Association Cardiac Imaging Committee of the Council on Clinical
Cardiology; Cardiovascular Imaging and Intervention Committee of the Council on Cardiovascular Radiology and Intervention: Role
of noninvasive testing in the clinical evaluation of women with suspected ischemic heart disease: A consensus statement from the
American Heart Association. Circulation 2014;130(4):350-379.

45
77. A 4-month-old female is brought to your office by her parents because she spits up after
most feedings. She is their first child and was born at term with no prenatal or postnatal
complications. She is formula fed. A review of her growth chart shows normal growth. Her
physical examination and vital signs are normal as well.
Which one of the following would be most appropriate at this point?
A) Reassurance only
B) Advising the parents to place the infant in a prone position for sleeping
C) An upper GI series
D) Omeprazole (Prilosec)
E) Ranitidine (Zantac)

ANSWER: A
Gastroesophageal reflux is very common in infants. Reflux in infants peaks at 4 months of
age, and two-thirds of infants regurgitate at least once a day at this age. The incidence declines
dramatically in the next few months, and by 1 year of age less than 5% of infants regurgitate
on a daily basis. If the infant is healthy and growing normally, reassurance is appropriate
(SOR C). Conservative measures are recommended if the reflux causes distress (SOR C).
Such measures include placing the infant on her side or prone while awake to reduce reflux.
However, infants should not be placed prone while sleeping, to prevent SIDS. Appropriate
conservative measures also include smaller, more frequent feedings, the addition of
thickening agents such as rice cereal to formula, and changing to amino acid formulas if
infants are allergic to cow’s milk protein.
If conservative measures and time do not alleviate the symptoms, a 4-week trial of H2-
blockers or proton pump inhibitors can be tried. The American Academy of Pediatrics and
the Society of Hospital Medicine (Pediatric) recommend not routinely treating reflux with
medication. Imaging, such as esophagogastroduodenoscopy, is not routinely used in the initial
workup of reflux in infants. It is reserved for recalcitrant cases, atypical symptoms, or
complications (SOR C).
Ref: Baird DC, Harker DJ, Karmes AS: Diagnosis and treatment of gastroesophageal reflux in infants and children. Am Fam Physician
2015;92(8):705-714.

46
78. A 50-year-old male presents to your office with a 4-day history of the rash.

It spread from the lower trunk to the lower extremities, including the genital area. He also
complains of pain and swelling of the testes. He considers himself to be in good health and
takes no medications. He is afebrile with a normal examination except for the pink-purple
maculopapular eruption and bilateral swollen testes. A CBC, urinalysis, and comprehensive
metabolic panel are normal.
Which one of the following is the most likely diagnosis?
A) Henoch-Schönlein purpura
B) Kawasaki disease
C) Polyarteritis nodosa
D) Rocky Mountain spotted fever
E) Thrombocytopenic purpura

ANSWER: A
Henoch-Schönlein purpura (HSP) presents most often in children but not infrequently in
adults. The purpuric rash is classically seen on the waist and extends to the legs, sparing the
proximal trunk and arms. Orchitis with testicular swelling occurs in 35% of men with HSP
and is often complicated by abdominal pain, arthritis, and renal insufficiency.
Kawasaki disease is a pediatric disease presenting with fever, conjunctivitis, and lesions of
the lips. Polyarteritis nodosa often presents with fever and multisystem symptoms and
findings. There are most often abnormalities on the CBC and chemistry profile. Rocky

47
Mountain spotted fever is associated with a petechial rash that involves the proximal trunk
and extremities, including the palms and soles. Thrombocytopenic purpura, by definition, is
associated with a low platelet count.
Ref: Reamy BV, Williams PM, Lindsay TJ: Henoch-Schönlein purpura. Am Fam Physician 2009;80(7):697-704. 2) Kasper DL, Fauci
AS, Hauser SL, et al (eds): Harrison’s Principles of Internal Medicine, ed 19. McGraw-Hill, 2015, pp 2190-2192.

79. A 90-year-old male with a history of metastatic lung cancer is admitted to hospice. You
agree to follow the patient. The following week the hospice nurse calls you because the patient
is complaining of significant dyspnea. His oxygen saturation is 91% on room air. A physical
examination reveals diminished but otherwise clear breath sounds.
Which one of the following is the treatment of choice for this patient's dyspnea?
A) Oxygen
B) Albuterol in normal saline by nebulizer
C) Prednisone orally
D) Morphine sulfate sublingually
E) Lorazepam (Ativan) orally

ANSWER: D
Opioids, given either orally or intravenously, are the treatment of choice for dyspnea and have
been studied thoroughly in patients with COPD and patients with cancer. They have been
found to be effective in alleviating dyspnea and, when used carefully, do not have serious side
effects such as respiratory depression. When the patient is experiencing anxiety, which
regularly occurs in association with breathlessness, benzodiazepines can be added, although
there is no evidence that they improve the dyspnea. Patients are regularly given supplemental
oxygen for dyspnea, but systematic reviews have found no benefit for patients with cancer or
heart failure who do not have hypoxemia. However, oxygen may provide some relief for
patients with COPD who do not have hypoxemia. Prednisone and albuterol are not indicated
for this patient.
Ref: Blinderman CD, Billings JA: Comfort care for patients dying in the hospital. N Engl J Med 2015;373(26):2549-2561.

48
80. A 58-year-old minister comes to your office accompanied by his wife for a follow-up
evaluation of personality changes. His wife says he has been making inappropriate comments
to females in the church and has been more withdrawn at social gatherings. He has also not
been preparing his sermons or balancing their checking account. These behaviors are
uncharacteristic for him and his symptoms have been progressively worsening over the past
6–12 months. He is quiet during this discussion. He has been on an SSRI for 3 months with
minimal to no improvement. The history is otherwise normal, as is a physical examination,
including a focused neurologic examination. Short-term memory is intact.
This presentation is most consistent with which one of the following diagnoses?
A) Alzheimer’s disease
B) Frontotemporal dementia
C) Lewy body dementia
D) Mixed dementia
E) Vascular dementia

ANSWER: B
Core features of the behavioral variant frontotemporal dementia (FTD) include an insidious
onset and gradual progression, an early decline in social and interpersonal conduct, early
impairment in regulation of personal conduct, early emotional blunting, and early loss of
insight. Common initial symptoms include apathy, lack of initiation, diminished interest, and
inactivity. Common features also include disinhibition and impulsivity. Examples include
socially inappropriate remarks, including sexual comments.
These types of symptoms are less common in early phases of other types of dementia. FTD
is frequently misdiagnosed as a primary psychiatric disorder such as depression. Alzheimer’s
disease presents with memory and visuospatial loss. Lewy body dementia tends to cause
memory loss, fluctuating cognition, visual hallucinations, and spontaneous parkinsonian
motor features. Vascular dementia patients usually have a history of cerebrovascular events.
Mixed dementias generally are a combination of Alzheimer’s and other types of dementias.
Ref: Cardarelli R, Kertesz A, Knebl JA: Frontotemporal dementia: A review for primary care physicians. Am Fam Physician
2010;82(11):1372-1377. 2) Warren JD, Rohrer JD, Rossor MN: Frontotemporal dementia. BMJ 2013;347:f4827

81. A 54-year-old male returns to your office for follow-up of chronic right knee pain that has
worsened over the past month. On examination, range of motion of the knee is normal with
no catching in the joint. There is a subtle effusion of the right knee and tenderness along the
medial joint line. A radiograph shows medial joint space narrowing and subchondral sclerosis
of the medial tibiofemoral joint.
Which one of the following should you recommend at this point?
A) Stretching and strengthening with physical therapy
B) Taping and therapeutic ultrasound with physical therapy
C) Lateral wedge insoles
D) Oral glucosamine and chondroitin

49
ANSWER: A
The mainstay of treatment for osteoarthritis of the knee is active rehabilitation and exercise
(SOR A). Active rehabilitation, such as stretching and strengthening, is more effective than
passive rehabilitation, such as taping, heat, electrostimulation, or therapeutic ultrasound (SOR
B). Lateral wedge insoles and glucosamine and chondroitin supplements are unlikely to
significantly improve pain in patients with knee arthritis (SOR B).
Ref: Jones BQ, Covey CJ, Sineath MH Jr: Nonsurgical management of knee pain in adults. Am Fam Physician 2015;92(10):875-883.

82. A 5-year-old male has a 10-day history of respiratory symptoms, including nasal
congestion. He seemed to improve around day 5 but acutely worsened on day 7 with a new
onset of fever, daytime cough, and persistent nasal drainage. On examination his oral
temperature is 38.1°C (100.6°F), heart rate 100 beats/min, respiratory rate 24/min, and blood
pressure 90/68 mm Hg. He has no sinus tenderness or cervical lymphadenopathy, and normal
tympanic membranes bilaterally. You note nasal mucosal swelling and erythema, and mild
pharyngeal erythema. Cardiac and lung examinations are normal.
Which one of the following would you recommend?
A) Saline nasal rinses, decongestants, fluids, and rest
B) A laboratory workup including a CBC, an erythrocyte sedimentation rate, and a C-reactive
protein level
C) Sinus CT
D) Amoxicillin
E) Azithromycin (Zithromax)

ANSWER: D
This child meets the criteria for acute bacterial sinusitis (ABS) and should be treated with
antibiotics. He exemplifies the concept of “double sickening,” in which a child initially has
typical symptoms of a viral upper respiratory infection and improves initially only to worsen
later, with daytime cough, persistent nasal discharge, and/or new fever. Other criteria for ABS
include persistence of URI symptoms without improvement after 7–10 days and “severe
onset” ABS with a high fever and purulent nasal discharge for at least 3 days. Evidence shows
that treatment with antibiotics in these situations improves outcomes (SOR B). The first-line
antibiotic is amoxicillin with or without clavulanate. The length of treatment can range from
10 to 28 days. Depending on risks, patients may be treated with either high-dose amoxicillin
or amoxicillin/clavulanate, with an amoxicillin dosage of 90 mg/kg/day. Many of the bacteria
causing ABS have been shown to be resistant to azithromycin and
trimethoprim/sulfamethoxazole and these antibiotics should be avoided. For patients allergic
to penicillins, cephalosporins should be used.
The diagnosis of ABS is based on the history (SOR C). The physical examination is not
particularly helpful and findings such as sinus tenderness, mucosal swelling, and
transillumination of the sinuses do not help differentiate ABS from a viral URI. Laboratory

50
studies are not indicated in the diagnosis. Imaging studies are likewise not indicated for the
initial diagnosis, as they are often abnormal in both viral URIs and ABS. If complications
such as orbital cellulitis or neurologic compromise are a concern, then CT may be indicated.
There is no good evidence to support adjuvant care for ABS. Saline nasal irrigation,
decongestants, or intranasal corticosteroids may be helpful but cannot replace antibiotic
therapy in children who meet the criteria for ABS.
Ref: Wald ER, Applegate KE, Bordley C, et al: Clinical practice guideline for the diagnosis and management of acute bacterial sinusitis
in children aged 1 to 18 years. Pediatrics 2013;132(1):e262-e280. 2) Marom T, Alvarez-Fernandez PE, Jennings K, et al: Acute
bacterial sinusitis complicating viral upper respiratory tract infection in young children. Pediatr Infect Dis J 2014;33(8):803-808. 3)
Hauk L: AAP releases guideline on diagnosis and management of acute bacterial sinusitis in children one to 18 years of age. Am Fam
Physician 2014;89(8):676-681.

83. According to the JNC 8 panel guidelines, which one of the following would be first-line
drug therapy for hypertension in an African-American male who has no other medical
problems?
A) An ACE inhibitor
B) An aldosterone antagonist
C) An "-blocker
D) Hydralazine
E) A thiazide-type diuretic

ANSWER: E
The 2014 evidence-based guideline from the JNC 8 panel recommends that in the general
African-American population, including those with diabetes mellitus, initial antihypertensive
treatment should include a thiazide-type diuretic or calcium channel blocker (for general
African-American population: SOR B; for African-American patients with diabetes: SOR C).
Ref: James PA, Oparil S, Carter BL, et al: 2014 Evidence-based guideline for the management of high blood pressure in adults: Report
from the panel members appointed to the Eighth Joint National Committee (JNC 8). JAMA 2014;311(5):507-520.

84. A 13-year-old female has been having difficulty breathing while playing soccer for the
last few weeks. She has no personal or family history of asthma and has never needed medical
treatments for breathing problems before. Her symptoms include coughing and wheezing only
when running intensely for long periods of time. These symptoms persist for up to an hour
after she stops exercising. She has no symptoms at other times during the day or at night. A
pulmonary function test shows better than average lung function with no change after
albuterol (Proventil, Ventolin) inhalation. Which one of the following treatments would be a
good first choice to help with this patient’s symptoms?
A) A 5-day burst of oral prednisone
B) Daily use of a cromolyn inhaler
C) Daily inhaled corticosteroids during soccer season
D) An albuterol inhaler 10–15 minutes prior to exercise
E) Inhaled salmeterol (Serevent Diskus) every morning during soccer season

51
ANSWER: D
Asthmatic symptoms during exercise are common. These can occur as exacerbations of
underlying airway inflammation or as bronchospasm in otherwise normal airways. This
patient seems to have bronchospasm that would be best managed by albuterol prior to
exercise. If she finds that she needs her inhaler frequently, the addition of an anti-
inflammatory agent such as inhaled corticosteroids or oral montelukast would be reasonable.
Ref: Krafczyk MA, Asplund CA: Exercise-induced bronchoconstriction: Diagnosis and management. Am Fam Physician
2011;84(4):427-434. 2) Spiro SG, Silvestri GA, Agusti A: Clinical Respiratory Medicine, ed 4. Elsevier Saunders, 2012, pp 501-520.
3) Boulet LP, O’Byrne PM: Asthma and exercise-induced bronchoconstriction in athletes. N Engl J Med 2015;372(7):641-648.

85. A 23-year-old male presents with moderate pain and stiffness of the low back and buttock
area that began 3–4 months ago. There was no known precipitating event and the symptoms
are worsening.
Which one of the following characteristics of his pain and stiffness would make ankylosing
spondylitis more likely?
A) An acute onset over days
B) Improvement with exercise and activity
C) Stiffness in the late afternoon and evening
D) Symptom relief at night

ANSWER: B
Ankylosing spondylitis is an inflammatory condition that affects the axial skeleton primarily,
but other joints may be involved. The pain begins insidiously in the lower back and gluteal
region. The symptoms improve with exercise and activity but they worsen at night. The
stiffness is most prominent in the morning and may last up to a few hours.
Ref: Kasper DL, Fauci AS, Hauser SL, et al (eds): Harrison’s Principles of Internal Medicine, ed 19. McGraw-Hill, 2015, pp 2169-
2173.

86. In an adult, the radiation exposure from a typical abdominal CT examination is


approximately the same as how many posteroanterior chest radiographs?
A) 4
B) 40
C) 400
D) 4000

ANSWER: C
Radiation exposure in humans is quantified by the sievert (Sv), which equals 1 joule of
radiation energy/kg of human tissue. Most clinical diagnostic test exposures measure in the
millisievert (mSv) range. Natural background exposure in the United States averages 3
mSv/year. Although exact exposures will vary according to patient size, type of equipment

52
used, and operator expertise, typical radiation doses for common radiographic studies include
the following:
Posteroanterior chest ——— 0.02 mSv
Skull ————————— 0.1 mSv
Lumbar spine ——————1.5 mSv
CT head ————————2 mSv
CT abdomen ——————8 mSv
This list shows that CT of the abdomen provides a radiation dose 400 times that of the typical
posteroanterior chest radiograph. A small increased relative risk of cancer mortality was
demonstrated in Japanese survivors of atomic bombs receiving doses in the range of 5–20
mSv, suggesting that consideration of the risk/benefit ratio of some radiographic studies is
warranted.
Ref: US Food and Drug Administration: What are the radiation risks from CT? 2016.

87. A 41-year-old male presents to your office for follow-up of recurrent nephrolithiasis. He
currently follows a normal diet and does not drink soft drinks very often.
Which one of the following would help prevent recurrent kidney stones?
A) A loop diuretic
B) A thiazide diuretic
C) A high-protein diet
D) Two 12-oz cola-flavored soft drinks per day

ANSWER: B
Patients with recurrent nephrolithiasis should first try to increase fluid intake to achieve a
daily urine output ≥2 L. Increasing fluid intake decreases the recurrence of stones by at least
50%. Reducing soft drink intake may also help but this seems to be limited to those who drink
colas, which are acidified by phosphoric acid. Soft drinks acidified with citric acid, such as
fruit-flavored drinks, do not appear to have the same effect.
There is little evidence that dietary changes help significantly. If a patient’s fluid increase is
not sufficient, treatment with a thiazide diuretic, citrate, or allopurinol is recommended (SOR
C).
Ref: Qaseem A, Dallas P, Forciea MA, et al: Dietary and pharmacologic management to prevent recurrent nephrolithiasis in adults: A
clinical practice guideline from the American College of Physicians. Ann Intern Med 2014;161(9):659-667. 2) Hauk L: Prevention of
recurrent nephrolithiasis: Dietary and pharmacologic options recommended by the ACP. Am Fam Physician 2015;92(4):311.

53
88. You see a 45-year-old white male smoker for follow-up of his hypertension. His past
medical history is otherwise unremarkable. At this visit he has a blood pressure of 136/84 mm
Hg. A lipid panel reveals the following:
Triglycerides ………………………… 226 mg/dL
Total cholesterol ……………………… 228 mg/dL
LDL-cholesterol …………………………159 mg/dL
HDL-cholesterol ………………………… 31 mg/dL
The ASCVD Risk Estimator yields a 10-year atherosclerotic cardiovascular disease risk of
15.4%. Based on the 2013 American College of Cardiology/American Heart Association
guideline for reducing atherosclerotic cardiovascular disease risk in adults, which one of the
following is the most appropriate medication regimen for this patient?
A) Moderate- or high-intensity statin therapy
B) Statin therapy titrated to reduce LDL-cholesterol by 20%–30%
C) Combined statin and fibrate therapy to reduce both LDL-cholesterol and triglycerides
D) Combined statin and niacin therapy to reduce LDL-cholesterol and raise HDL-cholesterol
E) Combined statin and PCSK9 inhibitor therapy to reduce the LDL-cholesterol level to <70
mg/dL

ANSWER: A
The 2013 American College of Cardiology/American Heart Association guideline for
reducing atherosclerotic cardiovascular disease (ASCVD) risk in adults recommended several
significant changes in the management of hyperlipidemia. This guideline recommends
looking at overall risk as estimated by a tool, the ASCVD Risk Estimator, which considers
not only lipid parameters but also age, sex, ethnicity, systolic blood pressure, and the presence
or absence of diabetes mellitus, treated hypertension, and smoking. Four major risk groups
were identified for treatment, one of which was adults ≥40 years of age with an estimated 10-
year risk of ASCVD ≥7.5%.
Therapy is graded by intensity (low-, moderate-, or high-intensity statin therapy), and
therapeutic targets for LDL-cholesterol were abandoned since there is no demonstrable
benefit from achieving a certain level of LDL-cholesterol in treated patients. This patient has
an estimated 10-year ASCVD risk well above 7.5% and is a candidate for moderate- or high-
intensity statin therapy under the guideline. Niacin and fibrate therapy do not have a
demonstrable impact on cardiovascular outcomes and are not recommended in the guideline.
PCSK9 inhibitors are also not yet recommended in any guideline.
Ref: ACC/AHA release updated guideline on the treatment of blood cholesterol to reduce ASCVD risk. Am Fam Physician
2014;90(4):260-265. 2) Stone NJ, Robinson JG, Lichtenstein AH, et al: 2013 ACC/AHA guideline on the treatment of blood
cholesterol to reduce atherosclerotic cardiovascular risk in adults: A report of the American College of Cardiology/American Heart
Association Task Force on Practice Guidelines. Circulation 2014;129(25 Suppl 2):S1-S45.

54
89. Acute altitude sickness consists of headache, nausea, dizziness, and sleep disturbance.
Risk factors include which one of the following?
A) Below-average physical fitness
B) Fast ascent
C) Age …65 years
D) Male sex

ANSWER: B
Major risk factors for acute mountain sickness include a history of previous mountain
sickness, fast ascent, and lack of acclimatization. Slow ascent with frequent stops at various
levels is the safest way to prevent altitude sickness. Females are at increased risk and good
physical fitness is not protective. Persons <46 years of age are at increased risk.
Ref: Bärtsch P, Swenson ER: Acute high-altitude illnesses. N Engl J Med 2013;368(24):2294-2302.

90. A 45-year-old Hispanic female presents to your office for follow-up of a blood pressure
measurement of 155/95 mm Hg at a health fair screening. She has no significant past medical
history. She reports that she exercises daily for 30 minutes, follows a low-salt diet, and rarely
drinks alcohol. Her blood pressure in the office today is 154/95 mm Hg.
Which one of the following medications would be most appropriate for this patient?
A) Chlorthalidone
B) Clonidine (Catapres)
C) Doxazosin (Cardura)
D) Metoprolol succinate (Toprol-XL)
E) Spironolactone (Aldactone)

ANSWER: A
Thiazide diuretics such as chlorthalidone are considered a first-line therapy for hypertension.
β-Blockers, aldosterone antagonists, and other antihypertensive medications may be used as
add-on therapy to reach blood pressure goals.
Ref: James PA, Oparil S, Carter BL, et al: 2014 evidence-based guidelines for the management of high blood pressure in adults: Report
from the panel members appointed to the Eighth Joint National Committee (JNC 8). JAMA 2014;311(5):507-520.

55
91. A 70-year-old male presents to the emergency department with confusion and a fever of
38.9°C (102.0°F). He has a history of decreased mental status, feeling weak and confused,
and low urinary output. On examination he has a blood pressure of 80/50 mm Hg, a pulse rate
of 100 beats/min, a respiratory rate of 24/min, and a temperature of 38.9°C (102.0°F).
He has mottling of the skin, and his capillary refill time is 5 seconds. A CBC reveals a WBC
count of 24,000/mm3 (N 4500–11,000) with 90% neutrophils. His urinalysis is remarkable
for WBCs too numerous to count, with 4+ bacteria. Aggressive intravenous fluid resuscitation
and intravenous antibiotics are initiated but the patient remains hypotensive.
Which one of the following would be most appropriate at this point?
A) High-dose intravenous corticosteroids
B) Hydroxyethyl starch (Hespan)
C) Low-dose dopamine
D) Norepinephrine (Levophed)
E) Dobutamine

ANSWER: D
Prompt recognition and treatment of sepsis increases the chances of survival. Aggressive fluid
resuscitation is the initial treatment for hypotension in patients in septic shock. Antibiotic
therapy should be administered within 1 hour of suspecting sepsis. If fluid resuscitation is not
successful in restoring blood pressure, norepinephrine is the currently recommended first-line
vasopressor. The use of hydroxyethyl starch is not recommended because the mortality rate
is higher in sepsis patients. Previously, dopamine was recommended, but low-dose dopamine
for renal perfusion has now been shown to be ineffective. For patients who are vasopressor
dependent, low-dose corticosteroids can be considered.
Ref: Gauer RL: Early recognition and management of sepsis in adults: The first six hours. Am Fam Physician 2013;88(1):44-53. 2)
Seymour CW, Rosengart MR: Septic shock: Advances in diagnosis and treatment. JAMA 2015;314(7):708-717.

92. A healthy 65-year-old female presents for a health maintenance visit. She has no record
of having received pneumococcal vaccine.
Which one of the following does the CDC recommend for this patient?
A) Both 13-valent pneumococcal conjugate vaccine (PCV13) and 23-valent pneumococcal
polysaccharide vaccine (PPSV23) now
B) PCV13 now and PPSV23 in 1 year
C) PPSV23 now and PCV13 in 1 year
D) Both PCV13 and PPSV23 now, and PPSV23 every 5 years
E) PCV13 only, now and every 5 years

ANSWER: B
In spite of an estimated 50%–80% reduction in cases of invasive pneumococcal disease (IPD)
as a result of vaccination with the 23-valent pneumococcal polysaccharide vaccine (PPSV23),
IPD remains a significant problem in the United States, with approximately 40,000 cases in

56
2010 resulting in about 4000 deaths. The introduction of the 13-valent pneumococcal
conjugate vaccine (PCV13) has been shown to further reduce vaccine-type cases of IPD by
as much as 75%, leading to the 2014 recommendation from the Advisory Committee on
Immunization Practices to administer PCV13 to adults age 65 and older. In immunocompetent
adults age 65 and older who are pneumococcal vaccine–naive, administering PPSV23 1 year
or more after PCV13 results in a better immune response than giving PPSV23 first, and
reduces local reactions to the vaccines compared to simultaneous administration or
administering PPSV23 first. For some immunocompromised patients the recommendation is
to wait a minimum of 8 weeks after giving PCV13 before administering PPSV23.
Ref: Shah AA, Fields H, Wallace MR: Navigating the changes in pneumococcal immunizations for adults. Am Fam Physician
2015;92(6):456-458. 2) Kobayashi M, Bennett NM, Gierke R, et al: Intervals between PCV13 and PPSV23 vaccines:
Recommendations of the Advisory Committee on Immunization Practices (ACIP). MMWR Morb Mortal Wkly Rep 2015;64(34):944-
947.

93. A 30-year-old male asks about colon cancer screening. He is healthy, but his father was
diagnosed with colon cancer at the age of 45.
At what age would you recommend that this patient have his first screening colonoscopy?
A) 30
B) 35
C) 40
D) 45
E) 50

ANSWER: B
Average-risk adults should be screened for colorectal cancer starting at age 50. People who
are at higher risk for developing colon cancer should be screened at either age 40 or 10 years
earlier than the age at which the youngest affected family member was diagnosed, whichever
is earliest. Risk factors for colon cancer include age, ethnicity, and family history. There is
good evidence that screening identifies premalignant lesions, which allows for early treatment
and reduced mortality.
Ref: Randel A: ACP releases best practice advice on colorectal cancer screening. Am Fam Physician 2012;86(12):1153-1154.

94. Which one of the following is the most common cause of hearing loss in the newborn?
A) Aminoglycosides
B) Genetic inheritance
C) Head trauma
D) Prematurity
E) Rubella

ANSWER: B

57
In the past, rubella was a common cause for reduced hearing. With the advent of vaccination,
genetic inheritance has become the most frequent cause for deafness. Aminoglycosides are
rarely a reason for hearing loss. The well-known association of aminoglycosides with hearing
loss has reduced the exposure risk from these drugs. Head trauma and prematurity remain
important causes for deafness but are still very small risk factors compared to simple
inheritance. Prematurity risk has diminished with improvements in the care of premature
infants.
Ref: Morton CC, Nance WE: Newborn hearing screening—A silent revolution. N Engl J Med 2006;354(20):2151-2164. 2) Rakel RE,
Rakel DP (eds): Textbook of Family Medicine, ed 9. Elsevier Saunders, 2016, p 322.

95. A 32-year-old female at 12 weeks gestation presents with an itchy vulvar lesion, which
you diagnose as external genital warts. Which one of the following would be accurate advice
regarding treatment?
A) The recurrence rate is low with treatment
B) Cryotherapy is the most effective treatment
C) Treatment should be based on patient preference
D) Treatment is effective in preventing congenital transmission

ANSWER: C
Treatment of genital warts should be based on patient preference and cost (SOR C). There is
no treatment that is more effective than others, and the recurrence rate for any treatment is
relatively high. Cryotherapy is not more effective than other treatments. Treatment in
pregnancy has not been found to decrease the risk of transmission.
Ref: Karnes JB, Usatine RP: Management of external genital warts. Am Fam Physician 2014;90(5):312-318.

58
96. An agitated 35-year-old male is brought to the urgent-care clinic by his wife. She reports
that her husband has been restless and tremulous since early this morning. Further history
reveals that he was seen 2 days ago for a work-related back injury and was given a prescription
for cyclobenzaprine, 10 mg 3 times daily as needed. He reports improvement in his back pain
but admits that last night he took an extra dose of a sleeping pill prescribed by his psychiatrist.
His chronic medical conditions include controlled essential hypertension and depression. On
examination the patient’s vital signs include a temperature of 38.1°C (100.6°F), a blood
pressure of 124/82 mm Hg, a heart rate of 98 beats/min, and a respiratory rate of 12/min. The
patient appears diaphoretic with a resting symmetrical tremor. He has a regular heart rate and
rhythm and his lungs are clear. His abdomen is soft with normal bowel sounds. His strength
is 5/5 bilaterally in the upper and lower extremities, and he has 4+ patellar and biceps reflexes
bilaterally.
Which one of the following is the most likely explanation for these findings?
A) Anticholinergic syndrome
B) Central cord syndrome
C) Malignant hyperthermia
D) Neuroleptic malignant syndrome
E) Serotonin syndrome

ANSWER: E
Serotonin syndrome is a potentially life-threatening condition caused by excessive
serotonergic activity, and certain medications are more likely to precipitate it. Early
recognition of symptoms is important, as most cases can be managed on an outpatient basis
with discontinuation of the precipitating medication/agent and supportive care. The Hunter
Serotonin Toxicity Criteria can be used to diagnose serotonin syndrome (SOR C).
The differential diagnosis includes anticholinergic syndrome, malignant hyperthermia, and
neuroleptic malignant syndrome. Anticholinergic syndrome is associated with tachycardia,
tachypnea, and hyperthermia. Malignant hyperthermia and neuroleptic malignant syndrome
are associated with hypertension, tachycardia, tachypnea, and hyperthermia. These conditions
are precipitated by other classes of medications. Central cord syndrome is a spinal cord
disease caused by spinal trauma, syringomyelia, and intrinsic cord tumors, and presents with
arm weakness greater than leg weakness.
Ref: Ables AZ, Nagubilli R: Prevention, recognition, and management of serotonin syndrome. Am Fam Physician 2010;81(9):1139-
1142. 2) Kasper DL, Fauci AS, Hauser SL, et al (eds): Harrison’s Principles of Internal Medicine, ed 19. McGraw-Hill, 2015, pp 2650-
2660.

59
97. A 30-year-old female is 3 months post partum. She had a recent upper respiratory infection
and now has anterior neck pain, anxiety, and palpitations. On examination her thyroid is
palpable and extremely tender. Laboratory testing reveals that free T4 is elevated and TSH is
suppressed.
Which one of the following is the most likely diagnosis?
A) Hashimoto thyroiditis
B) Postpartum thyroiditis
C) Subacute thyroiditis
D) Postpartum depression

ANSWER: C
Only subacute thyroiditis is associated with a painful and tender thyroid. It often follows an
upper respiratory viral illness that triggers inflammatory destruction of thyroid tissue. Thyroid
hormone is increased and TSH is suppressed. Treatment consists of corticosteroids or
NSAIDs. The condition is usually self-limited.
Ref: Sweeney LB, Stewart C, Gaitonde DY: Thyroiditis: An integrated approach. Am Fam Physician 2014;90(6):389-396.

98. An 82-year-old female with a history of diastolic heart failure, stage 3 chronic kidney
disease, and essential hypertension presents to your office with shortness of breath. She does
not have chest pain. The physical examination reveals an elderly female in mild distress. Her
blood pressure is 138/82 mm Hg, pulse rate 92 beats/min, respiratory rate 24/min, and oxygen
saturation 88% on room air, increased to 92% on 2 L/min of oxygen via nasal cannula. Her
lungs are clear to auscultation bilaterally. Her heart rate is regular and no murmurs are
detected. She has 1+ bilateral lower extremity edema that is slightly worse on the right. A
chest radiograph is normal and her D-dimer level is elevated.
Which one of the following would be most appropriate at this point?
A) A serum BNP level
B) A serum troponin I level
C) Echocardiography
D) A bilateral venous duplex study of the lower extremities
E) CT angiography of the chest

ANSWER: E
This patient has a high risk of pulmonary embolism based on her presentation and the elevated
D-dimer assay. CT angiography (CTA) of the chest would be the next step in the evaluation
of this patient (SOR A). The other tests may be helpful but should not delay chest CTA. If
chest CTA is negative a venous duplex study would be helpful in ruling out a DVT.
Ref: Qaseem A, Snow V, Barry P, et al; Joint American Academy of Family Physicians/American College of Physicians Panel on
Deep Venous Thrombosis/Pulmonary Embolism: Current diagnosis of venous thromboembolism in primary care: A clinical practice
guideline from the American Academy of Family Physicians and the American College of Physicians. Ann Fam Med 2007;5(1):57-
62. 2) Fesmire FM, Brown MD, Espinosa JA, et al; American College of Emergency Physicians: Critical issues in the evaluation and

60
management of adult patients presenting to the emergency department with suspected pulmonary embolism. Ann Emerg Med
2011;57(6):628-652.

99. A 68-year-old female sees you because she developed increased floaters followed by
flashes of lights after a sneezing attack. She then noted a dark spot in the periphery of her
right eye. Three weeks ago she underwent cataract removal with intraocular lens implantation
in this eye. Her examination is notable for decreased visual acuity and a loss of peripheral
field in the right eye.
Which one of the following is the most likely diagnosis?
A) Posterior vitreous detachment
B) Central retinal artery occlusion
C) Retinal detachment
D) Intraocular lens dislocation
E) Acute angle-closure glaucoma

ANSWER: C
Retinal detachments can be caused by a break in the retina, exudate or leakage from beneath
the retina, or traction on the retina. Retinal detachments are often preceded by a posterior
vitreous detachment, which can lead to a break in the retina, and patients may experience an
increase in floaters but not light flashes or loss of vision. With a retinal detachment, patients
typically experience floaters followed within 1 week by flashes of light as the retina tears. If
the retinal tear becomes large enough vision is impaired. Any patient with the typical history
and loss of visual acuity or peripheral field should be urgently referred to an ophthalmologist,
as urgent surgery may be required.
Risk factors for retinal detachment include age 50–75, ocular trauma, previous cataract
surgery, family history, and a past history of retinal detachment. A patient with a previous
retinal detachment has a 25% risk of developing a retinal detachment in the other eye. Myopia
is the other significant risk factor, associated with a tenfold increased risk in patients with >3
diopters of refractive error.
Central retinal artery occlusion is manifested by painless complete loss of vision in the
affected eye. Acute angle-closure glaucoma is associated with eye pain, photophobia,
headache, nausea and vomiting, and cloudy loss of vision, and the onset is not typically
associated with coughing or sneezing.
Ref: Gelston CD: Common eye emergencies. Am Fam Physician 2013;88(8):515-519.

100. Which one of the following does the 2014 Global Initiative for Chronic Obstructive Lung
Disease (GOLD) guideline recommend for the treatment of exacerbations of COPD?
A) Prednisone, 40 mg daily for 5 days
B) Prednisone, 40 mg daily for 10 days
C) A methylprednisolone (Medrol) dose pack
D) Methylprednisolone sodium succinate (Solu-Medrol), 100 mg intravenously, followed by
prednisone, 40 mg for 7 days

61
ANSWER: A
The 2014 Global Initiative for Chronic Obstructive Lung Disease (GOLD) guideline
recommends administration of prednisone, 40 mg daily for 5 days, for COPD exacerbations.
The 2011 GOLD guideline had recommended the same dosage of prednisone for 10–14 days.
The guideline does not recommend a methylprednisolone dose pack or intravenous
methylprednisolone sodium succinate. A recent multicenter study showed that the shorter
duration of low-dose prednisone was equivalent to the longer treatment. Corticosteroids are
associated with elevated blood glucose; the development of cataracts, diabetes mellitus, and
osteopenia; and thromboembolic complications.
Ref: Sonstein L, Clark C, Seidensticker S, et al: Improving adherence for management of acute exacerbation of chronic obstructive
pulmonary disease. Am J Med 2014;127(11):1097-1104.

101. When evaluating a patient with menorrhagia, which one of the following laboratory
findings would be most consistent with von Willebrand disease?
A) An isolated prolonged prothrombin time
B) An isolated prolonged partial thromboplastin time
C) A low serum iron level
D) A low platelet count
E) A low fibrinogen level

ANSWER: B
Von Willebrand disease (vWD) is a common coagulation disorder generally due to a
hereditary reduction in the quality or quantity of a protein complex required for platelet
adhesion, known as von Willebrand factor (vWF). The extent of deficiency varies greatly,
resulting in vWD subtypes ranging from asymptomatic to serious. A common problem
associated with vWD is menorrhagia, and the diagnosis should always be entertained in
women who experience excessive menstrual blood loss. Although tests measuring vWF are
easily obtained, interpretation of the results can be challenging since vWF levels can be
affected by blood type, inflammation, infection, trauma, and emotional stress. Confirmation
of vWD often requires the expertise of a hematologist. Although the results for all laboratory
tests listed can fall within their reference ranges in a patient with vWD, the finding most
suggestive of this diagnosis is an isolated prolonged partial thromboplastin time.
Ref: Yawn B, Nichols WL, Rick ME: Diagnosis and management of von Willebrand disease: Guidelines for primary care. Am Fam
Physician 2009;80(11):1261-1268. 2) Committee on Adolescent Health Care, Committee on Gynecologic Practice: Von Willebrand
disease in women. American College of Obstetricians and Gynecologists, Committee Opinion no 580, 2013 (reaffirmed 2015).

62
102. A 68-year-old female with well-controlled hypertension presents for a routine
evaluation. The physical examination is significant for a BMI of 35.4 kg/m2. A
comprehensive metabolic panel prior to the visit revealed the following:
Serum calcium ………… 9.2 mg/dL (N 8.5–10.2)
Albumin ………………… 4.0 g/dL (N 3.5–5.4)
Creatinine ………………… 0.6 mg/dL (N 0.6–1.1)
25-hydroxyvitamin D ………… 9 ng/mL (N >20)
Alkaline phosphatase ………… 151 U/L (N 47–147)
Which one of the following is true, based on these findings?
A) She has primary hyperparathyroidism
B) She should take 1,25-dihydroxyvitamin D (calcitriol)
C) She should take vitamin D supplements
D) She should take calcium supplements
E) She has rickets

ANSWER: C
This patient has slight elevations of her alkaline phosphatase along with a deficiency of
vitamin D and normal calcium levels. This constellation of findings is most consistent with
secondary hyperparathyroidism related to vitamin D deficiency. Vitamin D storage is best
reflected by the serum 25-hydroxyvitamin D level. While there is some disagreement
regarding normal levels, a level <10 ng/mL is clearly deficient. This would put the patient at
risk for osteomalacia but not rickets, which is a clinical diagnosis based on the effects of
insufficient bone mineralization secondary to low vitamin D activity before the closure of
growth plates. Supplementation with activated vitamin D (calcitriol) is generally only
necessary in patients with renal failure or other conditions associated with inadequate
activation of the storage forms of vitamin D. Supplementation with vitamin D should decrease
PTH activity and thus bone turnover in this patient, which would likely normalize the alkaline
phosphatase.
Ref: Kasper DL, Fauci AS, Hauser SL, et al (eds): Harrison’s Principles of Internal Medicine, ed 19. McGraw-Hill, 2015, pp 2454-
2466.

103. A 16-year-old male presents with acute testicular pain that has been constant for the last
2 hours. He has nausea and vomiting that started about the same time. On examination you
note that the left testis is situated higher in the scrotum than the right testis. The left
hemiscrotum is erythematous, slightly warm, and indurated. The patient is currently afebrile.
Which one of the following is the most appropriate management at this time?
A) NSAIDs and scrotal support
B) Doxycycline plus intravenous ceftriaxone (Rocephin)
C) Rotating the testicle from lateral to medial (like closing a book)
D) Immediate urologic consultation for surgical exploration

63
ANSWER: D
This patient presents with a classic description of torsion of the left testicle, and not an
infectious process requiring antibiotics. Surgical exploration is the immediate priority, with
ischemic damage starting between 4 and 8 hours after onset. Attempts at manual detorsion
should not delay surgical exploration and often require analgesia or sedation. If surgery is not
an immediate option, manual detorsion is performed by rotating the testicle from medial to
lateral, like opening a book. Doppler ultrasonography is the imaging modality of choice, but
it delays surgical exploration and should be used only when the history and physical
examination make the diagnosis questionable.
Ref: Sharp VJ, Kieran K, Arlen AM: Testicular torsion: Diagnosis, evaluation, and management. Am Fam Physician 2013;88(12):835-
840.

104. U.S. federal labor law requires companies with >50 employees to provide which one of
the following benefits for employees who are nursing mothers?
A) Extended Family and Medical Leave Act protection for a total of 6 months
B) Reasonable unpaid break time to express milk
C) Extended bathroom access time to express milk
D) A refrigerator dedicated to storage of expressed milk

ANSWER: B
The Patient Protection and Affordable Care Act amended the Federal Labor Standards Act of
1938 to require employers of >50 employees to provide nursing mothers reasonable break
time to express milk for up to 1 year following the birth of their children. The employer is not
required to compensate nursing mothers during the breaks and is not required to provide
refrigerated storage for the expressed milk. The law also requires that the employer provide a
place to express milk, other than a bathroom, that is shielded from view and free from
intrusion from coworkers and the public.
Ref: US Department of Labor, Wage and Hour Division: Section 7(r) of the Fair Labor Standards Act of 1938 (29 U.S.C. 207): Break
time for nursing mothers provision.

105. A 30-year-old ICU nurse has been caring for several patients infected with Clostridium
difficile. She is asymptomatic but is worried that she may also be infected.
Which one of the following is the most appropriate recommendation for this nurse?
A) No testing and no treatment
B) Testing for C. difficile toxin
C) Testing for C. difficile antigen
D) Empiric treatment with metronidazole
E) Probiotics

ANSWER: A

64
Laboratory testing for Clostridium difficile should be done only on symptomatic patients. A
diagnosis of C. difficile infection requires the presence of diarrhea (≥3 unformed stools in a
24-hour period) or radiographic evidence of ileus and toxic megacolon. In addition, the
diagnosis requires a positive stool test for toxigenic C. difficile or its toxins, or colonoscopic
or histopathologic findings showing pseudomembranous colitis. Laboratory testing cannot
distinguish between asymptomatic colonization and symptomatic infection. Test of cure is
not recommended after C. difficile treatment. Probiotics may prevent antibiotic-associated
diarrhea and may also reduce C. difficile diarrhea in children and adults younger than 65, but
are not specifically recommended for preventing or treating C. difficile infection.
Ref: Bagdasarian N, Rao K, Malani PN: Diagnosis and treatment of Clostridium difficile in adults: A systematic review. JAMA
2015;313(4):398-408.

106. A 36-year-old white male complains of episodic pain in the rectum over the past several
years. The pain occurs every 3–6 weeks and is sharp, cramp-like, and severe. It lasts from 1
to 15 minutes. He has no other gastrointestinal complaints. A physical examination, including
a digital rectal examination and anoscopy, is normal.
The most likely diagnosis is
A) fecal impaction
B) coccygodynia
C) anal fissure
D) proctalgia fugax
E) sacral nerve neuralgia

ANSWER: D
Symptoms consistent with proctalgia fugax occur in 13%–19% of the general population.
These consist of episodic, sudden, sharp pains in the anorectal area lasting several seconds to
minutes. The diagnosis is based on a history that fits the classic picture in a patient with a
normal examination. All the other diagnoses listed would be evident from the physical
examination, except for sacral nerve neuralgia, which would not be intermittent for years and
the pain would not be transitory.
Ref: Feldman M, Friedman LS, Brandt LJ (eds): Sleisenger and Fordtran’s Gastrointestinal and Liver Disease, ed 9. Saunders Elsevier,
2010, pp 2272-2273. 2) Fargo MV, Latimer KM: Evaluation and management of common anorectal conditions. Am Fam Physician
2012;85(6):624-630.

65
107. A patient presents with a lesion on her forearm that first appeared 6 months ago. The
lesion Is

Which one of the following is the most likely diagnosis?


A) A cutaneous horn
B) Keratoacanthoma
C) Pyogenic granuloma
D) Seborrheic keratosis
E) Verruca vulgaris

ANSWER: A
This is a cutaneous horn, sometimes referred to as a hypertrophic actinic keratosis. It is a
horn-like projection of keratin on a slightly raised base. These usually arise in areas subject
to photoaging, including the forearms. The differential diagnosis often includes
keratoacanthoma, which occurs most commonly on the face, grows very rapidly, and often is
more nodular with a central pit. Pyogenic granulomas are fleshy appearing, and a wart or
seborrheic keratosis would both look slightly different but would rarely become so large in
just months. Cutaneous horns should be removed due to the possible development of in situ
or invasive squamous cell carcinoma (SOR A).
Ref: Habif TP: Clinical Dermatology: A Color Guide to Diagnosis and Therapy, ed 6. Mosby Elsevier, 2015, p 794.

66
108. An 82-year-old female sees you for follow-up 6 weeks after fracturing her hip when she
tripped on a hose and fell in her garden. She underwent surgical repair and acute inpatient
rehabilitation. She has successfully recovered, is participating in an outpatient physical
therapy program, and is now walking with a cane. She reports gastroesophageal reflux
controlled with over-the-counter ranitidine (Zantac) as her only chronic medical condition.
She recalls having been told that she had only mild bone loss on a bone density test last year.
She has been taking a calcium and vitamin D supplement since then. She is concerned about
sustaining another fracture.
Which one of the following should you do now to reduce her risk of recurrent fracture?
A) Prescribe alendronate, 70 mg weekly
B) Prescribe raloxifene (Evista), 60 mg daily
C) Prescribe teriparatide (Forteo), 20 :g daily
D) Discontinue ranitidine

ANSWER: A
Undertreatment of osteoporosis occurs frequently after a hip fracture. Unless a
contraindication exists, patients should be treated with a bisphosphonate after a hip fracture,
regardless of bone mineral density (SOR C). Controlled gastroesophageal reflux is not a
contraindication to bisphosphonate therapy. Proton pump inhibitor use, but not H2-blocker
use, is also a modifiable risk factor for osteoporosis. Raloxifene and teriparatide are not
bisphosphonates or first-line therapy for prevention of recurrent hip fracture. Raloxifene has
not been shown to reduce the rate of nonvertebral fractures.
Ref: LeBlanc KE, Muncie HL Jr, LeBlanc LL: Hip fracture: Diagnosis, treatment, and secondary prevention. Am Fam Physician
2014;89(12):945-951.

109. A 28-year-old male hospital employee sees you for a mandatory workplace influenza
vaccination. He has never received influenza vaccine and is apprehensive about it because of
a history of egg allergy. He says he developed hives on his trunk after eating scramble eggs
on several occasions when he was in high school and since then has avoided eating cooked
eggs, but not all egg-containing food items. He has had no cases of hives since making this
change in his diet.
According to the Advisory Committee on Immunization Practices, which one of the following
would be the most appropriate action?
A) Administer live attenuated influenza vaccine
B) Administer an intradermal test dose of inactivated influenza vaccine and if the patient does
not develop hives administer a full dose of inactivated influenza vaccine 2 or more hours later
C) Administer inactivated influenza vaccine
D) Administer oral diphenhydramine (Benadryl) 1 hour before inactivated influenza vaccine
E) Do not vaccinate for influenza

ANSWER: C

67
All currently available influenza vaccines, with the exceptions of recombinant and cell-
culture–based inactivated influenza vaccines, are prepared using embryonated egg culture and
can potentially provoke allergic and anaphylactic reactions. Large studies of influenza
vaccine administration to egg-allergic patients have resulted in a few mild reactions but no
documented occurrences of anaphylaxis, although there are reports of serious reactions. The
data collected from these studies provided sufficient confidence for the Advisory Committee
on Immunization Practices to develop guidelines for administration of influenza vaccine in
individuals with egg allergy.
For those who report that they can eat lightly cooked scrambled eggs, vaccination can proceed
without precaution or observation. Those who have experienced only hives can also receive
any influenza vaccine appropriate for their age and health status. In the past the CDC
recommended observing these patients for 30 minutes afterward, but this recommendation
was changed in 2016. People who have experienced symptoms such as hypotension,
wheezing, nausea, or vomiting, or reactions requiring emergency attention or epinephrine
after eating eggs or egg-containing foods can also receive any influenza vaccine appropriate
for their age and health status and also do not need to be observed. However, the vaccine
should be administered by a provider who can recognize and manage severe allergic reactions.
Withholding vaccination because of egg-induced hives is not recommended.
Ref: Grohskopf LA, Sokolow LZ, Broder KR, et al: Prevention and control of seasonal influenza with vaccines: Recommendations of
the Advisory Committee on Immunization Practices—United States, 2016–17 influenza season. MMWR Recomm Rep 2016;65(RR-
5):1-54.

110. A 65-year-old female presents to your office with a 1-day history of severe low back
pain that began acutely after she moved some furniture. She is in good health otherwise, has
no previous history of back problems, and has not had a fever. She is having pain in the
bilateral low back region, bilateral buttock region, and upper thigh. She has not been able to
void since the pain started.
Which one of the following diagnoses should be considered in this patient?
A) Acute lumbar strain
B) Lumbar muscle spasm
C) Mechanical low back pain
D) Large midline disc herniation
E) Sciatica

ANSWER: D
Patients with low back pain should be evaluated for the presence of neurologic deficits.
Urinary retention is the most frequent finding in cauda equina syndrome (90% sensitivity),
caused by compression of nerve roots from the lower cord segments. This is usually due to a
massive, centrally herniated disc, which can result in urinary retention or incontinence from
loss of sphincter function, bilateral motor weakness of the lower extremities, and saddle

68
anesthesia. This problem should be addressed urgently. In patients without urinary retention,
the probability of the cauda equina syndrome is approximately 1 in 10,000.
Ref: Chou R, Qaseem A, Snow V, et al: Diagnosis and treatment of low back pain: A joint clinical practice guideline from the American
College of Physicians and the American Pain Society. Ann Intern Med 2007;147(7):478-491. 2) Casazza BA: Diagnosis and treatment
of acute low back pain. Am Fam Physician 2012;85(4):343-350.

111. A 35-year-old male has a 5-day history of cough and has had one episode of blood-
streaked sputum. He is otherwise healthy and has never smoked. He is afebrile and has normal
findings on examination. A chest radiograph is normal.
Which one of the following would be most appropriate at this point?
A) Observation
B) CT of the chest
C) Pulmonary function studies
D) Bronchoscopy
E) A trial of antibiotics

ANSWER: A
This patient has a low risk of cancer, based upon his age and medical history, and no
suggestion of a lower respiratory infection. With this presentation, a chest radiograph is
recommended as the first step in the workup, and if findings are normal he should be observed
for 2–6 weeks (SOR A). If there is a recurrence of hemoptysis further evaluation is indicated,
which should include an interval history, a repeat examination, and CT of the chest.
If the initial presentation had suggested a lower respiratory infection, treatment with oral
antibiotics rather than observation would have been appropriate.
Ref: Earwood JS, Thompson TD: Hemoptysis: Evaluation and management. Am Fam Physician 2015;91(4):243-249.

112. The Timed Up and Go test is used to evaluate geriatric patients for which one of the
following?
A) Risk of falling
B) Effects of peripheral neuropathy
C) Kinetic tremor
D) Neurocardiogenic syncope
E) Central causes of vertigo

ANSWER: A
The Timed Up and Go test is the most frequently recommended screening test for mobility.
It takes less than a minute to perform and involves asking the patient to rise from a chair, walk
10 feet, turn, return to the chair, and sit down. Any unsafe or ineffective movement with this
test suggests balance or gait impairment and an increased risk of falling. If the test is
abnormal, referral to physical therapy for complete evaluation and assessment should be

69
considered. Other interventions should also be considered, such as a medication review for
factors related to the risk of falling.
Ref: Tinetti ME, Kumar C: The patient who falls: “It’s always a trade-off.” JAMA 2010;303(3):258-266. 2) Herman T, Giladi N,
Hausdorff JM: Properties of the ‘Timed Up and Go’ test: More than meets the eye. Gerontology 2011;57(3):203-210.

113. Which one of the following can falsely elevate hemoglobin A1c?
A) Antiretroviral treatment for HIV infection
B) Chronic liver disease
C) Hemolytic anemia
D) Iron deficiency anemia
E) Pregnancy

ANSWER: D
Hemoglobin A1c (HbA1c) testing measures the percentage of glycosylation of the HbA1c
chain, and correlates to average blood glucose levels over the previous 2–3 months. However,
hypertriglyceridemia, hyperbilirubinemia, iron deficiency anemia, splenectomy, renal failure,
and aplastic anemia can all falsely elevate HbA1c levels. The other factors listed can all
falsely lower HbA1c levels, as can vitamins C and E, and acute blood loss.
Ref: Pippitt K, Li M, Gurgle HE: Diabetes mellitus: Screening and diagnosis. Am Fam Physician 2016;93(2):103-109

114. A 62-year-old male presents for a routine health maintenance visit. He has osteoarthritis
and controlled hypertension, but is otherwise healthy. He does not smoke and his alcohol
consumption consists of 2–3 drinks a week. His medications include lisinopril (Prinivil,
Zestril) and acetaminophen. His wife just had a DXA scan and he asks if he should also be
screened for osteoporosis.
For this patient, the U.S. Preventive Services Task Force
A) makes no recommendation for or against DXA
B) recommends DXA at this visit
C) recommends DXA at age 65
D) recommends DXA at age 70

ANSWER: A
The U.S. Preventive Services Task Force has a grade I recommendation for routine screening
for osteoporosis in men, meaning there is insufficient evidence to recommend for or against
routine screening. Men older than 50 with a minimal-trauma fracture and men with conditions
associated with bone loss could be considered for screening. The National Osteoporosis
Foundation recommends screening all men age 70 and above for osteoporosis.
Ref: US Preventive Services Task Force: Screening for osteoporosis: US Preventive Services Task Force recommendation statement.
Ann Intern Med 2011;154(5):356-364. 2) Jeremiah MP, Unwin BK, Greenawald MH, Casiano VE: Diagnosis and management of
osteoporosis. Am Fam Physician 2015;92(4):261-268.

70
115. A healthy 45-year-old female presents to your office to establish care. She has no
significant past medical history and is up to date on her immunizations. She has no chest pain,
shortness of breath, or exercise intolerance. She does not take any prescribed medications but
does take a low-dose aspirin daily for prevention of coronary artery disease. She does not
smoke and she exercises by walking for 45 minutes 4–5 times a week. She is concerned
because her mother had a fatal cardiac arrest at age 63 and her father was recently diagnosed
with end-stage renal disease at age 75.
On examination the patient’s blood pressure is 120/75 mm Hg and her BMI is 22.1 kg/m2.
Her cardiovascular and pulmonary examinations are unremarkable.
Which one of the following would you recommend to this patient?
A) A resting EKG
B) An exercise EKG
C) Discontinuing aspirin therapy
D) Increasing aspirin to 325 mg daily
E) A basic metabolic panel to screen for chronic kidney disease

ANSWER: C
Although this patient has a family history of coronary artery disease, she is under the age of
50 and thus aspirin therapy as primary prevention is not recommended and may increase the
risk for gastrointestinal bleeding, regardless of the dosage. The U.S. Preventive Services Task
Force found insufficient evidence for screening for chronic kidney disease even in individuals
with a positive family history. Neither a resting nor exercise EKG is recommended for
asymptomatic individuals to detect or prevent coronary artery disease.
Ref: Moyer VA; US Preventive Services Task Force: Screening for coronary heart disease with electrocardiography: US Preventive
Services Task Force recommendation statement. Ann Intern Med 2012;157(7):512-518. 2) Vandvik PO, Lincoff AM, Gore JM, et al;
American College of Chest Physicians: Primary and secondary prevention of cardiovascular disease: Antithrombotic therapy and
prevention of thrombosis, 9th ed: American College of Chest Physicians evidence-based clinical practice guidelines. Chest 2012;141(2
Suppl):e637S-668S. 3) Jumbo M, Dorsch MP, Ealovega MW, et al: Essential hypertension. University of Michigan Health System
Guidelines for Clinical Care, 2014.

116. A 19-year-old female comes to your office with lower abdominal pain that has increased
over the past 2 days. Her last menstrual period was 4 days ago. She has been nauseated and
has been vomiting. The physical examination reveals a temperature of 38.0°C (100.4°F) and
lower abdominal tenderness with mild rebound. She has a mucopurulent cervical discharge,
tenderness with cervical motion, a normal-size uterus, and left adnexal fullness. A serum hCG
is negative.
Which one of the following is the most appropriate management?
A) Ceftriaxone (Rocephin), 250 mg intramuscularly
B) Hospitalization for intravenous antibiotics
C) Surgical consultation for immediate appendectomy
D) Laparoscopy

71
ANSWER: B
This patient has signs and symptoms of acute salpingitis. This condition is commonly
confused with appendicitis, ectopic pregnancy, and other pelvic pathology. In this case the
findings are clearly pelvic in origin. Endocervical inflammation with a mucopurulent
discharge is noted in almost every case of acute salpingitis. The acute nature of this
presentation and the adnexal fullness suggest gonorrhea rather than chlamydial infection,
although the antibiotic regimen should probably cover Chlamydia as well. Admission to the
hospital and treatment with parenteral antibiotics is most appropriate in this case because of
the severity of the illness, the desire to maintain reproductive function, and the adnexal
fullness.
Routine laparoscopy for every case of salpingitis is considered too costly and dangerous. The
choice of intravenous antibiotic may vary, but usually consists of a β-lactam antibiotic
(cefoxitin or ceftriaxone) plus doxycycline, or gentamicin and clindamycin. Ceftriaxone, 250
mg intramuscularly, is appropriate for uncomplicated gonococcal infection.
Ref: Workowski KA, Bolan G; Centers for Disease Control and Prevention (CDC): Sexually transmitted diseases treatment guidelines,
2015. MMWR Recomm Rep 2015;64(RR-03):78-82.

117. An 18-year-old female presents with an intensely pruritic papular eruption in the vicinity
of her waist that began shortly after she spent a day walking in the woods with her boyfriend.
Her rash consists of multiple small excoriated papules and welts along her beltline. She says
she was wearing jeans and sandals.
Which one of the following is the most likely cause of her rash?
A) Bedbugs
B) Chiggers
C) Fleas
D) Deer ticks
E) Mosquitoes

ANSWER: B
Mite larvae called chiggers cause itchy bites. The chiggers crawl on skin until they reach
constrictive clothing like belts or socks and then bite there. Flea bites are usually at ankle
height because fleas jump. Mosquito bites would be diffuse on exposed areas. Bedbugs tend
to bite on the upper body and neck. This is not a typical presentation for Lyme disease, which
has an initial rash that is localized and not pruritic (SOR C).
Ref: Juckett G: Arthropod bites. Am Fam Physician 2013;88(12):841-847.

72
118. A homeless 47-year-old male visits the local health department and asks to be screened
for tuberculosis. He is not aware of any definite exposure to tuberculosis and is asymptomatic.
The nurse asks whether the patient should be screened with a tuberculin skin test or an
interferon-gamma release assay (IGRA, QuantiFERON-TB Gold).
Which one of the following would be an advantage of IGRA in this patient?
A) It helps to distinguish latent tuberculosis from active tuberculosis
B) The patient’s blood can be stored for up to 36 hours if needed, to allow transport to a
qualified laboratory to run the test
C) A follow-up visit is not required to obtain results
D) The IGRA will be positive within 2 weeks of exposure to an individual with active
tuberculosis
E) The IGRA can simultaneously detect resistance to rifampin (Rifadin)

ANSWER: C
It is important for health care professionals to be familiar with the various options for
screening and testing for latent or active tuberculosis. In the United States the tuberculin skin
test (TST) is the traditional screening test for tuberculosis. The interferon-gamma release
assay (IGRA) is a blood test that can also aid in the diagnosis of latent tuberculosis.
Advantages of IGRA include the ability to get results without follow-up and the fact that prior
bacille Calmette-Guérin (BCG) vaccination does not cause a false-positive test. With TST
testing, prior BCG vaccination, especially if given within the last 10 years, can result in a
false-positive test.
As with the TST, conversion (i.e., a positive test) of IGRA may not occur within the first 8–
12 weeks following exposure to an individual with active tuberculosis. In addition, neither
the TST nor the IGRA can distinguish between latent and active tuberculosis.
There are two IGRA tests available for use in the United States and both tests need to be
processed within 8–30 hours, depending on the specific test used. Although there is now a
test available that can detect Mycobacterium tuberculosis complex (MTBC) and resistance to
rifampin, it is a separate test called the Xpert MTB/RIF assay.
Ref: Tuberculosis: Testing & diagnosis. Centers for Disease Control and Prevention, 2013. 2) Hartman-Adams H, Clark K, Juckett G:
Update on latent tuberculosis infection. Am Fam Physician 2014;89(11):889-896.

73
119. During a routine health maintenance visit, a 24-year-old female admits that she is not
feeling well due to being overwhelmed with stress. She feels she has always worried more
than most people, but recent troubles at home and at work have made things much worse. She
says she is irritable with people around her, has trouble focusing at work, and feels fatigued
late in the day. Despite her fatigue, she has difficulty falling asleep at night. The patient denies
anhedonia, suicidal thoughts, or a persistently depressed mood. She limits her caffeine intake,
does not smoke or drink alcohol, and is not using any illicit drugs. In addition to
psychotherapy, which one of the following medications is recommended for this patient?
A) Alprazolam extended release (Xanax XR)
B) Clonazepam (Klonopin)
C) Gabapentin (Neurontin)
D) Quetiapine (Seroquel)
E) Sertraline (Zoloft)

ANSWER: E
This patient’s symptoms are consistent with the DSM-5 criteria for generalized anxiety
disorder. First-line treatments for this condition are SSRIs, SNRIs, and tricyclic
antidepressants. Quetiapine and gabapentin are considered second-line medications for
anxiety if control cannot be obtained with more traditional agents. Benzodiazepines such as
alprazolam and clonazepam are sometimes necessary for short-term control of anxiety
symptoms but are generally discouraged due to sedating side effects, the potential for abuse
or diversion, and gradual tolerance.
Ref: American Psychiatric Association: Diagnostic and Statistical Manual of Mental Disorders, ed 5. American Psychiatric Association,
2013, pp 189-264. 2) Combs H, Markman J: Anxiety disorders in primary care. Med Clin North Am 2014;98(5):1007-1023. 3) Locke
AB, Kirst N, Shultz CG: Diagnosis and management of generalized anxiety disorder and panic disorder in adults. Am Fam Physician
2015;91(9):617-624.

120. A 35-year-old female who is approximately 90 kg (200 lb) above her ideal body weight
comes to you for weight loss recommendations. Her mother, who had a BMI of 37.0 kg/m2,
recently suffered a fatal heart attack and the patient would like to avoid this. She has no other
medical problems except for well-controlled hypertension. Her medication list includes
lisinopril (Prinivil, Zestril), 20 mg daily, and an etonogestrel subdermal (Nexplanon) implant
for contraception.
Which one of the following strategies would be most effective for reducing her cardiac risk?
A) A low-fat diet
B) A high-protein diet
C) Orlistat (Xenical), 120 mg 3 times daily with meals
D) Phentermine (Suprenza), 30 mg daily
E) Referral for bariatric surgery

ANSWER: E

74
Bariatric surgery has been shown to reduce all-cause mortality in patients with morbid
obesity, mostly from reduced myocardial infarctions. Although orlistat and phentermine,
along with other weight loss drugs, have been shown to be associated with moderate weight
loss, there is no evidence that any of these agents reduce morbidity or mortality. A low-
carbohydrate diet has been associated with increased HDL-cholesterol levels and decreased
triglyceride levels when compared to a low-fat diet, which may indicate a reduction in cardiac
risk. No particular diet strategy has been shown to be more effective for weight loss than any
other strategy.
Ref: Sjöström L, Narbro K, Sjöström CD, et al: Effects of bariatric surgery on mortality in Swedish obese subjects. N Engl J Med
2007;357(8):741-752. 2) Shai I, Schwarzfuchs D, Henkin Y, et al: Weight loss with a low-carbohydrate, Mediterranean, or low-fat
diet. N Engl J Med 2008;359(3):229-241. 3) Bazzano LA, Hu T, Reynolds K, et al: Effects of low-carbohydrate and low-fat diets: A
randomized trial. Ann Intern Med 2014;161(5):309-318. 4) Yanovski SZ, Yanovski JA: Long-term drug treatment for obesity: A
systematic and clinical review. JAMA 2014;311(1):74-86.

121. Which one of the following is most likely to improve outcomes in schizophrenia?
A) Combining antipsychotic medication with psychosocial treatment
B) Prescribing two second-generation antipsychotic medications together in small dosages
C) Initial treatment in the outpatient rather than inpatient setting
D) Using only one first-generation, or typical, antipsychotic medication combined with an
antidepressant medication

ANSWER: A
The combination of antipsychotic medication and psychosocial treatments, including
cognitive-behavioral therapy, family therapy, and social skills training, is associated with the
best outcomes in patients with schizophrenia (SOR B). Antipsychotic medications should not
be combined. Hospitalization, especially for the first episode of schizophrenia, is also
recommended for the best outcome (SOR C). Antidepressant medication will treat depression
associated with schizophrenia but will not necessarily improve the symptoms of
schizophrenia.
Ref: Holder SH, Wayhs A: Schizophrenia. Am Fam Physician 2014;90(11):775-782.

122. The preferred method for diagnosing psychogenic nonepileptic seizures


(pseudoseizures) is
A) inducing seizures by suggestion
B) postictal prolactin levels
C) EEG monitoring
D) video-electroencephalography (vEEG) monitoring
E) MRI of the brain

ANSWER: D
Inpatient video-electroencephalography (vEEG) monitoring is the preferred test for the
diagnosis of psychogenic nonepileptic seizures (PNES), and is considered the gold standard

75
(SOR B). Video-EEG monitoring combines extended EEG monitoring with time-locked
video acquisition that allows for analysis of clinical and electrographic features during a
captured event. Many other types of evidence have been used, including the presence or
absence of self-injury and incontinence, the ability to induce seizures by suggestion,
psychologic tests, and ambulatory EEG. While useful in some cases, these alternatives have
been found to be insufficient for the diagnosis of PNES.
Elevated postictal prolactin levels (at least two times the upper limit of normal) have been
used to differentiate generalized and complex partial seizures from PNES but are not reliable
(SOR B). While prolactin levels are often elevated after an epileptic seizure, they do not
always rise, and the timing of measurement is crucial, making this a less sensitive test than
was previously believed. Other serum markers have also been used to help distinguish PNES
from epileptic seizures, including creatine phosphokinase, cortisol, WBC counts, lactate
dehydrogenase, pCO2, and neuron-specific enolase. These also are not reliable, as threshold
levels for abnormality, sensitivity, and specificity have not been determined.
MRI is not reliable because abnormal brain MRIs have been documented in as many as one-
third of patients with PNES. In addition, patients with epileptic seizures often have normal
brain MRIs.
Ref: Alsaadi TM, Marquez AV: Psychogenic nonepileptic seizures. Am Fam Physician 2005;72(5):849-856. 2) Ropper AH, Samuels
MA, Klein JP: Adams and Victor’s Principles of Neurology, ed 10. McGraw-Hill, 2014, p 332. 3) Goldman L, Schafer AI (eds):
Goldman’s Cecil Medicine, ed 25. Elsevier Saunders, 2016, p 2403.

123. A 27-year-old male calls and reports that he has an elevated temperature of 101.5°F
(38.6°C). He underwent a surgical splenectomy following an automobile accident 5 months
ago. He has no allergies and is otherwise healthy.
Which one of the following would be the most appropriate recommendation for this patient?
A) Temperature monitoring with follow-up if it exceeds 39.0°C (102.2°F)
B) Antipyretics to maintain a temperature <39.0°C (102.2°F)
C) Oral antibiotics, with follow-up if the elevated temperature persists
D) Urgent parenteral antibiotic therapy

ANSWER: D
In an asplenic patient, fever might be an initial manifestation of a catastrophic infection and
must be treated immediately with a parenteral antibiotic agent. Quick administration of
antibiotics might prevent sepsis. Mortality can be as high as 50% among patients with
postsplenectomy sepsis. Intravenous or intramuscular ceftriaxone is recommended for
patients who have normal laboratory test results and who do not appear ill. If the patient lives
more than 2 hours from a medical facility that can administer parenteral antibiotic therapy,
oral antibiotics should be given.
Ref: Rubin LG, Schaffner W: Care of the asplenic patient. N Engl J Med 2014;371(4):349-356.

76
124. A 4-year-old female is brought to your office with a 6-day history of fever to a maximum
temperature of 103.2°F. She was seen 2 days ago and treated empirically for streptococcal
pharyngitis because she has a close contact who had a streptococcal infection. She has a mild
cough, rhinorrhea, and a sore throat. On examination she is febrile and fussy. She has a
maculopapular rash over her trunk, bilateral eye injection, shotty bilateral lymphadenopathy,
erythematous lips, a strawberry tongue, and pharyngeal erythema. Her examination is
otherwise unremarkable.
Laboratory Studies
WBCs ………………16,200/mm3 (N 4500–11,000)
Hemoglobin …………10.9 g/dL (N 14.0–17.5)
Hematocrit …………… 32.3% (N 41.0–50.0)
Platelets …………………495,000/mm3 (N 150,000–350,000)
AST (SGOT) ……………78 U/L (N 10–30)
ALT (SGPT) …………… 66 U/L (N 10–40)
Alkaline phosphatase ………165 U/L (N 30–120)
Albumin …………………… 2.8 g/dL (N 3.5–5.5)
C-reactive protein ……………12.9 mg/L (N 0.08–3.1)
Urine dipstick ……………… normal
A full urinalysis with a microscopic examination reveals 20–25 WBCs/hpf, 0–4 RBCs/hpf,
0–4 bacteria/hpf, and 0–10 epithelial cells/hpf. Additional laboratory tests are ordered, with
the following results:
Sodium ……………… 132 mEq/L (N 136–142)
Potassium ………………4.0 mEq/L (N 3.5–5.0)
Chloride …………………104 mEq/L (N 96–106)
Bicarbonate ………………22 mEq/L (N 21–28)
BUN ……………………… 9 mg/dL (N 8–23)
Creatinine …………………0.6 mg/dL (N 0.6–1.2)
Which one of the following would be the most appropriate management of this patient?
A) Supportive measures only
B) Ceftriaxone (Rocephin)
C) Intramuscular penicillin
D) Aspirin and intravenous immunoglobulin
E) Prednisone

ANSWER: D
This child has atypical Kawasaki disease. The diagnostic criteria for classic Kawasaki disease
is fever for at least 5 days and at least four of five principal clinical features. The clinical
features include:
Changes of the oral cavity and lips: cracked erythematous lips, strawberry tongue

77
Polymorphous rash: maculopapular, erythema multiforme–like or scarlitiniform rash,
involving the extremities, trunk, and perineal regions
Bilateral nonpurulent conjunctivitis
Changes in the extremities (erythema of the hands and feet, desquamation of the hands and
toes in weeks 2 and 3)
Cervical lymphadenopathy (>1.5 cm in diameter and generally unilateral)
Some patients do not meet the classic criteria but are labeled as having incomplete or atypical
disease. While Kawasaki disease is generally a clinical diagnosis and there are no specific
diagnostic tests, supplemental laboratory testing can help in the diagnosis of these atypical
cases. The supplemental laboratory criteria include:
Anemia
Cerebrospinal fluid pleocytosis
Elevated C-reactive protein and erythrocyte sedimentation rate
Elevated liver enzymes
Hypoalbuminemia
Hyponatremia
Platelets >450,000/mm3 after 5 weeks
Sterile pyuria
WBCs ≥15,000/mm3
Patients who have a fever for 5 days or more and two or three of the classic criteria should be
treated for atypical Kawasaki’s disease if the C-reactive protein level is elevated and they
have three or more associated laboratory abnormalities. The treatment of choice is IVIG and
high-dose aspirin to reduce the risk of coronary abnormalities. Corticosteroids have been used
as an adjunct in refractory cases or with IVIG, but not alone.
Ref: Saguil A, Fargo M, Grogan S: Diagnosis and management of Kawasaki disease. Am Fam Physician 2015;91(6):365-371.

125. A 22-year-old male presents to the emergency department with a debilitating headache
that occurs 3–4 times per month. Over the past 6 hours he has developed a throbbing, sharp
pain over the right side of his head, with significant nausea and vomiting. Previous headaches
have not lasted more than 1 day. He did not get any relief with ibuprofen, 600 mg taken 2
hours ago.
Which one of the following is the most appropriate next step in the management of this
patient’s headache?
A) Oral acetaminophen
B) Oral butalbital/aspirin/caffeine (Fiorinal)
C) Intravenous hydromorphone (Dilaudid)
D) Intravenous metoclopramide
E) Supplemental oxygen

ANSWER: D

78
This patient has an acute migraine headache that did not respond to NSAIDs. Metoclopramide
is an effective treatment for migraine beyond its antiemetic benefit (SOR B) and intravenous
administration may be helpful for the patient unable to tolerate oral medications.
Acetaminophen has not been proven to be effective for migraine. Opiates and barbiturate-
containing medications should only be used for patients who have failed multiple other
treatments (SOR C). Supplemental oxygen has shown efficacy in the treatment of cluster
headaches. Triptan medications would be another evidence-based choice for abortive therapy
of migraine.
Ref: Gilmore B, Michael M: Treatment of acute migraine headache. Am Fam Physician 2011;83(3):271-280.

126. A 25-year-old female presents to your office to discuss abnormal menstrual periods. She
says that her cycles have always been irregular but she has not had any bleeding in 3 months.
She also says she has gained 20 lb over the past 6 months. She is not taking any medications.
You perform an examination and order laboratory tests. Her blood pressure is 110/72 mm Hg,
heart rate 84 beats/min, respiratory rate 12/min, and weight 78.0 kg (172 lb) with a BMI of
29.5 kg/m2. She is noted to have moderate cystic acne. Her examination is otherwise
unremarkable. A serum hCG measurement is negative and TSH, FSH, and LH levels are
normal. Follow-up laboratory evaluation is significant for a total testosterone level 3 times
the upper limit of normal and a normal 17-hydroxyprogesterone level.
Which one of the following would be most appropriate at this point?
A) An estradiol level
B) A dexamethasone suppression test
C) Karyotyping
D) CT of the abdomen and pelvis
E) MRI of the brain

ANSWER: D
This patient presents with mild symptoms of hyperandrogenism. Her initial laboratory results
rule out pregnancy, thyroid disorders, and primary ovarian failure. The follow-up laboratory
evaluation indicates significant hyperandrogenism. The rapid onset and high testosterone
level suggest an ovarian or adrenal tumor that should be evaluated by abdominal/pelvic
imaging.
MRI of the brain is not helpful in evaluating hyperandrogenism. It would be appropriate in
evaluating hypothalamic and pituitary causes of secondary amenorrhea such as the female
athlete triad or other causes of stress and malnutrition that are associated with weight loss. A
karyotype would be helpful in identifying the cause of primary amenorrhea. An estradiol level
is not helpful in evaluating hyperandrogenism. A dexamethasone suppression test aids in the
diagnosis of Cushing syndrome, which generally does not cause amenorrhea and is usually
associated with stigmata of hypercortisolism, which this patient does not have.

79
Ref: Rotterdam ESHRE/ASRM-Sponsored PCOS Consensus Workshop Group: Revised 2003 consensus on diagnostic criteria and
long-term health risks related to polycystic ovary syndrome. Fertil Steril 2004;81(1):19-25. 2) Azziz R, Carmina E, Dewailly D, et al:
Positions statement: Criteria for defining polycystic ovary syndrome as a predominantly hyperandrogenic syndrome: An Androgen
Excess Society guideline. J Clin Endocrinol Metab 2006;91(11):4237-4245. 3) Klein DA, Poth MA: Amenorrhea: An approach to
diagnosis and management. Am Fam Physician 2013;87(11):781-788.

127. Which one of the following is the most prevalent form of elder abuse?
A) Financial exploitation
B) Verbal abuse
C) Physical abuse
D) Sexual abuse

ANSWER: A
Recent studies suggest that financial exploitation is emerging as the most prevalent form of
elder abuse. By the time cases are detected the older adult’s financial resources have often
been drastically reduced, so early detection and intervention are critical. Financial
exploitation of older adults, which was explored only minimally in previous studies, has
recently been identified as a virtual epidemic and is a problem that may be detected or
suspected by an alert physician.
Ref:Lachs MS, Pillemer KA: Elder abuse. N Engl J Med 2015;373(20):1947-1956.

128. A 45-year-old female complains of debilitating fatigue that seemed to start suddenly 8
months ago, associated with symptoms of a viral infection. She denies symptoms of
depression or substance abuse.
Which one of the following, if present, would indicate a diagnosis other than chronic fatigue
syndrome (myalgic encephalomyelitis)?
A) Impairment of focus and concentration
B) Joint erythema
C) Orthostatic intolerance
D) Post-exertional malaise
E) Tender lymph nodes

ANSWER: B
The diagnosis of myalgic encephalomyelitis/chronic fatigue syndrome (ME/CFS) is made by
determining that a patient’s symptoms are consistent with a standard definition of this disorder
and by ruling out the presence of other systemic diseases that can result in similar symptoms.
Currently suggested criteria for ME/CFS require at least 6 months of pathologic fatigue, along
with a combination of other symptoms including post-exertional fatigue, impaired
focus/concentration, orthostatic intolerance, and unrefreshing sleep. Other symptoms
frequently encountered in patients with this condition include headache, diffuse myalgias,
tender lymph nodes, and gastrointestinal or genitourinary impairments. The diagnosis of
ME/CFS should not be considered in patients with red flag symptoms of a potentially more

80
severe condition, such as chest pain, focal neurologic deficits, joint erythema/swelling,
enlarged lymph nodes, or shortness of breath.
Ref:Yancey JR, Thomas SM: Chronic fatigue syndrome: Diagnosis and treatment. Am Fam Physician 2012;86(8):741-746. 2) IOM
(Institute of Medicine) Committee on the Diagnostic Criteria for Myalgic Encephalitis/Chronic Fatigue Syndrome: Beyond Myalgic
Encephalomyelitis/Chronic Fatigue Syndrome: Redefining an Illness. The National Academies Press, 2015, pp 213-225.

129. A previously healthy 6-month-old female is admitted to the hospital with bronchiolitis.
Which one of the following would be an appropriate treatment for this patient?
A) Albuterol (Proventil, Ventolin)
B) Epinephrine
C) Nebulized hypertonic saline
D) Systemic corticosteroids

ANSWER: C
The American Academy of Pediatrics (AAP) recommends nebulized hypertonic saline for
infants and children hospitalized with bronchiolitis (SOR B). They do not recommend
nebulized hypertonic saline for infants in the emergency department with a diagnosis of
bronchiolitis.
The AAP guideline also recommends that clinicians do not administer albuterol or
epinephrine to infants and children with a diagnosis of bronchiolitis (SOR B). In addition,
clinicians should not administer systemic corticosteroids to infants with a diagnosis of
bronchiolitis in any setting (SOR A).
Ref: Ralston SL, Lieberthal AS, Meissner HC, et al: Clinical practice guideline: The diagnosis, management, and prevention of
bronchiolitis. Pediatrics 2014;134(5):e1474-e1502.

130. A 41-year-old premenopausal female presents to your office with amenorrhea, headache,
and abnormal vision. Visual field testing reveals bitemporal hemianopsia. You order
laboratory tests and MRI of the brain. The MRI reveals a 15-mm mass in the pituitary gland.
Which one of the following laboratory results would suggest an ACTH-secreting adenoma?
A) Elevated LH and FSH
B) Elevated serum prolactin
C) Elevated serum insulin-like growth factor 1
D) Low free T4 with a normal TSH level
E) Elevated 24-hour urine free cortisol

ANSWER: E
Pituitary adenomas are the most common disorder of the pituitary gland and are responsible
for 10%–15% of all intracranial masses. They present with symptoms of hormone secretion
or a neurologic mass effect, or as an incidental finding on CT/MRI. Premenopausal women
often experience amenorrhea, while the most common neurologic symptoms are headache

81
and vision changes (classically bitemporal hemianopsia) due to compression of the optic
chiasm.
Tumors secreting prolactin (lactotrophs) are the most common, comprising 40%–57% of all
pituitary adenomas. Nonsecreting tumors make up 28%–37%, growth hormone–secreting
adenomas (somatotrophs) 11%–13%, and ACTH-secreting adenomas (corticotrophs) 1%–
2%. FSH-, LH-, and TSH-secreting tumors are rare. ACTH-secreting tumors result in an
increase in circulating cortisol, and the diagnosis is confirmed by 24-hour urine cortisol, late-
night salivary cortisol, or overnight dexamethasone suppression testing.
Gonadotrophs cause a resulting elevation of FSH and LH. Lactotrophs increase serum
prolactin, and somatotrophs increase serum insulin-like growth factor. Thyrotrophs result in
normal TSH with low free T4.
Ref: Lake MG, Krook LS, Cruz SV: Pituitary adenomas: An overview. Am Fam Physician 2013;88(5):319-327. 2) Hitzeman N,
Cotton E: Incidentalomas: Initial management. Am Fam Physician 2014;90(11):784-789.

131. The American College of Obstetricians and Gynecologists and the American Academy
of Pediatrics support the advance provision of emergency contraception to sexually active
women. Evidence shows that this practice
A) Decreases pregnancy rates on a population level
B) Decreases the time from unprotected sex to use of emergency contraception
C) Decreases contraception use by the patient prior to sexual activity
D) Increases rates of sexually transmitted infection
E) Increases rates of unprotected intercourse

ANSWER: B
Multiple studies, including randomized, controlled trials, have compared standard access to
emergency contraception (EC) with advance provision in which the patient is given a
prescription for the EC and encouraged to have it filled in order to have it immediately
available in case of unprotected intercourse. In multiple populations in the United States and
Europe advance provision compared to standard access has been shown to increase the rate
of use of EC and to reduce the interval between intercourse and use of EC.
However, no change has been shown for several outcomes when advance provision was
compared to standard access to EC. These include rates of sexually transmitted infections,
unprotected intercourse, use of routine contraceptives prior to sexual activity, and pregnancy
within the population studied.
Ref: American Academy of Pediatrics Committee on Adolescence: Emergency contraception. Pediatrics 2012;130(6):1174-1182. 2)
ACOG Committee Opinion Number 542: Access to emergency contraception. Obstet Gynecol 2012;120(5):1250-1253.

82
132. You see a 90-year-old patient with elevated blood pressure. Which one of the following
is an expected physiologic change in elderly patients?
A) Increased renal blood flow
B) Increased diastolic blood pressure
C) Decreased systolic blood pressure
D) Decreased peripheral resistance
E) Wider pulse pressure

ANSWER: E
Age-related physiologic differences such as wider pulse pressure should be a consideration
when treating hypertension in the elderly. These patients also have lower cardiac output,
higher peripheral resistance, lower intravascular volume, and lower renal blood flow
compared with younger patients. Pulse pressure (the difference between systolic blood
pressure and diastolic blood pressure) is a measure of the degree of age-related vascular
stiffness and is a risk factor for coronary artery disease events.
With increasing age the strongest predictor of coronary artery disease gradually shifts from
diastolic blood pressure to systolic blood pressure, and then to pulse pressure. Systolic blood
pressure rises gradually throughout adult life, whereas diastolic blood pressure peaks and
plateaus in late middle age, and declines slightly thereafter. Diastolic hypertension occurs in
<10% of all patients with hypertension after age 70.
Ref: Aronow WS, Fleg JL, Pepine CJ, et al: ACCF/AHA 2011 expert consensus document on hypertension in the elderly: A report of
the American College of Cardiology Foundation Task Force on Clinical Expert Consensus documents developed in collaboration with
the American Academy of Neurology, American Geriatrics Society, American Society for Preventive Cardiology, American Society
of Hypertension, American Society of Nephrology, Association of Black Cardiologists, and European Society of Hypertension. J Am
Coll Cardiol 2011;57(20):2037-2114. 2) Chaudhry KN, Chavez P, Gasowski J, et al: Hypertension in the elderly: Some practical
considerations. Cleve Clin J Med 2012;79(10):694-704.

133. At a routine health maintenance visit a 60-year-old male complains of urinary frequency.
A review of systems reveals nocturia but no dysuria. He is otherwise healthy. He has smoked
1 pack of cigarettes per day since age 18 and has a history of benign prostatic hyperplasia
(BPH). On examination his prostate is smooth and enlarged without nodules. A dipstick
urinalysis shows 1+ blood but is otherwise negative. Urine microscopy reveals 7 RBCs/hpf
and 2 WBCs/hpf.
Which one of the following would be most appropriate at this point?
A) Reassuring the patient that his BPH is causing microscopic hematuria
B) A repeat urinalysis with microscopy in 6 months
C) Urine cytology
D) BUN and creatinine levels, CT urography, and referral for cystoscopy
E) Empiric antibiotic treatment and a repeat urinalysis after completion of treatment

ANSWER: D

83
Asymptomatic microscopic hematuria is defined by the American Urological Association
(AUA) as ≥3 RBCs/hpf in the absence of an obvious cause such as menstruation, infection,
vigorous exercise, renal disease, trauma, a recent urologic procedure, or a viral illness. Urine
microscopy is required to confirm hematuria found on a dipstick examination.
This patient has risk factors for urothelial cancer, including smoking, his age, and his sex. In
a patient with no obvious cause for hematuria, the AUA does NOT recommend repeating the
urinalysis or treating empirically with antibiotics, as this may delay the diagnosis of cancer.
In addition, assuming that benign prostatic hyperplasia (BPH) is the cause for his hematuria
is inadvisable; patients with BPH usually also have risk factors for malignancy. The
recommended initial workup includes renal function testing, CT urography, and cystoscopy.
Ref: Sharp VJ, Lee DK, Askeland EJ: Urinalysis: Case presentations for the primary care physician. Am Fam Physician
2014;90(8):542-547.

134. A 32-year-old primigravida at 20 weeks gestation presents with a 5-day history of


gradually worsening left calf pain and swelling. She was placed in a walking boot 3 weeks
ago to immobilize a left foot fracture. She feels well otherwise, and she specifically denies
any chest pain, cough, palpitations, dyspnea, fever, chills, easy bruising, or bleeding. Her
examination is notable only for posterior tenderness and swelling of the left calf. A CBC,
prothrombin time, and partial thromboplastin time are all normal. Duplex Doppler
ultrasonography of the left leg is consistent with deep vein thrombosis.
Which one of the following do you recommend at this point?
A) Low molecular weight heparin (Lovenox)
B) Unfractionated heparin
C) Aspirin and clopidogrel (Plavix)
D) Rivaroxaban (Xarelto)

ANSWER: A
Moderate evidence of lower risk from randomized, controlled trials supports the use of low
molecular weight heparin over unfractionated heparin for treatment of venous
thromboembolism (VTE) in pregnancy. Aspirin is not a first-line treatment for VTE, as it
crosses the placenta and has a weak association with miscarriage. Clopidogrel is not indicated
for treatment of VTE. Rivaroxaban and other new non–vitamin K oral anticoagulants are not
recommended because of the lack of data regarding their use in pregnancy. Warfarin crosses
the placenta and is associated with fetal hemorrhage and loss.
Ref: Bates SM, Greer IA, Middeldorp S, et al: VTE, thrombophilia, antithrombotic therapy, and pregnancy: Antithrombotic Therapy
and Prevention of Thrombosis, 9th ed: American College of Chest Physicians Evidence-Based Clinical Practice Guidelines. Chest
2012;141(2 Suppl):e691S-e736S. 2) Marshall AL: Diagnosis, treatment, and prevention of venous thromboembolism in pregnancy.
Postgrad Med 2014;126(7):25-34.

84
135. A 60-year-old male sees you for a follow-up visit for degenerative disc disease that has
been causing mild to moderate pain for 6 months. He has had an extensive workup with no
reversible or worrisome findings. He has been taking acetaminophen for the past 3 months
but remains symptomatic. He asks about herbal supplements that may be beneficial, as he is
reluctant to start NSAIDs or opioid therapy at the present time.
Which one of the following supplements has the most evidence of benefit for this problem?
A) S-adenosyl-L-methionine
B) "-Lipoic acid
C) B-vitamin supplementation
D) Gamma-linolenic acid (evening primrose oil)
E) Glucosamine/chondroitin

ANSWER: A
S-adenosyl-L-methionine is found in all human cells and assists in producing a wide range of
compounds such as cartilage and neurotransmitters. It has been shown to be as effective as
celecoxib in relieving joint pain, but may take up to 2 months to take effect. Glucosamine and
chondroitin have not been shown to be effective. B-complex vitamins may have some benefit
in diabetic and alcoholic neuropathy. "-Lipoic acid and gamma-linolenic acid have also been
shown to improve symptoms of neuropathy.
Ref: Chang K-L, Fillingham R, Hurley RW, Schmidt S: Chronic Pain Management. FP Essentials, no 432, May 2015, pp 21-26. 2)
Onysko M, Legerski P, Potthoff J, Erlandson M: Targeting neuropathic pain: Consider these alternatives. J Fam Pract 2015;64(8):470-
475.

136. An 86-year-old female is in the emergency department with community-acquired


pneumonia confirmed on a chest radiograph. Physical findings include a temperature of
38.4°C (101.1°F), a pulse rate of 101 beats/min, a blood pressure of 101/50 mm Hg, an oxygen
saturation of 90% on room air, and a respiratory rate of 32/min. The patient is awake, alert,
and oriented times three. The physical examination is otherwise unremarkable except for
coarse breath sounds in the left lung base. Laboratory findings include a BUN level of 14
mg/dL (N 8–25), a serum creatinine level of 0.7 mg/dL (N 0.6–1.5), a blood glucose level of
144 mg/dL, and a WBC count of 15,000/mm3 (N 4300–10,800).
Which one of the following would be most appropriate?
A) Discharge to home and treatment with azithromycin (Zithromax)
B) Discharge to home and treatment with amoxicillin/clavulanate (Augmentin)
C) Discharge to home and treatment with amoxicillin/clavulanate plus azithromycin
D) Hospital admission and treatment with amoxicillin/clavulanate
E) Hospital admission and treatment with ceftriaxone (Rocephin) plus azithromycin

ANSWER: E
This patient requires hospitalization based on her CURB-65 score of 3 (age >65, diastolic
blood pressure <60 mm Hg, respiratory rate >30/min, BUN <19 mg/dL, no confusion), which

85
places her mortality risk at 14%. Although azithromycin has been associated with an increased
risk of myocardial infarction in elderly patients hospitalized with community-acquired
pneumonia, the combination of azithromycin with a β-lactam has been associated with
decreased mortality in this population. Azithromycin alone is acceptable treatment in the
outpatient setting, but not when the patient requires hospitalization. A macrolide plus a β-
lactam antibiotic has been shown to have a lower 30-day mortality rate than a β-lactam alone.
This combination also results in a greater proportion of hospitalized patients achieving clinical
stability at 7 days (defined as stable vital signs and oxygen saturation >90% on room air)
when compared with β-lactam therapy alone.
Ref: Aujesky D, Auble TE, Yealy DM, et al: Prospective comparison of three validated prediction rules for prognosis in community-
acquired pneumonia. Am J Med 2005;118(4):384-392. 2) Rodrigo C, Mckeever TM, Woodhead M, Lim WS; British Thoracic Society:
Single versus combination antibiotic therapy in adults hospitalized with community acquired pneumonia. Thorax 2013;68(5):493-495.
3) Garin N, Genné D, Carballo S, et al: β-Lactam monotherapy vs β-lactammacrolide combination treatment in moderately severe
community-acquired pneumonia: A randomized noninferiority trial. JAMA Intern Med 2014;174(12):1894-1901. 4) Mortensen EM,
Halm EA, Pugh MJ, et al: Association of azithromycin with mortality and cardiovascular events among older patients hospitalized with
pneumonia. JAMA 2014;311(21):2199-2208.

137. A 55-year-old female who works at a local day care center presents with a severe cough.
Her illness began 10 days ago with malaise, a low-grade fever, rhinorrhea, tearing, and a mild
cough. Over the past 3 days the cough has become much more severe and she coughs to the
point that vomiting is induced.
Which one of the following is most likely to lead to a definitive diagnosis?
A) A Gram stain and culture of sputum
B) Serologic studies
C) Polymerase chain reaction testing
D) A chest radiograph
E) Office pre- and post-bronchodilator spirometry

ANSWER: C
Pertussis has been increasing in incidence. The initial presentation usually involves
nonspecific symptoms including malaise, lacrimation, and rhinorrhea, which is referred to as
the catarrhal stage. The following stage, known as the paroxysmal stage, is manifested by
severe coughing that may lead to the characteristic high-pitched whooping sound when the
patient tries to catch his or her breath. Coughing to the point of emesis is also characteristic
of pertussis, and the cough can be severe enough to actually result in rib fractures. The cough
may last several weeks before it begins to wane during the convalescent phase, which usually
lasts 2–3 weeks.
Pertussis can be diagnosed clinically if there is a coughing illness of 2 weeks’ duration with
one of the classic signs of pertussis (post-tussive emesis, respiratory whoop, or paroxysmal
cough) and there is no other apparent cause. The CDC also recommends the use of both
cultures and polymerase chain reaction testing to confirm the diagnosis. Culture is not the
best choice, however, as it is often done improperly, and culture results may not be available

86
for several days. Polymerase chain reaction has the advantage of providing results in 1–2
days. It has good specificity and the sensitivity is much higher than that of a culture; its
sensitivity is highest during the first 2 weeks of symptoms.
Treatment with antibiotics usually does not improve clinical symptoms, although it does
decrease transmission. The CDC continues to recommend antibiotics for pertussis. Either
azithromycin or clarithromycin is currently recommended.
Ref: Kline JM, Lewis WD, Smith EA, et al: Pertussis: A reemerging infection. Am Fam Physician 2013;88(8):507-514

138. A mother presents to your office with her newborn son for a well child examination. She
states that he is healthy and she has no current concerns.
Which one of the following is appropriate in the evaluation of this child for developmental
dysplasia of the hip?
A) Limited hip abduction assessment
B) Barlow and Ortolani maneuvers
C) Measurement of leg length
D) Hip ultrasonography

ANSWER: B
Screening all infants for developmental dysplasia of the hip (DDH) has been a mainstay of
care for many years. DDH is an abnormality of the acetabulum or femoral head and their
congruence, presenting in infancy. The prevalence of instability on examination ranges from
1.6–28.5 per 1000 births. While many screening measures may be helpful in identifying DDH,
none has been found to improve long-term clinical outcomes (SOR C). The U.S. Preventive
Services Task Force found insufficient evidence to recommend routine screening to prevent
poor outcomes. However, the American Academy of Pediatrics and the Pediatric Orthopedic
Society of North America both recommend that the physical examination of all newborns
include screening for DDH. There are no studies recommending the abandonment of this
screening.
Ortolani (reducing a dislocated hip) and Barlow (dislocating an unstable hip) maneuvers are
commonly performed early in infancy. By 2–3 months of age these are less useful and limited
hip abduction assessment is more acceptable (SOR C). A clunk denotes a positive finding.
Leg length measurement is useful in the evaluation of a child presenting with a limp but is
not indicated in DDH.
More than 50% of patients with positive physical findings have been found to have normal
hips within 1 month on follow-up ultrasonography. Universal ultrasonography resulted in a
higher rate of detected DDH with subsequent treatment but did not reduce the need for
surgery. Most of these abnormalities resolve spontaneously, and ultrasonography should not
be used as a universal screening measure (SOR C). Plain films are inappropriate in infancy
because the hip is primarily cartilaginous and the associated radiation exposure is
unacceptable. It may be useful in older children to track progress after treatment.

87
Ref: Ewald E, Kiesel E: Screening for developmental dysplasia of the hip in newborns. Am Fam Physician 2013;87(1):10-11. 2) Lewis
ML: A comprehensive newborn examination: Part II. Skin, trunk, extremities, neurologic. Am Fam Physician 2014;90(5):297-302. 3)
Jackson JC, Runge MM, Nye NS: Common questions about developmental dysplasia of the hip. Am Fam Physician 2014;90(12):843-
850. 4) Naranje S, Kelly DM, Sawyer JR: A systematic approach to the evaluation of a limping child. Am Fam Physician
2015;92(10):908-918.

139. A 6-month-old male is brought to your office for a well child check. He has been healthy
except for mild eczema that resolved with about 7 days of a medium-potency corticosteroid
when he was 4 months old. He is the only child of a single mother. The mother has introduced
some solid foods including eggs, cheese, and bananas, and her son has tolerated these without
any trouble. However, she is concerned about introducing peanuts into her son’s diet. The
mother’s brother suffers from a severe peanut allergy and was recently hospitalized after an
accidental peanut ingestion. The mother herself does not eat nuts regularly because of her
brother’s allergy but has had peanuts in small amounts without any trouble.
Which one of the following would you recommend regarding the introduction of foods
containing peanuts and other nuts into this child’s diet?
A) Begin introducing these foods before 12 months of age
B) Start introducing these foods once he has reached 12 months of age
C) Wait until at least 5 years of age to introduce these foods
D) Introduce these foods only after allergy testing of the child is performed
E) Introduce these foods only after allergy testing of the mother is performed

ANSWER: A
The American Academy of Pediatrics now recommends early introduction of peanut-
containing products for most children to reduce the incidence of peanut allergy. For children
at high risk for an allergic reaction (e.g., those with severe eczema or a first degree relative
with peanut allergy) allergy testing can be considered first. For this child, it should be
recommended that the mother introduce peanut-containing products soon. There is no
evidence that testing a parent prior to introducing the child to peanut-containing products is
beneficial.
Ref: Fleischer DM, Sicherer S, Greenhawt M, et al: Consensus communication on early peanut introduction and the prevention of
peanut allergy in high-risk infants. Pediatrics 2015;136(3):600-604.

140. Which one of the following is the strongest modifiable risk factor for abdominal aortic
aneurysm?
A) Cigarette smoking
B) Excessive alcohol consumption
C) Hyperlipidemia
D) Hypertension
E) Type 2 diabetes mellitus

ANSWER: A

88
Smoking is the strongest modifiable risk factor for the development of an abdominal aortic
aneurysm. Nonmodifiable risk factors include older age, male sex, and a family history of the
problem. Other less prominent risk factors include hypertension, an elevated cholesterol level,
obesity, and preexisting atherosclerotic occlusive disease.
Ref: Kent KC: Abdominal aortic aneurysms. N Engl J Med 2014;371(22):2101-2108.

141. Which one of the following antidepressants can prolong the QT interval and should be
avoided with concomitant QT-prolonging agents such as atypical antipsychotics?
A) Bupropion (Wellbutrin)
B) Citalopram (Celexa)
C) Mirtazapine (Remeron)
D) Sertraline (Zoloft)
E) Venlafaxine

ANSWER: B
Prolongation of the QT interval is an important medication adverse effect to consider. This is
particularly true in patients taking multiple medications, because this effect can be additive
and increases the risk of life-threatening arrhythmias such as torsades de pointes. Among
commonly used antidepressants, citalopram and escitalopram may prolong the QT interval.
Other SSRIs, as well as bupropion, venlafaxine, and mirtazapine, do not have this effect. Both
tricyclic antidepressants and antipsychotics, commonly used in patients also taking SSRIs,
can cause QT prolongation, making their combined use problematic.
Ref: Muench J, Hamer AM: Adverse effects of antipsychotic medications. Am Fam Physician 2010;81(5):617-622. 2) Price AL,
Marzani-Nissen GR: Bipolar disorders: A review. Am Fam Physician 2012;85(5):483-493. 3) Kovich H, DeJong A: Common
questions about the pharmacologic management of depression in adults. Am Fam Physician 2015;92(2):94-100.

142. A healthy 50-year-old male consults you about preparations for a business trip to India.
He will be traveling in rural areas at times and has read that he should have antibiotics with
him in case of traveler’s diarrhea.
Which one of the following would be best for you to prescribe?
A) Trimethoprim/sulfamethoxazole (Bactrim)
B) Azithromycin (Zithromax)
C) Ciprofloxacin (Cipro)
D) Rifampin (Rifadin)
E) Doxycycline

ANSWER: B
Antibiotics shorten the course of moderate to severe traveler’s diarrhea. Azithromycin is
recommended as self-treatment for moderate to severe traveler’s diarrhea in South and
Southeast Asia, where Campylobacter species are a more common cause of the illness than
anywhere else. Campylobacter species are resistant to fluoroquinolones. Ciprofloxacin is

89
recommended for travel to South and Central America and to Africa. When symptoms are
mild (1–3 loose bowel movements per 24 hours without limiting activities), traveler’s diarrhea
can be treated with loperamide or bismuth subsalicylate. Rifampin and doxycycline are not
commonly prescribed for presumptive treatment of traveler’s diarrhea.
Ref: Steffen R, Hill DR, DuPont HL: Traveler’s diarrhea: A clinical review. JAMA 2015;313(1):71-80. 2) Freedman DO, Chen LH,
Kozarsky PE: Medical considerations before international travel. N Engl J Med 2016;375(3):247-260.

143. A 49-year-old female presents for recheck of an elevated serum calcium level. She
complains of constipation and reports that her brother has hypercalcemia. The remainder of
her review of systems, physical examination, and past medical, surgical, family, and social
histories is unremarkable. She takes no medications.
Six months ago the patient’s calcium level was 10.4 mg/dL (N 8.5–10.2) and a serum
parathyroid hormone (PTH) level was 58 pg/mL (N 9–77).
Repeat laboratory studies today reveal the following:
Calcium ………………………… 10.6 mg/dL
Serum PTH ………………………66 pg/mL
24-hour urine calcium …………… 205 mg/24 hour (N 100–300)
Creatinine ……………………………0.8 mg/dL (N 0.4–1.1),
Estimated…………………………….GFR 62 mL/min/1.73 m2
Serum albumin …………………… 4.7 g/dL (N 3.2–5.2)
25-hydroxyvitamin D ……………… 32 ng/mL (N 30–80)
Which one of the following is the most likely diagnosis?
A) Familial hypocalciuric hypercalcemia
B) Primary hyperparathyroidism
C) Secondary hyperparathyroidism
D) Secondary vitamin D deficiency
E) Renal insufficiency

ANSWER: B
Secondary hyperparathyroidism is most likely due to low vitamin D intake or low serum
vitamin D, often in the setting of renal disease. However, the serum vitamin D level and
estimated glomerular filtration rates are normal in this patient. This rules out secondary
hyperparathyroidism, as well as vitamin D deficiency and renal insufficiency. Primary
hyperparathyroidism is more likely in the presence of hypercalcemia and inappropriately
normal or high parathyroid hormone levels. Familial hypocalciuric hypercalcemia is ruled out
by a normal 24-hour urine calcium level.
Ref: Marcocci C, Cetani F: Primary hyperparathyroidism. N Engl J Med 2011;365(25):2389-2397. 2) Michels TC, Kelly KM:
Parathyroid disorders. Am Fam Physician 2013;88(4):249-257.

90
144. You are seated directly across from a patient who has very limited English proficiency.
The interpreter you have arranged for enters the examination room and waits next to the door.
In order to best facilitate the interview, you request that the interpreter
A) Maintain her current position by the door
B) Sit next to or slightly behind your patient
C) Sit next to you
D) Stand directly behind you
E) Switch places with you

ANSWER: B
In order to facilitate the most effective interview, the interpreter should be as inconspicuous
as possible. This is best achieved by having the interpreter seated next to or slightly behind
the patient.
Ref: Juckett G, Unger K: Appropriate use of medical interpreters. Am Fam Physician 2014;90(7):476-480.

145. A 6-month-old uncircumcised male is brought to your clinic by his parents, who are
concerned because his foreskin remains “tight” and cannot be retracted when they bathe him.
He has never been treated for a urinary tract infection or balanitis and his parents report he
has a good urine stream. On examination the skin at the preputial outlet appears healthy with
no scarring.
Which one of the following is the most appropriate intervention?
A) Reassurance and continued routine foreskin hygiene
B) A short course of a topical corticosteroid twice daily to the foreskin
C) Prophylactic topical antibacterial ointment twice daily
D) Forceful retraction of the foreskin twice daily to reduce adhesions
E) Referral to a pediatric urologist for circumcision

ANSWER: A
Phimosis is the inability of the foreskin (prepuce) to retract over the glans and it can be
physiologic or pathologic. Nonretractile foreskin is very common in young boys, and is seen
in up to 10% of uncircumcised 3-year-old boys. This physiologic phimosis is part of normal
development and over time the foreskin will become retractile due to intermittent erections
and keratinization of the inner foreskin. Pathologic phimosis is due to distal scarring and on
examination typically appears as a white, contracted fibrotic ring around the preputial outlet.
Pathologic phimosis, painful erections with a tight foreskin, recurrent bouts of balanitis, and
recurrent urinary tract infections in conjunction with phimosis are indications for urologic
consultation and consideration of circumcision. This child’s examination is consistent with
physiologic phimosis.
A short course of a topical corticosteroid (2–8 weeks of 0.05% betamethasone twice daily)
applied to the preputial outlet may result in accelerated resolution of physiologic phimosis.

91
However, given this child’s age and the absence of complications, reassurance and continued
good foreskin hygiene are recommended. The foreskin should not be forcibly retracted, as
this may lead to microtears and resultant scarring. In the absence of infection, neither
antibacterial nor antifungal ointment is indicated.
Ref: McGregor TB, Pike JG, Leonard MP: Pathologic and physiologic phimosis: Approach to the phimotic foreskin. Can Fam
Physician 2007;53(3):445-448. 2) Drake T, Rustom J, Davies M: Phimosis in childhood. BMJ 2013;346:f3678.

146. A 35-year-old female presents to your office with a 3-month history of dyspnea. She
does not smoke and has not had a productive cough. She has no other significant past medical
history and takes no medications. A chest radiograph reveals significant hilar adenopathy with
bilateral infiltrates. Which one of the following physical examination findings would be
consistent with the most likely diagnosis in this patient?
A) Bilateral conjunctivitis
B) Alopecia
C) Erythema nodosum
D) A malar rash
E) Xerostomia

ANSWER: C
This patient has sarcoidosis. Extrapulmonary manifestations are common in patients with
sarcoidosis and erythema nodosum is a common cutaneous sign. Ocular symptoms usually
include uveitis, not conjunctivitis. Xerostomia is associated with Sjögren’s syndrome. A
malar rash and alopecia are dermatologic findings associated with lupus erythematosus.
Ref: Soto-Gomez N, Peters JI, Nambiar AM: Diagnosis and management of sarcoidosis. Am Fam Physician 2016;93(10):840-848.

147. An 83-year-old male with moderate Alzheimer’s disease and a BMI of 32.6 kg/m2 is
admitted to the nursing home. He has significant osteoarthritis and has difficulty with
ambulation. In recent years he has become increasingly frail and has frequently fallen in his
home. He has no history of coronary artery disease or stroke, but has had type 2 diabetes
mellitus for the past 15 years.
Which one of the following goals would be most appropriate for managing this patient’s
diabetes?
A) A hemoglobin A1c of approximately 8%–9%
B) Limiting caloric intake to 1600 kcal/day
C) An LDL-cholesterol level <100 mg/dL
D) A systolic blood pressure <130 mm Hg

ANSWER: A
The treatment of diabetes mellitus in frail elderly patients, especially nursing home residents,
can be less stringent than with other patients. Sliding-scale insulin and diabetic diets should
both be avoided in nursing home residents. Lowering LDL-cholesterol levels and aggressive

92
blood pressure control are not indicated for frail elderly patients. The acceptable levels of
hemoglobin A1c can also be liberalized, with levels of 8%–9% being acceptable.
Ref: Sinclair A, Morley JE: How to manage diabetes mellitus in older persons in the 21st century: Applying these principles to long
term diabetes care. J Am Med Dir Assoc 2013;14(11):777-780.

148. A 66-year-old male is diagnosed with monoclonal gammopathy of undetermined


significance. This patient will require regular follow-up visits because of the risk his condition
will progress to:
A) aplastic anemia
B) multiple myeloma
C) chronic lymphocytic leukemia
D) acute myelogenous leukemia
E) idiopathic thrombocytopenic purpura

ANSWER: B
Monoclonal gammopathy of undetermined significance (MGUS) is present in approximately
2%–3% of the white population older than 50. It is associated with a risk of progression to
multiple myeloma at a rate of 1% per year. Most patients diagnosed with MGUS should be
reevaluated in 6 months with a medical history, physical examination, CBC, calcium and
creatinine levels, and serum electrophoresis, and then annually thereafter.
Ref: Rajkumar SV, Kyle RA: Protein electrophoresis and immunofixation for the diagnosis of monoclonal gammopathies. JAMA
2014;312(20):2160-2161.

149. A 59-year-old female with hypertension takes hydrochlorothiazide and amlodipine


(Norvasc). A routine basic metabolic panel is normal except for a mildly elevated calcium
level. Upon further questioning, she admits to a history of kidney stones in the past. After
stopping her hydrochlorothiazide, which one of the following laboratory evaluations would
be most appropriate?
A) Parathyroid hormone (PTH) and PTH–related peptide
B) 25-hydroxyvitamin D, magnesium, and creatinine
C) 24-hour urine calcium and creatinine
D) A repeat basic metabolic panel with ionized calcium
E) Sestamibi scintigraphy

ANSWER: D
Isolated elevated calcium levels should be confirmed before pursuing further testing. After
calcium elevation is confirmed, immediate treatment should be undertaken if hypercalcemia
is severe. Otherwise, a history and physical examination would be appropriate, as well as 25-
hydroxyvitamin D, magnesium, creatinine, and PTH levels. In patients with a normal or
elevated PTH level, 24-hour urine calcium and creatinine levels can help to differentiate
between primary hyperparathyroidism and familial hypocalciuric hypercalcemia. If the PTH

93
level is low, 25-hydroxyvitamin D, 1,25-dihydroxyvitamin D, and parathyroid hormone–
related peptide levels should be checked to evaluate possible causes of hypercalcemia
independent of the parathyroid. Sestamibi scintigraphy is indicated only after confirmation of
hyperparathyroidism, and typically in anticipation of surgical treatment.
Ref: Michels TC, Kelly KM: Parathyroid disorders. Am Fam Physician 2013;88(4):249-257.

150. A 10-year-old female is brought to your clinic complaining of toe pain after a playground
injury. A radiograph reveals a displaced fracture involving half the joint surface of the first
proximal phalanx.
Which one of the following is the most appropriate management of this fracture?
A) Buddy taping to the second phalanx for 6 weeks
B) A rigid-sole shoe for 6 weeks
C) A short leg walking cast with a toe plate for 6 weeks
D) A posterior splint with orthopedic referral at 1 week
E) Prompt orthopedic referral for surgical fixation

ANSWER: E
Phalangeal fractures of the second to fifth toes can usually be managed with buddy taping and
a rigid-sole shoe for 3 weeks followed by buddy taping for another 3 weeks. If the fracture is
significantly displaced, closed reduction in the office using local anesthesia may be
appropriate.
However, because of its importance in weight bearing and balance, fractures of the first toe
(hallux) have a higher potential for negative outcomes. Specifically, fractures of the hallux
can be managed nonsurgically if they are not displaced and involve less than 25% of the
articular surface of the joint. Nonsurgical management involves use of a short leg walking
cast with a toe plate (extending past the end of the great toe) for 3 weeks and then progression
to a rigid-sole shoe with buddy taping after that.
Since this patient’s fracture involves more than 25% of the articular surface of the joint of the
first toe, early referral for surgical pinning by an orthopedic surgeon would lead to the best
outcome.
Ref: Marx JA, Hockberger RS, Walls RM (eds): Rosen’s Emergency Medicine: Concepts and Clinical Practice, ed 8. Elsevier Saunders,
2014, pp 723-750. 2) Bica D, Sprouse RA, Armen J: Diagnosis and management of common foot fractures. Am Fam Physician
2016;93(3):183-191.

94
151. An 85-year-old female with hypertension and chronic kidney disease was admitted to
the hospital for a total knee replacement. The next evening she becomes agitated and
disoriented to place and time.
Which one of the following interventions would be most appropriate in this patient’s
management?
A) Haloperidol
B) Zolpidem (Ambien)
C) Avoiding opioids
D) Removing hearing aids
E) Early mobilization

ANSWER: E
Sedative hypnotics such as zolpidem are associated with increased delirium symptoms and
should not be used as first-line therapy (SOR C). Behavioral interventions such as early and
recurrent mobilization, continuous observation, improving sensory perception (such as with
hearing aids), initial reorientation, and removal of unnecessary irritations such as intravenous
lines and catheters have been shown to decrease symptoms of delirium (SOR B). While
opioids can affect mental status, untreated pain is also associated with delirium, and opioids
are likely indicated in this postoperative patient. Antipsychotic medications such as
haloperidol, as well as physical restraints, should be used as last resorts and only after an
appropriate medical evaluation (SOR A).
Ref: Kalish VB, Gillham JE, Unwin BK: Delirium in older persons: Evaluation and management. Am Fam Physician 2014;90(3):150-
158.

152. You are evaluating a 7-year-old male for long-standing nighttime bed wetting. He has
no daytime symptoms and no other health problems. A thorough physical examination is
normal.
Which one of the following would be most appropriate at this point?
A) Urinalysis alone
B) Urinalysis and a serum metabolic panel
C) Urinalysis and a lumbosacral radiograph
D) Urinalysis and renal ultrasonography

ANSWER: A
This child has monosymptomatic enuresis, meaning there are no other symptoms except
nighttime bed wetting. His physical examination is normal. In this setting, the recommended
initial workup is a urinalysis alone. If the results are normal and there are no other symptoms
suggestive of underlying behavioral or medical conditions, reassurance and instruction on
possible behavioral interventions is appropriate without further evaluation.
Ref: Baird DC, Seehausen DA, Bode DV: Enuresis in children: A case-based approach. Am Fam Physician 2014;90(8):560-568.

95
153. Which one of the following is the only intervention shown to be effective in the
prevention and treatment of pressure ulcers?
A) Vitamin C supplements
B) Protein supplements
C) Topical collagen dressings
D) Dextranomer paste applied to wounds
E) Negative-pressure wound therapy

ANSWER: B
The American College of Physicians recently published clinical practice guidelines for the
treatment of pressure ulcers. Many modalities were evaluated, but good studies and evidence
were available only for a few treatments. Several good-quality studies indicate that protein
supplementation improves wound healing and also appears to have some benefit in prevention
as well. Vitamin C supplementation made no difference in wound healing. Dextranomer paste
applied to wounds actually worsened wound healing compared to standard wound dressings.
Topical collagen dressings and negative-pressure wound therapy yielded no improvement in
wound healing compared to usual care.
Ref: Qaseem A, Humphrey LL, Forciea MA, et al: Treatment of pressure ulcers: A clinical practice guideline from the American
College of Physicians. Ann Intern Med 2015;162(5):370-379.

154. A healthy 30-year-old male presents to your office with a 2-cm abscess on his leg. There
is no surrounding erythema and he is afebrile. You incise and drain the abscess.
Which one of the following would be the most appropriate management for this patient?
A) Observation only
B) Amoxicillin/clavulanate (Augmentin)
C) Cephalexin (Keflex)
D) Clindamycin (Cleocin)
E) Trimethoprim/sulfamethoxazole (Bactrim)

ANSWER: A
A simple abscess with no overlying cellulitis that is treated with incision and drainage does
not require antibiotics. If there were overlying cellulitis, antibiotic use would be appropriate.
Purulent infections have a higher likelihood of being caused by MRSA, so coverage with
trimethoprim/sulfamethoxazole or clindamycin would be appropriate. In nonpurulent skin
infections, amoxicillin/clavulanate or cephalexin would be appropriate choices.
Ref: Ramakrishnan K, Salinas RC, Agudelo Higuita NI: Skin and soft tissue infections. Am Fam Physician 2015;92(6):474-483.

96
155. Which one of the following is most effective for treating fibromyalgia?
A) Tricyclic antidepressants
B) SSRIs
C) NSAIDs
D) Opioid analgesics
E) Corticosteroids

ANSWER: A
Pain in fibromyalgia is thought to be due to a centralized pain state. Medications that reduce
the activity of neurotransmitters or increase the activity of inhibitory neurotransmitters such
as norepinephrine and serotonin work best, and tricyclic antidepressants appear to be most
effective (SOR A). Older SSRIs have limited benefit. Oral analgesics only work in about one-
third of patients, and opioids may increase hyperalgesia. Corticosteroids are ineffective.
Ref: Clauw DJ: Fibromyalgia: A clinical review. JAMA 2014;311(15):1547-1555.

156. An 84-year-old female sees you for a follow-up visit for cardiovascular issues, including
a previous myocardial infarction. Her current medication regimen consists of Lisinopril
(Prinivil, Zestril) and carvedilol (Coreg). While she is not dyspneic at rest, she now becomes
short of breath after walking half a block. On examination her blood pressure is 122/74 mm
Hg, pulse rate 72 beats/min, respirations 18/min, and oxygen saturation 97% on room air. She
has no jugular venous distention and her lungs are clear. No edema is noted. A recent
echocardiogram showed a left ventricular ejection fraction of 30%.
Adding which one of the following would help to decrease both mortality and the risk of
hospitalization?
A) Digoxin
B) Furosemide
C) Isosorbide dinitrate/hydralazine (BiDil)
D) Losartan (Cozaar)
E) Spironolactone (Aldactone)

ANSWER: E
Systolic heart failure, or heart failure with a reduced left ventricular ejection fraction (<40%),
should be managed with an ACE inhibitor and a β-blocker. If there is volume overload, a
diuretic should be added. For those who cannot tolerate an ACE inhibitor, an angiotensin
receptor blocker can be used. However, an ACE inhibitor should not be given with an
angiotensin receptor blocker, as this combination increases mortality.
Digoxin will decrease symptoms and lessen the chance of hospitalization, but mortality is not
reduced. Mortality and hospitalization can be reduced by the addition of an aldosterone
antagonist such as spironolactone. In African-American patients, using a combination of

97
hydralazine and isosorbide dinitrate improves both symptoms and mortality for those with
New York Heart Association class III or IV heart failure with a reduced ejection fraction.
Ref: Drugs for chronic heart failure. JAMA 2015;313(10):1052-1053.

157. In a healthy full-term infant who is exclusively breastfed, iron supplementation should
begin at what age in order to prevent iron deficiency anemia?
A) 1 month
B) 4 months
C) 6 months
D) 9 months
E) 12 months

ANSWER: B
Healthy full-term infants receive 60%–80% of their iron stores from their mothers during the
third trimester of pregnancy. Thus, even an exclusively breastfed infant will not typically be
at risk of developing iron deficiency anemia until the age of 4 months. Iron supplementation
should then be started and continued until the child is eating foods containing sufficient
dietary iron. Preterm babies who are exclusively breastfed should begin iron supplementation
at 1 month of age due to a foreshortened third trimester.
Ref: Wang M: Iron deficiency and other types of anemia in infants and children. Am Fam Physician 2016;93(4):270-278

158. A previously healthy 46-year-old female presents with a 1-month history of leg swelling
and weight gain. She denies chest pain and shortness of breath. She takes no medications, has
never smoked, and drinks only an occasional glass of wine.
The physical examination is notable for 2+ pitting edema to the mid-thigh on both legs, and
mild ascites. The remainder of her examination is normal. A dipstick urinalysis shows 3+
protein. You suspect primary nephrotic syndrome.
In addition to proteinuria and low albumin, which one of the following is the most common
finding in patients with primary nephrotic syndrome?
A) Coagulopathy
B) Hypercalcemia
C) Hyperglycemia
D) Hyperlipidemia
E) Infection

ANSWER: D
Nephrotic syndrome is the triad of edema, proteinuria, and hypoalbuminemia, and it usually
is also associated with hyperlipidemia. The initial evaluation of this patient should include
testing for hyperlipidemia. Nephrotic syndrome is associated with an increased risk of deep
vein thrombosis but does not cause a coagulopathy. Diabetes mellitus can be a secondary
cause of nephrotic syndrome but is not a finding in primary nephrotic syndrome. Relative

98
hypocalcemia may be found in patients with nephrotic syndrome due to low protein, but
hypercalcemia is not associated with nephrotic syndrome. Children and patients with relapses
of nephrotic syndrome have an increased risk of infection.
Ref: Kodner C: Diagnosis and management of nephrotic syndrome in adults. Am Fam Physician 2016;93(6):479-485

159. A 62-year-old male with type 2 diabetes mellitus, hypertension, and


hypercholesterolemia is found to have a low serum vitamin B12 level. Which one of his
current medications is most likely to cause this?
A) Aspirin
B) Enalapril (Vasotec)
C) Metformin (Glucophage)
D) Simvastatin (Zocor)
E) Hydrochlorothiazide

ANSWER: C
Low vitamin B12 is seen with prolonged use of metformin, H2-blockers, and proton pump
inhibitors. The mechanism is unclear, and it is not known whether prophylactic
supplementation prevents deficiency. Metformin reduces absorption of vitamin B12 in 30%
of patients and reduces vitamin B12 concentrations in 5%–10% of those taking it. The dosage
and duration of metformin therapy is correlated with the risk of vitamin B12 deficiency. Given
this risk, it seems prudent to monitor vitamin B12 levels periodically in patients taking
metformin.
Ref: Langan RC, Zawistoski KJ: Update on vitamin B12 deficiency. Am Fam Physician 2011;83(12):1425-1430.

160. A 25-year-old gravida 1 para 0 presents for follow-up of gestational diabetes mellitus.
She had a positive screening test at her 24-week visit last week and has been checking her
blood glucose at home twice daily since that time. She is not currently on any diabetes
medications and has a nutrition visit scheduled for later today. She is currently unclear on
what her glucose level should be at various times throughout the day.
Which one of the following is the goal fasting blood glucose in this patient?
A) ≤ 75 mg/dL
B) ≤ 95 mg/dL
C) ≤ 120 mg/dL
D) ≤ 140 mg/dL
E) ≤ 180 mg/dL

ANSWER: B
Clinicians who care for pregnant women need to be familiar with the diagnosis and
monitoring parameters for gestational diabetes mellitus, as these help to determine the need
for management strategies outside of diet and exercise. The goal is ≤ 95 mg/dL for fasting

99
blood glucose, ≤140 mg/dL for 1-hour postprandial glucose, and ≤120 mg/dL for 2-hour
postprandial glucose.
Ref: Garrison A: Screening, diagnosis, and management of gestational diabetes mellitus. Am Fam Physician 2015;91(7):460-467.

161. Which one of the following is the strongest risk factor for primary hypertension in
children and adolescents?
A) A family history of hypertension
B) Elevated BMI
C) Male sex
D) African-American ethnicity

ANSWER: B
There is an increasing prevalence of hypertension in children and adolescents. BMI is the
strongest risk factor for hypertension in this age group. Other risk factors include a family
history of hypertension, low birth weight, male sex, and certain ethnic backgrounds. The
prevalence of hypertension in obese children is 11%, which is more than double that of the
general pediatric population (reported at 1%–5%).
Ref: Moyer VA; US Preventive Services Task Force: Screening for primary hypertension in children and adolescents: US Preventive
Services Task Force recommendation statement. Pediatrics 2013;132(5):907-914.

162. A 75-year-old female presents with a 6-week history of the gradual onset of pain behind
her right knee. She has a previous history of osteoarthritis and mild hypertension, but no other
significant problems. Examination of the knee reveals a soft mass in the medial popliteal fossa
that is present when the knee is extended, but is much smaller when the knee is flexed to 45°,
which also causes the pain to subside.
Which one of the following would be most appropriate to diagnose this problem?
A) A plain radiograph
B) MRI
C) Ultrasonography
D) Arteriography
E) No imaging

ANSWER: E
Popliteal cysts cause posterior knee pain, and are thought to be a result of increased intra-
articular pressure forcing fluid into the bursa and causing expansion and subsequent pain.
Risk factors include rheumatoid arthritis or osteoarthritis and advanced age. With the patient
lying down, a mass can be palpated in the medial popliteal fossa while the knee is extended.
When the knee is flexed to 45° the pain subsides and the mass is reduced in size or disappears.
Imaging is generally not needed for the diagnosis. Plain radiographs are obtained if there is a
possible fracture. Ultrasonography can be used if deep vein thrombosis is a consideration. If

100
significant internal derangement of the knee is suspected, MRI may be useful. An arteriogram
would be an option if aneurysm of the popliteal artery were suspected.
Treatment is generally conservative, although addressing the underlying knee pathology will
prevent the cyst from recurring. Pain is often decreased by keeping the knee in flexion and
the use of ice and NSAIDs. If this is not successful, intra-articular corticosteroids or cyst
aspiration is often effective.
Ref: Covey CJ, Hawks MK: Nontraumatic knee pain: A diagnostic & treatment guide. J Fam Pract 2014;63(12):720-728.

163. Which one of the following screening measures does the U.S. Preventive Services Task
Force recommend against (USPSTF D recommendation)?
A) Abdominal ultrasonography to evaluate for an abdominal aortic aneurysm in a 65-year-
old male smoker
B) BRCA testing in a high-risk 40-year-old female
C) A Papanicolaou test in a 58-year-old female who previously had an abdominal
hysterectomy for fibroids
D) Colonoscopy for a 70-year-old male with no history of rectal bleeding

ANSWER: C
Overscreening has become increasingly common. This inappropriate care costs the health
care system while providing no benefit and possibly causing harm to patients. All of the listed
screenings are appropriate with the exception of cervical cancer screening for a patient who
has had a hysterectomy. The U.S. Preventive Services Task Force recommends against
screening for cervical cancer in women over 65 who have had adequate recent screening, and
in women who have had a hysterectomy for benign disease (SOR A).
Ref: Ebell M, Herzstein J: Improving quality by doing less: Overscreening. Am Fam Physician 2015;91(1):22-24.

164. A 52-year-old male was admitted to the hospital with evidence of an acute myocardial
infarction. He underwent cardiac catheterization and percutaneous coronary intervention with
placement of two drug-eluting stents. Echocardiography revealed a left ventricular ejection
fraction of 30%. By the time of his discharge the patient had been started on several new
medications, including aspirin.
Which one of the following medications is more useful for symptom control than for
improving mortality in this situation?
A) Atorvastatin (Lipitor)
B) Clopidogrel (Plavix)
C) Lisinopril (Prinivil, Zestril)
D) Metoprolol tartrate (Lopressor)
E) Nitroglycerin

ANSWER: E

101
Evidence-based guidelines for the treatment of patients with acute coronary syndrome support
several medications in the subacute period. Dual antiplatelet therapy, such as clopidogrel
combined with aspirin, has been shown to reduce cardiovascular mortality (SOR B). ACE
inhibitors and statins should be initiated immediately after a myocardial infarction and
continued indefinitely to reduce mortality and the risk of repeat infarction (SOR A). β-
Blockers have been shown to improve mortality in patients with a left ventricular ejection
fraction <40% (SOR A). Nitroglycerin is often used to manage angina but has no
demonstrated mortality benefit.
Ref: Mercado MG, Smith DK, McConnon ML: Myocardial infarction: Management of the subacute period. Am Fam Physician
2013;88(9):581-588.

165. A 24-year-old female sees you for follow-up of her chronic abdominal pain. You have
diagnosed her with somatization disorder. You have scheduled regular, frequent visits and
she has been in counseling for a few months. She still has significant anxiety about her
symptoms and you would like to start her on a medication.
Which one of the following medications would be most appropriate for this patient?
A) Amitriptyline
B) Aripiprazole (Abilify)
C) Bupropion (Wellbutrin)
D) Carbamazepine (Tegretol)
E) Lamotrigine (Lamictal)

ANSWER: A
Somatic disorders usually require a multifaceted approach to treatment. It is important to
schedule regular visits at short intervals to establish a collaborative relationship with the
patient. It is also important to limit diagnostic testing and reassure the patient that serious
diseases have already been ruled out, and to screen for other mental illnesses. Referral for
counseling using cognitive-behavioral therapy and mindfulness-based therapies is also
effective. SSRIs and tricyclic antidepressants have been found to be the most effective
pharmacotherapy (SOR B) for somatic symptom disorders. Amitripityline is the most studied
tricyclic antidepressant, and trials have shown that it has a greater likelihood of success
compared to SSRIs. Other antidepressants, anticonvulsants, and antipsychotic medications
are ineffective and should be avoided (SOR B).
Ref: Kurlansik SL, Maffei MS: Somatic symptom disorder. Am Fam Physician 2016;93(1):49-54.

102
166. A 27-year-old male complains of severe back and left leg pain that started yesterday
afternoon. He states that just before the pain began he was moving some patio furniture around
his back porch when he felt a sudden “pop” in his left lumbar region. Over the subsequent
few hours he developed gradually increasing pain in his low back that radiates down the
posterolateral side of his left leg and onto the top of his left foot. He describes the pain as
lancinating and knife-like, rating it as 10/10 on a pain scale. He is unable to stand up straight
or sit down comfortably for an examination. He denies any bowel or bladder dysfunction and
appears to have otherwise intact strength and sensation. Prior to this event he was in good
health and took no routine medications.
Which one of the following is recommended at this time?
A) No imaging
B) Radiographs of the lumbar spine only
C) CT of the lumbar spine without contrast
D) MRI of the lumbar spine without contrast
E) MRI of the lumbar spine with contrast

ANSWER: A
This case can be best described as acute lumbar pain with radiculopathy. There are no red
flags that would suggest the presence of a time-sensitive condition such as cauda equina
syndrome, cancer, infection, or fracture. In cases like this, consensus agreements recommend
against any imaging studies. Instead, a 6-week course of pain medication, manipulation,
physical therapy, and self-care should be undertaken.
Ref: Chou R, Qaseem A, Snow V, et al: Diagnosis and treatment of low back pain: A joint clinical practice guideline from the American
College of Physicians and the American Pain Society. Ann Intern Med 2007;147(7):478-491. 2) Patel ND, Broderick DF, Burns J, et
al; Expert Panel on Neurologic Imaging: ACR Appropriateness Criteria: Low Back Pain. American College of Radiology, 2015.

167. The son of a 76-year-old female with metastatic cancer calls you stating that the patient
is having uncontrollable nausea. She takes long-acting morphine for pain and he has already
tried giving her promethazine and ondansetron (Zofran) without significant benefit. She has
been able to drink fluids, but feels “full” and has not eaten more than a few bites of food each
day. She has not been vomiting and her urine output has been normal. However, the son
reports that she has not had a bowel movement in about 5 days, which he assumes is due to
her low intake of food.
Which one of the following would be most appropriate at this point?
A) Increasing the dosage of oral morphine
B) Initiating subcutaneous hydration
C) Assessing for possible fecal impaction
D) Inserting a nasogastric tube
E) Suggesting oral or inhaled marijuana

ANSWER: C

103
Nausea is a common symptom near the end of life and can have many causes. One of the first
objectives in the palliative care of nausea is to identify and address the underlying cause. This
patient is taking a long-acting opiate that predisposes her to constipation. Furthermore, the
family seems to accept the fact that she is not having any bowel movements because she is
not eating very much, which is an incorrect understanding. There are no signs of dehydration
to warrant subcutaneous hydration, nor are there signs of bowel obstruction that would
warrant a nasogastric tube.
Ref: Walsh TD, Caraceni AT, Fainsinger R, et al (eds): Palliative Medicine. Saunders Elsevier, 2009, pp 921-930. 2) Clary PL, Lawson
P: Pharmacologic pearls for end-of-life care. Am Fam Physician 2009;79(12):1059-1065. 3) Rakel RE, Rakel DP (eds): Textbook of
Family Medicine, ed 9. Elsevier Saunders, 2016, pp 61-62.

168. A 90-year-old female is being considered for transcatheter aortic valve replacement due
to severely symptomatic critical aortic stenosis. In addition to a medication review, which one
of the following tests would be best to assess for frailty as a component of the preoperative
assessment?
A) Gait speed
B) The Geriatric Depression Scale
C) The Confusion Assessment Method (CAM)
D) The Mini-Cognitive Assessment Instrument (Mini-Cog)

ANSWER: A
Frailty is an age-related, multidimensional state of decreased physiologic reserves. Frail
patients are at increased risk of decline as a result of illness or stressors such as surgery. The
definition and assessment of frailty are still not uniformly agreed upon, but gait speed is
recognized as a highly reliable single measurement tool. The other options listed are useful
components of a geriatric assessment but are not used as a single test for the evaluation of
frailty.
Ref: Elsawy B, Higgins KE: The geriatric assessment. Am Fam Physician 2011;83(1):48-56. 2) Robinson TN, Walston JD, Brummel
NE, et al: Frailty for surgeons: Review of a National Institute on Aging Conference on Frailty for Specialists. J Am Coll Surg
2015;221(6):1083-1092.

169. Which one of the following topical antibiotics used in the treatment of acne vulgaris is
not known to increase bacterial resistance?
A) Benzoyl peroxide
B) Clindamycin (Cleocin)
C) Dapsone (Aczone)
D) Erythromycin
E) Metronidazole

ANSWER: A
Benzoyl peroxide is the only topical antibiotic without evidence to suggest that its use
contributes to bacterial antimicrobial resistance. For this reason it is recommended as first-

104
line therapy in mild to moderate inflammatory acne and as an adjunctive agent with other
topical antibiotics that can induce bacterial resistance, such as clindamycin and erythromycin.
Dapsone is an antibiotic but is thought to primarily improve acne via its anti-inflammatory
effect. Metronidazole is used chiefly in acne rosacea rather than acne vulgaris and it may
contribute to antimicrobial resistance.
Ref: Titus S, Hodge J: Diagnosis and treatment of acne. Am Fam Physician 2012;86(8):734-740.

170. Which one of the following therapies is most likely to reduce abdominal cramping in a
patient with irritable bowel syndrome?
A) Peppermint oil
B) Polyethylene glycol (MiraLAX)
C) Probiotics
D) Wheat bran

ANSWER: A
Peppermint oil is an antispasmodic that has moderate evidence of benefit in the relief of
abdominal cramping in patients with irritable bowel syndrome (IBS). Probiotics as a group
have some evidence of benefit in improving global IBS symptoms, but the studies have used
different probiotic preparations so it is difficult to draw definite conclusions about their
effectiveness or to recommend a specific preparation. Polyethylene glycol is an osmotic
laxative that has evidence of benefit for bowel symptoms, including improvement in stool
frequency and consistency, but it does not improve abdominal pain. Wheat bran is an
insoluble fiber that contains fructans that can exacerbate IBS symptoms and should be
avoided in patients with IBS.
Ref: Chey WD, Kurlander J, Eswaran S: Irritable bowel syndrome: A clinical review. JAMA 2015;313(9):949-958.

171. A 4-year-old male is brought to your office by his neighbor with a 1-day history of fever,
lethargy, stiff neck, and rash. You suspect meningitis. The neighbor reports that the patient’s
father is on his way to the office and should arrive within 30 minutes.
Which one of the following would be most appropriate in this situation?
A) Initiate treatment now
B) Obtain informed consent to treat from the neighbor
C) Call the father to obtain informed consent to treat
D) Wait until the father arrives to obtain informed consent to treat

ANSWER: A
The American Academy of Pediatrics recommends that medical care for children with an
urgent or emergent condition never be withheld or delayed because of problems obtaining
consent. All efforts to obtain consent should be documented. In the case of a 4-year-old male
who is suspected to have meningitis, even a brief delay in treatment could result in a bad
outcome for the patient.

105
Ref: Committee on Pediatric Emergency Medicine and Committee on Bioethics: Consent for emergency medical services for children
and adolescents. Pediatrics 2011;128(2):427-433.

172. A 35-year-old female is concerned about a persistent rash on her face. It waxes and
wanes in severity but she has persistent redness over her forehead, nose, cheeks, and chin.
She also reports an occasional flush that usually lasts less than 5 minutes, sometimes spreads
to her neck and chest, and is accompanied by a sensation of warmth. In addition to redness
and flushing, the affected areas often contain papules, pustules, and small red streaks. Your
examination confirms erythema over her centrofacial features with accompanying
papulopustular lesions on her nose and telangiectasia on her chin and nose.
Which one of the following agents is an FDA-approved first-line topical therapy for this
patient’s condition?
A) Benzoyl peroxide
B) Clindamycin (Cleocin)
C) Metronidazole
D) Selenium sulfide
E) Tretinoin

ANSWER: C
This patient presents with the classic symptoms and signs of rosacea. FDA-approved first-
line agents for treating rosacea include metronidazole 0.75% lotion, 0.75% cream, and 1%
gel; azelaic acid 15% gel; sulfacetamide 10%/sulfur 5% cream, foam, lotion, and suspension;
brimonidine 0.33% gel; and ivermectin 1% cream (SOR A).
Ref: Oge LK, Muncie HL, Phillips-Savoy AR: Rosacea: Diagnosis and treatment. Am Fam Physician 2015;92(3):187-196.

173. A 19-year-old female college student is referred to you by her dentist for a medical
evaluation related to dental problems. The patient states that she feels well and exercises at
least 2–3 hours every day. On examination her teeth are stained and there are enamel erosions.
She has nontender enlargement of both parotid glands. The remainder of the examination is
normal.
Which one of the following is associated with these findings?
A) Bacterial parotitis
B) Bulimia nervosa
C) Cat scratch disease
D) Mononucleosis
E) Mumps

ANSWER: B
Bulimia nervosa is an eating disorder associated with recurrent binge eating and induced
vomiting. There is also a history of excessive physical activity in many cases. Because of
vomiting, the teeth are stained and there is destruction of the enamel from stomach acid.
Parotid swelling is also noted frequently. With mumps and bacterial parotitis the patient feels

106
sick and parotid glands are tender. Cat scratch disease and mononucleosis affect lymph
glands.
Ref: Harrington BC, Jimerson M, Haxton C, Jimerson DC: Initial evaluation, diagnosis, and treatment of anorexia nervosa and bulimia
nervosa. Am Fam Physician 2015;91(1):46-52.

174. A drug available for over 20 years has been noted to have beneficial effects for a common
disease. Because it is a cheap, generically available drug, no pharmaceutical firm is willing
to fund a clinical trial. What type of study would be best suited to evaluate this treatment?
A) A double-blind crossover study
B) A meta-analysis
C) A non-inferiority trial
D) A case-control study
E) A single-blind trial

ANSWER: C
A new treatment may not be more effective than available treatments but it may have lower
costs, fewer adverse effects, or greater convenience compared to standard treatment. A
treatment with other advantages might be preferred to standard treatment for some patients.
A non-inferiority trial rigorously evaluates a treatment against accepted and effective
treatments, with the goal of demonstrating that it is at least almost as good. Single- and
double-blind studies (with or without crossover of the control and experimental groups) can
be done to demonstrate the drug is effective, but showing that a new treatment is not inferior
to standard treatments can be rigorously demonstrated by a non-inferiority trial. A non-
inferiority trial does not distinguish between a new treatment that is not inferior and one that
is truly superior, and cannot demonstrate equivalence. A case-control trial is not appropriate
to compare one medication to another; its retrospective design is better for evaluating risk
factors or exposures as possible causes or contributors to a disease or illness, as it involves no
intervention in either the control or the experimental group.
Ref: Kaji AH, Lewis RJ: Noninferiority trials: Is a new treatment almost as effective as another? JAMA 2015;313(23):2371-2372.

175. While working yesterday, a 53-year-old roofer stepped on a nail that went through his
shoe and penetrated the sole of his foot. He irrigated his foot thoroughly with water when he
got home. He comes to your office today because he has developed some erythema and pain
in his foot, with a small amount of drainage. While Staphylococcus aureus and Streptococcus
species are the most common causes of skin and soft-tissue infection, which organism should
also be considered in this situation?
A) Corynebacterium
B) Klebsiella
C) Pasteurella multocida
D) Pseudomonas aeruginosa
E) Vibrio vulnificus

107
ANSWER: D
Streptococcus species and Staphylococcus aureus are the most common causes of skin and
soft-tissue infections. With a puncture wound caused by an object penetrating the sole of the
shoe, another organism that should be considered as a possible cause of the infection is
Pseudomonas aeruginosa. Klebsiella is not a common cause of these infections. Pasteurella
multocida is a common cause of infection from a cat bite, and can also be seen in dog bites.
Corynebacterium is also associated with infections from animal bites. Vibrio vulnificus
should be considered when there is a skin infection after exposure to salt water.
Ref: Marx JA, Hockberger RS, Walls RM (eds): Rosen’s Emergency Medicine: Concepts and Clinical Practice, ed 8. Elsevier Saunders,
2014, pp 1851-1863. 2) Ramakrishnan K, Salinas RC, Agudelo Higuita NI: Skin and soft tissue infections. Am Fam Physician
2015;92(6):474-483.

176. A newborn female is found to have constricted ear deformity (cup ears). Her mother had
gestational diabetes mellitus.
Which one of the following studies would be most appropriate for this patient?
A) A single-view chest radiograph
B) Long bone radiographs
C) Echocardiography
D) Renal ultrasonography
E) Unenhanced head CT

ANSWER: D
If isolated ear anomalies such as cup ears, preauricular skin tags, or ear pits are associated
with maternal gestational diabetes mellitus, ultrasonography is warranted to screen for renal
abnormalities. Ear malformations are also associated with renal abnormalities in children with
other malformations, dysmorphic features, a family history of deafness, or teratogenic
exposures (SOR C).
Ref: Lewis ML: A comprehensive newborn exam: Part I. General, head and neck, cardiopulmonary. Am Fam Physician
2014;90(5):289-296.

177. The U.S. Preventive Services Task Force recommends routine screening for which one
of the following in adolescents?
A) Alcohol use
B) Cervical cancer
C) Chlamydia infection
D) Depression
E) Scoliosis

ANSWER: D
According to the U.S. Preventive Services Task Force, screening for major depression is
recommended if adequate treatment and follow-up can be provided. Sexually active females

108
should be screened for Chlamydia infection. Cervical cancer screening begins at age 21.
Because of undue harm from unnecessary treatment, scoliosis screening is not recommended.
There is insufficient evidence to recommend for or against routine screening for alcohol use
(SOR A, B).
Ref: Ham P, Allen C: Adolescent health screening and counseling. Am Fam Physician 2012;86(12):1109-1116.

178. In an adult patient with significant depression and no other health problems, which one
of the following is the best initial choice for pharmacotherapy?
A) Amitriptyline
B) Duloxetine (Cymbalta)
C) Fluoxetine (Prozac)
D) Mirtazapine (Remeron)
E) Trazodone (Oleptro)

ANSWER: C
Depression can be treated with counseling and/or medication. The choice of medication
depends on many factors including side effects, dosing schedule, cost, patient preference, and
comorbidities. In a patient with significant depression and no comorbidities, a second-
generation SSRI should be the initial medication choice. Fluoxetine is the only SSRI choice
listed.
Duloxetine is an SNRI that may lead to sweating, tachycardia, urinary retention, and elevation
of blood pressure. It is indicated if the patient has fibromyalgia. Trazodone is associated with
somnolence, orthostatic hypotension, and priapism. It may be used in low doses as an adjuvant
in patients with insomnia. Mirtazapine can cause sedation, weight gain, increased appetite,
dry mouth, dizziness, and constipation. It is helpful for patients with significant insomnia and
loss of appetite. Amitriptyline is a tricyclic antidepressant and has significant anticholinergic
side effects. It is also associated with conduction abnormalities that can lead to arrhythmias.
Ref: Drugs for psychiatric disorders. Treat Guidel Med Lett 2013;11(130):53-64. 2) Kovich H, DeJong A: Common questions about
the pharmacologic management of depression in adults. Am Fam Physician 2015;92(2):94-100.

179. A 29-year-old male presents with a 3-week history of multiple small, brownish-red
macules in his left inguinal fold that are now coalescing into larger macules. The rash does
not itch, and he has not used any new bath or laundry products. Wood’s lamp illumination of
the area reveals a reddish fluorescence.
Which one of the following topical treatments would be most appropriate for this patient’s
rash?
A) Erythromycin
B) Hydrocortisone
C) Nystatin
D) Selenium sulfide
E) Zinc oxide

109
ANSWER: A
An intertriginous rash of brownish-red macules that often coalesce is characteristic of a
bacterial infection caused by Corynebacterium minutissimum known as cutaneous
erythrasma. Coral-red fluorescence under a Wood’s lamp confirms the diagnosis. The best
treatment for erythrasma is oral or topical erythromycin, with the oral form being most
effective. Topical clindamycin and antibacterial soaps may also be of some benefit.
Ref: Kalra MG, Higgins KE, Kinney BS: Intertrigo and secondary skin infections. Am Fam Physician 2014;89(7):569-573.

180. Which one of the following is routinely required to establish a diagnosis of chronic
lymphocytic leukemia?
A) A lymph node biopsy
B) A bone marrow biopsy and aspirate
C) CT of the chest, abdomen, and pelvis
D) Flow cytometry and immunophenotyping of peripheral blood
E) Weight loss of more than 10%, unexplained fever, and night sweats

ANSWER: D
Flow cytometry and immunophenotyping of peripheral blood demonstrate clonality of the
circulating lymphocytes in chronic lymphocytic leukemia (CLL). Absolute lymphocytosis is
defined as >5000 cells/mm3.
Ninety percent of CLL patients are asymptomatic at the time of diagnosis. Imaging procedures
are not necessary for establishing the diagnosis. While a bone marrow biopsy or aspiration
and/or a lymph node biopsy may be necessary as the disease progresses, they are not routinely
needed to establish a CLL diagnosis.
Ref: Nabhan C, Rosen ST: Chronic lymphocytic leukemia: A clinical review. JAMA 2014;312(21):2265-2276.

181. A 43-year-old male presents to your office for an urgent visit because he has a feeling
that he is being followed and is fearful for his life. He tells you someone is listening to his
cell phone conversations and has stolen files from his laptop, and he has observed “shadowy
figures” watching him. He reports that the police have done nothing to protect him, and he
has considered hiring a private investigator. He reports that he has been living out of his car
for the past month. Upon further questioning the patient admits that he lost his job as an
accountant a year ago and is estranged from his family. His overall appearance is unkempt
and his speech is pressured and rapid. His heart rate is 88 beats/min, blood pressure 138/80
mm Hg, and temperature 37.0°C (98.6°F). In ruling out medical causes for his psychosis,
which one of the following would be most useful?
A) The PHQ-9 questionnaire
B) Urine toxicology
C) MRI of the brain
D) HIV antibody testing
E) A serum calcium level

110
ANSWER: B
Illicit substance use is the most common medical cause or secondary cause of acute psychosis.
Conditions diagnosed by brain imaging, blood chemistry, and HIV tests (intracranial mass,
paraneoplastic syndrome, and HIV infection, respectively) are less common medical causes
of acute psychosis. Major depression with psychosis would be considered a psychiatric or
primary cause of psychosis.
Ref: Griswold KS, Del Regno PA, Berger RC: Recognition and differential diagnosis of psychosis in primary care. Am Fam Physician
2015;91(12):856-863.

182. A 62-year-old male with a history of smoking comes to your office for evaluation of a
solitary 2-cm right upper lobe pulmonary nodule noted on CT. Which one of the following
radiologic features is most predictive of malignancy?
A) Smooth borders
B) A ground-glass appearance
C) Central calcification of the nodule
D) A solid density

ANSWER: B
In the evaluation of a solitary pulmonary nodule, a risk assessment is used to determine the
diagnostic evaluation of the nodule. Clinical risk factors for malignancy include older age, a
history of smoking, and a previous history of cancer. Radiologic features indicative of
malignancy include a nodule diameter >10 mm, an upper lung location, irregular or spiculated
borders, no calcifications, a ground-glass appearance, and increasing size over time.
Radiologic characteristics suggesting benign disease include a size <5 mm, central
calcifications, smooth borders, a solid appearance, and a stable size over 1 year.
Ref: Kikano GE, Fabien A, Schilz R: Evaluation of the solitary pulmonary nodule. Am Fam Physician 2015;92(12):1084-1091.

183. A 13-year-old male is brought to the emergency department because of acute worsening
of his asthma. His routine medications are inhaled fluticasone (Flovent) and an albuterol
metered-dose inhaler (Proventil, Ventolin), which he has used every hour for the past 3 hours
without significant improvement. He is unable to talk in complete sentences and his peak
expiratory flow is 200 L/min, down from a baseline of 480 L/min. He has no previous history
of intubations and only one short hospital admission.
Which one of the following should be given to reduce the likelihood of hospitalization?
A) Cromolyn
B) Epinephrine via nebulizer
C) Inhaled fluticasone
D) Montelukast (Singulair)
E) Oral prednisone

ANSWER: E

111
Rapid administration of systemic corticosteroids can reduce the frequency of hospitalization
in patients with severe asthma who present with an exacerbation (SOR A). There is no clear
benefit from parenteral versus oral administration (SOR B). The same benefit is not seen with
inhaled corticosteroids during an exacerbation. Montelukast and cromolyn are not appropriate
treatments for an asthma exacerbation. Nebulized epinephrine is used in the treatment of
croup.
Ref: Pollart SM, Compton RM, Elward KS: Management of acute asthma exacerbations. Am Fam Physician 2011;84(1):40-47.

184. A 52-year-old gravida 3 para 3 comes to your office with concerns about symptoms
related to menopause. Her last menstrual period was 18 months ago. Her main concern is a 1-
year history of chronic vaginal dryness, irritation, and painful intercourse. She has occasional
mild hot flashes that are minimally bothersome to her. Her obstetric and gynecologic history
is unremarkable. On examination you note thin, pale tissue around the introitus, and dry
vaginal mucosa.
Which one of the following treatments is safest and most effective for managing her
condition?
A) Oral estradiol (Estrace)
B) Oral raloxifene (Evista)
C) Oral conjugated estrogens/medroxyprogesterone (Prempro)
D) Vaginal estradiol (Estring)
E) Vaginal etonogestrel/ethinyl estradiol (NuvaRing)

ANSWER: D
The safest and most effective treatment for postmenopausal atrophic vaginitis is local estrogen
such as the estradiol vaginal ring, estradiol vaginal tablets, or estrogen cream. All of these
therapies are effective, and while there is a theoretical risk of small amounts of systemic
absorption, there is no evidence to suggest that there is an increased risk of endometrial cancer
or thromboembolic disease related to this type of estrogen use. These therapies are therefore
preferred over systemic estrogens such as estradiol or a combination of conjugated estrogens
and medroxyprogesterone. Raloxifene, a synthetic estrogen agonist/antagonist, has not been
found to be effective for the treatment of atrophic vaginitis. The vaginal ring is approved for
use as a contraceptive and is not intended for use in postmenopausal patients.
Ref: ACOG Practice Bulletin No. 141: Management of menopausal symptoms. Obstet Gynecol 2014;123(1):202-216

185. A newborn male is brought to your office for his first examination after hospital
discharge. He has been diagnosed with Down syndrome.
Which one of the following is indicated for all newborns with Down syndrome?
A) A cervical spine film
B) An echocardiogram
C) Polysomnography
D) Ultrasonography of both hips

112
ANSWER: B
Down syndrome is the most common chromosomal abnormality. The 5-year survival rate is
>90%, with most individuals living past 60 years of age, and there has been significant
progress in improving quality of life for these patients. Infants with Down syndrome present
with a wide variety of symptoms and signs, however, and there are certain health
complications that can increase the risk of hospitalization.
All infants with Down syndrome should be screened for cardiac, feeding, vision, hearing,
thyroid, and hematologic abnormalities. Up to 50% manifest congenital cardiac defects, and
a cardiology consultation and echocardiography are warranted. Hypotonia may affect feeding
and should be noted for assessment. Congenital cataracts are common, so a red reflex
examination is imperative. Cervical spine films, hip ultrasonography, and polysomnography
are not recommended in a newborn with Down syndrome. However, all children with Down
syndrome should have polysomnography between the ages of 1 and 4 years. Due to concerns
about atlantoaxial instability parents should be advised to monitor for any signs of spinal cord
impingement.
Screening for subclinical thyroid disease should also be performed, as well as a CBC to
evaluate for transient myeloproliferative disorder, leukemoid reaction, and polycythemia,
which all occur more frequently in Down syndrome patients.
It is important to congratulate the parents on their newborn and carefully explain the rationale
for each recommended test and discuss results and their meaning clearly. An Individual
Family Service Plan is also recommended, with a review every 6 months to evaluate specific
measures indicated for each age group.
Ref: Bunt CW, Bunt SK: Role of the family physician in the care of children with Down syndrome. Am Fam Physician
2014;90(12):851-858.

186. One of your patients is admitted to the hospital with an acute myocardial infarction and
requires a drug-eluting stent in his left circumflex artery. Which one of the following
antiplatelet regimens would be most appropriate for at least the next 6–12 months?
A) Clopidogrel (Plavix) alone
B) Clopidogrel plus aspirin, 81 mg daily
C) Aspirin alone, 325 mg daily
D) Aspirin/dipyridamole (Aggrenox)
E) Prasugrel (Effient) plus aspirin, 325 mg daily

ANSWER: B
Evidence strongly supports the use of dual antiplatelet therapy with aspirin and a P2Y12
inhibitor such as clopidogrel for a minimum of 6–12 months in patients with a drug-eluting
stent. Aspirin at a dosage of 81 mg is as effective as 325 mg, if not more effective. It is also
associated with a lower bleeding risk. Prasugrel is a second-line antiplatelet agent and can be
used when there is a contraindication to clopidogrel or if the patient is resistant to clopidogrel.
When it is used, it should be used with an aspirin dosage of 81 mg daily. Aspirin/dipyridamole

113
is effective for secondary stroke prevention but does not have a role in the treatment of
ischemic cardiovascular disease.
Ref: Montalescot G, Brieger D, Dalby AJ, et al: Duration of dual antiplatelet therapy after coronary stenting: A review of the evidence.
J Am Coll Cardiol 2015;66(7):832-847.

187. In hypothyroid patients, which one of the following makes TSH levels alone unreliable
for assessing the response to thyroid replacement?
A) A history of total surgical thyroidectomy
B) Treatment with desiccated thyroid USP
C) Treatment with liothyronine (T3)
D) Hypothyroidism due to pituitary disease
E) Graves disease

ANSWER: D
Secondary centrally mediated hypothyroidism is caused by hypothalamic or pituitary disease
in which TSH is not being produced, resulting in failure to stimulate the thyroid gland to make
thyroid hormone. Standard treatment for this type of hypothyroidism is levothyroxine, but
determining the response to therapy requires measuring free T4 rather than TSH. Desiccated
thyroid USP is a combination of T3 and T4 and response to treatment is usually assessed with
TSH levels. T3 levels can fluctuate in patients treated with desiccated thyroid, as well as those
who are on liothyronine (T3), and are therefore generally not used. TSH is also used to
monitor treatment in patients with surgical hypothyroidism. Graves disease is an autoimmune
disorder commonly causing hyperthyroidism, but patients usually become hypothyroid after
treatment. TSH is used to monitor these patients as well.
Ref: Drugs for hypothyroidism. Med Lett Drugs Ther 2015;57(1480):147-150.

188. A 65-year-old male is admitted to the hospital with diastolic heart failure and stage 3
chronic kidney disease. He is noted to have a confirmed serum potassium level of 6.8 mEq/L
(N 3.6–5.0).
Which one of the following is the best initial management to reduce his potassium level?
A) Intravenous calcium gluconate solution
B) Intravenous furosemide
C) Intravenous insulin and glucose
D) A rectal sodium polystyrene sulfonate (Kayexalate) retention enema
E) Hemodialysis

ANSWER: C
This patient has severe hyperkalemia and needs urgent treatment. Intravenous insulin
followed by glucose will shift potassium intracellularly and is an effective treatment for
severe hyperkalemia (SOR B). Sodium polystyrene sulfonate is not recommended as an
urgent treatment and would not be an initial treatment for severe hyperkalemia. Intravenous

114
calcium gluconate solution does not lower serum potassium but is indicated to prevent
arrhythmias in patients with hyperkalemia and EKG changes. Intravenous furosemide is not
a treatment for hyperkalemia, although hypokalemia is a common side effect. Hemodialysis
is a treatment for severe hyperkalemia but is not considered a first-line treatment.
Ref: Viera AJ, Wouk N: Potassium disorders: Hypokalemia and hyperkalemia. Am Fam Physician 2015;92(6):487-495.

189. A 2-year-old male is brought to the emergency department by his frightened mother
following the sudden onset of nasal stuffiness and a harsh, barking, nonproductive cough. The
child does not appear significantly distressed. His temperature is 37.9°C (100.2°F) orally.
Mild nasal flaring is present. The nasopharyngeal mucus appears mildly edematous and
injected, and upper airway noises are heard, but good air movement is evident on auscultation
of his chest.
Which one of the following is the most appropriate treatment for this patient?
A) A bedside humidifier
B) A single dose of oral dexamethasone
C) Amoxicillin for 7 days
D) Nebulizer treatment with racemic epinephrine
E) Tracheal intubation and oxygen administration

ANSWER: B
Croup is a common, self-limited illness caused by viral infection of the upper respiratory tract.
The diagnosis is based primarily on the clinical history and examination findings. A history
of the abrupt onset of a barking cough, inspiratory stridor, and hoarseness in a 2-year-old child
is typical of croup, although it can present at any age between 6 months and 12 years. Low-
grade fever, a barking cough, and varying degrees of respiratory distress (nasal flaring,
retractions, or stridor) are typically present on examination. Findings such as a toxic
appearance, wheezing, drooling, and difficulty swallowing are not consistent with the
diagnosis.
Treatment of mild disease with a corticosteroid has proven benefit (SOR A) even when
administered as a single oral dose (SOR B). Nebulized epinephrine has been shown to
improve outcomes in children with moderate to severe croup (SOR A). Humidification
therapy in the emergency department setting provides no benefit (SOR A). This child has mild
croup and a single dose of dexamethasone (0.15–0.60 mg/kg, usually given orally) followed
by close observation is the most appropriate treatment.
Ref: Zoorob R, Sidani M, Murray J: Croup: An overview. Am Fam Physician 2011;83(9):1067-1073.

115
190. Which one of the following outcomes is seen with intensive glycemic control in patients
with type 2 diabetes mellitus?
A) Fewer hypoglycemic events
B) Improved blood pressure control
C) A delay in the development of macroalbuminuria
D) A reduced need for dialysis
E) Lower all-cause mortality rates

ANSWER: C
Intensive glycemic control in type 2 diabetes mellitus results in a reduced rate of progression
to macroalbuminuria, which is an indicator of the development of diabetic nephropathy.
Intensive glycemic control does not reduce the likelihood of dialysis, nor does it improve
blood pressure control. Intensive control does increase the incidence of hypoglycemic events,
which is particularly hazardous in the elderly. As was demonstrated in the Action to Control
Cardiovascular Risk in Diabetes (ACCORD) trial, intensive glycemic control actually
increases all-cause mortality.
Ref: Roett MA, Liegl S, Jabbarpour Y: Diabetic nephropathy—The family physician’s role. Am Fam Physician 2012;85(9):883-889.

191. A 61-year-old truck driver is admitted to your service through the emergency department
in the evening because of possible cellulitis. When you examine him in the morning he has
blistering along his left forearm and dorsal hand that has not receded from the lines drawn in
the emergency department. It is not erythematous or tender. He does not recall any injuries,
insect bites, infections, or fever, and has been afebrile since admission. He denies pain in his
arm, but does complain of itching.
Laboratory studies reveal a normal CBC and comprehensive metabolic panel, with the
exception of an ALT (SGPT) level of 100 U/L (N 7–35) and an AST (SGOT) level of 75 U/L
(N 10–34). The patient reports no significant past medical problems but has had similar
blistering intermittently on the back of both hands that is worse in the summer. He admits to
drinking alcohol heavily on the weekends when he is not driving and has smoked a pack of
cigarettes per day for the last 45 years.
Which one of the following would be most appropriate at this point?
A) Intravenous antibiotics until clinical improvement is noted
B) Discharge on oral antibiotics
C) Total serum, plasma, or spot urine porphyrins
D) Ultrasonography of the arm to rule out an underlying abscess
E) Surgical consultation for possible debridement

ANSWER: C
Porphyria cutanea tarda (PCT) should be suspected in patients who present with blistering
lesions on sun-exposed skin. Factors associated with increased susceptibility to PCT include

116
those that cause liver damage, such as alcohol use and hepatitis C, as well as smoking,
estrogen use, HIV infection, and HFE mutations. The initial test in suspected PCT should be
total serum, plasma, or spot urine porphyrins. The clinical presentation of this patient is less
consistent with cellulitis, which usually presents with erythema, pain, and warmth, and when
severe may lead to fever and leukocytosis.
Ref: Habif TP: Clinical Dermatology: A Color Guide to Diagnosis and Therapy, ed 6. Elsevier, 2016, pp 407-410.

192. An 8-year-old male is diagnosed with attention-deficit/hyperactivity disorder. Which one


of the following is the most effective treatment?
A) Behavioral therapy
B) Atomoxetine (Strattera)
C) Bupropion (Wellbutrin)
D) Psychostimulants such as methylphenidate (Ritalin) and dextroamphetamine
(Dexedrine)
E) "2-Receptor agonists such as guanfacine (Tenex)

ANSWER: D
Psychostimulants are the most effective medication option for treating attention-
deficit/hyperactivity disorder (ADHD) and are widely accepted as first-line therapy.
Behavioral therapy is recommended before considering medications for children under the
age of 6, and may be considered for older children but may not add much benefit in this age
group, particularly when there is a good response to medication. Atomoxetine and the "2-
receptor agonist guanfacine are considered second-line agents because of lower effectiveness
against core ADHD symptoms and a greater frequency of side effects. Bupropion has been
used off-label for ADHD but is not FDA approved for this indication.
Ref: Felt BT, Biermann B, Christner JG, et al: Diagnosis and management of ADHD in children. Am Fam Physician 2014;90(7):456-
464.

193. A 17-year-old high school football player presents to your office with right fourth finger
pain that began suddenly after he made a tackle during last night’s game. Pain and tenderness
are greatest at the flexor crease of the distal interphalangeal (DIP) joint. He can fully flex the
fourth metacarpophalangeal and proximal interphalangeal (PIP) joints. Flexion of the DIP is
weak and cannot be held against applied resistance.
Which one of the following is most likely?
A) Rupture of the extensor tendon at the DIP
B) Rupture of the extensor tendon’s central slip at the PIP
C) Flexor digitorum profundus rupture
D) Rupture of both flexor tendons

ANSWER: C

117
This patient has ruptured the flexor digitorum profundus (FDP) tendon as a result of a sudden
force applied to the distal phalanx while in flexion. This happens when catching the finger in
an opponent’s jersey or belt loop while making a tackle, hence the name jersey finger. This
injury is also seen in wrestlers.
The FDP attaches at the base of the distal phalanx while the flexor digitorum sublimis (FDS)
attaches at the base of the middle phalanx. Thus, it can be determined by physical examination
which tendon is injured, providing for early and accurate diagnosis and proper treatment. With
the noninvolved fingers held in full extension, if the patient can flex the proximal
interphalangeal (PIP) joint the FDS is intact. With rupture of the FDP, distal interphalangeal
(DIP) joint flexion may still be weakly present, but the patient is unable to hold flexion against
resistance and this maneuver elicits pain. With rupture of both flexor tendons, the patient is
unable to flex either the PIP or the DIP joint. In the scenario presented, rupture of both flexor
tendons is unlikely. Rupture of the extensor digitorum tendons may cause mallet finger at the
DIP joint, or a boutonnière deformity of the PIP joint. Since many of these flexor tendon
injuries require surgical exploration and repair, early orthopedic consultation is
recommended.
Ref: Borchers JR, Best TM: Common finger fractures and dislocations. Am Fam Physician 2012;85(8):805-810. 2) Brukner P, Khan
K: Clinical Sports Medicine, ed 4. McGraw Hill, 2012, pp 435-448.

194. A 50-year-old female with no other significant past medical history undergoes a total
vaginal hysterectomy because of uterine fibroids. She has a pulmonary embolism 2 weeks
after the surgery.
How long should she remain on anticoagulant therapy?
A) 1 month
B) 3 months
C) 6 months
D) 9 months
E) Indefinitely

ANSWER: B
Anticoagulant therapy is the treatment for pulmonary embolism and should be started
immediately. A rapid-acting parenteral anticoagulant (heparin, enoxaparin) or a direct orally
active factor Xa inhibitor (rivaroxaban) should be started initially. After the initial treatment,
the factor Xa inhibitor should be continued or the parenteral anticoagulant should be
transitioned to an oral anticoagulant. Patients who develop a pulmonary embolism as a
complication of a reversible factor have a low risk of recurrence when the anticoagulant
therapy is stopped. A 3-month course of anticoagulant is adequate treatment.
Ref: Goldman L, Schafer AI (eds): Goldman’s Cecil Medicine, ed 25. Elsevier Saunders, 2016, pp 620-627.

118
195. Risk factors for sudden infant death syndrome include which one of the following?
A) Side sleeping
B) Being large for gestational age at birth
C) A history of an apparent life-threatening event
D) Pacifier use
E) Asian or Pacific Islander ethnicity

ANSWER: A
Proper sleep position is highly important in reducing the risk of sudden infant death syndrome
(SIDS). Side sleeping and prone sleeping increase the risk for SIDS (SOR A). Bed sharing
also increases the risk. The “Back to Sleep” campaign of the American Academy of Pediatrics
(AAP) was associated with a 53% decrease in the incidence of SIDS in the United States in
the 1990s.
The use of pacifiers during sleep decreases the risk for SIDS, as does exclusive breastfeeding.
The AAP recommends offering a pacifier for use during sleep for infants in whom
breastfeeding is well established.
There are also multiple invariable risk factors for SIDS. These include male sex, Native
American ethnicity, birth weight <2500 g, and birth before 37 weeks gestation. Large for
gestational age infants are not at increased risk for SIDS, and infants of Asian or Pacific
Islander ethnicity actually have a lower than average risk of SIDS. Apparent life-threatening
events are not a risk factor for SIDS and are not related to SIDS (SOR B).
Ref: Adams SM, Ward CE, Garcia KL: Sudden infant death syndrome. Am Fam Physician 2015;91(11):778-783.

196. An 86-year-old female presents to the emergency department with shortness of breath
and a nonproductive cough. On examination she is slightly tachypneic and tachycardic but
her temperature and blood pressure are normal. Bilateral crackles are noted on the pulmonary
examination, but the cardiac examination is unremarkable. A chest radiograph shows bilateral
fluffy infiltrates compatible with heart failure or pneumonia. A CBC, comprehensive
metabolic panel, and troponin level are normal. Her EKG shows sinus tachycardia.
Which one of the following laboratory studies would be best to help determine whether the
patient should be treated for pneumonia, heart failure, or both?
A) A serum D-dimer level
B) BNP and procalcitonin levels
C) Serial troponin and creatine phosphokinase levels
D) An erythrocyte sedimentation rate and C-reactive protein level
E) A serum lactic acid level

ANSWER: B
BNP is a sensitive marker for heart failure and procalcitonin is a reasonable marker for acute
infection. Serum lactate is used to help assess the severity of hypoperfusion, or shock. The

119
erythrocyte sedimentation rate and C-reactive protein level are not specific and would not be
good discriminators in this case. Serial troponin and creatine phosphokinase can help rule out
acute myocardial ischemia but would not help discriminate between heart failure and
pneumonia. D-dimer is a marker for thromboembolism and is not relevant in this situation.
Ref: Okorie ON, Dellinger P: Lactate: Biomarker and potential therapeutic target. Crit Care Clin 2011;27(2):299-326. 2) Maisel A,
Neath SX, Landsberg J, et al: Use of procalcitonin for the diagnosis of pneumonia in patients presenting with a chief complaint of
dyspnoea: Results from the BACH (Biomarkers in Acute Heart Failure) trial. Eur J Heart Fail 2012;14(3):278-286. 3) Chowdhury P,
Choudhary R, Maisel A: The appropriate use of biomarkers in heart failure. Med Clin North Am 2012;96(5):901-913. 4) Vincent JL
(ed): Annual Update in Intensive Care and Emergency Medicine 2013. Springer, 2013, pp 77-86.

197. A 52-year-old male runner with a long-standing history of essential hypertension


presents for a follow-up visit. A review of his medical record shows blood pressure readings
that are consistently 10–15 mm Hg above his goal. He is currently on hydrochlorothiazide,
25 mg/day, and is compliant with a low-sodium diet. An EKG is shown down.
Based on the EKG, which one of the following would be most appropriate?
A) A coronary calcium score
B) An exercise stress test
C) 24-hour Holter monitoring
D) Echocardiography
E) Coronary angiography

120
121
ANSWER: D
The diagnosis and treatment of left ventricular hypertrophy (LVH) in patients with
hypertension is important, as it is associated with increased morbidity and mortality from
heart failure, arrhythmias, and sudden cardiac death. LVH is often seen in patients with long-
standing uncontrolled hypertension. If the sum of the S wave in lead V1 and the R wave in
lead V5 or V6 is ≥35 mm or the R wave in aVL is ≥11 mm, it suggests the presence of LVH
(Sokolow-Lyon indices). The sensitivity of an EKG for LVH ranges from 7% to 50%,
however, so echocardiography is the test of choice to make the diagnosis. Echocardiography
helps to directly visualize and quantify left ventricle wall size, in addition to providing other
valuable information such as cardiac wall motion abnormalities, ejection fraction, and cardiac
chamber dimensions. There is no primary role for the other tests listed in this setting (SOR
B).
Ref: Drazner MH: The progression of hypertensive heart disease. Circulation 2011;123(3):327-334. 2) Goldberger AL, Goldberger
ZD, Shvilkin A: Goldberger’s Clinical Electrocardiography: A Simplified Approach, ed 8. Elsevier Saunders, 2012.

198. A 28-year-old gravida 2 para 1 visits your office for routine prenatal care at 36 weeks
gestation. Today she reports some mildly increased lower extremity edema without associated
headache, visual changes, or right upper quadrant pain. She has not noticed any leaking
amniotic fluid and has only had occasional mild uterine contractions. Her blood pressure has
been normal at past visits, but on examination today it is 144/92 mm Hg. Repeat measurement
confirms the blood pressure reading, and a urine dipstick shows trace protein.
Her fundal height measures 36 cm, fetal heart tones have a rate of 134 beats/min, and
Leopold’s maneuvers indicate that her fetus is in a cephalic position. Laboratory tests reveal
a normal platelet count, normal transaminase and creatinine levels, and a normal urine
protein/creatinine ratio.
Your plan at this time should include which one of the following?
A) Immediate induction of labor
B) Home blood pressure and symptom monitoring, and induction of labor at 37 weeks
gestation
C) Home blood pressure and symptom monitoring, weekly office visits and laboratory
evaluation, and induction of labor at 39 weeks gestation
D) Home blood pressure and symptom monitoring, weekly office visits and laboratory
evaluation, and induction of labor at 40 weeks gestation
E) Home blood pressure and symptom monitoring, weekly office visits and laboratory
evaluation, and induction of labor at 41 weeks gestation

ANSWER: B
This patient has gestational hypertension. She has no findings consistent with a diagnosis of
preeclampsia, although she is now at risk for this condition. Current evidence suggests that,
in the absence of preeclampsia with severe features (blood pressure >160/110 mm Hg,

122
thrombocytopenia, impaired liver function, renal insufficiency, pulmonary edema, cerebral
edema, or visual disturbance), pregnancy can safely be continued until 37 weeks gestation.
At that point delivery is recommended in order to avoid the risks associated with gestational
hypertension, which include abruption, intrauterine growth restriction, and progression to
preeclampsia and eclampsia.
Ref: American College of Obstetricians and Gynecologists; Task Force on Hypertension in Pregnancy: Hypertension in pregnancy.
Report of the American College of Obstetricians and Gynecologists’ Task Force on Hypertension in Pregnancy. Obstet Gynecol
2013;122(5):1122-1131.

199. A 35-year-old production worker has developed intermittent numbness and paresthesias
in the first three fingers of her right hand since being moved to a new workstation 6 weeks
ago. These symptoms usually occur mostly while she is working, but sometimes while she is
driving or sleeping. She is asymptomatic when you see her in your office. Your examination
reveals normal skin color, normal musculature, and no pain to palpation around the hand,
wrist, and forearm. Holding the affected wrist in flexion for 30 seconds reproduces her
symptoms.
Which one of the following would be most appropriate at this point?
A) Nerve conduction studies of the hand, wrist, and forearm
B) Arterial Doppler studies of the hand, wrist, and forearm
C) A Velcro compression strap to wear just below her right elbow while working
D) A cock-up wrist splint to wear during times that she is symptomatic
E) Referral for carpal tunnel release surgery

ANSWER: D
This patient has signs and symptoms of carpal tunnel syndrome. Her new task is likely causing
inflammation and/or hypertrophy of the flexor tendons of the wrist and hand, which in turn
are placing pressure on the median nerve within the closed space of the carpal tunnel. The use
of a wrist splint and NSAIDs has been shown to reduce pain and should be tried along with
exercises of the wrist and hand, and activity modification if possible. Nerve conduction
studies are not recommended unless conservative management fails and carpal tunnel release
is being considered.
Ref: Rakel RE, Rakel DP (eds): Textbook of Family Medicine, ed 9. Elsevier Saunders, 2016, pp 659-660.

200. Which one of the following is the preferred first-line treatment for chronic midsubstance
Achilles tendinopathy?
A) Oral NSAIDs
B) Oral corticosteroids
C) Local corticosteroid injection of the retrocalcaneal bursa
D) Eccentric strengthening of the gastrocnemius and soleus muscles
E) Tendinoplasty

123
ANSWER: D
The term Achilles tendinopathy reflects the chronic, noninflammatory, degenerative nature
of this disorder as opposed to the traditional term tendinitis. Eccentric strengthening programs
of the gastrocnemius and soleus muscles have demonstrated 60%–90% improvement in pain
and function and are considered the first-line treatment for chronic midsubstance Achilles
tendinopathy. Oral NSAIDs and oral corticosteroids are ineffective, and local corticosteroid
injection in or near a load-bearing tendon such as the Achilles tendon increases the risk of
tendon rupture. Surgical tendinoplasty is inconsistently beneficial and carries added risk, and
is not a first-line treatment.
Ref: Childress MA, Beutler A: Management of chronic tendon injuries. Am Fam Physician 2013;87(7):486-490

201. A 45-year-old male with hyperlipidemia presents to your office for follow-up. Because
of his cardiovascular risk level he has been on moderate-intensity statin therapy with
atorvastatin (Lipitor) for the past 6 months. His HDL-cholesterol level at this visit is 29
mg/dL.
Which one of the following steps is most appropriate?
A) Continue the statin
B) Add fibrates
C) Add niacin
D) Supplement with fish oil

ANSWER: A
Drug therapy aimed at increasing HDL-cholesterol levels when added to a statin treatment
does not decrease a patient’s cardiovascular risk. Such agents have no effect on all-cause
mortality, cardiovascular mortality, or the risk of stroke (SOR B). Current guidelines for
treatment of hyperlipidemia are based on individual cardiovascular risk stratification rather
than LDL-cholesterol levels, and recommend low-, medium-, or high-intensity therapy with
statins, based on an individual patient’s risk.
Ref: Ebell MH, Grad R: Top 20 research studies of 2014 for primary care physicians. Am Fam Physician 2015;92(5):377-383.

202. A 65-year-old asymptomatic female is found to have extensive sigmoid diverticulosis


on screening colonoscopy. She asks whether there are any dietary changes she should make.
In addition to increasing fiber intake, which one of the following would you recommend?
A) Limiting intake of dairy products
B) Limiting intake of spicy foods
C) Limiting intake of wheat flour
D) Limiting intake of nuts
E) No dietary limitations

ANSWER: E

124
Patients with diverticulosis should increase dietary fiber intake or take fiber supplements to
slow progression of the diverticular disease. Avoiding nuts, corn, popcorn, and small seeds
has not been shown to prevent complications of diverticular disease. Limiting intake of dairy
products, spicy foods, and wheat flour would be appropriate for other gastrointestinal
problems such as lactose intolerance, gastroesophageal reflux disease (GERD), and celiac
disease.
Ref: Wilkins T, Embry K, George R: Diagnosis and management of acute diverticulitis. Am Fam Physician 2013;87(9):612-620.

203. A 32-year-old male with type 1 diabetes mellitus presents to your office with a tender,
scaly lesion on his anterior left shin. It is a 5-cm reddish-brown plaque with well-defined
borders and what appears to be yellowish deposits in the center.
You perform a punch biopsy of the lesion to confirm your diagnosis of
A) granuloma annulare
B) necrobiosis
C) sarcoidosis
D) xanthoma

ANSWER: B
Necrobiosis lipoidica diabeticorum is seen in 0.3% of patients with diabetes mellitus or
impaired glucose tolerance. The lesions may precede the diagnosis of diabetes mellitus by
several years. The sharply demarcated reddish-brown plaque with central yellow deposits in
the pretibial area is characteristic and a biopsy is not always necessary. Topical corticosteroids
are sometimes helpful.
Granuloma annulare and sarcoidosis are unlikely on the leg. Early lesions of necrobiosis
lipoidica diabeticorum can be confused with granuloma annulare or sarcoidosis, however, and
a biopsy may be helpful. While xanthomas can be flat plaques up to several centimeters in
size, they usually occur on flexor surfaces of the limbs along with the trunk and face. They
lack the reddish-brown outer portion of the lesions described here.
Ref: Lee SY, Shen L, Keimig EL: A solitary red patch. JAMA 2014;311(22):2328-2329.

204. Pioglitazone (Actos) has been found to be associated with which one of the following
adverse effects?
A) Steatohepatitis
B) Fluid retention
C) Cognitive dysfunction
D) Increased stroke risk
E) Increased all-cause mortality

ANSWER: B
Thiazolidinediones (TZDs) improve hyperglycemia by improving insulin resistance and by
maintaining or improving β-cell secretory function. One of the side effects of TZDs is that

125
they can cause fluid retention, especially in patients with cardiac and renal disease. This may
lead to weight gain and peripheral edema. Because of this effect, TZDs are contraindicated in
patients with New York Heart Association class III or IV heart failure. They can also reduce
bone mineral density and are associated with a higher risk of non-osteoporotic bone fractures.
In addition to their ability to decrease glucose levels, however, there have been some other
favorable effects noted with their use. Clinical trials have shown that treatment with
pioglitazone resulted in a significant reduction in the composite outcome of nonfatal acute
myocardial infarction, stroke, and all-cause mortality. They may also help prevent central
nervous system insulin resistance–related cognitive dysfunction. TZDs are also useful in
patients with nonalcoholic steatohepatitis.
Ref: Handelsman Y, Bloomgarden ZT, Grunberger G, et al: American Association of Clinical Endocrinologists and American College
of Endocrinology—Clinical practice guidelines for developing a diabetes mellitus comprehensive care plan—2015. Endocr Pract
2015;21(Suppl 1):1-87. 2) Garber AJ, Abrahamson MJ, Barzilay JI, et al: Consensus statement by the American Association of Clinical
Endocrinologists and American College of Endocrinology on the comprehensive type 2 diabetes management algorithm—2016
Executive Summary. Endocr Pract 2016;22(1):84-113.

205. A 52-year-old male with a history of severe esophageal reflux and dysphagia undergoes
upper GI endoscopy that reveals a mid-esophageal stricture and severe erosive esophagitis.
The stricture is dilated by the gastroenterologist and he recommends long-term proton pump
inhibitor therapy.
While long-term proton pump inhibitor therapy should alleviate symptoms of reflux, it may
be associated with an increased risk of which one of the following?
A) Clostridium difficile infection
B) Helicobacter pylori infection
C) Type 2 diabetes mellitus
D) Iron deficiency anemia
E) Hypothyroidism

ANSWER: A
Proton pump inhibitor use has been shown to increase the risk for Clostridium difficile and
other enteric infections, and elderly patients and those with significant comorbid conditions
may already be at increased risk. Studies have not shown an increased risk for iron deficiency.
There is no increased risk for hypothyroidism, Helicobacter pylori infection, or type 2
diabetes mellitus.
Ref: Harnik IG: Gastroesophageal reflux disease. Ann Intern Med 2015;163(1):ITC1.

126
206. A 4-year-old female is brought to your office after a recent camping excursion and
presents with a large number of mosquito bites. She has developed a number of areas of
honey-colored crusting lesions you diagnose as nonbullous impetigo.
Which one of the following oral medications is preferred for this patient?
A) Cephalexin (Keflex)
B) Doxycycline
C) Erythromycin
D) Penicillin VK
E) Trimethoprim/sulfamethoxazole (Bactrim)

ANSWER: A
Nonbullous impetigo is most often caused by Streptococcus pyogenes and methicillin-
sensitive Staphylococcus aureus. Cephalexin is the most appropriate option, with good
coverage for both of these bacteria. Penicillin VK has been found to be no more effective than
placebo in the treatment of impetigo. Macrolide resistance limits the use of erythromycin.
Tetracycline should not be used in children <8 years old as it may cause staining of permanent
teeth. Trimethoprim/sulfamethoxazole has coverage against both methicillin-sensitive and
methicillin-resistant Staphylococcus aureus, but may have inadequate coverage for
Streptococcus.
Ref: Hartman-Adams H, Banvard C, Juckett G: Impetigo: Diagnosis and treatment. Am Fam Physician 2014;90(4):229-235.

207. Which one of the following medications blocks dopamine stimulation in the
chemoreceptor trigger zone, making it an effective antiemetic for patients with
gastroenteritis?
A) Diphenhydramine (Benadryl)
B) Meclizine (Antivert)
C) Metoclopramide (Reglan)
D) Ondansetron (Zofran)
E) Scopolamine (Transderm Scop)

ANSWER: C
Dopamine antagonists, such as metoclopramide, block dopamine stimulation in the
chemoreceptor trigger zone, thereby limiting emetic input to the medullary vomiting center
(SOR C). SSRIs, such as ondansetron, also work in the chemoreceptor trigger zone. They
inhibit serotonin at the 5-HT3 receptor in the small bowel, vagus nerve, and chemoreceptor
trigger zone. Antihistamines and anticholinergics limit stimulation of the vomiting center
through inhibition of the H1 receptor and acetylcholine, respectively. These medications are
particularly beneficial in vestibular-mediated nausea, such as motion sickness.
Ref: Flake ZA, Linn BS, Hornecker JR: Practical selection of antiemetics in the ambulatory setting. Am Fam Physician 2015;91(5):293-
296.

127
208. An asymptomatic 46-year-old male with autosomal dominant polycystic kidney disease
sees you for a routine visit. His vital signs in the office include a blood pressure of 152/93
mm Hg, a heart rate of 82 beats/min, a respiratory rate of 17/min, a temperature of 37.0°C
(98.6°F), and an oxygen saturation of 99% on room air. His glomerular filtration rate is 49
mL/min/1.73 m2. Which one of the following is the preferred initial therapy for controlling
this patient’s blood pressure?
A) Amlodipine (Norvasc)
B) Furosemide
C) Lisinopril (Prinivil, Zestril)
D) Metoprolol
E) Spironolactone (Aldactone)

ANSWER: C
Autosomal dominant polycystic kidney disease (ADPCKD) is the most common genetic
kidney disease and accounts for 4.7% of end-stage kidney disease cases in America. Many
patients with ADPCKD are asymptomatic, but early symptoms can include flank pain, gross
hematuria, or recurrent urinary tract infections. The most common extrarenal manifestation
of ADPCKD is hypertension, which can precipitate cardiovascular dysfunction, including left
ventricular hypertrophy. Thus, early diagnosis and management of hypertension is crucial.
The goal blood pressure should be <140/90 mm Hg in patients under the age of 60. All
ADPCKD patients eventually develop a loss of renal function, and approximately 80%
develop end-stage renal disease by age 70. An ACE inhibitor is the recommended first-line
therapy (SOR C), so lisinopril is the best choice for this patient. Angiotensin receptor blockers
are acceptable in patients who cannot tolerate ACE inhibitors.
Ref: Grantham JJ: Autosomal dominant polycystic kidney disease. N Engl J Med 2008;359(14):1477-1485. 2) Srivastava A, Patel N:
Autosomal dominant polycystic kidney disease. Am Fam Physician 2014;90(5):303-307.

209. A 70-year-old female presents to your office with a 1-month history of generalized
aching of both shoulders. She denies a specific injury but believes that her dog has been
pulling too much on the leash. She has also noticed joint stiffness in her hips and trouble
getting moving in the mornings. She denies headaches and visual disturbance. A physical
examination is normal except for mild pitting edema of the ankles. The patient has lost 5 kg
(11 lb) since her health maintenance visit 8 months ago. Laboratory studies reveal an
erythrocyte sedimentation rate of 66 mm/hr (N 1–25) and a platelet count of 450,000/mm3
(N 150,000–350,000). Her creatine kinase level is 50 U/L (N 40–150). Which one of the
following should you prescribe now?
A) Aspirin, 81 mg daily
B) Colchicine (Colcrys), 1.2 mg initially, 0.6 mg an hour later, then 0.6 mg daily
C) Hydroxychloroquine (Plaquenil), 200 mg twice a day for a month then 100 mg twice daily
D)Methotrexate,7.5mg weekly increased by2.5mg/week as needed to obtain symptom control
E) Prednisone, 15 mg daily with a slow taper

128
ANSWER: E
Polymyalgia rheumatica (PMR) without concurrent giant cell arteritis is treated with a slow
taper of low-dose corticosteroids (SOR C). Disease-modifying antirheumatic drugs and
antimalarial medications have no role in the treatment of PMR. Aspirin therapy is
recommended as adjuvant therapy for giant cell arteritis to decrease stroke risk (SOR C).
Colchicine may alleviate acute symptoms of gout and pseudogout.
Ref: Wasserman AM: Diagnosis and management of rheumatoid arthritis. Am Fam Physician 2011;84(11):1245-1252. 2) Caylor TL,
Perkins A: Recognition and management of polymyalgia rheumatica and giant cell arteritis. Am Fam Physician 2013;88(10):676-684.
3) Hainer BL, Matheson E, Wilkes RT: Diagnosis, treatment, and prevention of gout. Am Fam Physician 2014;90(12):831-836.

210. An active 76-year-old male presents to your office as a new patient. He recently fell
while hiking and was diagnosed with rib fractures in the emergency department. He says he
has always been healthy except he was told his blood pressure was mildly elevated 10 years
ago. He has not followed up with a doctor since that visit. Today his examination is normal
other than bruising and mild tenderness over his left 7th to 9th ribs laterally and a blood
pressure of 144/88 mm Hg.
You review his transition of care document from the emergency department and his problem
list states that he has chronic kidney failure stage 3A based on an estimated glomerular
filtration rate of 55 mL/min/1.73 m2. A urinalysis in your office is normal, with no hematuria
or proteinuria.
Which one of the following would be most appropriate at this time?
A) Reassurance and observation
B) Renal ultrasonography
C) CT of the abdomen
D) Magnetic resonance angiography of the renal artery

ANSWER: A
Chronic kidney disease appears to be overdiagnosed in the older population. Stage 3A kidney
disease is defined as an estimated glomerular filtration rate (GFR) of 45–59 mL/min/1.73 m2
and is predominantly seen in older patients. It is seldom progressive in the absence of
significant proteinuria. Older patients with chronic kidney disease are less likely to develop
end-stage renal disease than to die of complications related to aging and cardiovascular
disease. There is a decline in estimated GFR with normal aging, and the likelihood of patients
progressing to end-stage renal disease and dialysis is minimal if they have a GFR of 45–59
mL/min/1.73 m2.
Ref: Glassock R, Delanaye P, El Nahas M: An age-calibrated classification of chronic kidney disease. JAMA 2015;314(6):559-560.

129
211. A 13-year-old baseball pitcher develops pain in the upper arm at the shoulder when
throwing and is diagnosed with Little League shoulder. This entity is actually a
A) coracoclavicular ligament sprain
B) sprain or tear of the acromioclavicular ligament
C) deltoid muscle strain
D) proximal humeral epiphysitis
E) labral tear on the acromion

ANSWER: D
In contrast to Little League elbow, which is a complex of possible injuries, Little League
shoulder refers to one entity, proximal humeral epiphysitis, most often developing as an
overuse injury in baseball pitchers age 11–16.
Ref: Saltzman BM, Chalmers PN, Mascarenhas R, et al: Upper extremity physeal injury in young baseball pitchers. Phys Sportsmed
2014;42(3):100-111.

212. A 35-year-old white male who has had type 1 diabetes mellitus for 20 years begins
having episodes of hypoglycemia. His glucose levels had previously been stable and well
controlled, and he has not recently changed his diet or insulin regimen.
Which one of the following is the most likely cause of the hypoglycemia?
A) Spontaneous improvement of β-cell function
B) Renal disease
C) Reduced physical activity
D) Insulin antibodies

ANSWER: B
The most common cause of hypoglycemia in previously stable, well-controlled diabetic
patients who have not changed their diet or insulin dosage is diabetic renal disease. A
reduction in physical activity or the appearance of insulin antibodies (unlikely after 20 years
of therapy) would increase insulin requirements and produce hyperglycemia. Spontaneous
improvement of β-cell function after 20 years would be very rare.
Ref: Fauci AS, Braunwald E, Kasper DL, et al (eds): Harrison’s Principles of Internal Medicine, ed 17. McGraw-Hill, 2008, pp 1751-
1752. 2) Alsahli M, Gerich JE: Hypoglycemia, chronic kidney disease, and diabetes mellitus. Mayo Clin Proc 2014;89(11):1564-
1571.

130
213. A 63-year-old female presents with a complaint of a painful, red right eye. She says that
her vision has also been blurry. She has also noted a discharge. The patient wears contact
lenses.
When you examine the patient you note photophobia. The pupils are equal, round, and
reactive to light. You also note unilateral diffuse injection. Fluorescein staining reveals focal
corneal uptake.
Which one of the following is the most likely diagnosis?
A) Corneal abrasion
B) Subconjunctival hemorrhage
C) Uveitis
D) Acute angle-closure glaucoma
E) Herpes zoster ophthalmicus

ANSWER: A
Common causes of red eye include infectious conjunctivitis, allergies, corneal abrasion,
keratoconjunctivitis, subconjunctival hemorrhage, uveitis, blepharitis, iritis, acute angle-
closure glaucoma, and herpes zoster ophthalmicus.
Viral infections typically cause conjunctivitis with mild pain and no loss of vision. The
problem is usually unilateral in the beginning and a watery to serous discharge may be noted.
Adenovirus is the most common cause. Acute bacterial conjunctivitis has a similar
presentation and may include eyelid edema and a purulent discharge. Allergic conjunctivitis
is usually bilateral and painless, with intense itching, and a stringy or ropy watery discharge.
Herpes zoster ophthalmicus is associated with a vesicular rash, keratitis, and uveitis. The rash
is preceded by pain and a tingling sensation. Findings include conjunctivitis and dermatomal
involvement, which are usually unilateral.
With corneal abrasion there is usually a history of an injury involving a foreign object. Signs
and symptoms include severe eye pain; red, watery eyes; photophobia; and a foreign body
sensation. Vision is usually normal and pupils are equal and reactive to light.
Symptoms of uveitis include a red eye, loss of vision, and photophobia. It is associated with
many autoimmune diseases, including reactive arthritis, ankylosing spondylitis, and
inflammatory bowel disease.
Acute angle-closure glaucoma causes a significant loss of vision, with dilated pupils that don’t
react normally to light. Symptoms include severe pain and watery eyes, with halos around
lights. Patients may have nausea and vomiting. This form of glaucoma often has an acute
onset.
Ref: Cronau H, Kankanala RR, Mauger T: Diagnosis and management of red eye in primary care. Am Fam Physician 2010;81(2):137-
144. 2) Goldman L, Schafer AI (eds): Goldman’s Cecil Medicine, ed 25. Elsevier Saunders, 2016, pp 2556-2568.

131
214. A 54-year-old male smoker with a family history of coronary artery disease tells you that
he takes β-carotene and vitamin E regularly to help prevent cancer and heart disease. In
counseling this patient, you discuss smoking cessation and advise him that
A) he should continue both supplements
B) he should continue β -carotene and discontinue vitamin E
C) he should discontinue β -carotene and continue vitamin E
D) he should discontinue both supplements
E) evidence is insufficient to assess the risk and benefit of these supplements

ANSWER: D
The U.S. Preventive Services Task Force recommends against the use of $-carotene or
vitamin E supplementation for the prevention of cardiovascular disease or cancer. This is a
class D recommendation (do not recommend). Overall there is no beneficial effect on cancer
or heart disease from these vitamin supplements. In one study vitamin E appeared to increase
the risk of hemorrhagic stroke, and $-carotene has been found to increase the risk of lung
cancer in persons already at higher risk for lung cancer.
Ref: Moyer VA; US Preventive Services Task Force: Vitamin, mineral, and multivitamin supplements for the primary prevention of
cardiovascular disease and cancer: US Preventive Services Task Force recommendation statement. Ann Intern Med 2014;160(8):558-
564.

215. In its dental recommendations from birth to 5 years of age, the U.S. Preventive Services
Task Force recommends fluoride supplementation for which one of the following when the
primary water supply is deficient in fluoride?
A) All children starting at birth
B) All children starting at 6 months of age
C) All children starting at 2 years of age
D) All children with erupted primary teeth
E) Children with erupted primary teeth who have no access to fluoride varnish

ANSWER: B
The U.S. Preventive Services Task Force recommends that primary care clinicians prescribe
oral fluoride supplementation starting at 6 months of age for children whose water supply is
deficient in fluoride (<0.6 ppm) (B recommendation). The task force found evidence of
moderate benefit of oral fluoride supplementation for the prevention of dental caries in this
group.
Ref: Prevention of dental caries in children from birth through five years of age: Recommendation statement. Am Fam Physician
2015;91(3):190A-190E.

132
216. You see a 5-week-old female for a well child visit in December. She was delivered at 28
weeks gestation because of severe preeclampsia in the mother. Her parents state that she is
doing well, feeding well, and growing. The physical examination is normal.
At this time, you recommend immunoprophylaxis with
A) influenza vaccine
B) palivizumab (Synagis)
C) pertussis vaccine
D) rotavirus vaccine
E) intravenous immunoglobulin

ANSWER: B
For all infants born before 29 weeks gestation, palivizumab is recommended for the first year
of life during respiratory syncytial virus season to reduce the likelihood of hospitalization.
Immunization against pertussis and rotavirus is not recommended until the 2-month visit.
Influenza vaccine is not recommended for any infant until 6 months of age. There is no
indication for immunoglobulin in this infant.
Ref: Joffe S, Escobar GJ, Black SB, et al: Rehospitalization for respiratory syncytial virus among premature infants. Pediatrics
1999;104(4 Pt 1):894-899. 2) American Academy of Pediatrics Committee on Infectious Diseases, American Academy of Pediatrics
Bronchiolitis Guidelines Committee: Updated guidance for palivizumab prophylaxis among infants and young children at increased
risk of hospitalization for respiratory syncytial virus infection. Pediatrics 2014;134(2):415-420. 3) Gauer RL, Burket J, Horowitz E:
Common questions about outpatient care of premature infants. Am Fam Physician 2014;90(4):244-251.

217. A 25-year-old male presents to the emergency department with a complaint of his heart
racing for the past 45 minutes. He has no known medical problems. He does have occasional
episodes of palpitations, but they usually only last a short period. An EKG shows a narrow
QRS complex tachycardia at a rate of 180 beats/min. His condition is not improved by vagal
maneuvers and adenosine (Adenocard). He remains hemodynamically stable.
Which one of the following would be most appropriate at this point?
A) Amiodarone (Cordarone)
B) Procainamide
C) Sotalol (Betapace)
D) Verapamil (Calan, Verelan)
E) DC cardioversion

ANSWER: D
The treatment goal of narrow QRS complex tachycardia is to slow down the heart rate and to
convert to normal sinus rhythm by blocking, or increasing the refractoriness of, the
atrioventricular node. In a hemodynamically stable patient vagal maneuvers are a good first-
line treatment, followed by adenosine. If those do not work verapamil or diltiazem can be
used. DC cardioversion is used in narrow QRS complex tachycardia if the patient becomes
hemodynamically unstable. Amiodarone, procainamide, and sotalol are all used for the
treatment of wide QRS complex tachycardia but not for narrow-complex tachycardia.

133
Ref: Helton MR: Diagnosis and management of common types of supraventricular tachycardia. Am Fam Physician 2015;92(9):793-
800.

218. A 19-year-old female high school student is brought to your office by a friend who is
concerned about the patient cutting herself on the wrists. The patient denies that she was trying
to kill herself, and states that she did this because she “just got so angry” at her boyfriend
when she caught him sending a text message to another woman. She denies having a
depressed mood or anhedonia, and blames her fluctuating mood on everyone who “keeps
deserting her,” making her feel like she’s “nothing.” She admits that she has difficulty
controlling her anger. Her sleep quality and pattern appear normal, as does her appetite. She
denies hallucinations or delusions. The wounds on her wrists appear superficial and there is
evidence of previous cutting behavior on her forearms. Her vital signs are stable.
Which one of the following would be most beneficial for this patient?
A) Clonazepam (Klonopin)
B) Fluoxetine (Prozac)
C) Quetiapine (Seroquel)
D) Inpatient psychiatric admission
E) Psychotherapy

ANSWER: E
This patient displays most of the criteria for borderline personality disorder. This is a
maladaptive personality type that is present from a young age, with a strong genetic
predisposition. It is estimated to be present in 1% of the general population and involves equal
numbers of men and women; women seek care more often, however, leading to a
disproportionate number of women being identified by medical providers.
Borderline personality disorder is defined by high emotional lability, intense anger, unstable
relationships, frantic efforts to avoid a feeling of abandonment, and an internal sense of
emptiness. Nearly every patient with this disorder engages in self-injurious behavior (cutting,
suicidal gestures and attempts), and about 1 in 10 patients eventually succeeds in committing
suicide. However, 90% of patients improve despite having made numerous suicide threats.
Suicidal gestures and attempts peak when patients are in their early 20s, but completed suicide
is most common after age 30 and usually occurs in patients who fail to recover after many
attempts at treatment. In contrast, suicidal actions such as impulsive overdoses or superficial
cutting, most often seen in younger patients, do not usually carry a high short-term risk, and
serve to communicate distress.

Inpatient hospitalization may be an appropriate treatment option if the person is experiencing


extreme difficulties in living and daily functioning, and pharmacotherapy may offer a mild
degree of symptom relief. While these modalities have a role in certain patients,
psychotherapy is considered the mainstay of therapy, especially in a relatively stable patient
such as the one described.

134
Ref: Dean L, Falsetti SA: Treating patients with borderline personality disorder in the medical office. Am Fam Physician
2013;88(2):140-141.

219. Long-term treatment with which one of the following is known to reduce serum vitamin
D levels?
A) Estrogen replacement
B) Isoniazid
C) Phenytoin
D) Statins
E) Thiazide diuretics

ANSWER: C
Absorption and metabolism of vitamin D is known to be affected by interaction with other
medications. Isoniazid and thiazide diuretics can lead to increased blood levels or activity of
vitamin D. Estrogen replacement therapy can also increase levels of vitamin D in the blood,
although this potential benefit seems to diminish when progesterone is added. Vitamin D
absorption through the gut can be reduced by mineral oil, cholestyramine, and certain antacid
preparations, leading to lower blood levels. The metabolism of vitamin D is accelerated by
anticonvulsant drugs such as phenobarbital and phenytoin, which can also result in lower than
desired levels of vitamin D. Statins are not reported to have any known effect on vitamin D
levels.
Ref: NIH Office of Dietary Supplements. Vitamin D: Fact sheet for health professionals. 2) Hawkins EB, Ehrlich SD: University of
Maryland Medical Center. Possible interactions with: Vitamin D. 3) Gröber U, Kisters K: Influence of drugs on vitamin D and calcium
metabolism. Dermatoendocrinol 2012;4(2):158-166.

220. A 6-year-old male is brought to your office by his parents. He describes pain in the right
hip for the past month or two, and the parents report that for the past few weeks they have
noticed intermittent limping. The history is negative for trauma, injury, and fever. On
examination the patient indicates that the pain is over the anterior hip joint, with increased
pain on internal rotation, which is limited to 90°.
Of the options listed below, which one of the following is the most likely diagnosis in this
case?
A) Septic arthritis of the hip
B) Transient synovitis of the hip
C) Osteoarthritis of the hip
D) Osteonecrosis of the femoral head (Legg-Calvé-Perthes disease)
E) Slipped capital femoral epiphysis

ANSWER: D
This case presents a child with an insidious onset of pain and mild limitation of range of
motion of one hip joint. The case suggests involvement of the femoroacetabular joint, and all
of the listed options can affect this joint. Idiopathic osteonecrosis of the hip (Legg-Calvé-

135
Perthes disease) occurs most commonly in children at 2–12 years of age and has a male
predominance. Symptom onset is insidious, as in this case, with symptom severity and
functional limitations dependent on the level of disease progression. Slipped capital femoral
epiphysis occurs much more commonly in adolescents. Transient synovitis and septic arthritis
have a more acute onset, and typically cause fever. Osteoarthritis typically occurs in older
adults.
Ref: Wilson JJ, Furukawa M: Evaluation of the patient with hip pain. Am Fam Physician 2014;89(1):27-34.

221. Which one of the following is considered first-line therapy for nausea and vomiting of
pregnancy?
A) Ginger
B) Blue cohosh
C) Cranberry juice
D) Vitamin B6
E) Fenugreek

ANSWER: D
A number of alternative therapies have been used for problems related to pregnancy, although
vigorous studies are not always possible. For nausea and vomiting, however, vitamin B6 is
considered first-line therapy, sometimes combined with doxylamine. Other measures that
have been found to be somewhat useful include ginger and acupressure.
Cranberry products can be useful for preventing urinary tract infections, and could be
recommended for patients if this is a concern. Blue cohosh has been used as a partus
preparator, but there are concerns about its safety. Fenugreek has been used to increase milk
production in breastfeeding mothers, but no rigorous trials have been performed.
Ref: Low Dog T: The use of botanicals during pregnancy and lactation. Altern Ther Health Med 2009;15(1):54-58. 2) Herrell HE:
Nausea and vomiting of pregnancy. Am Fam Physician 2014;89(12):965-970.

222. A 38-year-old female calls your office and reports nausea, vomiting, abdominal cramps,
and diarrhea that developed 1 hour after she ate some chicken salad that had been sitting
outside at a picnic. Prior to that she was asymptomatic.
Which one of the following is the most likely cause of this patient’s foodborne illness?
A) Campylobacter jejuni
B) Escherichia coli O157:H7
C) Shigella species
D) Staphylococcus aureus
E) Hepatitis A

ANSWER: D
Most of the symptoms of foodborne illness are not specific to the causative organism.
However, the onset of vomiting and diarrhea within hours of consumption of contaminated

136
food results from the ingestion of preformed toxins, most often from Staphylococcus aureus
or Bacillus cereus in the United States. Campylobacter jejuni, Escherichia coli O157:H7,
Shigella species, and hepatitis A typically produce symptoms more than a day after ingestion.
Ref: Switaj TL, Winter KJ, Christensen SR: Diagnosis and management of foodborne illness. Am Fam Physician 2015;92(5):358-365.

223. A 76-year-old nursing home resident complains of constipation. He has had minimal
improvement with trials of scheduled toileting and increased fiber and fluids. A physical
examination is normal.
Which one of the following is the best intervention at this point?
A) Bisacodyl (Dulcolax)
B) Lactulose solution
C) Magnesium citrate solution
D) Polyethylene glycol (MiraLAX)

ANSWER: D
For functional constipation in older adults, behavioral changes should be first-line
management. These include scheduled toileting with proper positioning, increased intake of
fiber and fluids, and avoiding bedpan use. If there is not an adequate response to behavioral
interventions, osmotic laxatives should be initiated. Polyethylene glycol is more effective and
has fewer side effects compared to lactulose. Magnesium salts, including magnesium citrate,
do not have strong evidence for safety or efficacy, and magnesium toxicity is a concern with
long-term use. Due to possible adverse effects of stimulant laxatives in the long term,
particularly with older adults, these drugs should be used only if fiber and osmotic laxatives
are unsuccessful.
Ref: Bharucha AE, Pemberton JH, Locke GR 3rd: American Gastroenterological Association technical review on constipation.
Gastroenterology 2013;144(1):218-238. 2) Mounsey A, Raleigh M, Wilson A: Management of constipation in older adults. Am Fam
Physician 2015;92(6):500-504.

224. A 44-year-old male complains of feeling tired and sad for the last few months. He has a
past medical history of obesity, diabetes mellitus with painful peripheral neuropathy, and
seizure disorder. He has also noticed that he is not as interested in his usual hobbies and is
eating more than usual. You diagnose depression.
Which one of the following would be the most appropriate agent for this patient, considering
his comorbidities and symptoms?
A) Bupropion (Wellbutrin)
B) Citalopram (Celexa)
C) Duloxetine (Cymbalta)
D) Nortriptyline (Pamelor)

ANSWER: C

137
SSRIs and SNRIs are both effective in reducing depressive symptoms, but SNRIs have been
shown to be superior to SSRIs for management of neuropathic pain (SOR A). Bupropion
would effectively treat the patient’s depression and could cause weight loss, but it is
contraindicated in patients with seizure disorders (SOR A). Tricyclic antidepressants such as
nortriptyline could also help with the pain but might also worsen the patient’s obesity and
fatigue (SOR A).
Ref: Kovich H, DeJong A: Common questions about the pharmacologic management of depression in adults. Am Fam Physician
2015;92(2):94-100.

225. A 16-year-old male presents to your office with a 1-month history of paresthesias in the
ring finger and little finger of his left hand. He is the catcher on his high school baseball team.
The physical examination reveals no abnormalities of his neck, shoulder, or elbow. His hand
grip is normal, but maximal flexion of the wrist elicits paresthesias in the fourth and fifth
digits.
Which one of the following is the most likely diagnosis?
A) Carpal tunnel syndrome
B) Ulnar entrapment
C) Flexor carpi ulnaris tendinitis
D) De Quervain’s tenosynovitis
E) Stress fracture of the scaphoid

ANSWER: B
An ulnar neuropathy most commonly presents with sensory changes in the fourth and fifth
digits and usually does not involve weakness in hand grip. There is usually no specific injury,
but any activity that results in repetitive or prolonged wrist extension, as with cycling or
playing catcher, may increase the risk of this problem. It is important to examine the neck for
cervical disc disease and to examine the shoulder to see if motion elicits the pain, which would
indicate a brachial plexus problem. If symptoms are reproduced by compressing the ulnar
nerve at the elbow this could be the site of entrapment. Clinical tests may include a positive
Tinel sign on percussion of the ulnar nerve over Guyon’s canal. Also, there may be a positive
Phalen sign, with maximum passive flexion of the wrist for 1 minute inducing paresthesias in
the fourth and fifth fingers.
With ulnar neuropathy, plain radiographs are usually normal. Ultrasonography of the
peripheral nerves may be helpful in identifying compression etiologies. However,
electromyelography and nerve conduction velocities may be required to identify the area of
entrapment.
Ref: Shehab R, Mirabelli MH: Evaluation and diagnosis of wrist pain: A case-based approach. Am Fam Physician 2013;87(8):568-
573.

138
226. A 52-year-old male was recently diagnosed with mild essential hypertension. His
average blood pressure over several repeated weekly readings is 142/92 mm Hg. He now
comes to your office to discuss a management plan. He is particularly interested in trying to
manage his hypertension nonpharmacologically if possible.
Which one of the following nonpharmacologic interventions should you recommend for this
patient?
A) Limit table salt consumption to 500 mg/day
B) Consume 3 glasses of red wine daily
C) Consume a high-protein diet
D) Monitor home blood pressure readings
E) Start a yoga program

ANSWER: D
Self-measured blood pressure monitoring, with or without additional support (e.g., education,
counseling, telemedicine, home visits, Web-based logging), lowers blood pressure when
compared with usual care, although the benefits beyond 12 months are not clear (SOR A).
Limiting sodium intake to 2400 mg/day (approximately 1 teaspoon of table salt) is
recommended to lower blood pressure. Additional benefit occurs with a limit of 1500 mg/day
(SOR B). A diet that emphasizes vegetables, fruits, and whole grains is also recommended to
lower blood pressure (SOR A), as well as limiting alcohol consumption to no more than 2
drinks/day for men, and 1 drink/day for women (SOR C). Because of mixed results from
therapeutic trials and many limitations to the trials, the American Heart Association does not
recommend either yoga or acupuncture to lower blood pressure.
Ref: Oza R, Garcellano M: Nonpharmacologic management of hypertension: What works? Am Fam Physician 2015;91(11):772-776.

227. A 52-year-old male sees you at the request of his wife because of his snoring and her
concerns that he has obstructive sleep apnea. Further evaluation with a sleep study is
recommended if the patient reports which one of the following?
A) Insomnia
B) A history of hypertension
C) Snoring that awakens his wife
D) Unexplained daytime sleepiness

ANSWER: D
Daytime sleepiness is the clinically relevant symptom of obstructive sleep apnea (OSA) that
is most responsive to treatment. Other associated symptoms such as snoring, insomnia, and
fatigue are either less clinically relevant or less responsive to treatment. Hypertension,
diabetes mellitus, and coronary artery disease are associated with OSA, but evidence is
insufficient that continuous positive airway pressure (CPAP) improves outcomes for these
conditions, especially if they are not associated with daytime sleepiness.

139
Ref: Qaseem A, Dallas P, Owens DK, et al: Diagnosis of obstructive sleep apnea in adults: A clinical practice guideline from the
American College of Physicians. Ann Intern Med 2014;161(3):210-220.

228. A healthy 18-year-old male plans to play soccer and basketball and presents for a
preparticipation sports evaluation. The American Academy of Family Physicians
recommends AGAINST which one of the following measures to screen for cardiac disease
for this patient?
A) A detailed past medical history
B) A detailed family history
C) A detailed cardiovascular physical examination
D) An EKG

ANSWER: D
A baseline EKG is currently not recommended in an asymptomatic patient to screen for
cardiovascular disease. The American Academy of Family Physicians (AAFP) specifically
advises against performing a baseline EKG.
The AAFP does endorse performing a detailed past medical history to exclude a history of
hypertension, chest pain, prior heart murmur, or syncope. It also recommends obtaining a
family history that includes questions about any family members with prolonged QT
syndrome, Marfan syndrome, or sudden death before the age of 50. A physical examination
is recommended, including palpation of the femoral pulses simultaneously to detect
coarctation of the aorta, as well as heart auscultation performed with the patient both supine
and standing, and with the Valsalva maneuver, to detect a heart murmur suggestive of cardiac
disease.
Ref: Siwek J: Choosing wisely: Top interventions to improve health and reduce harm, while lowering costs. Am Fam Physician
2012;86(2):128-133. 2) Final Recommendation Statement: Coronary Heart Disease: Screening with Electrocardiography. US
Preventive Services Task Force, 2012. 3) Mirabelli MH, Devine MJ, Singh J, Mendoza M: The preparticipation sports evaluation. Am
Fam Physician 2015;92(5):371-376.

229. A full-term newborn female develops respiratory distress 1 hour after an uncomplicated
caesarean delivery. She has a respiratory rate of 70/min, and mild grunting and intercostal
retractions are noted on examination. The remainder of the examination is within normal
limits. A chest radiograph shows some hyperexpansion and fluid in the fissures.
Which one of the following is the most likely cause of her symptoms?
A) Respiratory distress syndrome of the newborn
B) Transient tachypnea of the newborn
C) Pneumothorax
D) Meconium aspiration syndrome

ANSWER: B
This patient has transient tachypnea of the newborn, which typically occurs within 2 hours of
birth. The chest radiograph usually shows hyperexpansion with perihilar densities and fluid

140
within the fissures. Respiratory distress syndrome of the newborn is most often seen in
premature infants, and the chest radiograph shows a classic diffuse ground-glass appearance.
With pneumothorax a chest radiograph would typically show a partial or complete lung
collapse. Meconium aspiration syndrome occurs in the setting of meconium-stained fluid and
is usually apparent immediately after delivery. The chest radiograph typically shows fluffy
densities with hyperinflation.
Ref: Hermansen CL, Mahajan A: Newborn respiratory distress. Am Fam Physician 2015;92(11):994-1002.

230. In a critically ill adult, which one of the following options for deep vein thrombosis
prophylaxis is associated with the greatest reduction in mortality risk?
A) Anticoagulation therapy
B) Antiplatelet therapy
C) Mechanical device use (pneumatic compression)
D) Inferior vena cava filter use

ANSWER: A
In critically ill adult patients, the only deep vein thrombosis prophylaxis that decreases
mortality risk is anticoagulation therapy. Mechanical device prophylaxis does not lower the
mortality risk compared to no prophylaxis. Data suggests that patients managed with both
prophylactic anticoagulation and mechanical device prophylaxis have a higher mortality risk
than those managed by prophylactic anticoagulation alone.
Ref: Lilly CM, Liu X, Badawi O, et al: Thrombosis prophylaxis and mortality risk among critically ill adults. Chest 2014;146(1):51-
57.

231. An 18-year-old male comes to your office because of the recent onset of recurrent,
unpredictable episodes of palpitations, sweating, dyspnea, gastrointestinal distress, dizziness,
and paresthesias. He says he is always concerned about when the next attack will occur. His
physical examination is unremarkable except for moderate obesity. Laboratory findings,
including a CBC, blood chemistry profile, and TSH level, reveal no abnormalities.
The most likely diagnosis is
A) mitral valve prolapse
B) paroxysmal supraventricular tachycardia
C) pheochromocytoma
D) generalized anxiety disorder
E) panic disorder

ANSWER: E
Panic disorder typically presents in late adolescence or early adulthood with unpredictable
episodes of palpitations, sweating, gastrointestinal distress, dizziness, and paresthesias. The
attacks are sporadic and last 10–60 minutes. Generalized anxiety disorder is more common,
and common symptoms include restlessness, fatigue, muscle tension, irritability, difficulty

141
concentrating, and sleep disturbance. Pheochromocytoma is associated with headache and
hypertension, and usually occurs in thin patients. Paroxysmal supraventricular tachycardia is
usually not associated with gastrointestinal distress or paresthesias. While mitral valve
prolapse can be associated with anxiety and panic disorder, the physical examination would
not be normal.
Ref: Kasper DL, Fauci AS, Hauser SL, et al (eds): Harrison’s Principles of Internal Medicine, ed 19. McGraw-Hill, 2015, pp 2708-
2709.

232. A 70-year-old male comes to your office for a wellness evaluation. He smoked for many
years, but quit 10 years ago. While searching through several health-related websites he read
that because of his smoking history he should undergo screening for an abdominal aortic
aneurysm. Which one of the following screening tests for abdominal aortic aneurysm is
recommended by the U.S. Preventive Services Task Force for this patient?
A) One-time low-dose abdominal CT
B) One-time diffusion-weighted MRI
C) One-time conventional abdominal duplex ultrasonography
D) Annual conventional abdominal duplex ultrasonography
E) No screening tests

ANSWER: C
The U.S. Preventive Services Task Force recommends one-time conventional abdominal
duplex ultrasonography for screening patients who are at risk of abdominal aortic aneurysm
(males 65–75 years of age who have smoked a total of 100 cigarettes or more during their
lifetime). This imaging modality has high sensitivity and specificity, and it is noninvasive,
easy to use, and low cost. None of the other imaging modalities have been formally evaluated
in clinical trials.
Ref: LeFevre ML; US Preventive Services Task Force: Screening for abdominal aortic aneurysm: US Preventive Services Task Force
recommendation statement. Ann Intern Med 2014;161(4):281-290. 2) Croswell J, Chu K: Screening for abdominal aortic aneurysm.
Am Fam Physician 2015;91(8):563-564.

233. A 35-year-old female presents to your office with a 2-week history of pain, mild
swelling, and point tenderness in the midportion of her anterior tibia. She has recently
increased her jogging from 6 miles per week to 30 miles per week. She is unable to hop on
the involved leg, as this induces significant pain at the site. A plain radiograph reveals a faint
lucency at the same location.
This patient would most likely benefit from which one of the following?
A) NSAIDs
B) Bisphosphonate therapy
C) Vitamin E supplementation
D) A bone stimulator
E) A walker boot

142
ANSWER: E
Repetitive high-intensity training places an individual at risk for developing a stress fracture.
A plain radiograph is the best initial test for a suspected stress fracture. If this is negative, then
the study should be repeated in 2–3 weeks. MRI is now considered the procedure of choice if
there is an urgent need for a diagnosis, although triple-phase bone scintigraphy has a similar
sensitivity.
Treatment of stress fractures usually consists of decreasing activity or, in some instances, such
as involvement of the anterior tibial cortex (where there is a risk of a complete fracture), non–
weight bearing with immobilization. Using a walker boot for tibial stress fractures reduces
the time to resumption of full activity. Most stress fractures should not be treated with a bone
stimulator. Supplementation with vitamin D and calcium has shown some benefit in
prevention, whereas bisphosphonates have not. NSAIDs are relatively contraindicated, as
limited studies have shown that they may actually inhibit healing of traumatic fractures.
Ref: Patel DS, Roth M, Kapil N: Stress fractures: Diagnosis, treatment, and prevention. Am Fam Physician 2011;83(1):39-46. 2) Davis
BJ, Estes AR: An adolescent with right shin pain. JAMA 2015;313(15):1566-1567.

234. A 42-year-old female continues to have elevated blood pressure while on three
antihypertensive agents. You are concerned that she may have idiopathic adrenal hyperplasia
or an aldosterone-producing adenoma.
Which one of the following would be the most appropriate laboratory test?
A) Serum cortisol
B) An aldosterone:renin ratio
C) 17-Hydroxyprogesterone
D) A dexamethasone suppression test
E) Renal ultrasonography

ANSWER: B
Up to 28% of patients may be defined as having resistant hypertension (not controlled on
three drugs or controlled on four or more drugs). Primary aldosteronism is present in up to
5%–10% of all hypertensive patients and 7%–20% of those with resistant hypertension. This
may be due to bilateral adrenal hyperplasia or a unilateral aldosterone-secreting adenoma,
which can be diagnosed if there is elevated serum aldosterone in the presence of suppressed
renin levels.
A cortisol level and a dexamethasone suppression test are appropriate tests for Cushing
syndrome. A 17-hydroxyprogesterone level tests for congenital adrenal hyperplasia. Renal
ultrasonography will not adequately screen for any of these conditions.
Ref: Viera AJ, Neutze DM: Diagnosis of secondary hypertension: An age-based approach. Am Fam Physician 2010;82(12):1471-1478.
2) Vongpatanasin W: Resistant hypertension: A review of diagnosis and management. JAMA 2014;311(21):2216-2224.

143
235. For the past 2 weeks a 16-year-old female has had an eruption on one breast,

She reports being troubled by the appearance of the lesions.


Of the following, the most appropriate management of this condition is to
A) curette each lesion to remove the top
B) apply a corticosteroid ointment locally twice daily for 10 days
C) advise the patient that the lesions will disappear in 2 weeks without treatment
D) prescribe penicillin, once daily for 10 days
E) order a serologic test for syphilis and treat if positive

ANSWER: A
This patient has molluscum contagiosum, which can be easily treated by curetting the lesions
and inducing an inflammatory reaction. The lesions may disappear spontaneously in a few
months; however, the best management of this condition in a patient who is bothered by it is
to induce resolution.
Ref: Habif TP: Clinical Dermatology: A Color Guide to Diagnosis and Therapy, ed 6. Elsevier, 2016, pp 219-223.

144
236. A 45-year-old male presents with what he suspects is a dislocated finger. He was playing
football with some friends 2 days ago and when he was trying to catch the ball it struck his
finger. On examination the distal interphalangeal (DIP) joint of the left fifth finger has full
passive range of motion, but he cannot actively extend the joint. A radiograph confirms an
avulsion fracture from the dorsum of the proximal fifth distal phalanx, with a mallet
deformity.
Which one of the following is the most appropriate treatment of this fracture?
A) Immobilization of the DIP joint in 30° flexion for 6–8 weeks
B) Immobilization of the DIP joint in extension for 6–8 weeks
C) Immobilization of both the DIP and proximal interphalangeal (PIP) joints in extension for
6–8 weeks
D) Referral for surgical pinning

ANSWER: B
The patient has a mallet fracture, an avulsion fracture of the distal phalanx with a bone
fragment on the terminal extensor tendon, resulting in unopposed flexion and the inability to
actively extend the distal interphalangeal (DIP) joint. Conservative treatment consists of
immobilization of the DIP joint in extension for 8 weeks and is recommended for most cases
of mallet fracture. If the joint is allowed to flex at any time during that period, then the
treatment period must be extended. Immobilization of the proximal interphalangeal (PIP)
joint is not required. Surgical pinning may be indicated in more complicated fractures or with
failure of conservative therapy.
Ref: Borchers JR, Best TM: Common finger fractures and dislocations. Am Fam Physician 2012;85(8):805-810

237. A 62-year-old male with severe COPD without hypoxia (based on spirometry within the
last year) comes to your office because of continued breathlessness. He has a 41-pack-year
smoking history but stopped 1 year ago and reports he is using his inhalers as prescribed,
which include albuterol (Proventil, Ventolin) as needed, budesonide/formoterol (Symbicort)
twice daily, and tiotropium (Spiriva) daily. He has previously demonstrated that he is able to
use the metered-dose inhalers appropriately. He is discouraged about his continued difficulty
breathing and asks if there is anything else that can be done to improve this problem. His
oxygen saturation on room air in the office is 92%.
Which one of the following would be most appropriate at this time?
A) Replace his inhaled medications with nebulized alternatives
B) Replace his tiotropium with ipratropium (Atrovent)
C) Order home oxygen
D) Refer for outpatient pulmonary rehabilitation

ANSWER: D

145
Pulmonary rehabilitation should be considered in patients with COPD who are optimally
medically managed and continue to have symptoms, particularly dyspnea. Pulmonary
rehabilitation improves exercise capacity, dyspnea, and health-related quality of life outcomes
in patients with COPD. Supplemental oxygen use has been shown to decrease mortality in
patients with COPD who have severe hypoxemia. Short-acting anticholinergics such as
ipratropium should be used for patients with mild disease requiring only as-needed
medications. The long-acting anticholinergic tiotropium has been shown to improve quality-
of-life scores.
Ref: Nici L, Lareau S, ZuWallack R: Pulmonary rehabilitation in the treatment of chronic obstructive pulmonary disease. Am Fam
Physician 2010;82(6):655-660. 2) Lee H, Kim J, Tagmazyan K: Treatment of stable chronic obstructive pulmonary disease: The
GOLD guidelines. Am Fam Physician 2013;88(10):655-663. 3) McCarthy B, Casey D, Devane D, et al: Pulmonary rehabilitation for
chronic obstructive pulmonary disease. Cochrane Database Syst Rev 2015;(2):CD003793.

238. A 68-year-old female who is a recent immigrant from Mexico is brought to your office
by her son with a complaint of headaches. The patient speaks English adequately, but diverts
her eyes to look at her son when answering your questions.
Which one of the following is the most likely reason for this patient not making eye contact?
A) Her son is overly controlling
B) She is a victim of abuse
C) She is being untruthful
D) She is showing respect to you
E) She is depressed

ANSWER: D
Nonverbal communication is important for identifying issues that a patient may be hiding or
be unwilling to divulge. Some nonverbal clues, however, are culturally based. Many older or
less-educated Mexican-Americans consider direct eye contact to be disrespectful. Because a
physician is held in high regard, these patients will often either look down or look at another,
more “equal” person in the room while being interviewed. Many Americans, on the other
hand, may consider a lack of eye contact to be negative, and that it indicates that a patient is
unsure of the information they are providing, has poor self-esteem, or is hiding something.
Ref: Purnell LD, Paulanka BJ: Guide to Culturally Competent Health Care. FA Davis Company, 2005, pp 339-351. 2) Rakel RE, Rakel
DP (eds): Textbook of Family Medicine, ed 9. Elsevier Saunders, 2016, pp 149-150.

239. Which one of the following is an absolute contraindication to combined oral


contraceptives in a 42-year-old female?
A) Varicose veins
B) Obesity (BMI >30.0 kg/m2)
C) Sickle cell disease
D) A history of ovarian cancer
E) Smoking 1 pack of cigarettes/day

146
ANSWER: E
In a female ≥35 years old, smoking 15 or more cigarettes per day poses an unacceptable health
risk with the use of combined oral contraceptives. Patients with varicose veins are not at
increased risk for deep vein thrombosis (DVT)/pulmonary embolism, which would be an
unacceptable health risk with the use of combined oral contraceptives. Combined oral
contraceptives actually lower the risk of ovarian cancer, and women may continue to use them
while awaiting treatment for ovarian cancer. Women with a BMI ≥30.0 kg/m2 who take oral
contraceptives are more likely to develop DVTs than those who do not use them, but the
advantages of oral contraceptives are considered to be greater than the disadvantages in these
patients, and obesity is not an absolute contraindication. A patient with sickle cell disease is
at a higher risk of adverse events from an unintended pregnancy than from the use of
combined oral contraceptives.
Ref: Curtis KM, Tepper NK, Jatlaoui TC, et al: US medical eligibility criteria for contraceptive use, 2016. MMWR Recomm Rep
2016;65(RR-3):1-103.

240. Which one of the following descriptions of an injury by an athlete is most consistent
with an isolated posterior cruciate ligament tear?
A) Hearing a pop in the knee during a pivoting motion
B) An immediate onset of pain after cutting on the knee
C) A direct blow to the anterior tibia while the knee is in flexion
D) Forceful twisting of the knee
E) An onset of pain after a running jump

ANSWER: C
An understanding of the anatomy and function of the components of the knee, coupled with
a clear description of the traumatic event, is essential for making an accurate initial clinical
assessment of sports-related knee injuries. The posterior cruciate ligament (PCL) connects
the medial femoral condyle to the posterior intercondylar area of the tibia and is affixed in
such a way that the anterolateral section is taut in flexion and the posteromedial section is taut
in extension, helping to maintain the correct anatomic relationship between the femur and
tibia. The PCL alone provides almost all of the resistance to posterior displacement of the
tibia and so is appropriately the strongest of the cruciate ligaments. Tearing or rupture of the
PCL can occur with hyperextension, hyperflexion, or rotation applied with a force that is so
great that other knee components are also generally injured.
The most common mechanism leading to an isolated injury of the PCL is a direct blow to the
anterior tibia with the knee in flexion, like that experienced when the proximal tibia impacts
the dashboard in an automobile crash or when an athlete is hit or kicked in the proximal tibia
while the knee is in flexion. Athletes with a PCL injury frequently complain of posterior knee
pain and pain when kneeling. The presence of painful limitation of flexion and a posterior sag
sign (posterior drawer sign) on examination strongly supports a diagnosis of isolated PCL
injury.

147
Ref:Grover M: Evaluating acutely injured patients for internal derangement of the knee. Am Fam Physician 2012;85(3):247-252.

148
‫‪American Board of Family Medicine‬‬

‫‪2017 Questions and answer‬‬

‫تم جمع االسئلة واألجىبة في ملف واحد لتسهيل المذاكزة‬


‫مع تحيبت متدربي بزنبمج سمبلة طب األسزة في صحة الشزقية‬

‫د‪.‬ووري الدوسري‬ ‫د‪. .‬حمد العمري‬


‫د‪.‬هادي ال قرٌه‬ ‫د‪.‬خدٌجة الفاضلً‬
‫د‪.‬افىان العواد‬ ‫د‪.‬عبدالرحمه الحربً‬
‫د‪.‬داوود الىاصر‬ ‫د‪.‬فٍصل باطوٌل‬
‫د‪.‬ابرار العٍسى‬ ‫د‪.‬دمحم الراضً‬
‫د‪.‬اٌمان البشر‬ ‫د‪.‬وجالء الدوسري‬
‫د‪.‬أشواق المري‬ ‫د‪.‬زٌىب الراشد‬
1. A 3-year-old male is brought to your office for a well child visit. The history indicates that the child has
a vocabulary of about 100 single words and has not begun to speak in 2-word phrases. An ear
examination is normal and the parents have no concerns regarding the child’s hearing. They report that
the child is interacting with others normally.

Which one of the following would you propose?

A) Reassessment in 6 months
B) Limiting screen time to 2 hours per day
C) A 4-week intensive parent-administered reading program
D) Referral to a local early developmental intervention program

Item 1
ANSWER: D
In a 3-year-old, red flags that would suggest the need for immediate speech-language evaluation include
the inability to understand prepositions or action words or the child having a vocabulary that consists of
less than 200 words (SOR C). A child should use 2-word phrases by 2½ years of age.
The evaluation should be performed through a local early developmental intervention program or a
speech-language pathologist. The therapeutic response to parent-administered programs varies greatly,
with programs lasting longer than 8 weeks having more success. Limiting screen time would not address
this child’s problem. Many family physicians would implement a parent-completed developmental
survey such as Ages and Stages.

2. A previously healthy 29-year-old pediatric nurse has a 3-day history of malaise, arthralgias, and a
nonpruritic rash. The rash is a faint, maculopapular, irregular, reticulate exanthem that covers her thighs
and the inner aspects of her upper arms. Symmetric synovitis is present in several distal and proximal
interphalangeal joints and in her metacarpophalangeal joints. Small effusions, warmth, and tenderness are
noted in her left wrist and right elbow. No other joints are affected.

The most likely cause of this problem is


A) varicella-zoster virus
B) measles (rubeola) virus
C) parvovirus B19
D) adenovirus
E) HIV

2
Item 2
ANSWER: C
Also known as erythema infectiosum or fifth disease, parvovirus B19 infection is a fairly common cause of
an exanthematous rash and arthritis in younger women. This infection should be particularly suspected
in health care workers who have frequent contact with children. By the age of 15 approximately 50% of
children have detectable IgG antibodies to the virus, and this figure rises to 90% in the elderly. Within
households and caregivers the secondary infection rate, especially among nonimmune children and
young adults, approaches 50%. The specific characteristics of the rash, the pattern of joint involvement,
and the place of employment in an otherwise healthy person all offer clues suggesting parvovirus B19 as
the infecting agent. Measles virus, adenovirus, and HIV rarely cause arthritis, although HIV infection
can cause a musculoskeletal syndrome later in the disease. Varicella-zoster virus may cause large-joint
arthritis, but the rash is distinctively vesicular and pruritic.

3. A previously healthy 5-year-old male is brought to your office because of painful swelling in his neck.
His mother noted the swelling about 3 or 4 days ago and it has grown rapidly. This morning he was sent
home from school with a temperature of 101.2°F.
On further questioning the patient reports no other symptoms and his mother confirms he has not
complained of anything besides the pain in his neck. He has been eating well and has had no weight loss.
They do not have a cat and he has not been around any cats. He is up to date on immunizations.
A physical examination reveals normal vital signs with the exception of a temperature of 38.5°C
(101.3°F). He has a tender, erythematous, slightly fluctuant, enlarged lymph node in the left anterior
cervical chain. He has no other enlarged lymph nodes and the examination is otherwise unremarkable.

Which one of the following would be the most appropriate next step?
A) Monitoring for up to 4 weeks to see if the problem resolves
B) Empiric antibiotic therapy directed at Staphylococcus aureus and group A Streptococcus
C) CT of the neck
D) Referral for a fine-needle aspiration biopsy of the enlarged lymph node
E) Referral to a hematologist for evaluation of lymphadenopathy

Item 3
ANSWER: B
Cervical lymphadenopathy in children may be due to several causes, and the evaluation should focus on
a complete history and physical examination to determine if observation or more urgent evaluation is
indicated. When signs of infection are present it is appropriate to treat the patient with antibiotics, with
evaluation for improvement in 2–3 days. If there are signs of malignancy (size >3 cm; a hard, firm,
immobile mass; associated type B symptoms) the child should be referred urgently to an
otolaryngologist. Fine-needle aspiration (FNA) of a lymph node can be helpful in some circumstances,
but the initial evaluation should determine whether referral for excision may be needed, particularly if
the history and examination suggest malignancy. In this patient, an infection is most likely and FNA

3
would not be indicated at this time. If imaging is needed in children under the age of 14, the
recommended initial study is ultrasonography. For those over 14 years of age, CT is recommended.

4. A 65-year-old Hispanic male with known metastatic lung cancer is hospitalized because of a 2-week
history of decreased appetite, lethargy, and confusion. Laboratory evaluation reveals the following:

Serum calcium 15.8 mg/dL (N8.4–10.0)


Serum phosphorus 3.9 mg/dL (N 2.6–4.2)
Serum creatinine 1.1 mg/dL (N 0.7–1.3)
Total serum protein 5.0 g/dL (N 6.0–8.0)
Albumin 3.1 g/dL (N 3.7–4.8)

Which one of the following would be the most appropriate INITIAL management?
A) Calcitonin-salmon (Miacalcin) subcutaneously
B) Pamidronate by intravenous infusion
C) Normal saline intravenously
D) Furosemide intravenously

Item 4
ANSWER: C
The initial management of hypercalcemia of malignancy includes fluid replacement with normal saline to
correct the volume depletion that is invariably present and to enhance renal calcium excretion. The use
of loop diuretics such as furosemide should be restricted to patients in danger of fluid overload, since
these drugs can aggravate volume depletion and are not very effective alone in promoting renal calcium
excretion. Although intravenous pamidronate has become the mainstay of treatment for the
hypercalcemia of malignancy, it should be considered only after the patient has been made euvolemic by
saline repletion. The same is true for the other calcium-lowering agents listed.

5. A 40-year-old white male has seen you in the past for hypertension and alcohol abuse. He comes to your
office for an acute visit due to a swollen, painful right knee that developed over the past few days. He
says he has been treated for gout flares in the past by another physician. He tells you his pain has always
been in his knee, and he always has marked swelling of the knee when the pain is severe. He has never
had any other swollen joints. The fluid from his knee has never been evaluated to his knowledge,
although it has been drained, which resulted in pain relief. No past record of a uric acid level is found,
and he does not recall having it checked. He has no fever, no systemic signs of illness, and no injury to
his knee.

A physical examination reveals a swollen knee with no evidence of internal derangement, no erythema,
and no other swollen joints. No nodules are appreciated. A knee radiograph reveals only the swelling
within the joint. His uric acid level is 5.1 mg/dL (N 4.0–8.0).

4
Which one of the following would be most appropriate at this point?

A) Dietary changes to decrease the risk of gout flares


B) Allopurinol (Zyloprim) for gout flare prevention
C) Colchicine (Colcrys) for the gout flare
D) Corticosteroid injection into the painful knee
E) Aspiration and evaluation of fluid from the knee joint, including crystal analysis

Item 5
ANSWER: E
Although this patient reports a history of gout, the diagnosis is not entirely clear. Gout can be diagnosed
clinically if at least six of the following findings are present:
• asymmetric swelling within a joint on radiography • an attack of monoarticular arthritis • a joint fluid
culture that is negative for microorganisms during an attack of joint inflammation • development of
maximal inflammation within 1 day • hyperuricemia • joint redness • more than one attack of acute
arthritis • pain or redness in the first metatarsophalangeal joint • a subcortical cyst without erosions on
radiography • a suspected tophus • a unilateral attack involving the tarsal joint
In the absence of a diagnosis based on clinical criteria, the diagnosis can be confirmed by the presence of
characteristic urate crystals in the joint fluid or the presence of a tophus proven to contain urate crystals
by chemical means or polarized light microscopy. As this patient does not meet these clinical criteria, it
would be appropriate to further evaluate whether his symptoms are truly from gout. It may also be
reasonable to start treatment while studies are pending, but the diagnosis should be confirmed. Should
gout be confirmed, dietary changes are recommended and allopurinol is a reasonable option for
preventing future flares.

6. Which one of the following patients should be tested for Helicobacter pylori infection and, if positive,
treated with eradication therapy without endoscopy?
A) A 45-year-old male with a 2-month history of epigastric burning after eating
B) A 45-year-old male who has progressive epigastric pain with associated anorexia and weight loss
C) A 53-year-old female with a 6-week history of burning in the chest after eating
D) A 60-year-old female with a 2-month history of constant epigastric burning
E) A 60-year-old male who takes daily aspirin and has developed epigastric burning and associated
vomiting over the past month

Item 6
ANSWER: A
The test-and-treat strategy is appropriate for patients with dyspepsia who are younger than 55 years of
age and have no alarm symptoms for gastric cancer. Testing for Helicobacter pylori in patients with
GERD is not recommended.

5
7. In patients with an acute ST-elevation myocardial infarction, fibrinolysis may be preferred as a
reperfusion strategy over percutaneous coronary intervention (PCI), depending on factors such as when
the symptoms began and the transport time to the nearest PCI-capable hospital. In which one of the
following situations would fibrinolysis be most appropriate?
A) An onset of symptoms 6 hours ago and a transport time of more than 2 hours
B) An onset of symptoms 10 hours ago and a transport time of approximately 30 minutes
C) An onset of symptoms 20 hours ago and evidence of ongoing ischemia when presenting to a PCI-capable
hospital
D) A sudden onset of symptoms on day 2 of a hospitalization for an ischemic stroke and a transport time of
more than 2 hours

Item 7
ANSWER: A
Once an ST-elevation myocardial infarction is identified, a reperfusion strategy should be chosen as quickly
as possible. In general, percutaneous coronary intervention (PCI) is preferred because it leads to
improved outcomes compared to fibrinolysis when performed in high-volume medical facilities without
treatment delays. If a patient’s first medical contact is at a PCI-capable hospital or the time from first
medical contact to device time is less than 120 minutes, PCI is the preferred intervention for patients
presenting with symptoms for less than 12 hours. If the transfer time to a PCI-capable hospital is not
short, such as the example of a patient with an onset of symptoms 6 hours ago and a transport time to the
nearest PCI-capable hospital of more than 2 hours, then fibrinolysis is the preferred management
strategy.
If there are absolute contraindications to fibrinolysis (such as a history of an ischemic stroke within the past 3
months), then PCI is preferred even if the transport time will not be short. In cases where the onset of
symptoms was more than 12 hours ago but less than 24 hours ago and evidence of ongoing ischemia
exists, it is still reasonable to pursue reperfusion therapy, and PCI would be the preferred strategy if it is
available.

8. Which one of the following hypoglycemic medications is proven to reduce mortality rates in patients
with type 2 diabetes mellitus?
A) Acarbose (Precose)
B) Glipizide (Glucotrol)
C) Metformin (Glucophage)
D) Rosiglitazone (Avandia)

Item 8
ANSWER: C
Metformin has been shown to reduce mortality rates in patients with type 2 diabetes mellitus (SOR A).
Acarbose, an -glucosidase inhibitor, reduces the risk of cardiovascular events, including myocardial
infarction, in patients with impaired glucose tolerance or type 2 diabetes mellitus (SOR B).

6
Rosiglitazone has been shown to be associated with an increased risk of myocardial infarction and death
from cardiovascular causes (SOR A). To date, there is insufficient evidence to make any conclusions
about the effect of sulfonylurea insulin secretagogues such as glipizide on cardiovascular morbidity and
mortality (SOR B).

9. A 74-year-old male with a history of diabetes mellitus, hypertension, and heart failure presents to the
clinic with shortness of breath with an unknown etiology. Laboratory results that show an elevated
procalcitonin level would help differentiate systolic heart failure from
A) diastolic heart failure
B) bacterial pneumonia
C) acute coronary syndrome
D) pulmonary embolus

Item 9
ANSWER: B
Procalcitonin is a biomarker that is elevated with bacterial infections but not with viral infections. The
laboratory test for procalcitonin has a high sensitivity and can help exclude bacterial pneumonia in
patients with acute heart failure, which can help expedite appropriate therapy with antibiotics. If the
procalcitonin level is low a bacterial infection is less likely and antibiotics should not be given.

10. A 38-year-old healthy female comes to your office for preventive care. Her medical history is significant
only for nephrolithiasis and controlled hypothyroidism. Her sexual history is significant for vaginal
intercourse with three male partners in the past year, including her current partner who uses intravenous
heroin. She has no fever, sore throat, swollen nodes, or other signs of acute illness. A fourth-generation
HIV antibody/antigen test is negative and her serum creatinine level is 0.6 mg/dL (N 0.5–1.1).

Which one of the following additional test results must be documented before offering preexposure
prophylaxis for HIV with emtricitabine/tenofovir (Truvada)?
A) Hepatitis B surface antibody and antigen
B) Hepatitis C antibody
C) Liver enzymes
D) HIV viral load
E) Her sexual partner’s HIV status

Item 10
ANSWER: A
Emtricitabine/tenofovir is the only currently approved regimen shown to be effective for HIV
preexposure prophylaxis (PrEP) (SOR A). This patient has multiple sexual partners, including one at
high risk for HIV infection due to intravenous drug use, and thus should be offered PrEP (SOR C). In

7
patients without signs of acute HIV, PrEP may be initiated after documentation of negative fourth-
generation HIV antibody/antigen testing, normal renal function, and hepatitis B infection and
immunization status. Tenofovir can be toxic to the kidneys and is not recommended in patients with an
estimated glomerular filtration rate <60 mL/min/1.73 m2.
Emtricitabine and tenofovir are both also active against hepatitis B virus (HBV) infection, so the use of
PrEP in patients with active HBV must be carefully considered. If a patient with active HBV stops
taking PrEP, reactivated HBV can cause liver damage. Patients susceptible to HBV infection should be
immunized. Hepatitis C testing would be prudent in this case, but the results are not needed to begin
therapy with emtricitabine/tenofovir.

11. .An 8-year-old female with a history of persistent asthma is interested in participating in a mile-long race
for a school fundraiser. Her mother is very concerned and thinks she should not run, and she brings in a
form to fill out to excuse the child from participation. The child states that she wants to participate. Her
medications include fluticasone (Flovent HFA), 44 g twice daily; loratadine (Claritin), 5 mg daily; and
albuterol (Proventil, Ventolin) as needed. She has no nighttime symptoms, has not used her inhaler at all
in the past week, and can keep up with other children during recess.

The best course of action is to

A) fill out the form as requested with no change in medication


B) recommend that she take 2 puffs of albuterol 30 minutes prior to the event and let her run
C) increase the fluticasone dosage to 110 g twice daily and let her run
D) add montelukast (Singulair) to her regimen and let her run
E) add a long-acting -agonist to her regimen and let her run

Item 11
ANSWER: B
This child has well controlled asthma, as evidenced by her normal daily activities, lack of nighttime
symptoms, and limited use of her rescue inhaler. There is no reason that the diagnosis of asthma should
limit her activities. Because her asthma is well controlled there is no need to add additional medications
or increase the dosage of her current medications. Long-acting -agonists are not recommended before
the age of 12. Prophylactic pretreatment with a short-acting -agonist has very little harm associated with
it and may prevent the need for a rescue inhaler during an athletic event. Children should have ready
access to their rescue inhalers at school and in other settings; this has been shown to reduce emergency
department visits.

8
12. A 28-year-old white male presents with pain in the right wrist since falling 2 weeks ago. On examination
he is tender in the anatomic snuffbox. A radiograph reveals a nondisplaced fracture of the distal third of
the carpal navicular bone (scaphoid).
Which one of the following would be most appropriate at this time?
A) A bone scan
B) A referral for physical therapy
C) A Velcro wrist splint
D) A short arm cast
E) A thumb spica cast

Item 12
ANSWER: E
Scaphoid fracture is the most common carpal bone injury. This injury tends to occur when the wrist is
hyperextended and the hand is pronated and radially deviated. The presentation can range from disabling
wrist pain to mild swelling and decreased range of motion. There is exquisite tenderness in the anatomic
snuffbox with axial loading of the thumb or a pincer grasp.
Radiographs should include PA, lateral oblique, and ulnar deviated views. Nondisplaced fractures can be
missed on radiographs, and if a fracture is suspected and the initial radiographs are negative for fracture,
the management is the same as it would be for a fracture until a fracture can be ruled out by advanced
imaging or by follow-up radiographs in 7 days. Treatment decisions depend upon fracture location and
displacement. A thumb spica cast (a short arm cast with the thumb immobilized) for 6–10 weeks is
appropriate for nondisplaced distal fractures. Surgical treatment should be considered for displaced or
proximal fractures.
There is some controversy about the wrist position for immobilization, whether neutral, in extension, or
in flexion, but the key treatment is cast immobilization. Follow-up evaluations should take place every
2–3 weeks, including out-of-cast radiographs, until union is confirmed. Nonunion occurs in
approximately 10% of all scaphoid fractures. It is more common with proximal scaphoid fractures due to
the precarious reverse blood supply. If union fails to occur the patient should be referred to an
orthopedist.

13. A 65-year-old male with a 40-pack-year history of smoking presents with shortness of breath on exertion.
Spirometry reveals the following:
FEV1/FVC 65% of predicted
FVC normal
Bronchodilator therapy no improvement in FEV1

Which one of the following is suggested by these results?


A) Normal spirometry
B) Reversible obstructive lung disease
C) Irreversible obstructive lung disease
D) Restrictive lung disease
E) Mixed obstructive and restrictive lung disease
9
Item 13
ANSWER: C
These spirometry results indicate an irreversible obstructive pattern. Patients with a restrictive
component to their lung disease have a decreased FVC. Reversible obstruction improves with
bronchodilator therapy.

14. A 52-year-old male presents with moderate symptoms of prostatism. A prostate examination is normal.
His post-void residual volume is 90 mL. His PSA level is 0.75 ng/mL (N 0.0–4.0). He says his nocturia
has become troublesome and you decide to initiate therapy.

This patient does NOT meet the criteria for use of which one of the following?

A) Doxazosin (Cardura)
B) Finasteride (Proscar)
C) Tadalafil (Cialis)
D) Tamsulosin (Flomax)
E) Silodosin (Rapaflo)

Item 14
ANSWER: B
Pharmacologic options for benign prostatic hyperplasia and lower urinary tract symptoms include an -
adrenergic blocker, a 5- -reductase inhibitor (if there is evidence of prostatic enlargement or a PSA level
>1.5 ng/mL), a phosphodiesterase-5 inhibitor, or antimuscarinic therapy. The first three have proven
efficacy as monotherapies.

15. A 30-year-old male presents with intermittent right upper quadrant pain after meals. He has been in
moderate pain for the past 3 hours. On examination the patient’s vital signs are normal except for a
temperature of 38.1°C (100.6°F). Examination of the abdomen reveals a positive Murphy’s sign.

Laboratory Findings

WBCs 8100/mm3 (N 4300–10,800)


ALT (SGPT) 42 U/L (N 10–55)
AST (SGOT) 28 U/L (N 10–40)
Alkaline phosphatase 128 U/L (N 45–115)
Bilirubin 1.0 mg/dL (N 0.0–1.0)
Lipase 12 U/dL (N 3–19)

Ultrasonography reveals cholelithiasis, an enlarged gallbladder, and thickening of the gallbladder wall.
10
The diameter of the common bile duct is normal.
Which one of the following is the most likely cause of this patient’s symptoms?
A) Choledocholithiasis
B) Acute cholecystitis
C) Acute cholangitis
D) Gallstone pancreatitis

Item 15
ANSWER: B
Gallstones are often asymptomatic and are found incidentally on imaging. However, they may become
symptomatic, usually causing pain in the right upper quadrant or epigastrium. Most patients with
symptomatic gallstones present with chronic cholecystitis, which causes recurrent attacks of pain. The
pain is constant and increases in severity at the beginning, and lasts from 1 to 5 hours. It often starts
during the night after a fatty meal and may be associated with nausea and vomiting. Abdominal
ultrasonography is the initial imaging method.
Patients with acute cholecystitis may have a history of symptoms consistent with chronic cholecystitis. With
acute cholecystitis, however, the pain does not remit and may last for days. The patient may also have a
fever on examination, and may have tenderness to deep palpation of the right subcostal area, known as
Murphy’s sign. The WBC count may be somewhat elevated. Ultrasonography will show thickening of
the bile duct wall (>4 mm).
Stones in the bile duct, or choledocholithiasis, typically lead to elevated transferase levels and bilirubin
levels that are elevated but <15 mg/dL. The pain may be either mild or severe, and may be intermittent
because of movement of the stones. Fever may be present. A bile duct >8 mm on ultrasonography in a
patient with gallstones, jaundice, and biliary pain indicates that stones may be present in the duct.
The two main complications of choledochal stones are cholangitis and pancreatitis. Acute cholangitis is a
bacterial infection. Bacterial growth is enhanced by obstruction of the duct. It may present as a mild self-
limited disease but can also lead to sepsis. Cases typically present with fever, pain, and jaundice.
Laboratory findings include an elevated WBC count, elevated bilirubin, and elevated transaminases and
alkaline phosphatase. Ultrasonography will show a dilated bile duct in many cases, although it might not
be dilated in acute obstruction.
Pancreatitis presents with pain, nausea, and vomiting. The pain is usually epigastric and radiates to the back.
It reaches its maximum intensity within an hour and may last for days. The physical examination may
reveal tachycardia, hypotension, tachypnea, and fever. The abdomen may be distended and is typically
tender to palpation. The diagnosis requires two of three primary features: abdominal pain, elevation of
serum amylase or lipase, and findings on imaging studies that are consistent with the diagnosis.
Ultrasonography can show pancreatic enlargement or edema, and visualization of gallstones will suggest
choledocholithiasis as the cause of the pancreatitis.

16. A 62-year-old male with diabetes mellitus recently underwent angioplasty with placement of a drug-
eluting stent for the treatment of left main coronary artery disease and acute coronary syndrome. The
patient is not considered at high risk for bleeding and you initiate dual antiplatelet therapy with aspirin and
clopidogrel (Plavix).

11
For how long should this patient continue dual antiplatelet therapy?

A) 1 month
B) 3 months
C) 6 months
D) 9 months
E) At least 12 months

Item 16
ANSWER: E
Dual antiplatelet therapy should extend beyond 1 year for patients with acute coronary syndrome who
are not considered at high risk of bleeding, especially those with risk factors associated with high
ischemic risk such as diabetes mellitus, peripheral artery disease, left main stenting, or a history of a
cardiovascular event. For dual antiplatelet therapy that continues beyond a year, either ticagrelor, 60 mg
twice daily, or clopidogrel, 75 mg daily, is recommended in addition to aspirin. The patient’s bleeding
and ischemic risk should be reevaluated at least annually.
Dual antiplatelet therapy should continue for at least 1 year in patients who are considered at high risk of
bleeding. For patients who are at very high risk of bleeding or who experience significant bleeding while
on dual antiplatelet therapy, a duration of less than 1 year is recommended.

17. A 67-year-old female sees you because of a cough she has had for the past few days and a fever that
started today. She is short of breath and generally does not feel well. She has no history of lung disease
and is a nonsmoker. Her medical history is significant for hypertension, hyperlipidemia, and type 2
diabetes mellitus, all of which are well managed with medications and diet.
A physical examination reveals a mildly ill-appearing female with a temperature of 38.2°C (100.8°F), a
pulse rate of 90 beats/min, a respiratory rate of 21/min, a blood pressure of 110/60 mm Hg, and an
oxygen saturation of 98% on room air. Her heart has a regular rhythm and her respirations appear
unlabored. She has rhonchi in the left lower lung field but has good air movement overall. A chest
radiograph reveals a left lower lobe infiltrate.

Which one of the following is the most appropriate setting for the management of this patient’s
pneumonia?

A) Home with close monitoring


B) An inpatient medical bed without telemetry monitoring
C) An inpatient medical bed with telemetry monitoring
D) An inpatient intensive care bed

12
Item 17
ANSWER: A
For community-acquired pneumonia, an important decision point is the severity of illness that indicates the
need for inpatient care. There are multiple tools for evaluation of pneumonia severity, including
SMART-COP (predicts the likelihood of the need for invasive ventilation or vasopressor support), the
Pneumonia Severity Index (predicts the risk of 30-day mortality and the need for admission to the
intensive-care unit), and CURB-65 or CRB-65. In an outpatient setting, CURB-65 and CRB-65 are easy
to use, although they have weaker predictive values for 30-day mortality. In addition, clinical judgment
should always be used. In this scenario, the patient does not clinically appear markedly ill, and her vital
signs and physical examination do not fit any criteria for increased risk in any of the scoring systems.
-65 or CRB-65 can
be managed as outpatients.

18. A 67-year-old female reports hearing a ringing sound when she is in a quiet room. The ringing is not
bothersome to her, but she wonders what is causing it. She has not noticed any hearing loss.
According to the American Academy of Otolaryngology—Head and Neck Surgery, neurologic imaging
(such as contrast-enhanced MRI of the brain) would be indicated if:
A) the patient requests imaging
B) the tinnitus is nonpulsatile
C) the tinnitus is unilateral
D) treatment with an antidepressant such as fluoxetine (Prozac) fails
E) audiology testing identifies symmetric, mild, high-frequency hearing loss

Item 18
ANSWER: C
Tinnitus that is bilateral and not bothersome can be treated conservatively with cognitive-behavioral
therapy, sound therapy, and, if appropriate, hearing aids. Antidepressants are not recommended.
Pulsatile tinnitus, unilateral tinnitus, or tinnitus associated with asymmetric hearing loss is more likely to
be associated with a pathologic cause. Symmetric, mild, high-frequency hearing loss is common in
elderly patients. Imaging should not be part of the routine management of tinnitus that does not have
warning signs, and patients should be counseled on conservative measures as described.

19. A pregnant 30-year-old female comes to your office in October and asks if she should receive influenza
vaccine. She is at 12 weeks gestation by dates and has a past history of a severe egg allergy.

Which one of the following would you recommend?


A) No immunization, because of the pregnancy
B) No immunization, because of the egg allergy
C) Standard inactivated seasonal vaccine now
D) High-dose inactivated seasonal vaccine now
E) Immunization during the second trimester

13
Item 19
ANSWER: C
Influenza vaccine is recommended for pregnant women regardless of trimester. This gives protection to
the mother, and the infant may be protected up to 6 months. High-dose vaccines are recommended
starting at age 65. An egg allergy is not a contraindication, but the vaccine should be given in a health
care setting.

20. A 2-year-old child stumbles, but his mother keeps him from falling by pulling up on his right hand. An
hour later the child refuses to use his right arm and cries when his mother tries to move it.

The most likely diagnosis is

A) dislocation of the ulna


B) dislocation of the olecranon epiphysis
C) subluxation of the head of the radius
D) subluxation of the head of the ulna
E) anterior dislocation of the humeral head

Item 20
ANSWER: C
Subluxation of the radial head, or nursemaid’s elbow, is one of the most common injuries in children under 5
years of age. It occurs when the child’s hand is suddenly jerked up, forcing the elbow into extension and
causing the radial head to slip out from the annular ligament.

21. A 56-year-old female comes to your office for evaluation of fatigue and shortness of breath. She has a
history of type 2 diabetes mellitus, hypertension, hyperlipidemia, and obesity. Her diabetes has been well
controlled, and a recent hemoglobin A1c was 6.7%.
She reports that she has been more tired than usual for the past several months and that walking more
than a block or going up a flight of stairs has now become difficult. She has no chest pain, palpitations,
dizziness, or cough. She has had mild, stable lower extremity edema for years, and this is unchanged.
She lives alone and is not sure if she snores. She has had difficulties with sleep for years and does not
feel refreshed upon awakening. She does not use tobacco or drink alcohol.

On examination she has a blood pressure of 128/78 mm Hg, a pulse rate of 76 beats/min, a respiratory
rate of 14/min, a temperature of 37.1°C (98.8°F), an oxygen saturation of 95% on room air, and a BMI
of 38.2 kg/m2. Auscultation of the heart reveals a regular rate and rhythm with no murmur. Her lungs
are clear to auscultation bilaterally. She has 1+ pitting edema of both lower extremities.

A chest radiograph is normal and an EKG reveals normal sinus rhythm. Echocardiography shows a left
ventricular ejection fraction of 60% without impaired diastolic function.

14
Which one of the following evaluations is most likely to reveal the cause of her fatigue?

A) 24-hour ambulatory blood pressure monitoring


B) Spirometry
C) A sleep study
D) CT angiography of the chest
E) Left heart catheterization

Item 21
ANSWER: C

This patient has pulmonary hypertension that, based on her history, is most likely related to obstructive
sleep apnea (OSA). Most patients with pulmonary hypertension have an underlying disease of the heart
or lungs that leads to elevated pulmonary artery pressures. Common underlying conditions include
chronic
lung disease such as COPD, OSA, and left heart failure (with a reduced or preserved ejection fraction).
Additional considerations include chronic thromboembolic disease and primary pulmonary arterial
hypertension.
This patient’s obesity and unrefreshing sleep make OSA the likely underlying cause of her pulmonary
hypertension. She does not have clinical features of thromboembolic disease or a history of COPD. Her
echocardiogram does not show heart failure, and she has no symptoms to suggest obstructive coronary
disease. Ambulatory blood pressure monitoring can aid in the diagnosis and optimal treatment of
hypertension, but this would be unlikely to relate directly to her pulmonary hypertension.

22. A 2-year-old white male is seen for a well child visit. His mother is concerned because he is not yet able
to walk. A routine physical examination, including an orthopedic evaluation, is unremarkable. Speech
and other developmental milestones seem normal for his age.

Which one of the following tests would be most appropriate?

A) A TSH level
B) Random urine for aminoaciduria
C) Phenylketonuria screening
D) A serum creatine kinase level
E) Chromosome analysis

Item 22
ANSWER: D

The diagnosis of Duchenne muscular dystrophy, the most common neuromuscular disorder of
childhood, is usually not made until the affected individual presents with an established gait abnormality
at the age of 4–5 years. By then, parents unaware of the X-linked inheritance may have had additional
children who would also be at risk. The disease can be diagnosed earlier by testing for elevated creatine

15
kinase in boys who are slow to walk. The mean age for walking in affected boys is 17.2 months, whereas
over 75% of developmentally normal children in the United States walk by 13.5 months. Massive
elevation of creatine kinase from 20 to 100 times normal occurs in every young infant with the disease.
Early detection allows appropriate genetic counseling regarding future pregnancies.
Hypothyroidism and phenylketonuria could present as delayed walking. However, these diseases cause
significant intellectual disability and would be associated with global developmental delay.
Furthermore, these disorders are now diagnosed in the neonatal period by routine screening. Disorders of
amino acid metabolism present in the newborn period with failure to thrive, poor feeding, and lethargy.
Gross chromosomal abnormalities would usually be incompatible with a normal physical examination at
18 months of age.

23. A 20-year-old female with a history of persistent depression sees you for follow-up. She reports
unplanned weight loss, chronic insomnia, fatigue, a loss of interest in hobbies that she previously enjoyed,
and decreased concentration. She has seen other physicians and has tried antidepressant medications,
including paroxetine (Paxil), bupropion (Wellbutrin), and citalopram (Celexa). She reports that there was
a week when her mood felt much more normal. She needed only about 3 hours of sleep each night
during that week. She admits she was somewhat impulsive, however, and experienced racing thoughts.
She does not abuse any substances but has several family members who are drug abusers. Her mother
and older sister committed suicide. Her examination, laboratory work, and drug screen are all
unremarkable.

Which one of the following is most likely to help this patient?

A) Escitalopram (Lexapro)
B) Mirtazapine (Remeron)
C) Quetiapine (Seroquel)
D) Trazodone (Oleptro)
E) Venlafaxine (Effexor XR)

Item 23
ANSWER: C

Bipolar disorders often present in late childhood or early adolescence. Outcomes can be improved by
early recognition. Manic episodes that occur with bipolar I disorder are usually easy to identify.
However, patients with bipolar II disorder may have a hypomanic episode that goes unrecognized, and
the patient may present with persistent depression. There is usually a family history of bipolar disorder
or multiple relatives having persistent depression, obsessive-compulsive disorder, attention-deficit
disorder, or panic disorder. There may be multiple instances in the family of suicide, drug abuse, alcohol
abuse, or incarceration. The patient has likely failed to respond to at least three antidepressant drugs and
may have a history of multiple divorces.
There are options for the treatment of bipolar depression. Quetiapine usually leads to a response after 1
week of therapy but is associated with weight gain and extrapyramidal side effects. Olanzapine may also
be used but should be combined with an SSRI. Lithium may also be effective for acute depression.

16
Lamotrigine is effective but titration should be spread over at least 6 weeks in order to decrease the risk
of Stevens-Johnson syndrome.

24. A 62-year-old male with a history of COPD sees you because of fever, chills, and redness and swelling
in his right lower extremity that has been progressing. He has a temperature of 38.0°C (100.4°F), a blood
pressure of 112/72 mm Hg, and a pulse rate of 94 beats/min. The physical examination is remarkable for
an erythematous area with increased warmth that extends approximately 18 cm (7 in) between the knee
and ankle. You note no apparent abscess formation. The patient does not recall any specific injury that
could have caused this problem.

According to the guidelines of the Infectious Diseases Society of America, which one of the following
would be the most appropriate antibiotic regimen for this patient?

A) Ceftriaxone (Rocephin)
B) Piperacillin/tazobactam (Zosyn)
C) Vancomycin (Vancocin)
D) Vancomycin plus ceftriaxone
E) Vancomycin plus piperacillin/tazobactam

Item 24
ANSWER: A

The most common pathogen for nonpurulent cellulitis is -hemolytic streptococci. Guidelines from the
Infectious Diseases Society of America recommend treating moderate nonpurulent cellulitis with
penicillin, ceftriaxone, cefazolin, or clindamycin alone. Vancomycin would be indicated if the patient
had a history of illicit drug use, purulent drainage, concurrent evidence of MRSA infection elsewhere,
nasal colonization with MRSA, or severe cellulitis.

25. When you arrive at your office in the morning your nurse asks you to see an 80-year-old white female
who has come in without an appointment. The patient has a long history of hypertension and has felt
very nauseated and lightheaded since last night. She denies chest pain and dyspnea.
Physical Findings

Blood pressure 100/60 mm Hg


Temperature 36.5°C (97.7°F)
Pulse 40 beats/min
Respirations 18/min
Appearance generalized pallor
HEENT within normal limits
Chest bibasilar rales
Heart 40 beats/min; no gallop, no murmur
Abdomen soft, no masses
Rectal stool negative for occult blood
Extremities no edema
17
The patient’s EKG is shown on the following page. Which one of the following does the EKG show?

A) Pericarditis
B) Acute anteroseptal myocardial infarction
C) Acute inferior wall myocardial infarction
D) Idioventricular rhythm

Item #25

Item 25
ANSWER: C

This patient’s EKG shows marked ST-T elevation in the inferior leads consistent with an acute inferior
wall myocardial infarction. Pericarditis almost always presents with severe chest pain, and the ST
segment elevation is more diffuse. With anteroseptal infarction, ST elevation is seen only on leads V1–
V3.

26. The parents of a 40-day-old infant bring her to your clinic because she has had a persistent fever for the
past 2 days with rectal temperatures up to 101.0°F. She has been fussy and wants to be held, but has
been nursing well. She is crying when you enter the room, and on examination she has good skin turgor
and capillary refill. The examination does not reveal any obvious source of infection. By the time you
complete the examination the infant is resting quietly in her father’s arms.

You obtain a CBC and urinalysis. The WBC count is 12,500/mm3 (N 5000–19,500) with an absolute

18
neutrophil count of 8500/mm3 (N 1000–9000). The urinalysis is within normal limits.

Which one of the following would be most appropriate at this time?

A) Home care and parental observation only, as long as the temperature remains under 39.0°C (102.2°F)
B) Home care and reevaluation in 24 hours
C) Oral antibiotics and reevaluation in 24 hours
D) A complete sepsis workup, including blood cultures, stool studies, a chest radiograph, and cerebrospinal
fluid studies

Item 26
ANSWER: B

Most children will be evaluated for a febrile illness before 36 months of age, with the majority having a
self-limited viral illness. Nontoxic-appearing febrile infants 29–90 days of age who have a negative
screening laboratory workup, including a CBC with differential and a normal urinalysis, can be sent
home
and followed up in 24 hours (SOR B). A second option is to obtain blood cultures and stool studies, or a
chest film if indicated by the history or examination, and spinal fluid studies if empiric antibiotics are to
be given. This infant’s clinical status did not indicate that any of these additional studies should be
performed, and empiric antibiotic treatment is not planned.
Observation with no follow-up is an appropriate strategy in nontoxic children, but only if the child is 3–
36
months of age and the temperature is under 39.0°C (102.2°F) (SOR B).
Nontoxic children 3–36 months of age should be reevaluated in 24–48 hours if their temperature is
39.0°C. Although a positive response to antipyretics has been considered an indication of a lower risk
of serious bacterial infection, there is no correlation between fever reduction and the likelihood of such
an infection. Any infant younger than 29 days, and any infant or child with a toxic appearance regardless
of age, should undergo a complete sepsis workup and be admitted for observation until culture results
are obtained or the source of the fever is found and treated (SOR A).

27. A 56-year-old white male reports lower leg claudication that occurs when he walks
approximately one block and is relieved by standing still or sitting. He has a history of diabetes mellitus
and hyperlipidemia. His most recent hemoglobin A1c was 5.9% and his LDL-cholesterol level at that
time was 95 mg/dL. Current medications include glyburide (DiaBeta), metformin (Glucophage),
simvastatin (Zocor), and daily aspirin. He stopped smoking 1 month ago and began a walking program.
A physical examination is normal except for barely palpable dorsalis pedis and posterior tibial pulses.
Femoral and popliteal pulses are normal. Noninvasive vascular studies of his legs show an ankle-
brachial index of 0.7 bilaterally and decreased flow.

Which one of the following would be most appropriate for addressing this patient’s symptoms?

19
A) Fish oil
B) Warfarin (Coumadin)
C) Cilostazol (Pletal)
D) Dipyridamole (Persantine)
E) Clopidogrel (Plavix)

Item 27
ANSWER: C

The patient described has symptomatic arterial vascular disease manifested by intermittent claudication.
He has already initiated the two most important changes: he has stopped smoking and started a walking
program. His LDL-cholesterol is at target levels; further lowering is not likely to improve his symptoms.
In the presence of diffuse disease, interventional treatments such as angioplasty or surgery may not be
helpful; in addition, these interventions should be reserved as a last resort. Cilostazol has been shown to
help with intermittent claudication, but additional antiplatelet agents are not likely to improve his
symptoms. Fish oil and warfarin have not been found to be helpful in the management of this condition.
.

28. A 78-year-old female with advanced dementia is brought to your office by her family for a wellness
examination. Other than appearing chronically ill and showing significant cognitive impairment, she has
an unremarkable examination.

Which one of the following should be offered to this patient?

A) A clinical breast examination


B) A Papanicolaou test
C) Mammography
D) Colonoscopy
E) No cancer screening

Item 28
ANSWER: E

The Society of General Internal Medicine does not recommend cancer screening in adults with a life
expectancy of less than 10 years. Other organizations have similar recommendations for specific
cancers, usually based on a life expectancy of less than 10 years or an age greater than 65. For patients
who have had negative screening results for cervical cancer in the past, this screening may be stopped at
age 65. For patients who are 76–85 years of age, screening for colorectal cancer and breast cancer
should be performed on an individual basis, taking into account the patient’s overall health and
screening history.

20
29. A 4-year-old male is brought to your office by his maternal aunt, who is his new guardian. She is
concerned that he is exhibiting problems with behavior and attention. On examination you note long,
wide, protruding ears, an elongated face, and frontal bossing.

Which one of the following is the most likely cause of these dysmorphic features?

A) Angelman syndrome
B) Fragile X syndrome
C) Klinefelter syndrome
D) Marfan syndrome
E) Prader-Willi syndrome

Item 29
ANSWER: B

The prepubescent male child with fragile X syndrome can be recognized by large ears, an elongated
face,
macrocephaly, or frontal bossing. This dysmorphic presentation can be subtle in young children, with an
average age at diagnosis of 8 years. After puberty, a prominent jaw and macro-orchidism is
characteristic.
Although a child with Marfan syndrome has an elongated face, the frontal bossing and large ears are not
characteristic of that condition. Prepubescent boys with Klinefelter syndrome do not have facial
dysmorphic features. They appear similar to prepubescent boys with normal karyotypes. Facial
dysmorphic
features associated with Angelman syndrome include microbrachycephaly, maxillary hypoplasia, a large
mouth, and prognathism. Facial dysmorphic features associated with Prader-Willi syndrome include a
narrow distance between the temples, almond-shaped eyes, and a thin upper lip.

30. A 54-year-old female presents with painful sores in her mouth that appeared a few days ago. She has had
some trouble eating due to the pain, but she is able to swallow without difficulty. She also began to have
some pain around her right ear today. She has no fever, chills, nasal congestion, cough, or difficulty
hearing. Her medical history is significant only for an anxiety disorder treated with sertraline (Zoloft).
On examination her vital signs are all normal. You see vesicles on the right side of the hard palate and
she has a swollen, red right pinna, with vesicles in the external auditory canal.

The organism responsible for this condition is

A) coxsackievirus
B) Epstein-Barr virus
C) group A Streptococcus
D) herpes simplex virus
E) varicella zoster virus

21
Item 30
ANSWER: E

This patient has herpes zoster oticus, which is also known as Ramsay Hunt syndrome when associated
with a facial nerve palsy. It is caused by reactivation of the varicella-zoster virus (VZV) in the geniculate
ganglion of the facial nerve. Typical symptoms include painful vesicles on one side of the palate and the
ipsilateral ear. When the reactivation involves other branches of the facial nerve it can result in a
unilateral facial herpetiform rash that may also involve the anterior two-thirds of the tongue, taste
disturbance, and reduced lacrimation. If the nearby cochlear and vestibular nerves become involved,
patients may also experience hearing loss, tinnitus, nausea, vomiting, and vertigo. The diagnosis is
usually made clinically, but if confirmation is needed polymerase chain reaction testing of vesicular
fluid or of a swab of the base of an ulcer may be done. Treatment includes antivirals (acyclovir,
valacyclovir) and prednisone, and is more effective when started sooner in the course of illness. Herpes
simplex virus (HSV) can cause oral vesicles and ulcers, but the distribution of vesicles in the ear and the
mouth of this patient is not typical for HSV. Epstein-Barr virus can cause leukoplakia of the mouth but
not vesicles and is typically associated with systemic signs of illness. Group A Streptococcus causes
throat pain and fever, not vesicles. Coxsackievirus causes oral vesicles and ulcers but is usually
associated with fever and does not typically involve the ear.

31. A 75-year-old female with a 10-year history of type 2 diabetes mellitus presents with moderate bilateral
burning pain in the distal portion of her feet. Her hemoglobin A1c is 8.1%.
Which one of the following is recommended as first-line therapy for improvement of this patient’s pain?

A) Amitriptyline
B) Ibuprofen
C) Pregabalin (Lyrica)
D) Tramadol (Ultram)

Item 31
ANSWER: C

This patient’s condition is consistent with distal symmetric polyneuropathy (DSPN). It may be present in
up to 10%–15% of newly diagnosed patients with type 2 diabetes mellitus and in up to 50% of patients
within 10 years of diagnosis. Pregabalin or duloxetine is recommended as the initial approach in the
symptomatic treatment of neuropathic pain in diabetes (SOR A). There is no significant evidence
supporting glycemic control or lifestyle interventions as effective treatment for the condition. Narcotics,
including tramadol, are not first- or second-line choices, and although tricyclic antidepressants such as
amitriptyline are effective, they present a higher risk for serious side effects, especially in the elderly.
There are no recommendations for the use of NSAIDs.
.

32. A 50-year-old female reports vaginal dryness, burning, and pain with penetration during sexual

22
intercourse. On examination she is noted to have pale, dry vaginal epithelium that is smooth and shiny
with loss of most rugation.

Which one of the following treatments is most likely to be effective for her sexual dysfunction?

A) Cognitive-behavioral therapy
B) Vaginal estrogen
C) Testosterone therapy
D) Bupropion (Wellbutrin)
E) Sildenafil (Viagra)

Item 32
ANSWER: B

This patient has genitourinary syndrome of menopause (formerly termed vulvovaginal atrophy) based on
her symptoms and examination. Estrogen therapy is highly effective for dyspareunia related to
genitourinary syndrome of menopause, with the vaginal route preferred over systemic therapy if vaginal
dryness is the primary concern. Bupropion and sildenafil may benefit women with sexual dysfunction
induced by antidepressant medications. Data on the benefit of testosterone therapy is limited and
inconsistent and lacks long-term information about safety. Cognitive-behavioral therapy has been shown
to effectively treat low sexual desire, but does not affect the physiologic changes associated with
genitourinary syndrome of menopause.

33. A 62-year-old female with known systolic heart failure has a 2-month history of increased fatigue and
worsening shortness of breath with ambulation. She says she has adhered to her medication regimen.
Her oxygen saturation is 96% on room air and a physical examination is within normal limits.
Laboratory studies, chest radiographs, and an EKG are ordered. The echocardiogram shows an ejection
fraction of 35% and normal right heart function. Her estimated pulmonary pressure is 45 mm Hg.

The best option for treatment of her pulmonary hypertension at this point is to

A) add a vasodilator
B) begin oxygen therapy
C) recommend lifelong anticoagulation
D) maximize treatment for heart failure
E) schedule right heart catheterization

23
Item 33
ANSWER: D

This patient has pulmonary hypertension due to left heart failure. The recommended treatment is to
maximize treatment for her heart failure and any other comorbidities. Vasodilators are not recommended
in the treatment of pulmonary hypertension due to left heart failure and may be harmful (SOR C).
Oxygen therapy is recommended only for patients with hypoxia (SOR C). Lifelong anticoagulation is
recommended if pulmonary hypertension is due to chronic thromboembolic disease but not if it is due to
left heart failure (SOR C). Anticoagulation is not recommended in systolic left heart failure unless there
is another indication.
Right heart catheterization is not recommended for pulmonary hypertension due to left heart disease
because vasodilators are not a treatment option. Right heart catheterization is recommended in
pulmonary hypertension prior to initiating vasodilator therapy in appropriate patients (SOR C).

34. An 11-year-old female who plays in a local youth soccer league presents with right heel pain that has
persisted for several months. She does not recall a specific injury that could have caused the pain. On
examination the skin is intact and she is tender over the right posterior heel. Examination of the ankle
and forefoot is unremarkable and a neurovascular examination is normal. You decide that rest and
physical therapy would be the best initial management.

Which one of the following is CONTRAINDICATED for this patient?

A) Ice packs
B) Moist heat
C) Whirlpool therapy
D) An exercise prescription
E) Therapeutic ultrasound

Item 34
ANSWER: E

Calcaneal apophysitis, also called Sever’s disease, is a common cause of heel pain in young athletes,
especially those who participate in basketball, soccer, track, and other sports that involve running.
Typically the heel apophysis closes by age 15. Treatment options include activity modification, the use
of ice packs and/or moist heat, stretching, analgesics, and orthotic devices. The use of therapeutic
ultrasound on the active bone growth plates in children is contraindicated.

35. An 18-year-old female presents with a painful right ankle after twisting it during a basketball game. On
examination she has no tenderness over the lateral malleolus or posterior distal fibula, which she has
identified as the location of the pain.

24
According to the Ottawa ankle rules, which one of the following would indicate that an ankle radiograph
should be performed?

A) Moderate ankle swelling


B) The inability to bear weight on the right foot with the left foot elevated for 5 seconds
C) The inability to stand for 5 seconds with weight evenly distributed on both feet, at the time of injury and
during the evaluation
D) The inability to take 4 steps at the time of the injury and during the evaluation
E) The inability to walk 6 feet during the evaluation

Item 35
ANSWER: D

The Ottawa ankle rules are 99% sensitive and 58% specific for identifying a fracture. They state that
ankle radiography should be performed when a patient presents with pain in the malleolar region and has
either point tenderness over the tip of the malleolus or the posterior edge of the affected bone (distal 6
cm), or is unable to bear weight at the time of injury and while being evaluated in the emergency
department or office. Inability to bear weight is defined as the inability to take four steps. A limp when
weight is transferred to the affected extremity still counts as being able to bear weight.

36. A 70-year-old female presents with a complaint of recent fatigue and ―not feeling well.‖ She is unable to
provide further details about her fatigue but states that she has not felt like doing anything recently. She
does not have shortness of breath, chest pain, weight gain or loss, hair loss, or constipation. Her past
medical history is significant only for hypertension that is well controlled on hydrochlorothiazide. She is
a nonsmoker, does not drink alcohol, and has no history of drug use. She is retired and lives alone. Her
husband died 3 years ago and she has no family in the area but is involved in her church, although she
says she has not enjoyed her church activities over the past few months.

In addition to laboratory testing, which one of the following would be appropriate at this time?

A) Referral to Adult Protective Services


B) Evaluation with the Geriatric Depression Scale or the PHQ-9
C) Paroxetine (Paxil)
D) An EKG and stress testing
E) Pulmonary function testing

25
Item 36
ANSWER: B

While some workup may be indicated, it is also important to realize that depression may present with
fatigue and vague symptoms, particularly in the geriatric population. The Geriatric Depression Scale and
PHQ-9 are good screening tests for depression and would help to determine if more focus should be
placed on a mood disorder. If further evaluation leads to a diagnosis of depression an SSRI may be
indicated but paroxetine is not the first choice in the elderly because of its long half-life. In addition, its
concentrations are increased 70%–80% in the elderly compared to younger adults, its clearance is
decreased, and it has multiple side effects and drug interactions.
An EKG and stress testing are unlikely to be beneficial in a patient who has no significant symptoms and
few risk factors for coronary artery disease. Pulmonary function testing may be helpful later in the
evaluation if the initial workup does not reveal a diagnosis, but would not be indicated at this time given
the lack of respiratory symptoms. A referral to Adult Protective Services is not warranted for suspected
depression but would be an option if there were a reasonable suspicion of abuse in a patient and medical
causes have been ruled out.

37. A 23-year-old gravida 1 para 0 at 35 weeks gestation presents with a 2-day history of ankle swelling and
headache. She denies any abdominal pain or visual disturbances. On examination you note a fundal
height of 35 cm, a fetal heart rate of 140 beats/min, 2+ lower extremity edema, and a blood pressure of
144/92 mm Hg. A urine dipstick shows 1+ proteinuria. A cervical examination reveals 2 cm dilation,
90% effacement, –1 station, and vertex presentation.

You send her to labor and delivery triage for further evaluation. Over the next 4 hours she has a
reactive nonstress test and her blood pressure ranges from 142/90 mm Hg to 148/96 mm Hg.
Laboratory results show a urine protein to creatinine ratio of 0.4 (N <0.3), normal BUN and
creatinine levels, normal liver enzyme and LDH levels, normal hemoglobin and hematocrit levels,
and a platelet count of 95,000/mm 3 (N 150,000–350,000).

Which one of the following would be the most appropriate next step in the management of this patient?

A) A biophysical profile
B) Ultrasonography to check for fetal intrauterine growth restriction
C) Initiation of antihypertensive treatment
D) Immediate induction of labor
E) Immediate cesarean delivery

Item 37
ANSWER: D

This patient most likely has preeclampsia, which is defined as an elevated blood pressure and proteinuria
26
after 20 weeks gestation. The patient needs further evaluation, including a 24-hour urine for quantitative
measurement of protein or a spot urine protein to creatinine ratio, blood pressure monitoring, and
laboratory evaluation that includes hemoglobin, hematocrit, a platelet count, and serum levels of
transaminase, creatinine, albumin, LDH, and uric acid. A peripheral smear and coagulation profiles also
may be obtained. Antepartum fetal testing, such as a nonstress test to assess fetal well-being, would also
be appropriate. Ultrasonography should be performed to assess for fetal intrauterine growth restriction,
but only after an initial laboratory and fetal evaluation. Delivery is the definitive treatment for
preeclampsia. The timing of delivery is determined by the gestational age of the fetus and the severity of
preeclampsia in the mother. Vaginal delivery is preferred over cesarean delivery, if possible, in patients
with preeclampsia. It is not necessary to start this patient on antihypertensive therapy at this point. An
obstetric consultation should be considered for patients with preeclampsia.

38. A 46-year-old male comes to your office to discuss smoking cessation. You have advised him to quit
smoking at past visits, and after several months of contemplation he is now ready to address this
problem. He has smoked 1 pack of cigarettes per day for the past 25 years. He is highly motivated and
wants to use the most effective regimen.
In addition to behavioral counseling, which one of the following interventions is associated with the
most successful outcome?

A) ―Cold turkey‖ nicotine withdrawal


B) Electronic nicotine delivery systems
C) Hypnotherapy
D) Clonidine (Catapres)
E) Varenicline (Chantix)

Item 38
ANSWER: E

The U.S. Preventive Services Task Force (USPSTF) recommends that clinicians screen all adults for
tobacco use, advise smoking cessation, and provide behavioral therapy and FDA-approved
pharmacotherapy if appropriate (A recommendation). Varenicline is an FDA-approved pharmacotherapy
that is an effective option for smoking cessation with or without behavioral therapy. Although clonidine
has been used for smoking cessation it is considered a second-line agent and is not FDA approved for
smoking cessation. The USPSTF concluded that there was not enough evidence on the effect on
smoking cessation to recommend an electronic nicotine delivery system (I recommendation). Abrupt and
complete (―cold-turkey‖) nicotine withdrawal is less effective than pharmacotherapy (nicotine
replacement therapy, bupropion hydrochloride, and varenicline). There is a lack of evidence regarding
the efficacy of hypnotherapy.

27
39. A 30-year-old female is planning a trip to Australia and is concerned about motion sickness. Which one
of the following medications is most effective for preventing this problem?

A) Dimenhydrinate (Dramamine)
B) Diphenhydramine (Benadryl)
C) Meclizine (Antivert)
D) Promethazine
E) Scopolamine

Item 39
ANSWER: E

Motion sickness is a syndrome that includes nausea and other symptoms, including vague subtle
symptoms of stomach awareness, malaise, fatigue, and irritability. The most effective medication is
scopolamine; transdermal scopolamine is more effective than oral scopolamine.

40. A 34-year-old female consults you because of excessive body and facial hair. She has a normal body
weight, no other signs of virilization, and regular menses. She had a bilateral tubal ligation 4 years ago.

Which one of the following would be the most appropriate treatment for her mild hirsutism?

A) Leuprolide
B) Metformin (Glucophage)
C) Prednisone
D) Spironolactone (Aldactone)

Item 40
ANSWER: D

Antiandrogens such as spironolactone, along with oral contraceptives, are recommended for the
treatment of hirsutism in premenopausal women (SOR C). Women should avoid becoming pregnant
while on spironolactone because of the potential for teratogenic effects. In addition to having side
effects, prednisone is only minimally helpful for reducing hirsutism by suppressing adrenal androgens.
Leuprolide, although better than placebo, has many side effects and is expensive. Metformin can be used
to treat patients with polycystic ovary syndrome, but this patient does not meet the criteria for this
diagnosis.

28
41. An 85-year-old male nursing home resident with a past history of a stroke has developed a pressure
ulcer over his right greater trochanter. The ulcer is 2 cm in size and is noted to be shallow with a
reddish-pink wound base. There is no evidence of secondary infection.

Which one of the following would be best for cleansing the wound?
A) Tap water
B) Aluminum acetate (Burow’s solution)
C) Hydrogen peroxide
D) Povidone/iodine solution (Betadine)
E) Sodium hypochlorite (Dakin’s solution)

Item 41
ANSWER: A

This patient has a stage 2 pressure ulcer. It is recommended that pressure ulcers not be cleaned with
povidone/iodine, Dakin’s solution, hydrogen peroxide, wet-to-dry dressings, or any solutions that may
impede granulation tissue formation. These sites should be cleaned with either saline or tap water and
covered with hydrocolloid, foam, or another nonadherent dressing that promotes a moist environment.

42. You evaluate a healthy 4-year-old female about an hour after she had 1 minute of generalized tonic-
clonic jerking. She has no previous history of similar symptoms. Additional history reveals that last
night she developed a runny nose, a cough, body aches, and a fever. Her past medical, social, and family
histories are unremarkable. She takes no medications.
A thorough physical examination is notable only for a temperature of 39.2°C (102.6°F) and a clear nasal
discharge. An influenza A test is positive.

Which one of the following would be most appropriate for the seizures?

A) Reassurance only
B) A loading dose of valproic acid (Depakene)
C) A neurology consultation and an EEG
D) CT of the brain
E) A lumbar puncture and cerebrospinal fluid analysis

Item 42
ANSWER: A

Febrile seizures occur in 2%–


as being general, tonic-clonic, and less than 15 minutes in duration, and occur in patients with no prior
history of neurologic disease. Prospective cohort studies support reassurance after a simple febrile
seizure. Retrospective cohort studies fail to show a benefit from neuroimaging or
electroencephalography. Two large case review studies showed no risk or a very low risk for meningitis
in the absence of altered mental status or meningeal signs, so a lumbar puncture is not indicated.
Randomized, controlled trials failed to show benefit and did show a potential side-effect risk from
starting antiepileptic medication.

29
43. A 34-year-old sexually active female consults you about contraception options. She has late-stage
kidney disease and her nephrologist has notified you that she will likely be recommended for kidney
transplantation soon.
Which one of the following would you recommend for safety and efficacy?
A) Condoms
B) Combined oral contraceptive pills
C) Medroxyprogesterone acetate (Depo-Provera) injections
D) An IUD
E) An etonogestrel/ethinyl estradiol vaginal ring (NuvaRing)

Item 43
ANSWER: D

Family physicians are often asked to provide primary care for organ transplant recipients. Pregnancy
should be avoided during the 12 months following transplantation because of the increased risk of
preterm delivery and graft rejection. Female fertility typically increases post transplant. The use of an
IUD avoids interactions with medications, does not increase the risk of infection, and is not affected by
typical immunosuppressive therapies. The remaining options are incorrect because of their higher failure
and discontinuation rates. The CDC cites failure rates with typical use of 9% for combined oral
contraceptives and the etonogestrel/ethinyl estradiol vaginal ring, 6% for injectable progesterone, 0.2%
for levonorgestrel IUDs, and 0.08% for the copper IUD. Barrier method failure rates exceed 18%.
Combined oral contraceptives and the vaginal ring also have potential estrogen-related side effects, and
injectable progesterone use increases the risk for osteoporosis.

44. A 26-year-old female has a 4-month history of continuous right-sided headache. The headache is
associated with tearing and nasal congestion only on the right, and has not responded to over-the-counter
analgesics. The patient went to the emergency department a few nights ago because of the pain, and CT
of the head at that visit was normal. On examination you note conjunctival injection on the right.
Findings are otherwise normal.
Which one of the following would be the most appropriate treatment at this time?
A) Sumatriptan (Imitrex)
B) Amitriptyline
C) Indomethacin
D) Topiramate (Topamax)

Item 44
ANSWER: C

There are several types of chronic headaches, and they often respond to different treatments. Migraine is
very prevalent and is characterized by headaches that are periodic, often unilateral, and frequently
pulsatile. Migraine is familial and typically starts in childhood, adolescence, or young adulthood, and the
headaches decrease in frequency over time. Some are associated with aura, which causes visual
disturbances. In mild cases, over-the-counter medications may control symptoms. For most patients,
however, treatment to control the attack can include triptans such as sumatriptan, and/or ergot alkaloids
such as ergotamine. Treatment to prevent attacks may also be appropriate, and could include a -blocker,
antiepileptic drugs, or amitriptyline.
Tension headaches are usually bilateral and are typically described as dull or aching, but patients often
30
describe tightness or pressure. They are not associated with symptoms such as throbbing, nausea, or
photophobia. Tension headaches are more frequent than migraine but patients often treat them at home
without seeking medical treatment. Frequent or persistent tension headaches can be treated with several
drugs used for anxiety or depression, including amitriptyline. Stronger analgesics and ergotamine are not
helpful.
Cluster headache is another type of chronic headache. This occurs most frequently in adult males, and
often occurs over a period which may extend over many weeks, with repeated episodes or clusters. It
most often occurs at night, and may recur several times during the night. The headache is unilateral and
is associated with orbital pain and vasomotor phenomenon such as blocked nasal passages, rhinorrhea,
conjunctival injection, and miosis. The headache can be treated with inhalation of 100% oxygen, and the
headache cycle can be terminated with verapamil. Ergotamine or sumatriptan can be used at night to
prevent attacks. There are also variants of cluster headaches, including chronic paroxysmal hemicrania,
which resembles cluster headache but has some important differences. Like cluster headaches, these
headaches are unilateral and accompanied by conjunctival hyperemia and rhinorrhea. However, these
headaches are more frequent in women, and the paroxysms occur many times each day. This type of
headache falls into a group of headaches that have been labeled indomethacin-responsive headaches
because they respond dramatically to indomethacin.

45. A 20-year-old female who is living alone for the first time reports that she has been having difficulty
falling asleep for the past several months. She denies obsessive thoughts, fears, or intrusive ambient
noise as obstacles to achieving sleep. She would prefer not to use a medication and hopes you can offer
suggestions to improve her sleep.

Of the following, which one is most likely to improve this patient’s sleep?
A) A television in the bedroom that stays on during the night
B) Cool temperatures in the bedroom
C) Vigorous physical exercise 1 hour before bedtime
D) A cup of warm coffee 30 minutes before bedtime
E) Sleeping with her pet cat

Item 45
ANSWER: B

Cognitive-behavioral therapy for chronic insomnia is known to be superior to pharmacologic therapies


(SOR A) and can be effectively administered by primary care physicians (SOR B). An important
component of cognitive-behavioral therapy is sleep hygiene education to identify behaviors that can
interfere with sleep, such as pets in the bedroom, caffeine consumption after 4 p.m., exercising within 2
hours of bedtime, and nicotine use. It is also important to identify factors that can facilitate sleep, such as
maintaining an environment conducive to sleep, including a cool room and a comfortable bed.
Addressing misunderstandings about normal sleep, reinforcing factual sleep-related information, and
addressing stimulus control, such as limiting use of the bedroom to sleep and sex and delaying going to
bed until sleepy, are among the other key components of a comprehensive cognitive-behavioral therapy
program for chronic insomnia.

46. A 33-year-old female comes to your office for follow-up of irritable bowel syndrome. You ruled out
other causes of her abdominal bloating, abdominal pain, and diarrhea at earlier visits. She has no change
in symptoms, such as constipation or blood in her stool. She has resisted treatment in the past, but her
symptoms are becoming more frequent and she would now like to consider treatment.
Evidence shows that which one of the following would most likely be beneficial for this patient?
31
A) Acupuncture
B) Increased insoluble fiber in her diet
C) Fluoxetine (Prozac), 20 mg daily
D) Neomycin, 1000 mg every 6 hours for 7 days
E) Polyethylene glycol (MiraLAX), 17 g daily

Item 46
ANSWER: C

This patient has diarrhea-predominant irritable bowel syndrome (IBS). There are many treatments
available, with varying degrees of evidence. SSRIs, along with tricyclic antidepressants, have been
shown to decrease abdominal pain and improve global assessment scores in those with IBS.
Polyethylene glycol is a treatment for constipation and would not help this patient. Acupuncture has not
been shown to be superior to sham acupuncture in improving IBS symptoms. Neomycin has been shown
to improve symptoms in constipation-predominant IBS but would not be helpful in diarrhea-
predominant IBS. Soluble fiber such as psyllium improves symptoms and decreases abdominal pain
scores in patients with IBS. Insoluble fiber has not been shown to improve any IBS outcomes.

47. A 24-year-old male complains of feeling on edge all of the time. For the past 2 years he has had
difficulty controlling his worrying about work, school, and relationships. He has had more difficulty
concentrating at work and school, is more irritable, and has difficulty staying asleep all night. He drinks
alcohol moderately and does not use drugs. You recommend regular exercise and refer him to a therapist
for cognitive-behavioral therapy to help manage his symptoms.

Which one of the following would be first-line medical therapy for this patient?
A) Bupropion (Wellbutrin)
B) Fluoxetine (Prozac)
C) Lorazepam (Ativan)
D) Methylphenidate (Ritalin, Concerta)
E) Quetiapine (Seroquel)

Item 47
ANSWER: B

Though symptoms of generalized anxiety disorder (GAD) overlap with other psychiatric and medical
conditions, the case presented is most consistent with GAD. SSRIs are first-line therapy for GAD (SOR
B). Benzodiazepines such as lorazepam can improve anxiety-related symptoms, but due to the side
effects and addiction potential they are recommended for short-term use (SOR B). Bupropion is
approved for the treatment of depression but is not used to treat GAD. Quetiapine may be considered as
second-line therapy for GAD (SOR B). Methylphenidate is first-line therapy for attention-
deficit/hyperactivity disorder but is not indicated to treat GAD. Psychotherapy, especially cognitive-
behavioral therapy, is also first-line treatment for GAD (SOR A), and exercise can also improve
symptoms (SOR B).

48. A 64-year-old male presents with increasing dyspnea on exertion. He feels well otherwise and has no
chronic medical problems. A physical examination is normal. Pulmonary function testing reveals normal
spirometry, with no evidence of an obstructive or restrictive pattern. However, his lung carbon
monoxide diffusing capacity (DLCO) is low. Based on these results, which one of the following is the
most likely diagnosis?
32
A) Asthma
B) Bronchiectasis
C) Chronic pulmonary emboli
D) COPD
E) Pulmonary fibrosis

Item 48
ANSWER: C

Low diffusing capacity of the lungs for carbon monoxide (DLCO) with normal spirometry indicates a
disease process that disrupts gas transfer in the lungs without causing lung restriction or airflow
obstruction. Common causes include chronic pulmonary emboli, heart failure, connective tissue disease
with pulmonary involvement, and primary pulmonary hypertension. Asthma, bronchiectasis, COPD, and
pulmonary fibrosis are associated with abnormalities on spirometry.

49. A 46-year-old male with a prosthetic heart valve requests your advice regarding antibiotic
prophylaxis for an upcoming dental procedure. The patient is allergic to penicillin.
Which one of the following would be most appropriate?
A) Ciprofloxacin (Cipro)
B) Clindamycin (Cleocin)
C) Doxycycline
D) Rifampin (Rifadin)
E) Trimethoprim/sulfamethoxazole (Bactrim)

Item 49
ANSWER: B

Patients with an artificial heart valve should be given antibiotic prophylaxis prior to dental procedures to
prevent infectious endocarditis. The organisms that most frequently cause infectious endocarditis include
Staphylococcus aureus (31%), Streptococcus viridans (17%), coagulase-negative staphylococci (11%),
Enterococcus (11%), Streptococcus bovis (7%), and other streptococci (5%). Amoxicillin is the
preferred medication for prophylaxis. Clindamycin or azithromycin can be used in patients with a
penicillin allergy. If the penicillin allergy is not associated with anaphylaxis, angioedema, or urticaria,
then cephalexin would be an appropriate antibiotic choice. Ciprofloxacin, doxycycline, rifampin, and
trimethoprim/ sulfamethoxazole are not used for bacterial endocarditis prophylaxis.

50. An elderly female who has been your patient for several years is discovered lying on the floor of her
kitchen by a Meals on Wheels volunteer. She is transported to the hospital in an unresponsive state.
After a thorough evaluation you diagnose a massive cerebral infarct. On several previous occasions the
patient verbalized to you her desire to not be subjected to life-prolonging treatments should she ever be
rendered incapacitated but she declined your suggestion that she confirm this in writing. The patient is
admitted to the hospital with ―do not resuscitate‖ (DNR) orders and supportive measures are instituted.
A neurology consultant evaluates her and agrees that her condition is terminal and irreversible. The
patient’s nephew, who is her closest relative, is angered by the DNR status and, noting that she has
Medicare coverage, demands every medical treatment that might prolong his aunt’s life, including
resuscitation.
Which one of the following would be the best course of action in terms of legality and ethics?

33
A) Institution of aggressive medical therapies, including full resuscitation in the event of
cardiac arrest and ventilator support in the event of respiratory arrest
B) Continuation of a supportive treatment plan, provided full resuscitation is initiated in the event of
cardiac arrest and ventilator support is provided in the event of respiratory arrest
C) Performance of full CPR for cardiopulmonary arrest, but no ventilator support for
respiratory failure
D) Continuation of the current treatment plan

Item 50
ANSWER: D

Adult patients or their legally authorized representatives have the right to refuse any medical treatment,
regardless of its likelihood of success; however, there is no legal right to receive any and all treatment
demanded. When patients have explicit advance directives in writing, their wishes are clear. When no
written document exists but the patient’s desire is well known to his or her physician, the physician is
ethically bound to honor these wishes. This responsibility must be balanced against the physician’s
ethical obligation not to perform futile treatments of no benefit to the patient. Family members acting as
a medical decision-making proxy are obligated to represent what they believe to be the patient’s wishes,
even if this conflicts with their personal beliefs.
This case highlights two commonly encountered issues: honoring the patient’s wishes when there is
conflicting evidence of what their wishes may be, and withholding medically futile treatment. In this
case, the futility of treating the patient’s condition overrides any confusion as to her wishes, and the
admission treatment plan is the most appropriate. Nevertheless, it would be best to discuss this with the
nephew and attempt to arrive at a consensus, with the patient’s best interest being the primary concern.

51. Which one of the following community health programs best fits the definition of secondary
prevention?
A) An antismoking education program at a local middle school
B) Blood pressure screening at a local church
C) A condom distribution program
D) Screening patients with diabetes mellitus for microalbuminuria

Item 51
ANSWER: B

Prevention traditionally has been divided into three categories: primary, secondary, and tertiary. Primary
prevention targets individuals who may be at risk to develop a medical condition and intervenes to
prevent the onset of that condition. Examples include childhood vaccination programs, water
fluoridation, antismoking programs, and education about safe sex. Secondary prevention targets
individuals who have developed an asymptomatic disease and institutes treatment to prevent
complications. Examples include routine Papanicolaou tests and screening for hypertension, diabetes
mellitus, or hyperlipidemia. Tertiary prevention targets individuals with a known disease, with the goal
of limiting or preventing future complications. Examples include screening patients with diabetes for
microalbuminuria, rigorous treatment of diabetes mellitus, and post–myocardial infarction prophylaxis
with -blockers and aspirin.

34
52. A 34-year-old male presents with a painful great toe after his foot was stepped on during a soccer
game. He has significant bruising and tenderness over the first distal phalanx and has no rotational
deformity. You obtain a radiograph and see a nondisplaced fracture of the distal first phalanx that
involves 40% of the joint space.
Which one of the following would be the preferred treatment for this fracture?
A) Buddy taping
B) A rigid-sole shoe
C) A short leg walking cast with a toe plate
D) Surgical fixation

Item 52
ANSWER: D

Toe fractures are the most common fracture of the foot and they generally involve minimal displacement
and are usually treated nonsurgically. Nondisplaced lesser toe fractures are generally treated with a
rigid-sole shoe or buddy taping to an adjacent toe. The great toe has an increased role in weight bearing
and balance, and fractures of this toe have a greater potential morbidity. Because of this, these fractures
are generally treated with a short leg walking cast with a toe plate. The foot should be placed in a cast
for 2–3 weeks, and if there are no significant symptoms at that time the patient can then use buddy
taping or a rigid-sole shoe for the next 3–4 weeks. Because there was intra-articular involvement in this
patient, he should also get a repeat radiograph after 1 week. Intra-articular fractures with 25% or greater
involvement of the joint surface should be referred for surgical treatment.

53. A 72-year-old female taking hydrochlorothiazide for hypertension develops trigeminal neuralgia and
you start her on carbamazepine (Tegretol). She is at risk for which one of the following metabolic
consequences?
A) Calcium pyrophosphate deposition
B) Hypercalcemia
C) Hyponatremia
D) Hyperuricemia

Item 53
ANSWER: C

Elderly patients, especially those taking hydrochlorothiazide, are at risk for developing hyponatremia
while taking carbamazepine. Carbamazepine is one of the medications that can cause the syndrome of
inappropriate antidiuretic hormone secretion, as it interferes with the ability to dilute the urine. It does
not lead to the other derangements listed (SOR A).

54. A 43-year-old female presents with an 8-month history of posttraumatic stress disorder following a
motor vehicle accident that severely injured her and a friend. She has had a positive response to
counseling and SSRI treatment but continues to have sleep disturbances and nightmares.
Which one of the following medications is most likely to decrease the frequency of her
nightmares?
A) Clonazepam (Klonopin)
B) Divalproex (Depakote)
35
C) Prazosin (Minipress)
D) Propranolol

Item 54
ANSWER: C

Prazosin is an -adrenergic receptor antagonist and is recommended for the treatment of nightmares in
posttraumatic stress disorder (SOR A). It is thought to reduce sympathetic outflow in the brain.
Although clonidine may be tried, evidence of its effectiveness is sparse (SOR C). Clonazepam,
propranolol, and divalproex have not been recommended.

55. A 27-year-old male has been treating his plaque psoriasis with high-potency topical
corticosteroids for several years. He comes to your office to discuss other options since the lesions on his
trunk and extremities are becoming resistant to this therapy.
Which one of the following treatment strategies would be most appropriate?
A) Switch to topical tazarotene (Avage, Tazorac)
B) Add topical calcipotriene (Dovonex, Sorilux)
C) Begin oral acitretin (Soriatane)
D) Begin an oral corticosteroid
E) Begin etanercept (Enbrel) injections

Item 55
ANSWER: B

Oral corticosteroids are not indicated in the treatment of plaque psoriasis. All of the other options are
indicated only if topical treatments fail. Of the options listed, the combination of a topical corticosteroid
and topical calcipotriene is considered the most appropriate for this patient. Another option would be to
add topical tazarotene to the topical corticosteroid. However, when tazarotene is used as monotherapy it
often fails to clear plaques and increases the incidence of skin irritation.

56. A 78-year-old male presents for a routine health maintenance examination and is concerned about a
gradual loss in his vision during the past year. He has smoked 1 pack of cigarettes per day for the past 60
years. He has no other medical problems. On Amsler grid testing he notes distorted grid lines.
Which one of the following would you recommend for this patient?
A) Watchful waiting
B) Avoiding all vitamin supplements
C) Treatment to reverse his visual changes
D) Smoking cessation to prevent further vision loss

Item 56
ANSWER: D

This patient presents with signs and symptoms that suggest age-related macular degeneration. Smoking
is a modifiable risk factor and smokers should be counseled to quit (SOR C). The patient should be
referred to an ophthalmologist for further evaluation and management. Watchful waiting would not be
appropriate. Vitamin supplements with Age-Related Eye Disease (AREDS) and AREDS2 formulations
have been shown to delay visual loss in patients with age-related macular degeneration (SOR A). Age-
related macular degeneration is not reversible but treatment can delay progression or stabilize the
changes (SOR A).
36
57. A patient is admitted to the hospital with severe acute pancreatitis, based on diagnostic criteriafor
severity. After appropriate intravenous hydration, which one of the following is associated with shorter
hospital stays and lower mortality?
A) Parenteral nutrition
B) Nothing by mouth until the pain has resolved
C) Clear liquids by mouth after 48 hours
D) Bolus nasogastric enteral nutrition
E) Continuous nasogastric enteral nutrition

Item 57
ANSWER: E

The American College of Gastroenterology recommends that patients with severe acute pancreatitis
receive enteral nutrition. Enteral feedings help prevent infectious complications, such as infected
necrosis, by maintaining the gut mucosal barrier and preventing translocation of bacteria that may seed
pancreatic necrosis. Currently, continuous enteral feeding is preferred over bolus feeding. A meta-
analysis has shown that continuous nasogastric enteral feeding started in the first 48 hours decreases
mortality and the length of hospital stay. Total parenteral nutrition is not recommended because of
infectious and line-related complications. It should be avoided unless the enteral route cannot be used.

58. An 8-year-old male is brought to your office because of a rash on his arms and legs that has been
present for the last several weeks. It seems to be spreading gradually, according to his parents. No other
family members have similar symptoms. A physical examination reveals excoriated erythematous
papules on both forearms and both lower legs that drain a small amount of serous fluid. The child says
that these are itchy. There are no signs of deeper inflammation and no lesions are present on the scalp,
hands, thorax, or groin.

Which one of the following is the most likely cause of this problem?
A) Bedbugs
B) Brown recluse spiders
C) Mites/chiggers
D) Scabies
E) Ticks

Item 58
ANSWER: A

This child’s presentation appears most consistent with bites from an insect. Having multiple exposures
on skin often not covered by clothing would be typical of household fleas or bedbugs. Tick bites are
typically identified by the presence of an actively feeding insect or a single papular lesion. Similarly,
brown recluse spider bites would not be expected to be multiple or recurrent. A chigger is the larval
form of a mite, which is an eight-legged arthropod. The larval form has only six legs, and tends to crawl
into spaces near constricted clothing and cause welts from their bites along the neckline, waistline, sock
line, or more rarely on the genitals. A scabies infestation often presents as an eczematous rash in
semiprotected folds of skin such as the web spaces of fingers, the umbilicus, the axillae, or the genital
region.

37
59. A 72-year-old male sees you for an annual follow-up visit. He has well controlled type 2 diabetes
mellitus, hypertension, and chronic kidney disease. His hemoglobin A1c is 6.2% today and his blood
pressure is 122/76 mm Hg. Historically, his serum creatinine level has been rising by 0.1–0.2 mg/dL per
year. Last year his creatinine level was 1.9 mg/dL (N 0.6–1.2), which translated to an estimated
glomerular filtration rate (eGFR) of 39.8 mL/min/1.73 m2 (N 90–120).

Which one of the following findings on this year’s laboratory testing should prompt a referral
to a nephrologist for management?
A) A phosphorus level of 5.0 mg/dL (N 2.5–4.5)
B) A hemoglobin level of 9.2 g/dL (N 13.5–17.5) with normal iron studies
C) A serum creatinine level of 2.1 mg/dL (eGFR 36.0 mL/min/1.73 m2)
D) A urine microalbumin to creatinine ratio of 160 g/mg (N <30)
E) A serum vitamin D level of 10 ng/mL (N 20–100)

Item 59
ANSWER: B

Chronic kidney disease is defined as an estimated glomerular filtration rate (eGFR) <60 mL/min/1.73
m2 for at least 3 months, or other evidence of kidney damage such as albuminuria, abnormal imaging, or
an abnormal biopsy. Current guidelines recommend referral to a nephrologist if a patient’s renal disease
is either of unknown etiology, is deteriorating quickly (eGFR decreasing by >5 mL/min/1.73 m 2 per
year), or is severe. Thresholds used to define severe chronic kidney disease include an eGFR <30
mL/min/1.73 m2, a urine albumin to creatinine ratio >300 g/mg, persistent acidosis or potassium
imbalance, non–iron deficiency anemia with a hemoglobin level <10 g/dL, and evidence of secondary
hyperparathyroidism. The elevated phosphorus in this patient most likely indicates metabolically
significant renal disease and warrants consultation.

60. A 24-year-old asymptomatic female has a chest radiograph that incidentally shows bilateral hilar
adenopathy. Additional evaluation supports a diagnosis of sarcoidosis.
Which one of the following would be most appropriate at this point?
A) Monitoring only
B) Treatment with corticosteroids
C) Treatment with methotrexate
D) A transbronchial lung biopsy

Item 60
ANSWER: A

According to an international consensus statement, there are three criteria for diagnosing sarcoidosis: (1)
a compatible clinical and radiologic presentation, (2) pathologic evidence of noncaseating granulomas,
and (3) exclusion of other diseases with similar findings. The main exceptions to the need for histologic
confirmation are the presence of bilateral hilar adenopathy in an asymptomatic patient (stage I) and the
presentation of sarcoid-specific Lofgren syndrome—with fever, erythema nodosum, and bilateral hilar
adenopathy that can be diagnosed based on clinical presentation alone. An asymptomatic patient with
38
stage I sarcoidosis (bilateral hilar lymphadenopathy on chest radiography) without suspected infection or
malignancy does not require an invasive tissue biopsy because the results would not affect the
recommended management, which is monitoring only. Treatment is not indicated because spontaneous
resolution of stage I sarcoidosis is common.
Reliable biomarkers for diagnosing sarcoidosis do not exist. Although the serum angiotensin converting
enzyme level may be elevated in up to 75% of untreated patients, this lacks sufficient specificity, has
large interindividual variability, and fails to consistently correlate with disease severity, all of which
limit its clinical utility.
Pathologic evidence of noncaseating granulomas from the most accessible and safest biopsy site should
be pursued only if there is an indication for treatment, such as significant symptomatic or progressive
stage II or III pulmonary disease or serious extrapulmonary disease. If treatment is indicated,
corticosteroids are the first-line treatment for sarcoidosis. Second- and third-line treatments include
methotrexate, azathioprine, leflunomide, and biologic agents.

61. A 62-year-old female presents with numbness and tingling in her feet. She first noticed tingling in
the toes of her right foot several months ago; it is now present in both feet and is causing numbness. She
has not experienced any weakness, or any changes in vision, speech, or memory. Her medical history
includes hypertension controlled by lisinopril (Prinivil, Zestril), 20 mg daily, and she also takes aspirin,
81 mg daily. She drinks a glass of wine nightly and does not smoke. She does not have a family history
of neurologic disorders.
On examination she has symmetric decreased sensation to light touch and vibration in her feet. Reflexes
and strength are intact bilaterally. Laboratory findings include a normal CBC and normal TSH and
vitamin B12 levels. Her erythrocyte sedimentation rate is 32 mm/hr (N 0–20). A comprehensive
metabolic panel is normal except for a total protein level of 8.5 g/dL (N 6.0–8.3).

Which one of the following tests would be most useful for making a diagnosis?

A) An angiotensin converting enzyme level


B) Serum protein electrophoresis
C) A chest radiograph
D) A lumbar puncture with cerebrospinal fluid analysis
E) MRI of the lumbar spine

Item 61
ANSWER: B

This patient has a peripheral neuropathy. A review of the patient’s history and specific laboratory testing
was performed to evaluate for the most common treatable causes of peripheral neuropathy, which
include diabetes mellitus, hypothyroidism, and nutritional deficiencies. Additional causes of peripheral
neuropathy include chronic liver disease and renal disease. It is important to consider medications as a
possible cause, including amiodarone, digoxin, nitrofurantoin, and statins. Excessive alcohol use is
another important consideration. In this patient, the mildly elevated total protein and erythrocyte
sedimentation rate, which suggest a monoclonal gammopathy such as MGUS (monoclonal gammopathy
of unknown significance) or multiple myeloma, should direct her workup. Serum protein electrophoresis
is indicated to assess for this. Other less common causes of peripheral neuropathy include carcinoma

39
causing a paraneoplastic syndrome,lymphoma, sarcoidosis, AIDS, and genetic disorders such as
Charcot-Marie-Tooth disease. Approximately 25% of patients with peripheral neuropathy have no
clearly defined cause after a thorough evaluation and are diagnosed with idiopathic polyneuropathy.
MRI of the lumbar spine can identify central lesions causing spinal cord or nerve root compression but is
not indicated in the evaluation of peripheral neuropathy. Serum angiotensin converting enzyme levels
and a chest radiograph can assist in the diagnosis of sarcoidosis, which can cause peripheral neuropathy
but is less likely in this patient. Cerebrospinal fluid analysis is important in assessing for chronic
inflammatory demyelinating polyradiculoneuropathy, a more rare cause of peripheral neuropathy.

62. A 20-year-old female presents to your office with questions about her contraceptive method. She has
been using a combined oral contraceptive pill for the past 2 years without any complications. She has
learned that several of her friends recently switched to an IUD. She is concerned about the efficacy of
her current method and asks about the failure rate. You tell her that with typical use, the annual failure
rate of a combined oral contraceptive pill is
A) 0.2%
B) 2%
C) 9%
D) 18%
E) 22%

Item 62
ANSWER: C

The annual failure rate of combined oral contraceptive pills with typical use is 9%. Typical failure rates
for other contraceptive methods are 0.2% for the levonorgestrel IUD, 6% for injectable progestin, 18%
for male condoms, and 22% for the withdrawal method.

63. According to recommendations by the U.S. Preventive Services Task Force, physicians should
screen all adolescents and adults for which one of the following disorders?
A) Bipolar disorder
B) Generalized anxiety disorder
C) Major depressive disorder
D) Posttraumatic stress disorder
E) Schizophrenia

Item 63
ANSWER: C

Due to the disease prevalence and impact, effectiveness of screening instruments, and benefits of
available treatment, the U.S. Preventive Services Task Force (USPSTF) recommends screening for
major depressive disorder when adequate systems are in place to ensure accurate diagnosis, effective
treatment, and appropriate follow-up. The USPSTF has made no recommendations regarding screening
for bipolar disorder, generalized anxiety disorder, posttraumatic stress disorder, or schizophrenia.

40
64. You see a 38-year-old male 11 days after his cat bit him on the thumb. He went to an urgent care
clinic and was given a ceftriaxone (Rocephin) injection and 10 days of oral amoxicillin/clavulanate
(Augmentin). He says the redness and pain in his thumb and hand have improved some, but the thumb
remains very painful. He received a tetanus booster recently. His cat is well. His vital signs are normal
and examination of the thumb reveals a swollen, erythematous, tender, warm interphalangeal joint with
decreased range of motion. There are healing 2-mm dorsal skin
wounds over the joint.

Which one of the following would you recommend at this point?

A) Continued amoxicillin/clavulanate
B) Azithromycin (Zithromax)
C) Clindamycin (Cleocin)
D) Rabies vaccine and continued amoxicillin/clavulanate
E) Consultation for surgery

Item 64
ANSWER: E

This patient’s cat is well more than 10 days after the bite, so rabies vaccine is not necessary.
Azithromycin is indicated for cat scratch disease, but the presentation does not suggest this. Severe
infections may require incision, drainage, and intravenous antibiotics. A surgery consultation is
recommended to evaluate for tendon sheath or joint infection.

65. An 85-year-old female is admitted to the hospital for surgery and develops confusion
postoperatively. The patient is a widow and lived independently prior to admission. Her daughter
says the confusion is atypical for her mother. She does not have a history of memory loss,
forgetfulness, or confusion prior to admission.

Which one of the following cognitive assessment tests should be used to assess her acute change
in mental status?

A) Confusion Assessment Method (CAM)


B) Mini-Cog
C) Mini-Mental State Examination (MMSE)
D) Montreal Cognitive Assessment test (MoCA)
E) Saint Louis University Mental Status exam (SLUMS)

41
Item 65
ANSWER: A

The patient is experiencing an acute cognitive change from baseline, indicating possible delirium. The
Confusion Assessment Method (CAM) is a delirium diagnosis tool useful for evaluating acute cognitive
changes. The other tests listed, including the Mini-Mental State Examination, Mini-Cog, Montreal
Cognitive Assessment, and Saint Louis Mental Status exam, test chronic baseline cognitive function and
are not designed to test for acute changes.

66. Which one of the following is true regarding direct observational therapy (DOT) in the treatment
of active tuberculosis?

A) It guarantees patient compliance with the prescribed regimen


B) It is recommended only in the office or clinic setting
C) It decreases drug-resistant tuberculosis
D) Patients require less monitoring for signs of treatment failure

Item 66
ANSWER: C

In the treatment of active tuberculosis, direct observational therapy (DOT) involves providing the
antituberculosis drugs directly to patients and watching them swallow the medication. It is the preferred
care management strategy for all patients with tuberculosis. The use of DOT does not guarantee the
ingestion of all doses of every medication, as patients may miss appointments, may not actually swallow
the pills, or may regurgitate the medication, sometimes deliberately. Due to these limitations, the use of
DOT does not remove the need to monitor patients for signs of treatment failure. DOT is effective in a
wide variety of settings, including in the community with health nurses. It even shows benefit when the
observation makes use of telehealth settings or mobile phones.

Among the important benefits of DOT are that it has been shown to decrease both the acquisition and
transmission of drug-resistant tuberculosis and to increase treatment success in HIV-positive patients.

67. Beyond short-term pain relief, local corticosteroid injection provides the best long-term
improvement for which one of the following?

A) Greater trochanteric bursitis


B) Knee osteoarthritis
C) Lateral epicondylitis
D) Subacromial impingement syndrome
E) Trigger finger

42
Item 67
ANSWER: E

Reported cure rates for trigger finger after corticosteroid injection range from 54% to 86%.
Corticosteroid injection for the other conditions listed results in temporary pain relief, but the underlying
conditions are not improved by the injection.

68. An 11-year-old female is brought to your office for a routine well child examination. The American
Academy of Pediatrics recommends screening this patient for which one of the following?

A) Anemia
B) Diabetes mellitus
C) Dyslipidemia
D) HIV

Item 68
ANSWER: C

Because of concerns about the growing epidemic of obesity in this population, the American Academy
of Pediatrics now recommends screening for elevated serum cholesterol levels in children 9–11 years of
age (SOR C). This patient should also be screened annually for depression beginning at 11 years of age
and continuing through 21 years of age. Universal screening for iron deficiency anemia is recommended
at 12 months of age and again at 15–30 months of age if the patient is determined to be high risk. HIV
screening is recommended in adolescents 16–18 years of age, and age 21 is now the recommended
starting age for screening for cervical dysplasia. Universal screening for diabetes mellitus is not
recommended for children or adolescents.

69. A 51-year-old female comes to your office for follow-up of fibromyalgia. She is currently taking
amitriptyline, 10 mg at bedtime, and naproxen (Naprosyn), 500 mg twice daily, for her
symptoms. A member of her fibromyalgia support group recommended fluoxetine (Prozac) to
her and she asks you if it would be helpful. It would be appropriate to tell her that SSRIs for the
treatment of fibromyalgia

A) do not affect depression scores


B) reduce fatigue
C) provide some pain reduction
D) help with sleep problems
E) are superior to tricyclics for pain control

Item 69

43
ANSWER: C

Fibromyalgia is a chronic complex condition characterized by muscle pain, fatigue, muscle tenderness,
and sleep disorders, often accompanied by mood disorders. SSRIs have been studied in the treatment of
these symptoms, and while they have been shown to produce up to a 30% reduction in pain scores in
patients with fibromyalgia, they have not been shown to affect fatigue or sleeping problems. They also
have not been shown to be superior to tricyclics when treating pain. As with other patient populations,
SSRIs have been shown to improve depression in those with fibromyalgia.

70. A 77-year-old female with widely metastatic lung cancer is seen in her home with increased
confusion, hallucinations, and combative behavior for the past 2 days. She has not slept and has
a very poor appetite.
Which one of the following should be the first step in the management of this patient?

A) Place her in gentle restraints for safety


B) Determine and treat the underlying cause of the behavior change
C) Start diphenhydramine (Benadryl) at bedtime for sleep
D) Start mirtazapine (Remeron) at bedtime for sleep and appetite
E) Start lorazepam (Ativan) as needed for agitation

Item 70
ANSWER: B

This patient has hyperactive delirium. The first step in management is to determine and treat the
underlying cause if possible. There are multiple causes of delirium such as medications, infections,
metabolic abnormalities, and underlying diseases. The first step in treatment is behavioral management
with strategies to orient the patient. Haloperidol or antipsychotics may be used if the patient is at risk of
harm. Lorazepam and anticholinergics should both be avoided, as they can worsen delirium. Restraints
can also worsen the agitation and should not be used. Mirtazapine is an antidepressant and is not used in
the treatment of delirium.

71. You are called to a rapid response on the inpatient medical ward at your local hospital. A
66-year-old male who was admitted for pneumonia complains of the sudden onset of lightheadedness.
Evaluation of his mental status shows that he is coherent. His heart rate is 175 beats/min and his blood
pressure is 120/62 mm Hg. A cardiac rhythm strip reveals a widened, regular, monomorphic QRS.

Which one of the following would be the best initial management of this patient?

44
A) Chest compressions
B) An isotonic fluid bolus
C) Adenosine (Adenocard)
D) Defibrillation

Item 71
ANSWER: C

Adenosine may be both diagnostic and therapeutic, and is a safe option in stable, regular, monomorphic
wide-complex tachycardia (SOR B). A fluid bolus may also be appropriate, depending on the underlying
disease state, but would not be the initial management for this patient. Defibrillation and chest
compressions would be appropriate in this patient if he did not have a measurable pulse. If adenosine
infusion is ineffective, antiarrhythmic medication would be recommended in this patient.

72. A 52-year-old male sees you for the first time for a health maintenance visit. He previously
tested positive on a hepatitis C antibody test performed by another physician. He seeks your
opinion regarding follow-up testing.

Which one of the following tests would you suggest at this point?

A) -Fetoprotein
B) Hepatitis B antigen
C) Hepatitis C RNA
D) Ultrasonography of the liver
E) A liver biopsy

Item 72
ANSWER: C

Screening for hepatitis C virus (HCV) with an anti-HCV antibody test is recommended for all adults at
high risk of infection, and one-time screening is recommended in adults born between 1945 and 1965. If
the anti-HCV antibody test result is positive, current infection should be confirmed with a qualitative
HCV RNA test.

73. A 26-year-old male graduate student presents with concerns about having attention-deficit
disorder (ADD). He reports inattentiveness and a lack of concentration for the last 2–3 years,
resulting in poorer academic performance than when he was an undergraduate. He says his wife
also complains of his lack of focus and attentiveness when attending to household
responsibilities. He does not recall having similar symptoms in grade school or high school.

45
You explain that in order to diagnose adult ADD

A) evidence of symptoms of ADD must be present before age 6


B) evidence of symptoms of ADD must be present before age 12
C) a therapeutic trial of a short-acting dextroamphetamine/amphetamine combination
(Adderall) is indicated
D) a therapeutic trial of an SNRI such as atomoxetine (Strattera) is indicated

Item 73
ANSWER: B

Diagnosing attention-deficit disorder in adults requires symptoms that interfere with social, academic, or
occupational functioning and are present in more than one setting. DSM-5 states that a history of
symptoms before age 12 is required for the diagnosis. DSM-IV specified that symptoms must have been
present before age 7.

74. A 20-year-old male who is in college in another state calls to report that he has developed
wheezing, oral itching, and a swollen lower lip after kissing his girlfriend. The symptoms
reminded him of an allergic reaction to peanuts he had when he was a child, so he
self-administered a dose of epinephrine with his auto-injector 15 minutes ago. His itching and
wheezing have improved, and he asks what else he should do.

What advice should you provide?

A) He should take oral diphenhydramine (Benadryl) now and prednisone for 3 days
B) He should go to the nearest emergency department
C) He should schedule a comprehensive reevaluation by an allergist
D) No further action is needed

Item 74
ANSWER: B

Patients with a peanut allergy can have reactions to infinitesimal amounts of peanut protein, including
residue on the lips of other people. This patient has successfully interrupted the course of anaphylaxis.
Diphenhydramine can help reduce subsequent symptoms, and prednisone is generally given, although its
value is unproven. However, the patient is at risk of a biphasic reaction and should go to an emergency
department where additional epinephrine and resuscitation facilities are available. The American
Academy of Pediatrics guideline recommends that all peanut-allergic patients who require a dose of
adrenalin be observed in an emergency department. Patients who have not already had a full allergy
evaluation need to see an allergist, but this patient’s peanut allergy has been established. Peanut-allergic
patients tend to have accidental exposure about once every 5 years in spite of efforts at avoidance.

46
75. A 30-year-old male presents to your office with a 10-day history of rhinorrhea, nasal congestion,
cough, and headache. He initially had a low-grade fever that resolved, but over the past 2 days all of his
symptoms have gotten worse. His past medical history is unremarkable and he does not smoke.
On examination there is a purulent secretion noted from the right nasal cavity and tenderness over the
right maxillary sinus region.

The most appropriate treatment is

A) amoxicillin/clavulanate (Augmentin)
B) azithromycin (Zithromax)
C) cefuroxime (Ceftin)
D) levofloxacin (Levaquin)
E) trimethoprim/sulfamethoxazole (Bactrim)

Item 75
ANSWER: A

In the first 3 to 4 days of illness, viral rhinosinusitis cannot be distinguished from early acute bacterial
rhinosinusitis. If the patient seems to be improving and then symptoms start to worsen on days 5–10 of
the illness (double sickening), acute bacterial rhinosinusitis should be suspected. The color of the nasal
discharge should not be used as the sole indication for antibiotic therapy. One study showed that
unilateral predominance with purulent rhinorrhea had an overall reliability of 85% for diagnosing
sinusitis. After 10 days of upper respiratory symptoms, the probability of acute bacterial rhinosinusitis is
60%. Antibiotic therapy should be considered if the patient does not improve after 7–10 days from the
onset of symptoms or if the symptoms worsen at any time. According to most guidelines, the first-line
antibiotic for treatment of adults with sinusitis is amoxicillin/clavulanate. Respiratory fluoroquinolones
are not recommended as first-line medications, as they offer no additional benefits and have significant
side effects. Second- and third-generation cephalosporins, trimethoprim/sulfamethoxazole, and
macrolide antibiotics are no longer recommended for initial therapy. This is due to high rates of
resistance in Streptococcus pneumoniae and Haemophilus influenzae.

76. A 7-month-old infant is hospitalized for the third time with lower-lobe bronchopneumonia.
Findings include a weight and height below the 10th percentile. A sibling died of sudden infant
death syndrome. Laboratory testing reveals a hemoglobin level of 9.0 g/dL (N 10.5–14.0), a mean
corpuscular volume of 85 m3 (N 72–88), and a serum calcium level of 9.0 mg/dL (N 9.0–10.5). A sweat
chloride level is 20 mEq/L (N <60).

Which one of the following is the most likely cause of this infant’s failure to thrive?

47
A) Cystic fibrosis
B) DiGeorge’s syndrome
C) Battered child syndrome
D) Gastroesophageal reflux
E) β-Thalassemia

Item 76
ANSWER: D

Gastroesophageal reflux accounts for a significant number of cases of failure to thrive, crib death, and
recurrent pneumonia. Features of gastroesophageal reflux include a history of recurrent pneumonia, a
low growth curve, a family history of sudden infant death syndrome, and normocytic anemia. A sweat
chloride level of 20 mEq/L rules out cystic fibrosis. Normal serum calcium excludes DiGeorge’s
syndrome. The battered child generally presents with more than just a single recurring medical problem.
β –Thalassemia would be indicated by a microcytic anemia.

77. Which one of the following treatments for cough has evidence of efficacy and safety when used
for children from 1 to 2 years of age?

A) Honey
B) Dextromethorphan
C) Diphenhydramine (Benadryl)
D) Codeine
E) Guaifenesin

Item 77
ANSWER: A

Honey has been shown to decrease the frequency and severity of cough in children. Honey is safe in
children 2 years of age or older, but should be avoided before then due to the risk of botulism. Safety
and/or efficacy data is not available for the other agents listed in children under 2 years old. Codeine in
particular has serious safety problems in young children and should be avoided.

78. While on call for your group practice you are called to admit a 23-year-old female with a history
of sickle cell disease who presented to the emergency department with chest pain, a cough, and
shortness of breath. She has no history of recent hospitalization.
Physical Findings
Blood pressure ……………………….. 176/86 mm Hg

48
Pulse ………………………………….. 103 beats/min
Respiratory rate ………………………. 20/min
Temperature …………………………....37.8°C (100.0°F)
Oxygen saturation……………………… 89% on room air

A chest radiograph shows consolidation in the right lower lobe.

In addition to oxygen, intravenous fluids, an intravenous third-generation cephalosporin, and


pain management, which one of the following is important to include in the patient’s treatment
plan?

A) Azithromycin (Zithromax)
B) Daptomycin (Cubicin)
C) Ertapenem (Invanz)
D) Gentamicin
E) Vancomycin (Vancocin)

Item 78
ANSWER: A

This patient has acute chest syndrome (ACS), a serious vaso-occlusive complication of sickle cell
disease (SCD). Its cause may be multifactorial, but infections are common and antimicrobials are
indicated. However, the clinical course of ACS is significantly different from infectious pneumonia in
patients without SCD, due to the damaged microvasculature that occurs in ACS. Studies have shown
that atypical pathogens predominate in ACS and it is therefore important to treat all patients with ACS
with antibiotics that cover Mycoplasma and Chlamydophila. Viral infections are also common,
especially in children with ACS. Other possible pathogens include Staphylococcus aureus,
Streptococcus pneumoniae, and Haemophilus influenzae. Therefore, the use of a third-generation
cephalosporin along with azithromycin is the recommended antibiotic coverage.

In addition to antimicrobials, treatment includes supportive care with supplemental oxygen, intravenous
fluids, pain control, and incentive spirometry. Depending on the degree of anemia seen, a simple blood
transfusion or exchange transfusion is often indicated as well. Consultation with a hematologist is
recommended in the care of patients with ACS. Even with appropriate care, mortality rates in ACS are
as high as 3%.

79. A 32-year-old obese female from Saudi Arabia presents with muscle aches. Her clothing limits
sun exposure and you decide to test her for vitamin D deficiency.

Which one of the following is the most appropriate test for this condition?

49
A) Calcium
B) Alkaline phosphatase
C) 25-hydroxyvitamin D
D) 1,25-dihydroxyvitamin D
E) Parathyroid hormone

Item 79
ANSWER: C

Recommendations to screen for vitamin D deficiency apply only to patients at risk and not to the general
population. This patient’s obesity and her clothing style, which limits sun exposure to the skin, puts her
at increased risk. Additionally, this patient’s muscle aches may be a symptom of vitamin D deficiency.
The recommended test for this condition is a 25-hydroxyvitamin D level. A 1,25-dihydroxyvitamin D
level is recommended to monitor, not diagnose, certain conditions. Parathyroid hormone, calcium, and
alkaline phosphatase levels are poor indicators of vitamin D status.

80. A 22-year-old female with polycystic ovary syndrome comes in to discuss contraception. She
has no other health conditions and takes no medications. Her menses are somewhat irregular,
occurring every 28–42 days. She also asks about treatment for her mild hirsutism.

Which one of the following medications would be most likely to address her need for
contraception and also improve her hirsutism?

A) Spironolactone (Aldactone)
B) Cyclic progesterone
C) Progesterone-only contraceptive pills
D) Oral combined hormonal contraceptives
E) A levonorgestrel-releasing IUD (Mirena)

Item 80
ANSWER: D

Management of polycystic ovary syndrome is typically aimed at addressing patient symptoms, as well as
irregular menses and the risk of endometrial hyperplasia. Infertility may become a therapeutic target for
women who desire pregnancy at some point in their lives. In this patient, who needs contraception and
hopes to address her hirsutism, combined oral contraceptives are most likely to address both concerns. In
addition to suppressing ovulation they also suppress gonadotropin and ovarian androgen production. The
estrogen component increases hepatic production of sex hormone binding globulin, thus decreasing

50
androgen bioavailability. Progestin-only pills and the levonorgestrel IUD protect against pregnancy but
will not improve hirsutism. Cyclic progesterone every 1–3 months can be used to prevent endometrial
hyperplasia but will not provide contraception or address hirsutism. Spironolactone is an androgen
receptor antagonist that can decrease hair growth, but it will not provide contraception.

81. Which one of the following coexisting conditions could require the use of a much higher than
expected dose of levothyroxine (Synthroid) to adequately treat hypothyroidism?

A) Chronic kidney disease


B) Diabetes mellitus
C) Helicobacter pylori gastritis
D) Hepatitis C infection
E) Hyperparathyroidism

Item 81
ANSWER: C

Absorption of levothyroxine is impaired by several gastrointestinal conditions, including atrophic


gastritis, chronic proton pump inhibitor use, and Helicobacter pylori infection. Treatment of H. pylori
infection reverses this effect, and following eradication of the infection a reduction of the levothyroxine
dosage by 30% or more will often be required.

82. A 46-year-old female has a 3-day history of dysuria with burning, frequency, and urgency. She
reports no fever, weakness, or hematuria. Her chronic health problems include obesity and
prediabetes. She has no known allergies.

Which one of the following would be the most appropriate treatment regimen for this patient?

A) Ibuprofen for 3 days


B) Trimethoprim/sulfamethoxazole (Bactrim) for 3 days
C) Ciprofloxacin (Cipro) for 3 days
D) Levofloxacin (Levaquin) for 7 days
E) Azithromycin (Zithromax) for 7 days

Item 82
ANSWER: B

Acute uncomplicated cystitis responds well to 3 days of trimethoprim/sulfamethoxazole, 160/800 mg


twice daily. Increasing resistance to fluoroquinolones has been seen and they are therefore less likely to
be successful. Because of the association with tendon rupture they are also not considered first-line
treatment. B-Lactam agents have similar resistance issues. Azithromycin is not indicated for urinary
tract infections.

Ibuprofen alone has produced good symptom relief, but antibiotics are frequently needed for a definitive
cure. The presence of diabetes or prediabetes should not change treatment (SOR A, SOR C).
51
83. A 13-year-old female sees you for the first time. Her mother has brought her in because the
patient has never menstruated. She feels well and has no other health concerns. Her medical
history is unremarkable, and she takes no medications.

A physical examination is significant for the absence of both breast development and pubic hair.
Her height is below the 2nd percentile and her weight is at the 6th percentile for her age. -hCG,
prolactin, and TSH levels are all normal. LH and FSH levels are elevated.

Which one of the following would you order at this point?

A) A trial of combined oral contraceptive pills


B) A serum testosterone level
C) Pelvic ultrasonography
D) Karyotyping
E) MRI of the brain and brainstem

Item 83
ANSWER: D

Brain MRI would be useful in ruling out brain and pituitary tumors, but this patient has no brain tumor
symptoms and normal prolactin and TSH levels. A trial of combined oral contraceptive pills may
precipitate withdrawal bleeding, but this has poor sensitivity as a marker of ovarian function (SOR C)
and will not lead to a diagnosis.

There are no signs or symptoms of hyperandrogenism, so a serum testosterone level would not be
helpful. Pelvic ultrasonography to confirm the presence of a uterus is not indicated by the physical
findings at this point.

Short stature and the lack of pubertal changes are characteristics of primary ovarian failure and Turner
syndrome. A karyotype to rule out Turner syndrome and the presence of Y chromosome material is
indicated at this point.

84. A 22-year-old gravida 1 para 0 with a history of homelessness and recent intravenous drug use
presents for prenatal care. Mantoux tuberculin testing is performed and produces 20 mm of
induration. She had a negative tuberculin test 1 year ago. A chest radiograph is normal. You
decide to treat her with isoniazid for latent tuberculosis infection.

Which one of the following should also be given to prevent the development of peripheral neuropathy?

A) Folate
B) Vitamin B6
C) Vitamin D
D) Duloxetine (Cymbalta)
E) Gabapentin (Neurontin)

Item 84
ANSWER: B

52
Most pregnant women with a positive tuberculin skin test are asymptomatic and have no evidence of
active tuberculosis, and therefore have latent tuberculosis infection. The risk of reactivation of
tuberculosis and progression to active disease is the highest in the first 2 years of conversion. In women
with a known conversion to a positive PPD in the last 2 years, treatment with isoniazid (INH), 300 mg
daily, is recommended starting after the first trimester. Treatment should last 6–9 months. Pregnant
women are at an increased risk for peripheral neuropathy when treated with INH. Vitamin B6
supplementation decreases the risk of developing peripheral neuropathy with the use of INH.

85. An 85-year-old female presents to your office with her daughter to discuss the benefits and risks of
oral anticoagulation, and to address her fall risk. She has chronic atrial fibrillation and mild
cognitive impairment, and meets the criteria for frailty. She lives with the daughter and uses a
walker but is independent for basic activities of daily living (ADLs) such as feeding, bathing,
and toileting. She does need assistance with paying bills and other instrumental ADLs. Her
quality of life is good overall and she enjoys interacting with friends and family. She has been
falling about once a month but has not sustained a serious injury.

Her blood pressure is 140/70 mm Hg. She does not use tobacco, alcohol, or illicit drugs. She
takes alendronate (Fosamax), 70 mg weekly, and hydrochlorothiazide, 12.5 mg daily. She has
been on warfarin (Coumadin) for about 3 years for the atrial fibrillation, with an INR of 2–3.
She has not had a stroke. Laboratory findings are significant for a serum creatinine level of 0.9
mg/dL (N 0.6–1.2) and normal liver enzyme levels.

You engage in shared decision making with the patient and her daughter regarding oral anticoagulation.
Which one of the following would you advise them about the risks and benefits of oral anticoagulation
for this patient?

A) The benefits outweigh the risks


B) The risks outweigh the benefits because of her age
C) The risks outweigh the benefits because of her frequency of falls
D) The risks outweigh the benefits because of her cognitive impairment
E) The risks outweigh the benefits because of her frailty

Item 85
ANSWER: A

The European Society of Cardiology 2016 Guidelines for Atrial Fibrillation state that the benefits of oral
anticoagulation outweigh the risks in the majority of patients with atrial fibrillation who meet
CHA2DS2-VASc criteria for oral anticoagulation. This includes the elderly and patients with cognitive
impairment, frailty, or frequent falling. Oral anticoagulation is superior to aspirin for the prevention of
stroke, while the bleeding risk with aspirin is not different than that of oral anticoagulation.

Use of the CHA2DS2-VASc criteria significantly increases the number of patients eligible for
anticoagulation therapy compared with the CHADS2 scoring system. If there is concern about bleeding
risk, particularly in patients older than 65 years of age, the HAS-BLED scoring system has been well
validated, with a score of 3 or more indicating that a patient has a high likelihood of hemorrhage. This
patient’s HAS-BLED score is 1 (age) and her estimated risk of major bleeding with 1 year of
anticoagulation is 1.88%–3.3%. Her adjusted stroke risk is high (4.8% per year), as she has a
CHA2DS2-VASc score of 4 (age 75, female, history of hypertension).

53
86. A 75-year-old male presents with a 12-month history of chest pressure radiating to his left arm
that occurs predictably after he walks briskly for 2 blocks and goes away with rest. A treadmill
stress test suggests coronary artery disease. The patient would prefer medical therapy over
revascularization if possible.

The patient’s blood pressure is 120/85 mm Hg. His heart rate is 52 beats/min and has been in
the low 50s at past visits. You initiate daily aspirin and a high-intensity statin, and prescribe
sublingual nitroglycerin to use as needed for chest pain.

Which one of the following additional treatments is recommended for management of his angina?

A) Isosorbide mononitrate
B) Ivabradine (Corlanor)
C) Metoprolol succinate (Toprol-XL)
D) Ranolazine (Ranexa)
E) Verapamil (Calan)

Item 86
ANSWER: A

In addition to aspirin, a high-intensity statin, and sublingual nitroglycerin as needed, patients with
chronic stable angina may be treated with -blockers, calcium channel blockers, and/or long-acting
nitrates. B-Blockers and heart rate–lowering calcium channel blockers should be avoided in this patient
who already has bradycardia. Ranolazine, which affects myocardial metabolism, is not used as a first-
line agent. Ivabradine is not a first-line agent and is used only in patients with heart failure. A long-
acting nitrate or a dihydropyridine calcium channel blocker would be appropriate for this patient.

87. A 42-year-old male hair stylist presents with numbness and tingling of the right fifth finger that
has been progressive over the last few months. He is particularly bothered after a long day of
prolonged elbow flexion. He has no pain but has noticed some clumsiness and difficulty with
fine coordination of his fingers. An examination clearly shows intrinsic muscle weakness.

Which one of the following is the most likely diagnosis?

A) Anterior interosseous nerve syndrome


B) Carpal tunnel syndrome
C) Cubital tunnel syndrome
D) Pronator syndrome
E) Wartenberg syndrome

Item 87
ANSWER: C

This patient has signs and symptoms of cubital tunnel syndrome, which is the second most common
peripheral neuropathy. Symptoms develop because of ulnar nerve compression in the upper extremity,
leading to sensory paresthesias in the ulnar digits and intrinsic muscular weakness. Vague motor
problems, including poor coordination of the fingers and hand clumsiness, are frequent complaints.
54
Provocative testing includes demonstration of Tinel’s sign over the cubital tunnel, and the elbow flexion
test with paresthesias elicited over the ulnar nerve.

Carpal tunnel syndrome causes paresthesias in the distal median nerve distribution. Wartenberg’s
syndrome reflects compression of the superficial radial nerve. Pronator syndrome is a proximal median
nerve neuropathy, while anterior interosseous nerve syndrome, a rare clinical entity, causes paresis or
paralysis of the flexor pollicis longus, and the flexor digitorum profundus of the index and long fingers.

88. One of your patients recently went into atrial fibrillation and you order an echocardiogram. The
diameter of which one of the following structures best predicts the likelihood that sinus rhythm
will be maintained after successful cardioversion?

A) The left atrium


B) The right atrium
C) The left ventricle
D) The right ventricle
E) The aortic root

Item 88
ANSWER: A

If atrial fibrillation is converted back to sinus rhythm, the likelihood of the patient staying in sinus
rhythm is best predicted from the diameter of the left atrium on the patient’s echocardiogram. Significant
left atrium enlargement means the patient is unlikely to stay in sinus rhythm after successful conversion.

Other factors that predict a lack of success in maintaining sinus rhythm after cardioversion include a
longer time in atrial fibrillation before cardioversion, or the presence of underlying heart disease,
especially rheumatic heart disease.

89. A 16-year-old male is hit on his shoulder while playing football. A radiograph subsequently
reveals a midclavicular fracture that is minimally displaced.

Appropriate management would be

A) a weekly radiograph to monitor for displacement


B) a sling for 2–6 weeks
C) a shoulder immobilizer for 6 weeks
D) a figure-of-eight bandage for 8 weeks
E) immediate referral to an orthopedist

Item 89
ANSWER: B

Fractures of the clavicle are common in young individuals, usually from sports injuries or direct trauma.
Eighty percent of these fractures occur in the midclavicle. Unless significantly displaced, these fractures
do not require referral. They can be treated with just a sling for 2–6 weeks. A sling is more comfortable
and less irritating than a figure-of-eight bandage. Passive range of motion of the shoulder is indicated as
soon as the pain allows. Physical therapy may be started at 4 weeks after the injury.
55
90. A 63-year-old male hospitalized for treatment of metastatic lung cancer develops a proximal deep
vein thrombosis of the leg. Which one of the following anticoagulants should be prescribed when he is
discharged from the hospital?

A) Apixaban (Eliquis)
B) Dabigatran (Pradaxa)
C) Enoxaparin (Lovenox)
D) Rivaroxaban (Xarelto)
E) Warfarin (Coumadin)

Item 90
ANSWER: C

In a patient with cancer, deep vein thrombosis of the leg or a pulmonary embolus is considered to be
cancer-associated thrombosis. Low molecular weight heparin (LMWH), such as enoxaparin, should be
chosen over the other anticoagulant options listed. (LMWH over warfarin is a Grade 2B
recommendation; LMWH over dabigatran is a Grade 2C recommendation; LMWH over rivaroxaban is a
Grade 2C recommendation; and LMWH over apixaban or edoxaban is a Grade 2C recommendation).

91. A 36-month-old male has persistent deficits in social communication and interaction across
multiple contexts. He displays restricted and repetitive patterns of behavior, interest, and
activities.

According to the DSM-5, which one of the following is the most appropriate diagnosis?

A) Asperger syndrome
B) Autism spectrum disorder
C) Autistic disorder
D) Childhood disintegrative disorder
E) Pervasive developmental disorder not otherwise specified

Item 91
ANSWER: B

Key diagnostic features of autism spectrum disorder include deficits in social communication and
interaction across multiple contexts and restricted, repetitive patterns of behavior, interests, or activities.
The DSM-5, which came out in 2013, created an umbrella diagnosis of autism spectrum disorder to
consolidate four previously separate disorders: autistic disorder, Asperger’s disorder, childhood
disintegrative disorder, and pervasive developmental disorder not otherwise specified. Any individuals
with a previous diagnosis of one of these disorders should now be given a diagnosis of autism spectrum
disorder.

92. A 65-year-old male presents for follow-up for his chronic hypertension. He mentions that he has a
rough spot on his arm that he has scratched off a few times, but it keeps returning. He asks
if this spot should be removed. On examination you find several scaly, slightly raised lesions on
his forearm as shown on the following page.

56
Which one of the following would be the most appropriate management?

A) Observation only
B) Application of moisturizers twice daily
C) Topical corticosteroids
D) Destruction or removal

Item #92

Item 92
ANSWER: D

This lesion is consistent with actinic keratosis, which is considered to be a premalignant lesion with the
potential to progress to squamous cell carcinoma, so treatment is generally indicated. This can be
accomplished with destruction or removal, including with topical medication therapies.

93. A 2-month-old male is brought to your office for a well child check. The parents tell you that
he regurgitates with every meal, and they are getting very frustrated. He takes about 4–5 ounces
of formula every 4 hours. With every feeding he will regurgitate a large amount of formula.
They have tried frequent burping and elevating his head during and after feeding, but he
continues to regurgitate. He does not appear to be uncomfortable and overall is a happy baby.
He is growing well and a complete examination is normal.

Which one of the following is the most appropriate management for this patient?

A) Reassurance
B) Metoclopramide (Reglan)
C) Omeprazole (Prilosec)
D) Ranitidine (Zantac)
E) Abdominal ultrasonography
57
Item 93
ANSWER: A

This child has gastroesophageal reflux. This is a normal physiologic process that occurs in infants. Most
reflux events are caused by transient lower esophageal sphincter relaxation that is triggered by
postprandial gastric distention. This relaxation can continue into childhood, but with growth and an
upright eating position it generally improves. Reflux in infants can be treated by implementing body
position changes while awake, lower volume feedings if they are overfed, thickening agents, and
antiregurgitant formula. It is recommended to avoid the use of medication in infants that have
regurgitation that is effortless, painless, and not affecting growth. When the infant is not growing well or
appears to be in pain, it would be appropriate to initiate pharmacotherapy with an acid-suppressing
medication. Abdominal ultrasonography would be indicated if there were forceful vomiting and
concerns about possible pyloric stenosis.

94. While making rounds at a nursing home you see a 70-year-old female with dementia. The staff tells
you that she has recently developed serious aggressive behaviors that include lashing out physically at
caregivers on a regular basis. Nonpharmacologic interventions have not curbed her violent outbursts.
Your evaluation does not reveal any treatable underlying conditions.

After a conversation about risks and benefits with her family and the nursing home staff, which
one of the following would you recommend for this patient?

A) Diphenhydramine (Benadryl)
B) Aripiprazole (Abilify)
C) Clonazepam (Klonopin)
D) Mirtazapine (Remeron)
E) Ziprasidone (Geodon)

Item 94
ANSWER: B

Although the FDA has not approved the use of antipsychotics for aggressive behavior associated with
dementia, they are often used to treat refractory behavioral and psychological symptoms of dementia.
Their off-label use should be considered only when nonpharmacologic therapies are ineffective and the
behaviors pose a risk of harm to the patient or others (SOR C), and the drug should be discontinued if
there is no evidence of symptom improvement (SOR A). In a meta-analysis of three atypical
antipsychotics, only aripiprazole showed small average reductions in behavioral and psychological
symptoms of dementia. Olanzapine has demonstrated inconsistent results and ziprasidone is ineffective.
Diphenhydramine is an anticholinergic agent and could exacerbate behaviors. Mirtazapine is indicated
for depression. The American Geriatrics Society recommends against the use of benzodiazepines in
older adults as a first choice for insomnia, agitation, or delirium.

95. A 56-year-old male presents to your office with the rash shown on the following page. It started
under his left arm 2 days ago and has spread slightly. It is itchy and burns a little. He has been treating it
with cortisone 1% cream twice daily with some relief. He has no significant past
medical history.
58
Which one of the following would be the most appropriate treatment?

A) Topical clobetasol
B) Topical mupirocin (Bactroban)
C) Oral cephalexin (Keflex)
D) Oral prednisone
E) Oral valacyclovir (Valtrex)

Item #95

Item 95
ANSWER: E

This patient presents with the classic rash associated with varicella zoster (shingles). The rash is
unilateral along a dermatome. Treatment of varicella zoster with antiviral therapy should be initiated if
the patient can be treated within 72 hours of the onset of rash. This will decrease the severity of the
outbreak and may decrease the risk of persistent neuropathic pain. Treatment can include topical
corticosteroids and capsaicin cream, as well as oral pain medications and a corticosteroid.

This rash is not a contact dermatitis or a heat reaction, and topical corticosteroids are not indicated as the
sole treatment. Oral corticosteroids alone should not be used to treat varicella zoster.

Varicella zoster can be complicated by a bacterial superinfection, in which case oral antistaphylococcal
antibiotics are indicated. This rash is not impetigo and there is no role for topical antibiotics.

96. A 58-year-old male comes to your office for a routine health maintenance examination. He has
smoked 1 pack of cigarettes per day for the last 35 years.

The U.S. Preventive Services Task Force recommends which one of the following lung cancer
screening strategies for this patient?

59
A) A chest radiograph annually
B) Low-dose CT annually
C) Sputum cytology every 3 years
D) Bronchoscopy every 5 years

Item 96
ANSWER: B

Lung cancer is the leading cause of cancer-related deaths in the United States and the third most
common cause of death overall. Smoking causes approximately 85% of all U.S. lung cancer deaths.
Thirty-seven percent of U.S. adults are current or former smokers. While nearly 90% of people
diagnosed with lung cancer will die from the disease, early-stage non–small cell lung cancer has a better
prognosis and can be treated with surgical resection. The largest randomized, controlled trial of low-dose
CT (LDCT) for lung cancer detection, the National Lung Screening Trial, enrolled 50,000 people age
55–74 with at least a 30-pack-year smoking history and showed a reduction in lung cancer mortality of
16% and a reduction in all-cause mortality of 6.7%. Based on this study and several other randomized,
controlled trials, the U.S. Preventive Services Task Force has concluded that LDCT has a high
sensitivity and an acceptable specificity for the detection of lung cancer in high-risk persons. The other
testing modalities listed have not been validated as acceptable screening strategies for lung cancer.

97. A pregnant 20-year-old female is diagnosed with a Chlamydia infection. She is treated and the
infection is cured, but she tests positive again at a follow-up visit.

In this situation, contacting her sexual partners is considered a reasonable breach of


confidentiality because
A) there is a public health risk
B) there is a risk to the fetus
C) the patient’s Chlamydia infection has become resistant to antibiotics
D) there is a risk of pelvic inflammatory disease

Item 97
ANSWER: A

There are three situations when a breach of confidentiality is justified: abuse of a vulnerable person
(child or elderly), a public health risk (communicable disease), or substantial danger to the patient or
others. While Chlamydia is not usually considered life-threatening or dangerous, it is communicable.
Contacting sexual partners to notify and treat them to stem the spread of disease is recommended.

98. A 22-year-old male presents with shoulder pain after sustaining an injury while playing
basketball. A radiograph of the shoulder is shown on the following page.

Which one of the following is the most likely diagnosis?

60
A) Acromioclavicular separation
B) Glenohumeral dislocation
C) Sternoclavicular dislocation
D) Clavicular fracture

Item 98 Item #98


ANSWER: B

The radiograph shows anterior glenohumeral (shoulder) dislocation. The glenohumeral articulation is the
most common joint to be dislocated, with over 90% dislocating anteriorly. The injury typically occurs as
a result of forced external rotation when the arm is abducted. On presentation there is a loss of normal
contour of the shoulder and the arm is typically held in an abducted and externally rotated position.
Associated findings may include injury to the axillary nerve (loss of sensation over the lateral shoulder),
axillary artery (diminished radial pulse on the ipsilateral side), rotator cuff (tear), or humerus (Hill-Sachs
deformities, Bankart lesions, other fractures). In posterior shoulder dislocations the arm is held adducted
and internally rotated (SOR C).

99. A 75-year-old male is admitted to the intensive-care unit with sepsis. His past medical history
is significant for diabetes mellitus and coronary artery disease.

Which one of the following would be the most appropriate maximum blood glucose goal?

A) 100 mg/dL
B) 140 mg/dL
C) 180 mg/dL
D) 220 mg/dL

Item 99
ANSWER: C

Sepsis is a severe life-threatening disorder that has a 25%–30% mortality rate. Early aggressive
management has been shown to decrease the mortality rate. The initial step in the management of sepsis
is respiratory stabilization. Fluid resuscitation should be started and followed by vasopressor therapy if
there is an inadequate blood pressure response. Antibiotics should be initiated within 1 hour of
presentation. Other interventions in early goal-directed therapy that have been shown to improve
mortality rates include blood transfusions, low-dose corticosteroid therapy, and conventional (not
intensive) glycemic control with a target glucose level of <180 mg/dL. Intensive management of glucose
in critically ill adult patients (a target glucose level of 80–110 mg/dL) has been shown to increase
mortality.

61
100. A 10-year-old female is brought to your office by her parents on the recommendation of the
school counselor. Her parents report that for at least the past year, both at home and at school,
their daughter often loses her temper, is easily annoyed, and is ―very touchy.‖ She is sullen and
angry, arguing frequently with her parents and teachers. At school she irritates and annoys
classmates, then blames them when they react negatively toward her. She also disrupts the
classroom by refusing to comply with classroom rules and expectations or with the teacher’s
authority.

This child’s behavior is most consistent with which one of the following?

A) Attention-deficit/hyperactivity disorder
B) Bipolar disorder
C) Conduct disorder
D) Intermittent explosive disorder
E) Oppositional defiant disorder

Item 100
ANSWER: E

This child presents with oppositional defiant disorder (ODD). To meet the DSM-5 criteria for ODD, the
child must demonstrate at least four symptoms from any of the following categories: angry/irritable
mood (often loses temper, is often touchy or easily annoyed, is often angry and resentful),
argumentative/defiant behavior (often argues with authority figures or with adults, often actively defies
or refuses to comply with requests from authority figures, often deliberately annoys others, often blames
others for his/her mistakes or misbehavior), and vindictiveness (has been spiteful or vindictive at least
twice within the past 6 months). These behaviors must be directed toward at least one person other than
a sibling.

Behavioral problems associated with conduct disorder are more severe, including aggression toward
animals or other persons, destruction of property, and a pattern of theft or deceit. The person’s conduct
frequently leads to conflict with authority figures.

Attention-deficit/hyperactivity disorder involves difficulty in following rules, struggles with authority


figures, and possibly annoying others. In ODD, however, defiance of authority figures occurs in settings
other than those where sustained attention or sitting quietly is required.

Bipolar disorder can include irritability and negative affect but the argumentative, defiant behavior or
vindictiveness that occur in ODD do not routinely occur in mood disorders.
Intermittent explosive disorder involves repeated, sudden episodes of impulsive, aggressive, violent
behavior or angry verbal outbursts in which the person reacts grossly out of proportion to the situation.
Road rage, domestic abuse, throwing or breaking objects, or other temper tantrums may be signs of
intermittent explosive disorder. Serious aggression toward others does not occur in ODD.

101. A 43-year-old female is very distressed about symptoms of postprandial fullness and early satiety.
She has seen several physicians over the last 6 months but said she was always told ―nothing’s wrong.‖
You review her most recent workup, including normal blood tests, normal gallbladder testing, and upper
endoscopy that shows no abnormalities, including negative testing for Helicobacter pylori. She has tried
multiple antacid medications, including omeprazole
62
(Prilosec), lansoprazole (Prevacid), and ranitidine (Zantac), with no success.
Which one of the following medications has the best chance of providing this patient with symptom
relief?

A) Clonazepam (Klonopin)
B) Escitalopram (Lexapro)
C) Metoclopramide (Reglan)
D) Ondansetron (Zofran)
E) Sucralfate (Carafate)

Item 101
ANSWER: C

This patient’s history fits the diagnosis of functional dyspepsia. Two subtypes of this disorder have been
described. The first, epigastric pain syndrome, is described as intermittent pain and burning in the
epigastrium. The second, postprandial distress syndrome, is more typical of the symptoms this patient
describes: postprandial fullness and early satiety. Although there is considerable benefit from
reassurance and ―naming‖ a patient’s condition, empiric treatment is also warranted. Patients with
epigastric pain syndrome are more likely to respond to proton pump inhibitors or H2-blockers. Patients
with predominantly postprandial distress symptoms are more likely to improve with a motility agent
such as metoclopramide.
Sucralfate, antacids, and selective antidepressants have not been shown to be more effective than
placebo in functional dyspepsia, whereas tricyclic antidepressants and buspirone have shown some
benefit and are reasonable next steps for this patient.

102. A 12-year-old male has a 1-week history of fever, headache, sore throat, and a mildly productive
cough. He also began having ear pain yesterday. On examination he does not appear to be toxic. He has
a temperature of 37.8°C (100.0°F). Examination of his ears shows a bulla on the right tympanic
membrane, as well as mild to moderate erythema of the posterior pharynx.
The neck is supple. The lungs have a few scattered crackles. The remainder of the examination is
unremarkable. A chest radiograph reveals thickened bronchial shadows, as well as interstitial infiltrates
in the lower lobes.
The most appropriate treatment at this time would be

A) amoxicillin
B) azithromycin (Zithromax)
C) ceftriaxone (Rocephin)
D) cefuroxime (Zinacef)
E) vancomycin

Item 102
ANSWER: B

Community-acquired pneumonia in children is treated based on age. The most likely etiologic agents in
a school-age child are Mycoplasma pneumoniae, Chlamydia pneumoniae, and Streptococcus
pneumoniae. Group A Streptococcus and Haemophilus influenzae are less common causes.
Staphylococcus aureus that is methicillin-resistant has become increasingly common. The preferred
treatment for community-acquired pneumonia is a macrolide antibiotic such as azithromycin.
63
In children ages 5–16, Mycoplasma pneumonia tends to have a gradual onset of symptoms and seldom
causes respiratory distress. Signs and symptoms may vary. The patient may develop a rash,
musculoskeletal symptoms, or gastrointestinal symptoms. Radiographs may reveal bronchopneumonia,
nodular infiltrates, hilar adenopathy, pleural effusions, or plate-like atelectasis. Ear pain may be due to
bullous myringitis, although this may be viral as well. Laboratory findings may not be helpful, as the
WBC count may be normal or slightly elevated. There may be thrombocytosis, an elevated erythrocyte
sedimentation rate, an elevation of cold agglutinins, or an elevated reticulocyte count. A Coombs test is
seldom needed, although it might be helpful at times. The diagnosis is generally made on a clinical basis.

103. A 36-year-old female has had elevated blood pressure readings since establishing care with you 6
months ago. You have increased her antihypertensive therapy monthly in an attempt to treat her
hypertension, and she is currently taking the maximum dosage of three antihypertensive medications
from different classes. She confirms that her blood pressure is also elevated at home, typically ranging
from 155/92 mm Hg to 165/98 mm Hg. She is otherwise well. She does not have chest pain, shortness of
breath, headaches, daytime sleepiness, or lower extremity edema. Her family history is significant for
hypertension diagnosed in her father in his 50s.
On examination the patient has a blood pressure of 168/95 mm Hg. Her pulse rate is 78
beats/min and her BMI is 28.1 kg/m2. She has a normal cardiac examination and no peripheral
edema.

You order laboratory testing, with the following significant findings:


144 mEq/L (N 136–142)
3.0 mEq/L (N 3.5–5.0)
0.72 mg/dL (N 0.6–1.2)
98 mg/dL
1.46 U/mL (N 0.4–4.2)
normal

Which one of the following additional tests is most likely to reveal the cause of her hypertension?

A) A dexamethasone suppression test


B) Plasma aldosterone/renin activity
C) Plasma free metanephrines
D) A sleep study
E) Renal ultrasonography

Item 103
ANSWER: B

In young patients with hypertension it is important to consider secondary causes in addition to the more
common essential hypertension. This patient’s relatively young age and elevated home blood pressure
readings despite drug therapy warrant further evaluation. The initial evaluation showed hypokalemia,
which suggests an endocrine cause of hypertension, specifically hyperaldosteronism. Other potential
causes of secondary hypertension include coarctation of the aorta, renal artery stenosis, thyroid
disorders, obstructive sleep apnea, pheochromocytoma, and Cushing syndrome. Each of these presents
with clinical findings that help to distinguish them from other potential causes, and the laboratory
evaluation would depend on the suspected cause
64
104. A 45-year-old female comes to your office for follow-up of palpitations and anxiety. Her pulse rate
is 112 beats/min and her heart rate is regular with no audible murmurs. Her thyroid is
nontender with no palpable nodules. You note bilateral exophthalmos as well as pretibial
myxedema. Her TSH level is 0.05 U/mL (N 0.5–5.0); free T4 and total T3 levels are elevated.
A pregnancy test is negative.

Which one of the following would be the best option for limiting the progression of this patient’s
ophthalmopathy?

A) Atenolol (Tenormin)
B) Cholestyramine (Questran)
C) Methimazole (Tapazole)
D) Prednisone
E) Radioactive iodine

Item 104
ANSWER: C

Based on her clinical presentation and classic ophthalmopathy, this patient has Graves disease. Unlike
radioactive iodine, methimazole has been shown to decrease the risk of development or progression of
ophthalmopathy in Graves disease (SOR B). Atenolol is used for symptomatic control in
hyperthyroidism. Cholestyramine can help lower thyroid hormone acutely but is not a long-term
treatment. Prednisone is used for severe hyperthyroidism and not long-term treatment. Atenolol,
cholestyramine, and prednisone do not have any effect on the long-term complications of Graves
disease.

105. A 46-year-old female with bipolar I disorder in remission seeks your advice regarding
discontinuation of her medications, which include sertraline (Zoloft), 50 mg daily; quetiapine
(Seroquel), 200 mg nightly; and hydroxyzine, 25 mg taken occasionally as needed for anxiety.
Which one of the following would you advise this patient?

A) Continue all current medications without change


B) Discontinue all medications except for as-needed use of hydroxyzine
C) Discontinue only quetiapine
D) Discontinue either sertraline or quetiapine
E) Discontinue quetiapine and start an alternative mood stabilizer

Item 105
ANSWER: A

Accepted mood stabilizers used for maintenance therapy in patients with bipolar disorder include
lithium, valproate, lamotrigine, and some atypical antipsychotics such as olanzapine, quetiapine, and
risperidone. The atypical antipsychotics are associated with weight gain and adverse metabolic changes.
Annual testing for diabetes mellitus is recommended. Long-term maintenance therapy with a mood
stabilizer is recommended in patients with bipolar I disorder due to the high risk of recurrent mania.
Monotherapy with antidepressants is contraindicated. Although the typical antipsychotics may cause QT
prolongation, atypical antipsychotics such as quetiapine do so much less frequently, and most SSRIs do
not cause this. Coadministration with sertraline is not contraindicated.
65
106. A 54-year-old male plumber presents with a 2-month history of nonpainful swelling in the
elbow. He has no history of injury or trauma. An examination is significant for a nontender, soft
nodule over the olecranon process with no erythema or warmth.
Which one of the following would be most appropriate at this point?

A) Conservative management with ice, compression wraps, and activity modification


B) Broad-spectrum antibiotics with coverage for MRSA
C) An intrabursal corticosteroid injection
D) An intrabursal hyaluronic acid (Synvisc) injection
E) Aspiration of the bursa fluid under sterile conditions

Item 106
ANSWER: A

Conservative treatment is the recommended initial management for olecranon bursitis when there is no
history of trauma or signs of septic bursitis. Aspiration of the bursal fluid is not recommended initially
due to the risk of iatrogenic infection, but can be considered for symptomatic relief if there is significant
enlargement or symptoms, or for diagnosis and culture if septic bursitis is suspected. Antibiotics are not
recommended for aseptic bursitis and should be delayed in septic bursitis until after aspiration for
culture. MRSA coverage may be indicated if the patient is at high risk for MRSA infection. An
intrabursal corticosteroid injection is not routinely recommended for bursitis unless an underlying
inflammatory condition is suspected, such as gout or rheumatoid arthritis. An intrabursal hyaluronic acid
injection is not a recommended treatment for bursitis.

107. A 74-year-old female comes to the emergency department with the acute onset of severe painin the
middle to lower back after lifting a small piece of furniture. Imaging demonstrates an acute nondisplaced
wedge compression fracture of the T12 vertebra. An examination confirms that there is no neurologic
deficit. She is admitted to the hospital.
In addition to pain management, which one of the following would you recommend?

A) Early mobilization as tolerated


B) Bed rest until a back brace is obtained
C) Evaluation for kyphoplasty
D) Evaluation for vertebroplasty
E) A neurosurgical evaluation

Item 107
ANSWER: A

Generally, the goal for patients with vertebral compression fractures is early mobilization when
tolerated. Bed rest is ordered only if movement is not tolerated. The evidence for back bracing is limited
but it can be used after weighing the risks and benefits. Current evidence supports initial conservative
treatment before considering vertebroplasty or kyphoplasty (SOR C). Neurosurgical consultation is not
required in this case.

66
108. A 69-year-old female presents with postmenopausal bleeding. You consider whether to begin your
evaluation with transvaginal ultrasonography to assess the thickness of her endometrium.

In evaluating the usefulness of this test to either support or exclude a diagnosis of endometrial cancer,
the most useful statistic is the

A) likelihood ratio
B) number needed to treat
C) prevalence
D) incidence
E) relative risk

Item 108
ANSWER: A

There has been a large increase in the number of diagnostic tests available over the past 20 years.
Although tests may aid in supporting or excluding a diagnosis, they are associated with expense and the
potential for harm. In addition, the characteristics of a particular test and how the results will affect
management and outcomes must be considered. Clinically useful statistics for evaluating diagnostic tests
include the positive predictive value, negative predictive value, and likelihood ratio.

The likelihood ratio indicates how a positive or negative test correlates with the likelihood of disease.
Ratios greater than 5–10 greatly increase the likelihood of disease, and those less than 0.1–0.2 greatly
decrease it. In the example given, if the patient’s endometrial stripe is >25 mm, the likelihood ratio is
15.2 and her post-
ratio is 0.02 and her post-test probability of endometrial cancer is 0.2%.

The number needed to treat is useful for evaluating data regarding treatments, not diagnosis. Prevalence
is the existence of a disease in the current population, and incidence describes the occurrence of new
cases of disease in a population over a defined time period. The relative risk is the risk of an event in the
experimental group versus the control group in a clinical trial.

109. A 65-year-old male presents to your office with tremor, difficulty sleeping, nausea, agitation,
and transient visual hallucinations 3 days after he stopped drinking alcohol following a 3-day
binge. His wife has noted increasing confusion over the past several hours.
Which one of the following medications would be best for treatment of his alcohol withdrawal
delirium?

A) Carbamazepine (Tegretol)
B) Haloperidol
C) Lorazepam (Ativan)
D) Phenobarbital
E) Propofol (Diprivan)

67
Item 109
ANSWER: C

A patient with alcohol withdrawal delirium (delirium tremens) is best treated as an inpatient, usually in
an intensive-care unit. Evaluation of these patients consists of a workup to identify comorbid medical
conditions, and treatment includes supportive care, intravenous thiamine, and medications to control
agitation, promote sleep, and raise the seizure threshold. The first-line pharmacologic agent for the
treatment of alcohol withdrawal delirium is a benzodiazepine, such as lorazepam. The other medications
listed are used as adjunctive treatment for severe symptoms or for patients who do not have an adequate
response to high doses of benzodiazepines. They all may have significant side effects and offer few
advantages for most patients compared to benzodiazepines.

110. Which one of the following strategies has been shown to improve the therapeutic alliance with the
patient and can aid in the management of difficult patient encounters?

A) Ignoring and putting aside the clinician’s negative feelings about the encounter
B) Motivational interviewing to explore the patient’s desire, ability, reason, and need to make a change
C) Setting a very limited amount of time for challenging appointments to keep from
interfering with the rest of the clinician’s schedule
D) Explaining why the patient’s findings from an online search of symptoms is inaccurate
and irrelevant

Item 110
ANSWER: B

Difficult patient encounters involve many factors including both patient and clinician characteristics,
behaviors, feelings, and attitudes. Motivational interviewing has been shown to improve the therapeutic
alliance and result in positive behavior change. Clinicians may need to adjust schedules to allow
appropriate time with such patients, but judgmental or dismissive remarks are not productive.

111. A 40-year-old African-American female brings you her lipid panel results from a work health fair
and asks if she should start taking medication to lower her cholesterol. The results include a total
cholesterol level of 295 mg/dL, an LDL-cholesterol level of 170 mg/dL, an
HDL-cholesterol level of 42 mg/dL, and a triglyceride level of 200 mg/dL. The patient does not
smoke and except for a BMI of 30.4 kg/m2 she is otherwise healthy. Her blood pressure is
132/76 mm Hg and she takes no medications.

Which one of the following would be consistent with current guidelines?

A) Initiating treatment with red yeast rice supplements


B) Initiating treatment with a statin medication only if she has a cardiovascular event or
develops diabetes mellitus
C) Initiating treatment based on her calculated risk of a cardiovascular event in the next 10
years
D) Initiating treatment with a statin based on her current LDL-cholesterol level

68
Item 111
ANSWER: C

The 2013 American College of Cardiology/American Heart Association cholesterol guideline suggests
statin therapy for individuals with an estimated 10-year risk of atherosclerotic cardiovascular disease of
7.5% or greater. The U.S. Preventive Services Task Force and the National Institute for Health and Care
Excellence recommend statin therapy when the patient’s 10-year risk of cardiovascular disease is 10% or
greater. While international guidelines differ somewhat, other major guidelines would support similar
recommendations.

112. An adolescent male is concerned because he is the shortest boy in his class. His age is 14.3 years
and his parents are of normal height. He has a negative past medical history and no symptoms.
On examination he is 151 cm (59 in) tall. The average height for his age is 165 cm. His weight
is 43 kg (95 lb). His sexual maturity rating is 3 for genitalia and 2 for pubic hair. A wrist
radiograph shows a bone age of 12.2 years. The average height for this bone age is 152 cm.

On the basis of this evaluation you can tell the patient and his parents that

A) he should have a growth hormone stimulation test


B) his adult height will be below average
C) his sexual development is about average for his age
D) he will begin to grow taller within approximately a year
E) an underlying nutritional deficiency may be the cause of his short stature
Item 112
ANSWER: D

Constitutional growth delay, defined as delayed but eventually normal growth in an adolescent, is
usually genetic. If evaluation of the short adolescent male reveals no evidence of chronic disease, if his
sexual maturity rating is 2 or 3, and if his height is appropriate for his skeletal age he can be told without
endocrinologic testing that he will begin to grow taller within a year or so. His adult height may be
below average but cannot be predicted reliably. Average sexual maturity ratings for a male of 14.3 years
are 4 for genitalia and 3–4 for pubic hair. The history and physical examination would have given clues
to any illnesses or nutritional problems.

113. While you are volunteering at a shelter for victims of domestic violence, a mother asks you to
examine her 12-year-old daughter who has an earache. During the examination you learn that
the child grew up in a household where there was substance abuse, mental illness, and violent
treatment of her mother. You are told that the child’s father is now incarcerated.

Considering how adverse childhood experiences affect behavior and health, this child is at greatest risk
for which one of the following?

A) Alcoholism
B) Attention-deficit disorder
C) Borderline personality disorder
D) Dissociative disorder
E) Schizophrenia
69
Item 113
ANSWER: A

Family physicians should know about health risks associated with adverse childhood experiences
(ACEs). Many risk factors are associated with cumulative ACEs. As ACEs increase so do the risks for
alcoholism, drug abuse, depression, suicide attempts, smoking, poor self-
sexually transmitted disease, physical inactivity, severe obesity, and several chronic medical conditions
that are leading causes of death in adults. Of the options listed, this child is at greatest risk for
alcoholism.

114. A 41-year-old male presents to the emergency department with severe left-sided back pain. He is
afebrile and a urinalysis shows red blood cells. Imaging reveals a 6-mm stone in the lower
ureter and no hydronephrosis.

In addition to NSAIDs and narcotics for pain control, which one of the following would be most
effective for hastening passage of the stone?

A) An 1-blocker such as tamsulosin (Flomax)


B) A 5- -reductase inhibitor such as finasteride (Proscar)
C) A nonselective -blocker such as propranolol
D) A phosphodiesterase inhibitor such as sildenafil (Viagra)
E) A thiazide diuretic such as hydrochlorothiazide

Item 114
ANSWER: A

alpha1-Blockers such as doxazosin, prazosin, and tamsulosin have been shown to hasten the passage of
ureteral stones (level 2 evidence). They are probably more effective than calcium channel blockers (level
2 evidence). -Blockers, phosphodiesterase inhibitors such as sildenafil, 5- alpha reductase inhibitors such
asfinasteride, and thiazide diuretics have not been shown to hasten stone passage. However, thiazide
diuretics have been shown to decrease stone formation in patients with hypercalciuria.

115. A 42-year-old female who has been your patient for 5 years has uncontrolled type 2 diabetes
mellitus. She asks your opinion about bariatric surgery to reduce her BMI of 41.1 kg/m2 and
improve her diabetes because she is ―sick of all the pills and doctor visits.‖

You inform her that bariatric surgery

A) decreases the risk of dying from obesity-related illness


B) usually results in weight loss similar to that seen with diet and exercise
C) usually allows patients to lose weight without changing their diet
D) often allows patients to stop all medications and supplements

70
Item 115
ANSWER: A

In 2013 nearly 180,000 bariatric surgery procedures were performed in the United States. Bariatric
surgery does result in reduced all-cause mortality and more weight loss. National Institutes of Health
Consensus Development Conference eligibility criteria include comprehension of risks, benefits,
expected outcomes, alternatives, and required lifestyle changes, including required postoperative
lifelong supplements, diet changes, and follow-up appointments.

116. A 29-year-old male presents to your office with a 2-week history of anal pain and bright red
blood on his stool with bowel movements. He says he typically has bowel movements every 3–5
days and his stool is usually hard. He has not noted any purulent drainage or perianal masses.

Which one of the following is the most likely diagnosis?

A) Anal fistula
B) Anal fissure
C) Internal hemorrhoids
D) External hemorrhoids
E) Perirectal abscess

Item 116
ANSWER: B

Posterior midline fissures cause pain during and after defecation. Most are caused by the passage of hard
stool and when stretched cause bleeding. Conservative therapy consisting of bulk agents and stool
softeners usually allows these to heal.

Internal hemorrhoids can cause bleeding with the passage of stool but are typically painless. External
hemorrhoids can bleed with trauma but typically cause pain with thrombosis, independent of bowel
movements. Anal fistulas and perirectal abscesses may intermittently drain purulent material. Abscesses
can cause continuous pain, and a perianal mass may be noted on examination.

117. A 48-year-old female is treated appropriately for MRSA bacteremia. An echocardiogram is


negative for endocarditis. There are no indwelling devices such as prosthetic heart valves or
vascular grafts.

Assuming that the patient improves with an excellent response to antibiotics, which one of the
following is recommended?

A) No repeat blood cultures


B) Repeat blood cultures when the antibiotic course is completed

D) Repeat blood cultures 2–4 days after the initial set and as needed thereafter
E) Repeat blood cultures 2 weeks after the antibiotic course is completed
71
Item 117
ANSWER: D

This patient’s MRSA bacteremia is considered uncomplicated due to the effectiveness of the antibiotic
therapy and the lack of endocarditis or implanted prostheses such as heart valves. Therefore, the
Infectious Diseases Society of America recommends that follow-up cultures of blood samples be
obtained 2–4 days after the initial cultures and as needed thereafter to document clearance of bacteremia
(SOR A; Quality of Evidence II).

118. GLP-1 agonists such as exenatide (Byetta) can be used as second-line agents to help improve
glycemic control in patients with type 2 diabetes mellitus. Which one of the following is a
CONTRAINDICATION to their use?

A) Hypothyroidism
B) Thyroid cancer
C) Coronary artery disease
D) Heart failure

Item 118
ANSWER: B

GLP-1 agonists are contraindicated in patients with medullary thyroid cancer or multiple endocrine
neoplasm syndrome, or with a family history of these conditions. They are not associated with heart
failure, coronary artery disease, or hypothyroidism. They have been associated with pancreatitis in rare
cases, but this is not a contraindication to prescribing them.

119. A 4-year-old male is brought to your office for evaluation of fever, coryza, and cough. On
examination the child appears mildly ill but in no respiratory distress. His temperature is 37.4°C
(99.3°F) and other vital signs are within the normal range. An HEENT examination is
significant only for light yellow rhinorrhea and reddened nasal mucous membranes. Lung
auscultation reveals good airflow with a few coarse upper airway sounds. While performing the
examination you note multiple red welts and superficial abrasions scattered on the chest and
upper back. When you question the parents they tell you the marks are where ―the sickness is
leaving his body,‖ and were produced by rubbing the skin with a coin.

This traditional healing custom is practiced principally by people from which geographic region?

A) Sub-Saharan Africa
B) Southeast Asia
C) The Middle East
D) The Caribbean
E) The Andes Mountains

72
Item 119
ANSWER: B

Coin rubbing is a traditional healing custom practiced primarily in east Asian countries such as
Cambodia, Korea, China, and Vietnam. The belief is that one’s illness must be drawn out of the body,
and the red marks produced by rubbing the skin with a coin are evidence of the body’s ―release‖ of the
illness. These marks may be confused with abuse, trauma from some other source, or an unusual
manifestation of the illness itself.

120. A 48-year-old female presents to the emergency department with chest pain. The evaluation,
including CT angiography, reveals a pulmonary embolus.

Which one of the following initial findings would be the strongest indication for thrombolytic
therapy?

A) Elevated troponin
B) Hypotension
C) Hypoxia
D) Bilateral pulmonary emboli
E) Right ventricular dysfunction on echocardiography

Item 120
ANSWER: B

Recent guidelines have suggested that hypotension (a systolic blood pressure <90 mm Hg or a diastolic
blood pressure <60 mm Hg, for 15 minutes or longer) should be treated with thrombolysis in patients
who are not at high risk for bleeding. Patients who have other indicators of cardiopulmonary impairment
without signs of hypotension should be given anticoagulation therapy and aggressive supportive care,
but should not be treated with thrombolytic therapy. If the patient’s condition continues to deteriorate as
evidenced by the development of hypotension or other clinical indicators of cardiopulmonary
compromise, thrombolysis may be considered.

121. A 50-year-old male sees you for a health maintenance visit. He has a 20-year history of smoking 1
pack of cigarettes per day but his history is otherwise unremarkable. He is not aware of any disease that
runs in his family, including diabetes mellitus. He has not visited a physician for the past 5 years. A
physical examination reveals a BMI of 28.2 kg/m2, normal blood pressure, and no other
significant findings. Laboratory testing reveals a fasting plasma glucose level of 107 mg/dL. According
to the U.S. Preventive Services Task Force, which one of the following would be
most appropriate at this point?
A) No further diabetes screening unless his BMI increases to equal or more 30 kg/m2
B) A repeat fasting plasma glucose level in 1–2 weeks
C) A repeat fasting plasma glucose level in 1 year
D) A repeat fasting plasma glucose level in 3 years
E) Treatment with metformin (Glucophage)

73
Item 121
ANSWER: B

The U.S. Preventive Services Task Force (USPSTF) recommends screening for abnormal blood glucose
levels as part of a cardiovascular disease risk assessment for adults who are 40–70 years of age and who
are overweight (BMI 25.0–29.9 kg/m2) or obese (BMI equal or above 30.0 kg/m2). Since his fasting
blood glucose result was in the range consistent with impaired fasting glucose (100–125 mg/dL), the
USPSTF recommends confirming the diagnosis of the abnormal glucose level soon by performing the
same test on a different day. Appropriate treatment should begin once the diagnosis is confirmed.

122. You are discharging a mother and her infant after delivery. The infant was born at 35 weeks
gestation, is exclusively breastfed, and has a weight that is appropriate for her gestational age. No blood
transfusions were necessary and all evaluations have been unremarkable. According to the guidelines of
the American Academy of Pediatrics, at what age should you
recommend initiation of iron supplementation for this infant?
A) Now
B) 1 month
C) 3 months
D) 6 months
E) 9 months

Item 122
ANSWER: B

According to the guidelines of the American Academy of Pediatrics, elemental iron supplementation (2
mg/kg per day) should begin at 1 month of age for exclusively breastfed infants born before 37 weeks
gestation and should continue until 12 months of age, unless the infant had multiple blood transfusions.

123. A nursing home resident is hospitalized, and shortly before she is to be discharged she develops a
skin ulcer that proves to be infected with MRSA.
Which one of the following is most important in terms of infection control when she returns to the
nursing home?

A) Surveillance cultures of residents with rooms near the patient


B) Aggressive housekeeping in the patient’s room
C) Masks, gowns, and gloves for anyone entering the patient’s room
D) Strict handwashing practices by all staff, visitors, and residents
E) Isolation of the patient in a room by herself

Item 123
ANSWER: D

All staff, visitors, and nursing home residents should observe strict handwashing practices when a
resident has a MRSA infection.
74
Barrier precautions for wounds and medical devices should also be initiated. Surveillance cultures are
not warranted. Aggressive housekeeping practices play little, if any, role in preventing the spread of
MRSA. Isolating the patient is not practical or cost effective.

124. A 46-year-old runner presents with left heel pain. The pain has been occurring mostly with
running, but more recently it is painful with walking. On examination there is tenderness and a palpable
nodule on the midsubstance of the left Achilles tendon.
Which one of the following therapeutic options is most likely to be effective for long-term recovery?
A) Oral NSAIDs
B) Eccentric calf-strengthening exercises

C) Corticosteroid injection of the Achilles tendon sheath


D) Surgical debridement or excision of the tendon nodule
E) Fixation in a walking boot

Item 124
ANSWER: B

Achilles tendinopathy is among the most common injuries in middle-aged distance runners. Oral
NSAIDs may be helpful for temporary pain relief, but they contribute little to recovery from this injury.
Corticosteroid injection is contraindicated due to the risk of tendon rupture. Surgical debridement and
fixation in a walking boot may be considered as a last resort for difficult cases, but the most effective
treatment overall is eccentric calf-strengthening exercises.

125. A 65-year-old white female is diagnosed with hypertension. She has no significant past medical
history. According to the JNC 8 panel, which one of the following would be an appropriate systolic
blood pressure goal for this patient?
A) <120 mm Hg
B) <130 mm Hg
C) <140 mm Hg
D) <150 mm Hg
E) <160 mm Hg

Item 125
ANSWER: D

In 2013 the members of the Eighth Joint National Committee (JNC 8) updated guidelines for the
management of hypertension, recommending treatment of systolic blood pressure to <150 mm Hg in
patients >60 years of age without comorbid conditions, based on evidence that a systolic blood pressure
150 mm Hg reduces rates of cardiovascular disease, stroke, and death (SOR A). While this was a recent
recommendation, the evidence supporting the beneficial effects of lowering blood pressure is decades
old .The JNC 8 panel concluded that there is insufficient evidence to support a specific systolic blood
pressure goal in adults <60 years of age. A Cochrane review concluded that treatment of patients with
mild hypertension (systolic blood pressure of 140–159 mm Hg and/or diastolic blood pressure of 90– 99
mm Hg) did not reduce morbidity or mortality compared with placebo (SOR A).

75
126. A mother who recently immigrated to the United States from Mexico brings her 4½-year-old son
to your clinic for his pre- kindergarten examination. The child’s examination is normal except for a
hemangioma located on his left arm. His mother says that the lesion appeared at about 4 weeks of age,
continued to grow until he was about 5 months of age, and then began to flatten, shrink, and fade. She is
concerned because it has not improved in the past 18 months. When you examine the lesion more
closely you note telangiectasia, fibrofatty tissue, dyspigmentation, and scarring where involution has
occurred.
Which one of the following would be the most appropriate management?

A) Oral propranolol
B) Corticosteroid injection
C) Referral for laser therapy
D) Referral for surgical excision

Item 126
ANSWER: D

Infantile hemangiomas usually appear by 4 weeks of age and stop growing by 5 months of age. As
manyas 70% leave residual skin changes, including telangiectasia, fibrofatty tissue, redundant skin,
atrophy, dyspigmentation, and scarring. Systemic corticosteroids were the mainstay of treatment for
hemangiomas during infancy until 2008, when the FDA approved oral propranolol for this indication.
Intralesional corticosteroids can be effective for small, bulky, well localized lesions in infants. Laser
therapy can also be used to treat early lesions or residual telangiectasia. Once involution is complete,
however, as is the case with this child, elective surgical excision is the treatment of choice, producing
better outcomes.

127. A 53-year-old male sees you for follow-up of his hypertension. His medical history includes
prediabetes and gout, and he is currently taking lisinopril (Prinivil, Zestril), 40 mg daily, to control his
blood pressure. His blood pressure after resting is 148/86 mm Hg. Laboratory findings include a serum
creatinine level of 0.8 mg/dL (N 0.6–1.2) and a serum potassium level of 4.5 mEq/L (N 3.5–5.1). Which
one of the following would be the most appropriate management of this patient’s hypertension?

A) No change in medication
B) Add amlodipine (Norvasc)
C) Add hydrochlorothiazide
D) Add losartan (Cozaar)
E) Add metoprolol succinate (Toprol-XL)

Item 127
ANSWER: B

First-line agents for hypertension include ACE inhibitors,


and calcium ,angiotensin receptor blockers, thiazide diuretics channel blockers. Hydrochlorothiazide
would be relatively contraindicated due to the patient’s gout. Losartan, an angiotensin receptor blocker,
should not be added because the patient is taking an ACE inhibitor. Metoprolol succinate, a -blocker, is
not a first-line agent for blood pressure unless there is another indication
.such as systolic heart failure or migraine prophylaxis
76
128. In a study to evaluate a test as a screen for the presence of a disease, 235 of the 250 people with
the disease had a positive test and 600 of the 680 people without the disease had a negative test. Based
on this data, the specificity of the test for the disease is

A) 235/250 = 94%
B) 15/250 = 6%
C) 600/680 = 88%
D) 80/680 = 12%
E) 15/80 = 19%

Item 128
ANSWER: C

The specificity of a test for a disease is the proportion or percentage of those without the disease who
have a negative test. In this case, option A is the sensitivity, which is the proportion of those with the
disease who have a positive test. Option B is the false-negative rate and option D is the false-positive
rate. Option E is the ratio of false-negative tests to false-positive tests, a meaningless ratio. The
predictive values of positive and negative tests are extremely important characteristics of a screening
test. Determination of these values requires knowledge of the prevalence of the disease in the population
screened, as well as the sensitivity, specificity, and false-positive and false-negative rates. Since the
prevalence of most diseases is low, the percentage of those with a positive test (the predictive value of a
positive test) is relatively low, even when sensitivity and specificity are high. When prevalence is low,
however, the predictive value of a negative test is very high and may approach 100%.

129. A 45-year-old female presents with a 6-month history of fatigue and arthralgias. When asked
about recent illnesses the only thing she can recall is that she developed a rash on her face after a picnic
about 1 month ago. An examination reveals swelling and tenderness in her left knee and over the
proximal interphalangeal joints of the second and third digits of her right hand. An antinuclear antibody
test and an anti–double-stranded DNA test are both positive. A CBC and blood chemistries are within
normal limits.

Which one of the following medications would be the best initial treatment for this patient?
A) Azathioprine (Imuran)
B) Cyclosporine
C) Hydroxychloroquine (Plaquenil)
D) Mycophenolate (CellCept)
E) Rituximab (Rituxan)

Item 129
ANSWER: C

According to the American College of Rheumatology criteria, this patient has systemic lupus
erythematosus, with photosensitivity, arthritis, a positive antinuclear antibody test, and a positive anti–
double-stranded DNA test. She has a mild form of the disease.
77
Hydroxychloroquine reduces arthritis pain in lupus patients (SOR A) and is the preferred initial
treatment for lupus arthritis. Cyclosporine and azathioprine are indicated for severe lupus or lupus
nephritis. Mycophenolate is indicated for refractory lupus or lupus nephritis. Rituximab is indicated for
severe refractory lupus. Ref: Lam NC, Ghetu MV, Bieniek ML: Systemic lupus erythematosus: Primary
care approach to diagnosis and management. Am Fam Physician 2016;94(4):284-294

130. A 64-year-old male presents to your office after vomiting blood twice over the last 2 hours. He is
healthy otherwise, except for hyperlipidemia. On the initial examination he has mild hypotension and
tachycardia, which normalize after fluid resuscitation. He has no further vomiting episodes.

What level of hemoglobin should be the threshold for transfusion of red cells in this situation?
A) 6.0 g/dL
B) 7.0 g/dL
C) 8.0 g/dL
D) 9.0 g/dL
E) 10.0 g/dL

Item 130
ANSWER: B

In otherwise healthy stable patients with upper gastrointestinal bleeding, a transfusion of red cells is
recommended when the hemoglobin level falls below 7.0 g/dL. In hypotensive patients with severe
bleeding, a blood transfusion before the hemoglobin level reaches 7.0 g/dL is needed to prevent
significant decreases below this level that would occur with just fluid resuscitation. In hemodynamically
stable patients with known cardiovascular disease and significant upper gastrointestinal bleeding, 8.0
g/dL should be the threshold for a blood transfusion.

131. You order an NT-pro BNP level in a patient with symptoms and signs of heart failure. Which one
of the following would contribute to a result that is higher than expected?
A) Male sex
B) Elevated BMI
C) Elevated albumin
D) Elevated creatinine

Item 131
ANSWER: D

Elevated levels of NT-pro BNP are known to indicate an increased likelihood of heart failure, and lower
levels can rule out heart failure. However, certain patient characteristics can lead to higher levels of NT-
pro BNP even in healthy individuals. The use of one normal cutoff level for elevated NT-pro BNP may
not be appropriate. Even healthy female patients and those >65 years of age will have higher levels of
NT-pro BNP than younger male patients (SOR A).
NT-pro BNP is negatively correlated with kidney function as measured by the estimated glomerular
filtration rate (GFR) and albumin levels. Patients with a low GFR or a low level of albumin have higher
NT-pro BNP levels (SOR A). Interestingly, grip strength is negatively correlated with NT-pro BNP as
well. A higher BMI is associated with a lower NT-pro BNP. Thus, the utility of NT- pro BNP to rule out
heart failure in obese patients is decreased (SOR A)

78
132. While you are examining a 3-day-old for a routine newborn checkup, his mother, who is also your
patient, tells you that she has been crying very easily and feeling overwhelmed, and that some of her
friends have suggested she ask about an antidepressant medication. She has no previous history of
depression or other mood disorders. She does not have any thoughts of self-harm or intent to harm the
baby or anyone else. She is breastfeeding and is otherwise healthy. Which one of the following would be
most appropriate at this point?
A) Reassurance that she likely has baby blues that will resolve in a few days
B) Encouraging her to make an appointment with a therapist if she does not feel better in 1 week
C) Referral to a counselor for psychotherapy
D) Use of a validated screening tool to evaluate for postpartum depression
E) Starting an SSRI for postpartum depression

Item 132
ANSWER: D

While it is possible that this patient has baby blues, given the short duration of symptoms proximate to
delivery, more evaluation is needed to differentiate between baby blues, which are very common and
unlikely to contribute to the development of postpartum depression, and an episode of postpartum
depression that has much more significant implications. There is more than one screening tool for
postpartum depression, including the Edinburgh Postnatal Depression Scale, the PHQ-9 questionnaire,
and the Postpartum Depression Screening Scale. An SSRI is the appropriate treatment for confirmed
moderate to severe postpartum depression along with psychotherapy, while mild to moderate postpartum
depression can be treated with psychotherapy or an SSRI. Referral to psychotherapy may be appropriate,
but further evaluation should be pursued in order to determine the best treatment plan. Due to the risks
associated with postpartum depression, an evaluation should be performed at this visit rather than asking
the patient to monitor symptoms.

133. In the development of clinical guidelines, which one of the following is rated as the strongest and
highest-quality evidence?
A) Evidence from randomized, placebo-controlled studies
B) Evidence from nonrandomized, double-blind, placebo-controlled studies
C) Evidence from nonrandomized, double-blind, crossover, placebo-washout–controlled studies
D) Evidence from well designed cohort or case-control analytical studies from more than one center
or research group
E) Evidence based on reports of expert committees or opinions of respected authorities in the
appropriate specialty area

Item 133
ANSWER: A

Randomized, controlled studies yield stronger evidence than other types of studies, especially case-
control or cohort studies, because randomization provides the greatest safeguard against unanticipated
study bias. Evidence obtained from randomized, controlled studies is considered level 1 (strongest) by
the U.S. Preventive Services Task Force. Evidence obtained from nonrandomized, controlled studies is
considered level 2a, well-designed case-control and cohort studies are considered level 2b, and reports of
expert committees or respected authorities are considered level 3 (weakest).

79
134. A 62-year-old male sees you the day after returning from a 4- day cruise. He says he developed a
fever and a productive cough on the day before the ship returned to Los Angeles following a trip down
the coast of Baja California. He tells you that several other passengers had similar symptoms. The
examination is remarkable for tachypnea and you hear crackles in both lungs. This patient’s history
should raise concerns about infection with which one of the following pathogens?
A) Asian avian influenza A virus
B) Coxiella burnetii
C) Hantavirus
D) Histoplasma capsulatum
E) Legionella species

Item 134
ANSWER: E

Legionella should be considered as a pathogen for community- acquired pneumonia when the patient has
a history of a hotel stay or cruise ship travel within the past couple of weeks. Travel to or residence in
Southeast Asia or East Asia is a risk factor for avian influenza, exposure to farm animals or parturient
cats is a risk factor for Coxiella burnetii infection, exposure to bird or bat droppings is a risk factor for
Histoplasma capsulatum infection, and travel to or residence in desert Southwest states with deer mouse
exposure is a risk factor for Hantavirus infection.

135. You see a 12-year-old female for a well child check. She is healthy without any medical problems
and neither she nor her father have any concerns today. She is up to date on her
immunizations except for her third dose of HPV vaccine. She received the first dose of the vaccine at her
11-year-old well child check and her second dose 1 month later. However, she was told by a few friends
that they only needed to get two doses of HPV vaccine, so she is very excited that she does not need any
shots today. According to the CDC, which one of the following is true regarding HPV vaccine for this
patient?
A) She has completed her HPV vaccine series
B) She should receive a third dose of HPV vaccine today
C) She needs an HPV booster at age 21 but does not need a third dose of vaccine today
D) She should have HPV titers drawn today and receive a third dose of vaccine only if the titers are
low

Item 135
ANSWER: B

In 2016 the CDC changed the recommendation for the number of HPV vaccine doses for children ages
11–14. Children in this age group need only two doses of HPV vaccine 6–12 months apart.
However, if they received two doses of HPV vaccine less than 5 months apart, they still need to have the
third dose. Children and young adults over the age of 14 and those with certain immunocompromising
conditions still require three doses of HPV vaccine. There is no indication for a booster dose at a later
date, nor is there clinical data to support using titers to gauge immunogenicity to HPV.

80
136. A 90-year-old female with severe dementia is seen in the emergency department for a left knee
strain. She was at home alone for 2 hours and no fall or injury was witnessed. A radiograph of the knee
is negative, and she is referred to you for follow-up the next day. At the follow-up visit the patient is
confused and agitated, and cries out at any attempt to examine her. She is unable to bear weight on her
left leg and it appears to be externally rotated.

Which one of the following would be most appropriate at this point?


A) Reassurance and pain medication
B) A repeat radiograph of the left knee
C) A radiograph of the left hip
D) A radiograph of the lumbosacral spine
E) MRI of the left knee

Item 136
ANSWER: C

A fractured hip is possible and must be ruled out since there is difficulty bearing weight and the leg is
externally rotated. Examination of a patient with severe dementia can be extremely difficult. Other
findings with a fractured hip would include pain elicited on rotation and groin pain when applying an
axial load. If the hip radiograph is negative, MRI of the knee may be considered.

137. In which one of the following scenarios would additional consent from a child’s parent or
guardian be necessary before proceeding with treatment in nearly every U.S. state?

A) A 6-year-old female with divorced parents who lives primarily with her mother is brought to the
clinic by her father to discuss his concerns of possible abuse
B) An 8-year-old unconscious male is brought to the emergency department by a neighbor after
falling out of a tree and striking his head
C) A 13-year-old male is brought to the clinic by a babysitter with a note giving permission to treat
signed by a parent
D) A 15-year-old female who is considered emancipated under state law comes to your office to
discuss family planning
E) A 16-year-old female who has driven herself to her clinic appointment reports a 2-day history of
ear pain; she says her mother made this appointment for her

Item 137
ANSWER: E

Informed consent to treat is considered an important ethical and legal part of caring for children and
adolescents. Some situations can become confusing when trying to balance the need for treatment, a
child’s consent, and a parent or guardian’s permission. In most states, 18 is the age when legal decisions
can be made; however, in some states 21 years old is the age of legal consent Children below the age of
majority must have proof of permission to treat from a parent or guardian for non-emergent care. This
does not apply to emergency situations in which a delay in care could result in serious harm. Another
exception to parental consent is when a child is considered emancipated under state law. This can
happen with a court order, or (in some states) if the child is married, is a parent, is in the military, or is
living independently. Either biologic parent can consent to treatment unless one of them is explicitly
81
denied guardianship. If a child presents with a non-emergent condition and does not have evidence of
permission from a parent or guardian, permission should be sought before the physician interaction takes
place.

138. A 30-year-old female presents to your office for evaluation of a 5-mm pulmonary nodule noted on
CT of the chest performed after a motor vehicle accident 2 weeks ago. She has had no symptoms and she
is not a smoker. A physical examination is unremarkable. Which one of the following would be most
appropriate at this point?
A) Non contrast chest CT in 1 year
B) A PET scan
C) Referral for a needle biopsy
D) Referral for bronchoscopy
E) Referral for wedge resection

Item 138
ANSWER: A

By definition, a pulmonary nodule is a circumscribed, round lesion that may measure up to 3 cm in size
and is surrounded by aerated lung. Management is based on the size of the nodule and the probability of
malignancy. Risk factors for lung cancer include a previous malignancy, a positive smoking history, and
age equal or above 65. Only 1% of nodules between 2 mm and 5 mm in size are malignant. Nodules <8
mm are difficult to biopsy, and a PET scan is not reliable. The risk of surgery outweighs the benefits in
nodules of this size. For a low-risk patient with a nodule 4 mm to <6 mm in size, a repeat non contrast
CT at 12 months is recommended. If it is unchanged, no further follow-up is needed.

139. A 45-year-old female visits your office for her annual health maintenance visit and mentions that
her hair has been thinning over the past few years. She is now concerned that it may be noticeable. She
takes no medications and is otherwise healthy with normal menstrual cycles. On examination she has a
negative pull- away test. You note diffuse thinning in the parietal regions, with sparing of the frontal
hairline. She has no scalp scarring, scale, or erythema.

Which one of the following would be the most appropriate pharmacotherapy?


A) Finasteride (Propecia), 1 mg daily
B) Griseofulvin, 500 mg daily
C) Hydroxychloroquine (Plaquenil), 200 mg twice daily
D) Minoxidil 2% (Rogaine), applied to the scalp twice daily
E) Triamcinolone 0.05% (Trianex), applied to the scalp twice daily

Item 139
ANSWER: D

Female pattern hair loss is categorized as diffuse and nonscarring. It presents with parietal hair thinning
with preservation of the frontal hairline. Minoxidil 2% produces regrowth of hair in female pattern hair
loss (SOR B). Oral finasteride is appropriate only for men with male pattern hair loss (SOR A).
Hydroxychloroquine is used for inflammatory hair loss associated with discoid lupus erythematosus,
which is focal and scarring. Topical corticosteroids are appropriate for alopecia areata (SOR B) but not
for female pattern hair loss. Griseofulvin is used to treat tinea capitis, which presents as focal scale with
erythema.
82
140. A 16-year-old male is brought to your office by his mother for ―stomachaches.‖ On the review of
systems he also reports headaches, occasional bedwetting, and trouble sleeping. His examination is
within normal limits. His mother says that he is often in the nurse’s office at school, and doesn’t seem to
have any friends. When you discuss these problems with him he admits to being teased and called names
at school.
Which one of the following would be most appropriate?

A) Explain that he must try to conform to be more popular


B) Explain that these symptoms are a stress reaction and will lessen with time
C) Explore whether his school counselor has a process to address this problem
D) Order a TSH level

Item 140
ANSWER: C

Childhood bullying has potentially serious implications for bullies and their targets. The target children
are typically quiet and sensitive, and may be perceived to be weak and different. Children who say they
are being bullied must be believed and reassured that they have done the right thing in acknowledging
the problem. Parents should be advised to discuss the situation with school personnel. Bullying is
extremely difficult to resolve. Confronting bullies and expecting victims to conform are not successful
approaches. The presenting symptoms are not temporary, and in fact can progress to more serious
problems such as suicide, substance abuse, and victim-to-bully transformation. The signs and symptoms
in the patient described here are consistent with bullying and not thyroid disease. The Olweus Bullying
Prevention Program developed in Norway is a well documented, effective program for reducing bullying
among elementary and middle- school students by altering social norms and by changing school
responses to bullying incidents, including efforts to protect and support victims. Students who have been
bullied regularly are more likely to carry weapons to school, be in frequent fights, and eventually be
injured.

141. A 67-year-old male is admitted to the intensive-care unit with complications of sarcoma of the
leg, including multiple pulmonary emboli. He most likely will require an urgent above-the-knee
amputation.
A spiritual assessment in this patient
A) should be delegated to the hospital chaplain
B) can be performed by the physician
C) is not appropriate if the patient has not listed a religion in his demographics
D) should be postponed until the patient is in less critical condition

Item 141
ANSWER: B

The Joint Commission for Hospital Accreditation now requires a patient spiritual assessment upon
hospital admission. Using the FICA Spiritual History Tool or HOPE questions for making the spiritual
assessment is appropriate for the physician. HOPE is a mnemonic for sources of Hope, Organized
religion, Personal spirituality and practices, and Effects on medical care and end-of-life issues. The
83
FICA tool includes questions in the categories of Faith and beliefs, Importance, Community, and how to
Address these issues when providing care. It is very appropriate for a physician to conduct a spiritual
assessment in older, hospitalized patients with critical or terminal illnesses. Some patients may consider
themselves spiritual but not necessarily religious.

142. A 60-year-old male with COPD returns for a follow-up visit. He has been treated twice for
pneumonia in the past year and is using his albuterol metered-dose inhaler (Proventil, Ventolin)
several times a day. His FEV1 is 60% of predicted.
To reduce symptoms and prevent hospitalization,
which one of the following would be the most appropriate medication to add to this patient’s treatment
regimen at this time?

A) Roflumilast (Daliresp)
B) A long-acting β 2-agonist bronchodilator
C) A long-acting β 2-agonist and an inhaled corticosteroid
D) A long-acting anticholinergic inhaler
E) Long-acting theophylline

Item 142
ANSWER: D

To decrease the patient’s frequent use of a short-acting bronchodilator, the addition of a long-acting
anticholinergic inhaler for maintenance is most reasonable. Such medications have been demonstrated to
improve quality of life and reduce hospitalizations. Evidence has demonstrated that a -agonist combined
with an inhaled corticosteroid would not be as preferable in this case, as they can increase the incidence
of pneumonia. Long-acting theophylline and roflumilast may be added to the regimen of patients still
symptomatic on triple therapy with long-acting β 2-agonists, inhaled corticosteroids, and anticholinergic
bronchodilators.

143. A 2-year-old female is brought to your office because of a round lesion on her lip that appeared
2 days ago. Her temperature and all vital signs are normal. She has no past medical history and
takes no medications. Further history reveals that she was playing with a toy trumpet in a busy
store a few days before the lesion appeared. A physical examination reveals a 1-cm round lesion
with crusting, and no other skin abnormalities.
Which one of the following would be the best treatment at this time?
A) Bacitracin
B) Mupirocin (Bactroban)
C) Neomycin
D) Cephalexin (Keflex)
E) Clindamycin (Cleocin)

Item 143
ANSWER: B

This patient has physical findings and a history consistent with impetigo, a skin infection caused by
Staphylococcus aureus and/or Streptococcus pyogenes. Since she has only one lesion, systemic
84
antibiotics are not required as they would be for a patient with extensive disease or multiple lesions.
Although bacitracin and neomycin are commonly used, they are much less effective for impetigo than
mupirocin, despite some reports of resistance to mupirocin (level A-1 evidence).

144. A 9-year-old female presents with a 4-week history of right knee pain with activity. There is no
history of trauma or recent illness. Your examination reveals lateral patellar tracking with
extension of the knee.
Which one of the following is the most likely diagnosis?
A) Patellofemoral pain syndrome
B) Osgood-Schlatter disease
C) Growing pains
D) Patellar tendinopathy
E) Sever’s disease

Item 144
ANSWER: A

Patellofemoral pain syndrome is one of the most common causes of knee pain in children, particularly
adolescent girls. Pain beneath the patella is the most common symptom. Squatting, running, and other
vigorous activities exacerbate the pain. Walking up and down stairs is a classic cause of the pain, and
pain with sitting for an extended period is also common. The physical examination reveals isolated
tenderness with palpation at the medial and lateral aspects of the knee, and the grind test is also positive.
Osgood-Schlatter disease is seen in skeletally immature patients. Rapid growth of the femur can cause
tight musculature in the quadriceps across the knee joint. It typically appears between the ages of 10 and
15, during periods of rapid growth. Pain and tenderness over the tibial tubercle and the distal patellar
tendon is the most common presentation. The pain is aggravated by sports participation, but also occurs
with normal daily activities and even at rest. Growing pains most often affect the thigh and quadriceps
and occur during late afternoon or evening, or wake the patient at night. The joints are not affected. The
pain typically goes away by morning, and may sometimes occur the day after vigorous or unusual
activity.Patellar tendinopathy is an overuse injury often seen in those who participate in jumping sports
such as volleyball, and is also related to frequent stops and starts in football players. It typically causes
infrapatellar pain, and findings include extensor mechanism malalignment, weakness of ankle flexors,
and tightness of the hamstring, heel cord, and/or quadriceps.
Sever’s disease is an overuse syndrome most often seen between the ages of 9 and 14, and is related to
osteochondrosis at the insertion of the Achilles tendon on the calcaneal tuberosity. It occurs during
periods of rapid growth, causes heel pain during and after activity, and is relieved with rest. It is often
related to beginning a new sport or the start of a season.

145. When evaluating a 52-year-old farmer for complaints of possible visual loss, headaches, and a
suspected corneal lesion, you note the funduscopic findings shown on the following page. These
findings are highly suggestive of
A) optic neuritis
B) glaucoma
C) papilledema
D) optic atrophy
E) rubeosis

Item #145
85
Item 145
ANSWER: B

This funduscopic picture shows a 70% ―cupping‖ or abnormal enlargement of the light-colored area
around the central vascular area in the disc, which is a phenomenon that highly suggests increased
intraocular pressure or glaucoma. Primary care physicians, especially family physicians, who often
examine the eyes for other symptoms must be aware of these early changes in order to allow patients to
obtain treatment to reverse or ameliorate the disease process.

146. A 36-year-old female calls your office because of a 2-day history of dysuria, urinary urgency,
and urinary frequency. She has not had any fever, nausea, or vaginal discharge. She tells you
her symptoms are similar to a previous urinary tract infection. She uses subdermal etonogestrel
(Nexplanon) for contraception, takes no oral medications, and has no drug allergies.
Which one of the following would be most appropriate at this point?
A) Prescribe ciprofloxacin (Cipro)
B) Prescribe nitrofurantoin (Macrobid, Macrodantin)
C) Prescribe a urinary analgesic such as phenazopyridine (Pyridium)
D) Ask the patient to come in today for evaluation
E) Ask the patient to submit a urine specimen before you prescribe antibiotics

Item 146
ANSWER: B

Urinary tract infection (UTI) is the most common bacterial infection in women. The annual incidence of
UTI in women is 12%. Women who have had a UTI in the past are usually quite adept at diagnosing
their own subsequent UTIs. Prospective studies have shown that women who suspect they have a UTI
are more than 85% accurate based on culture results (SOR B). This is more accurate than dipstick
testing, which has a sensitivity of 75% and a specificity of 82%. Nonpregnant female patients who have
typical UTI symptoms without signs of pyelonephritis (i.e., fever and nausea) or vaginitis can be treated
safely and effectively by phone. Urine culture testing is not indicated for uncomplicated UTIs. It has
been found that the traditional criterion for infection (100,000 colony-forming units/mL) is not sensitive
for women with a UTI. Urine testing should be reserved for patients suspected of having pyelonephritis.
There are three first-line antibiotics for uncomplicated UTI. These include nitrofurantoin for 5 days,
trimethoprim/sulfamethoxazole for 3 days, and fosfomycin as a single dose (SOR A). Fluoroquinolones
are second-line agents and are best reserved for more serious infections such as pyelonephritis.
Urinary analgesics can be helpful with UTI symptoms but are not the preferred method of treatment, as
antibiotics rapidly reduce the symptoms of infection.

147. A 50-year-old female comes to your office for routine health maintenance. She jogs 2 miles a
day and has had left medial knee pain for the last 6 months. Radiographs reveal moderate
degenerative arthritis of the knee. Her BMI is 24.1 kg/m2 and her physical examination,
including an examination of the knee, is otherwise normal.

Which one of the following would be most effective for this patient’s arthritis?
A) Weight loss
B) A knee brace
C) Foot orthoses
D) Hyaluronic acid injection
E) Exercise-based physical therapy
86
Item 147
ANSWER: E

Exercise-based therapy is the foundation for treating knee osteoarthritis. Foot orthoses can be helpful for
anterior knee pain but this patient’s pain is located medially. The benefit of hyaluronic acid injections is
controversial, and recommendations vary; recent systematic reviews do not support a clinically
significant benefit. Weight loss is recommended for patients with a BMI >25.0 kg/m2. Wearing a knee
brace has shown little or no benefit for reducing pain or improving knee function.
148. A home health nurse calls you about a 62-year-old male who is recovering at home several days
after spinal surgery. His recovery was going well until he became unable to urinate despite the
sensation of needing to do so. His last normal void was about 12 hours ago and felt incomplete.
Catheterization produced 900 mL of clear-appearing urine that she will send for urinalysis. His
bowel movements have been normal and his need for pain medications has been decreasing.
In addition to stopping medications that may be promoting his urinary retention, which one of
the following management strategies would be most appropriate?

A) Leaving the indwelling catheter in place for 48–72 hours


B) Starting finasteride (Proscar), 5 mg daily
C) Starting oxybutynin, 10 mg daily
D) Sending the patient to the emergency department

Item 148
ANSWER: A

This patient is suffering from acute urinary retention, likely due to mild benign prostatic hyperplasia
exacerbated by pain medication and a lack of activity. Acute urinary retention could also possibly be due
to irritation of sympathetic and/or parasympathetic nerves near the spine. Placing an indwelling bladder
catheter is appropriate. It would also be reasonable, although impractical in the short term, to teach the
patient or his caretakers to intermittently catheterize him.
The likelihood of a successful return to voiding spontaneously will increase over time. However, the risk
of catheter-associated urinary tract infection is estimated to be about 5% per day. Therefore, catheter
removal and a trial of spontaneous voiding should be attempted after 48–72 hours. There is good
evidence that starting an α-blocking medication such as tamsulosin during the time the catheter is in
place will nearly double the success of the trial of spontaneous voiding. Finasteride in isolation is not
recommended and oxybutynin would be contraindicated.
149. A 65-year-old female comes to your office in June for her Welcome to Medicare visit. She has
no health concerns but believes she is due for some immunizations. She received Tdap 10 years
ago. She has never had MMR and she did not receive influenza vaccine this past winter.

According to the CDC adult immunization schedule, which one of the following is indicated for
this patient?

A) Influenza vaccine
B) MMR
C) 13-valent pneumococcal conjugate vaccine (PCV13, Prevnar 13)
D) 23-valent pneumococcal polysaccharide vaccine (PPSV23, Pneumovax 23)
E) Tdap

87
Item 149
ANSWER: C

Patients 65 years of age and older should be vaccinated against Pneumococcus. The recommended
sequence for a previously unvaccinated person is to give the 13-valent pneumococcal conjugate vaccine
(PCV13) first, followed by the 23-valent pneumococcal polysaccharide vaccine (PPSV23) in 12 months.
The interval was increased from 6 months in 2015. This patient has already received a Tdap booster so
she requires only a Td booster. She does not need an MMR booster, as anyone born prior to 1957 is
considered immune. Influenza vaccine should be delayed until fall. She is also due for herpes zoster
vaccine if that was not offered when she turned 60.

150. A 29-year-old mother of three young children asks your opinion on giving probiotics to her
children. Which one of the following is a benefit of the use of probiotics in children?
A) They reduce diarrhea associated with irritable bowel syndrome
B) They reduce colic symptoms in formula-fed infants
C) They prevent the development of allergies
D) They prevent antibiotic-associated diarrhea

Item 150
ANSWER: D

A wealth of evidence-based research has established the benefits of probiotics, especially in children.
Probiotics, particularly Saccharomyces boulardii, have been shown to prevent the antibiotic-associated
diarrhea that occurs in 5%–30% of children who receive antibiotics (SOR A). The number needed to
treat to prevent one case of diarrhea is 10.
Probiotics reduce the pain associated with irritable bowel syndrome (IBS) but have not been shown to be
helpful in reducing diarrhea or constipation in pediatric IBS patients (SOR A).
In breastfed infants, probiotics reduce daily crying time by up to an hour. Similar benefits have not been
found in formula-fed infants or infants who are combining breastfeeding and formula. Probiotics have
not been shown to prevent colic in any infants. Other benefits of probiotics include the prevention of
eczema and upper respiratory infections (SOR A). However, their use has not been shown to prevent
allergies and asthma.
151. A 30-year-old male is taking a motorcycle trip in 6 weeks to Colorado, including a ride to the
top of Pikes Peak (elevation 14,100 ft). He has never been above 5000 ft prior to this trip and
is concerned about developing acute mountain sickness (AMS). He is generally healthy and takes
no medications, but smokes 1 pack of cigarettes per day. He is allergic to penicillin.

Which one of the following is the best option for this patient to reduce his risk of developing
AMS?

A) An intensive aerobic fitness program


B) Varenicline (Chantix) for smoking cessation
C) Prophylaxis with acetazolamide
D) Prophylaxis with Ginkgo biloba

Item 151
ANSWER: C

Acute mountain sickness (AMS) occurs in at least 25% of persons traveling to destinations over 8000
feet above sea level. Risk factors include rapid ascent, living at low altitudes (<2000 ft), a prior history
88
of altitude illness, and strenuous physical exertion during the ascent. AMS is most often manifested by
headache, fatigue, lightheadedness, and/or nausea. The best way to prevent AMS is gradual ascent, but
medications may also be effective in prophylaxis, especially if a rapid ascent such as in motorcycling,
driving, or flying to altitude is planned.
The drug of first choice in preventing AMS is acetazolamide, a carbonic anhydrase inhibitor, starting the
day before ascent. It is, however, contraindicated in patients with sulfa allergy. The second-line drug for
prevention is dexamethasone, which should be used for prophylaxis in sulfa-allergic patients. It is also
used in the treatment of AMS and high-altitude cerebral or pulmonary edema, but immediate descent of
at least 2000 feet is imperative if either of those more serious complications develop. While advocated
as a prophylactic and treatment option for AMS, the results for ginkgo are mixed and it is therefore not
recommended for use in this situation. Smoking cessation and physical conditioning are both good ideas
for this patient, but neither will reduce his risk for developing AMS.

152. A 21-year-old female asks you about Papanicolaou (Pap) testing recommendations. You
determine she is at average risk for cervical cancer and recommend which one of the following?
A) Pap testing without HPV co-testing now and in 1 year if results are normal
B) Pap testing without HPV co-testing now and in 3 years if results are normal
C) Pap testing with HPV co-testing now and in 3 years if results are normal
D) Pap testing with HPV co-testing now and in 5 years if results are normal
E) HPV testing only, now and in 5 years if results are normal

Item 152
ANSWER: B

Family physicians should be familiar with age-appropriate cancer screening recommendations. Deaths
from cervical cancer have been significantly reduced through screening. HPV testing is not
recommended for screening in average-risk women younger than 30 years old. Cytology without HPV
testing is recommended for screening every 3 years for an average-risk 21-year-old female.

153. A 34-year-old female with systemic sclerosis sees you for a follow-up visit. She is afebrile, with
a blood pressure of 132/76 mm Hg, a heart rate of 82 beats/min, and an oxygen saturation of
94% on room air. On examination you note that the patient is thin and has fibrotic skin changes
proximal to the elbows and knees, and facial tightening. She does not have increasing shortness
of breath but does have ongoing chronic musculoskeletal pain. She is currently taking
cyclophosphamide prescribed by her rheumatologist. Pulmonary function tests reveal an FVC
<50%, consistent with restrictive lung disease. CT of the chest shows ground-glass opacities
and honeycombing of the lower lobes of the lungs.

Which one of the following do these findings suggest?


A) Emphysema
B) Idiopathic pulmonary fibrosis
C) Interstitial lung disease
D) Pulmonary edema
E) Sarcoidosis

89
Item 153
ANSWER: C

Patients with systemic sclerosis (SS) in its final stages often develop a restrictive lung disease (SOR C).
Interstitial lung disease and pulmonary artery hypertension are common. While the restrictive pattern is
similar to idiopathic pulmonary fibrosis, this condition is characteristic of SS and is not idiopathic.
Emphysema presents with an obstructive pattern on pulmonary function tests. Pulmonary edema can
develop from cardiac malfunction and heart failure, but it is not present in this patient. Sarcoidosis is not
related to SS. There is a 10-year mortality of 42% in patients with SS who have an FVC <50%.
Cyclophosphamide may be helpful in some cases to improve lung function, decrease dyspnea, and
improve the patient’s quality of life (SOR B).

154. While percussing the chest of a 38-year-old male during his annual health maintenance
examination, you notice the lesion shown on the following page on the upper left side of his
back. The lesion is approximately 1.2 cm across the long axis.
Which one of the following would be the most appropriate initial intervention?
A) A superficial shave biopsy
B) A punch biopsy of the peripheral margin
C) A punch biopsy at the center of the lesion
D) An excisional biopsy with a 1- to 3-mm border around the lesion
E) Wide excision with a 2-cm border around the lesion

Item 154 Item #154


ANSWER: D

The appearance of this lesion strongly suggests malignant melanoma. Although any of the options listed
can provide tissue adequate for pathologic confirmation, the American Academy of Dermatology
recommends an excision with narrow margins (1–3 mm) that includes the entire gross lesion with
clinically negative margins to a depth that ensures the lesion is not transected (SOR B, level of evidence
2). For smaller lesions punch excision with sutures or shave removal to an adequate depth are acceptable
alternatives if the recommended excision objectives can be met. Once the tissue diagnosis is obtained,
appropriate staging and definitive surgery should be undertaken.

155. Which one of the following variables is the most important risk factor for being a victim of
domestic abuse?

A) Educational background
B) Psychological problems
C) Race
D) Gender
E) Socioeconomic status

Item 155
ANSWER: D
90
Domestic violence can affect children, intimate partners, and older adults. It is a serious medical
problem that should be considered in the care of patients and families. There is great variation in the
profiles of patients affected by domestic violence. Neither demographic factors nor psychological
problems have been found to be consistent predictors of victimization or violence. Domestic violence
cuts across all racial, socioeconomic, religious, and ethnic lines. The only consistent risk factor for being
a victim of domestic violence is female gender.

156. A 50-year-old female undergoes screening colonoscopy and a 1.2-cm polyp is discovered and
removed. She is told that it is benign but she is concerned about her future risk for developing
colon cancer. Which histologic type of colonic polyps >1.0 cm in size has the highest likelihood of
becoming
malignant?

A) Hamartomatous polyps
B) Hyperplastic polyps
C) Inflammatory polyps
D) Tubular adenomas
E) Villous adenomas

Item 156
ANSWER: E

Colon cancer arises from adenomatous polyps, and generally requires at least 5 years of growth before
malignant transformation. Villous adenomas carry a threefold increased risk for becoming malignant
compared with other adenomatous types such as tubular or tubulovillous adenomas. The larger the polyp
the greater the chance of malignancy, although malignant polyps <1.5 cm are rare. Hamartomas
(juvenile polyps) and inflammatory polyps (often associated with inflammatory bowel disease) are
benign. Hyperplastic polyps are the most common histologic type by far, but only rarely become
cancerous.

157. Which one of the following is diagnostic for type 2 diabetes mellitus?
A) A fasting plasma glucose level ≥126 mg/dL on two separate occasions
B) An oral glucose tolerance test (75-g load) with a 2-hour glucose level ≥160 mg/dL
C) A random blood glucose level ≥200 mg/dL in an asymptomatic person
D) A hemoglobin A1c ≥6.0% on two separate occasions

Item 157
ANSWER: A

The American Diabetes Association recommends screening for all asymptomatic adults with a BMI
>25.0 kg/m2 who have one or more additional risk factors for diabetes mellitus, and screening for all
adults with no risk factors every 3 years beginning at age 45. Current criteria for the diagnosis of
diabetes mellitus include a hemoglobin A1c ≥6.5%, a fasting plasma glucose level ≥126 mg/dL, a 2-hour
plasma glucose level ≥200 mg/dL, or, in a symptomatic patient, a random blood glucose level ≥200
mg/dL. In the absence of unequivocal hyperglycemia, results require confirmation by repeat testing.

91
158. A 66-year-old male with nonalcoholic steatohepatitis, hyperlipidemia, and hypertension sees you
for follow-up. He was recently diagnosed with cirrhosis. He has no new health complaints today.
His medications include metoprolol, simvastatin (Zocor), and low-dose aspirin. His vital signs
include a blood pressure of 120/75 mm Hg, a heart rate of 72 beats/min, and an oxygen
saturation of 96% on room air. The physical examination is notable for mild ascites and trace
lower extremity edema.

Which one of the following recommendations would be most appropriate for this patient?

A) Fluid restriction
B) Limiting sodium intake
C) A low-protein diet
D) Discontinuation of metoprolol
E) Discontinuation of simvastatin

Item 158
ANSWER: B

Dietary counseling is key in the management of cirrhosis. Patients with ascites should be limited to 2000
mg of sodium daily (SOR A). Fluid restriction is not recommended unless the serum sodium level is
<120 mEq/L, and it is extremely difficult to achieve. Patients would need to limit fluid intake to less
than their urinary output, which is often reduced.
Up to 60% of patients with cirrhosis suffer from malnutrition, so a high-protein diet is recommended
(1.0–1.5 g/kg dry body weight). High-protein diets are tolerated well and result in improved mental
status (SOR B). Protein restriction does not benefit those with hepatic encephalopathy.
β-Blockers should be stopped when the patient’s mean arterial pressure is <82 mm Hg (blood pressure
100/73 mm Hg) (SOR B). β -Blockers are used in early cirrhosis for hypertension, reducing portal
pressures, and prevention of varices. They are no longer effective when the patient becomes hypotensive
or has refractory ascites, spontaneous bacterial peritonitis, or other conditions associated with significant
changes in hemodynamic status. This patient is normotensive on his current dosage of metoprolol. As
his cirrhosis progresses the metoprolol will likely need to be discontinued.
Statins can be safely used in patients with cirrhosis. Their cardiovascular benefits are well established
and the risk of associated liver failure is extremely low (SOR A).

159. A 26-year-old female presents with a 3-month history of abdominal pain, lightheadedness with
standing, and some hyperpigmentation. Her CBC is normal, but a basic metabolic panel reveals
a slightly low sodium level and a slightly high potassium level.

Which one of the following would be the most appropriate next step in your evaluation of this
patient?

A) A serum aldosterone level


B) A corticotropin (ACTH) stimulation test
C) Paired morning cortisol and melanocyte stimulating hormone levels
D) 21-hydroxylase antibodies and 17-hydroxyprogesterone levels

92
Item 159
ANSWER: B

This patient has signs and symptoms consistent with primary adrenal insufficiency (PAI). In Western
countries autoimmunity is responsible for 90% of these cases. Because the corticotropin (ACTH)
stimulation test has a higher degree of sensitivity and specificity than morning cortisol and ACTH
concentrations, it is the preferred test in all patients with possible primary adrenal insufficiency. Serum
aldosterone paired with plasma renin activity is used to screen for adrenal hyperplasia in hypertensive
patients and also for establishing the existence of mineralocorticoid insufficiency in patients with PAI.
Once the diagnosis is established, 21-hydroxylase antibodies and 17-hydroxylase progesterone levels are
used to determine the etiology of PAI.

160. A 54-year-old male presents to your office with a 2-day history of mild right anterior chest pain
with deep breathing. He reports that it has been sharp and constant and is preventing him from
sleeping. He also describes shortness of breath and a cough productive of white sputum. He
reports that prior to this episode he had been in good health, and he has not experienced these
symptoms in the past.
On examination his blood pressure is 140/88 mm Hg, his temperature is 37.1°C (98.8°F), his
pulse rate is 88 beats/min, and his oxygen saturation is 95% on room air. Heart auscultation
reveals a regular rhythm with no murmur. His lungs are clear. Examination of the lower
extremities reveals no edema or tenderness. A chest radiograph is normal. An EKG reveals right
bundle branch block.

Which one of the following would you order next?

A) A D-dimer level
B) Compression ultrasonography
C) Echocardiography
D) A ventilation-perfusion scan of the lungs
E) CT angiography of the lungs

Item 160
ANSWER: A

Validated clinical prediction rules can be used to estimate the pretest probability of deep vein thrombosis
(DVT) and pulmonary embolism in a patient with dyspnea and chest pain, and to guide further
evaluation (SOR C). Factors used for calculating the pretest probability include elevated heart rate
without hemoptysis, a diagnosis of cancer, recent surgery/immobilization, previous thromboembolism,
and signs and symptoms of DVT. Based on these rules the patient described in the scenario has a low
score and therefore a low probability of pulmonary embolism.
A D-dimer level is the next most appropriate test for this low-probability scenario. Compression
ultrasonography would be the next test for a patient with an intermediate or high pretest probability for
DVT. CT angiography would be the next test for a clinically stable patient with an intermediate or high
pretest probability of pulmonary embolism. A ventilation-perfusion scan would be the next test if a CT
angiogram were indicated in a patient with a contraindication such as contrast allergy, renal disease, or
pregnancy. Echocardiography would be the next test for a critically ill patient with a high pretest
probability of pulmonary embolism.

93
161. Which one of the following has the best evidence supporting its use for acute low back pain
without radicular symptoms?
A) Acupuncture
B) Bed rest
C) Lumbar support
D) Oral corticosteroids
E) Cyclobenzaprine

Item 161
Answer : E

NSAIDs, acetaminophen, and muscle relaxants are effective for the treatment of acute low back pain
(SOR A). There is moderate-quality evidence that nonbenzodiazepine muscle relaxants are beneficial in
the treatment of acute low back pain. There is also moderate-quality evidence that NSAIDs combined
with nonbenzodiazepine muscle relaxants may have additive benefit for decreasing pain. Bed rest is not
helpful in the treatment of acute back pain and is not recommended (SOR A). There is no evidence that
lumbar support is helpful. Oral corticosteroids have not been found to be beneficial for isolated low back
pain, but there is questionable benefit when there are associated radicular symptoms. There are several
low-quality trials that show acupuncture has minimal or no benefit over sham treatment in acute back
pain.

162. A 70-year-old male is being treated with medication for type 2 diabetes mellitus. Which one of
the following hemoglobin A1c values is associated with the lowest mortality in this situation?
A) 5.0%–5.9%
B) 6.0%–6.9%
C) 7.0%–7.9%
D) 9.0%–9.9%

Item 162
Answer : C

values of 7%–8% have shown the greatest reduction in mortality in multiple studies. It is suggested that
frequent hypoglycemia is associated with lower hemoglobin A1c values, and that presents a greater risk.
Values over 9% are associated with greater mortality (SOR B). Thus, while the risk of complications
increases linearly with hemoglobin A1c, mortality has a V-shaped curve.

94
163. An 80-year-old male presents with a 4-hour history of generalized abdominal pain, vomiting,
and fever to 101°F. On examination you note normal cardiovascular findings, generalized
moderate abdominal tenderness, absent bowel sounds, and a normal rectal examination.

The diagnostic abdominal film shown on the following page suggests


A) a leaking abdominal aortic aneurysm
B) toxic megacolon
C) small bowel obstruction
D) diverticulitis
E) a perforated viscus

Item 163
Answer : C
Item #163

Radiographs confirm the clinical diagnosis of small-bowel obstruction in


most patients and more accurately define the site of obstruction. Small-bowel obstruction typically
occupies the more central portions of the abdomen. Patients with mechanical small-bowel obstruction
usually have minimal or no colonic gas. Films taken in the upright or lateral decubitus position in
patients with small-bowel obstruction usually show multiple gas-filled levels, with the distended bowel
resembling an inverted U.
Patients with small-bowel obstruction are likely to be depleted of fluids and electrolytes, and require
intravenous fluids, electrolyte management, and surgical evaluation.

164. A 64-year-old male presents with a 2-week history of a worsening constant burning pain in his
right foot. The physical examination reveals an absent dorsalis pedis pulse, and pallor develops
with elevation of the foot. The resting right ankle-brachial index is 0.45 and Doppler waveform
analysis indicates an isolated severe stenosis of the right posterior tibial artery.
Which one of the following therapeutic interventions would be most appropriate at this point?
A) A supervised walking program
B) Cilostazol (Pletal)
C) Warfarin (Coumadin)
D) Revascularization

Item 164
Answer : D

Medication and/or a walking program have been shown to improve functional capacity in patients with
symptomatic peripheral artery disease (PAD). However, this patient has critical limb ischemia and needs
urgent revascularization. Endovascular therapy of isolated disease below the knee is not recommended.
These patients should undergo femoral-tibial bypass. Warfarin is not recommended for the treatment of
PAD.

95
165. An otherwise healthy 32-year-old female sees you for a routine health maintenance examination
after having required laboratory screening at work. She has no significant past medical history
and the physical examination is normal. Laboratory results are unremarkable except for an
elevated alkaline phosphatase level.
Which one of the following would be most appropriate at this point?
A) A gamma-glutamyl transferase level
B) A parathyroid hormone level
C) Hepatitis C antibody testing
D) Ultrasonography of the liver
E) Bone scintigraphy

Item 165
Answer : D

When an otherwise healthy patient’s alkaline phosphatase level is elevated, it is helpful to determine
whether the source of elevated isoenzymes is the liver or bone. One method is fractionation of the
alkaline phosphatase by electrophoresis. A gamma-glutamyl transferase or 5 -nucleotidase level can also
indicate whether the elevation of the alkaline phosphatase is from the liver. These enzymes are rarely
elevated in conditions other than liver disease. Severe vitamin D deficiency can lead to osteomalacia
with an elevated alkaline phosphatase.

166. A rural community college has requested your guidance in offering a preventive health program
to its students. The most appropriate plan would include which one of the following?
A) Mammograms for female students
B) Lead poisoning screening for all students
C) Stool occult blood kits for all students
D) Smoking cessation programs for students who smoke
E) An annual routine physical examination for all students

Item 166
Answer : D

The U.S. Preventive Services Task Force does not recommend routine physical examinations but
recommends blood pressure screenings every 3–5 years for adults 18–39 years of age who are at low
risk for hypertension. Mammograms are not recommended until age 40. Lead screening is recommended
for at-risk individuals between 6 months and 6 years of age. Colorectal cancer screening for average-risk
individuals is recommended at age 50. Counseling on tobacco use and other substance abuse is
recommended as part of all routine preventive care.

167. A 72-year-old male has newly diagnosed systolic heart failure due to hypertensive
cardiomyopathy. The patient has an estimated left ventricular ejection fraction of 30% and was
dyspneic at rest and with minimal exertion at the time of diagnosis.
Which one of the following drugs is indicated to reduce mortality in this patient?
A) Atenolol (Tenormin)
B) Digoxin
C) Furosemide (Lasix)
D) Lisinopril (Prinivil, Zestril)
E) Nifedipine (Procardia)

96
Item 167
Answer : D

Medications shown to improve mortality in patients with heart failure with a reduced ejection fraction
include ACE inhibitors, angiotensin receptor blockers, -blockers, aldosterone antagonists, and in
African-American patients, direct-acting vasodilators. Among the -blockers, carvedilol, bisoprolol, and
metoprolol succinate have this indication. Diuretics, along with digoxin, may improve symptoms but do
not alter disease mortality.

168. A 65-year-old female develops gram-negative septicemia from a urinary tract infection. Despite
the use of fluid resuscitation she remains hypotensive, with a mean arterial pressure of 50 mmHg
.
Which one of the following would be the most appropriate treatment for this patient?
A) Vasopressin
B) Phenylephrine (Neo-Synephrine)
C) Epinephrine
D) Norepinephrine (Levophed)
E) Low-dose dopamine

Item 168
Answer : D Item #168

In a patient with sepsis, vasopressors are indicated when fluid resuscitation does not restore organ
perfusion and blood pressure. Norepinephrine and dopamine are the preferred pressor agents; however,
norepinephrine appears to be more effective and has a lower mortality rate. Norepinephrine is the
preferred drug for shock due to sepsis. Its relative safety suggests that it be used as an initial vasopressor.
It is a potent vasoconstrictor and inotropic stimulant and is useful for shock. As a first-line therapy
norepinephrine is associated with fewer adverse events, including arrhythmia, compared to dopamine.
Phenylephrine, epinephrine, or vasopressin should not be used as first-line therapy. Vasopressin is
employed after high-dose conventional vasopressors have failed. The use of low-dose dopamine is no
longer recommended based on a clinical trial showing no benefit in critically ill patients at risk for renal
failure. If an agent is needed to increase cardiac output, dobutamine is the agent of choice.

169. A 5-year-old male fell while playing yesterday and complained that his wrist hurt. He is brought
to your office today because he refuses to use his arm.
Which one of the following best describes the condition seen in the radiographs of his wrist,
shown on the following page?
A) A normal appearance
B) A distal radial buckle fracture
C) An ulnar greenstick fracture
D) A radioulnar diaphyseal fracture
E) A Salter-Harris II radial physeal fracture
97
Item 169
Answer : B

Even though they are the most common fracture in this age group, radial fractures can be missed by
clinicians. The bend in the cortex of the distal radius indicates the fracture. Sometimes referred to as a
buckle or torus fracture, it will heal with almost any choice of treatment. Most clinicians opt for casting
to reduce the chance of reinjury during the first few weeks of healing, but parent preference in this
regard is important. Some pediatric long-bone fractures involve growth plates, and the results can be
indeterminate, requiring either more advanced imaging or comparison views of the opposite limb.

170. A 20-year-old female visits your office for advice regarding contraceptives. She expresses
interest in long-acting reversible contraception (LARC).
Which one of the following is the recommended timing for LARC placement?
A) Any time during the menstrual cycle
B) 14 days before the anticipated onset of menses
C) 7 days before the anticipated onset of menses
D) 7 days after the onset of menses
E) 14 days after the onset of menses

Item 170
Answer : A

Long-acting reversible contraception (LARC) includes the copper IUD, levonorgestrel IUDs, and
subdermal implants. LARCs can be placed at any point in the patient’s menstrual cycle (SOR A). There
should be evidence that the patient is not pregnant prior to placement.

171. A 55-year-old male with chronic paranoid schizophrenia, mild intellectual disability, and
hypertension is brought to your office with foot pain. He resides in a personal care home and
is accompanied by his caregiver, who says the patient had pain this morning and could not walk.
His medications include hydrochlorothiazide, lisinopril (Prinivil, Zestril), risperidone
(Risperdal), and metformin (Glucophage). On examination he has an erythematous, warm,
tender metacarpophalangeal joint of his right great toe. He refuses joint aspiration but does agree
to allow blood work. A CBC and metabolic profile are both normal and his uric acid level is 6.8
mg/dL (N 3.4–7.0).
Which one of the following is the most likely diagnosis?
A) Gout
B) Osteoarthritis
C) Reactive arthritis
D) Rheumatoid arthritis
E) Trauma

98
Item 171
Answer : A

Gout is definitively diagnosed by aspiration of synovial fluid that is examined under polarized light
microscopy to identify negatively birefringent uric acid crystals. In primary care settings the diagnosis of
gout is often made clinically, and a clinical diagnostic rule has been developed that increases the
accuracy of the diagnosis. Points are scored as follows:

• acute onset with maximum symptoms within 1 day 0.5


• joint erythema 1.0
• cardiovascular disease or hypertension 1.5
• male sex 2.0
• previous attack of joint pain or arthritis 2.0
• first metatarsophalangeal joint involved 2.5
• serum uric acid >5.88 mg/dL 3.5
The maximum score is 13.0 points
–7.5 calls for
either joint aspiration for polarized light microscopy or referral to a rheumatologist.

172. A 33-year-old female with a BMI of 35.2 kg/m2 presents to your office for follow-up of her
recently documented blood pressure elevation. Her blood pressure is elevated again today. You
diagnose hypertension and decide to start pharmacologic therapy in addition to lifestyle
modifications.
Which one of the following agents would be the best first-line choice for this patient?
A) Verapamil (Calan)
B) Clonidine (Catapres)
C) Hydrochlorothiazide
D) Metoprolol tartrate (Lopressor)
E) Spironolactone (Aldactone)

Item 172
Answer : C

The 2014 evidence-based guideline from the panel members appointed to the Eighth Joint National
Committee (JNC 8) makes few suggestions regarding preferred initial agents for the treatment of
hypertension. Of the options listed, only hydrochlorothiazide is a reasonable option for first-line
treatment. Verapamil is rarely used for blood pressure control.

173. A 60-year-old male presents with a 6-week history of worsening bilateral shoulder, upper arm,
and neck pain. He has morning stiffness that lasts at least an hour. The review of systems is
otherwise negative. There is no localized tenderness or motor weakness on physical examination.
His erythrocyte sedimentation rate is 55 mm/hr.

Which one of the following is the best treatment option for this patient at this time?
A) Aspirin, 1000 mg 3 times daily
99
B) Indomethacin, 25–50 mg 3 times daily
C) Methotrexate, 7.5 mg once a week
D) Prednisone, 10–20 mg once daily
E) Prednisone, 20 mg 3 times daily

Item 173
Answer : D

The patient described has polymyalgia rheumatica (PMR). The hallmark of this condition is the rapid
and often dramatic response, typically within a few days, to low-dose corticosteroids. In fact, the lack of
response to low-dose prednisone in such a case should prompt the physician to consider another
diagnosis. A related condition, giant cell arteritis, is associated with transient or even permanent vision
loss, typically unilateral but sometimes bilateral. This condition usually presents with headache and
tenderness of the affected artery, most commonly the temporal artery. Prompt recognition and the
initiation of high-dose corticosteroids are keys to preventing blindness.
The other options listed are not pertinent to the management of PMR. While prompt response to low-
dose corticosteroids confirms the diagnosis, they are usually continued for 1–2 years, with gradual
tapering beginning several months after initiation of treatment (SOR C).

174. A 20-year-old male college student comes to your office to ask what he can do to prevent
meningitis. His roommate was just hospitalized with invasive meningococcal disease. The patient
has no symptoms at this time, a physical examination is normal, and he received meningococcal
vaccine at the recommended times. He lives in a campus dormitory that houses 22 students.

Which one of the following would be most appropriate?

A) Reassure him that as long as he did not have direct contact with respiratory secretions
he is at low risk of infection and does not need prophylaxis
B) Reassure him that because of his immunization status he is at low risk of infection and
does not need prophylaxis
C) Tell him there is no preventive treatment so he should monitor his symptoms very
closely and return immediately if he develops a fever, headache, or stiff neck
D) Treat him and the rest of the students in the dormitory with a single dose of
ciprofloxacin (Cipro)

Item 174
Answer : D

Postexposure prophylaxis after exposure to invasive meningococcal disease is indicated for any close
contact regardless of immunization status. Close contacts include those exposed in households,
dormitories, or day care centers, and those who have direct contact with oral secretions. There are
several options for prophylaxis, including ciprofloxacin, 500 mg orally one time; azithromycin, 500 mg
orally one time; ceftriaxone, 250 mg intramuscularly one time; or rifampin, 600 mg orally twice daily
for 2 days. Treatment should begin as soon as possible after exposure but no later than 14 days. While
rifampin can be used for postexposure prophylaxis, a one-time dose is not adequate. This patient is at
risk of infection due to his close contact with the source patient. Postexposure prophylaxis is indicated
even for vaccinated patients.

100
175. A 43-year-old female presents with marked proximal muscle weakness, dysphagia, and pain in
the shoulders and hips, all beginning within the past 5 weeks. She reports difficulty getting out
of a chair. On examination she has a violaceous rash involving the periorbital skin, and macular
erythematous lesions over the anterior chest and upper lateral thighs.

Which one of the following additional findings would you expect?


A) Hyperkeratotic plaques in intertriginous areas
B) Macules over the extensor surfaces of her joints
C) Polygonal papules on the flexor surface of her wrists
D) Distal onycholysis
E) Atrophic cuticles with contracted nail-fold capillaries on dermoscopy

Item 175
Answer : B

This patient’s symptoms and findings suggest dermatomyositis. This disease is distinguished from
autoimmune myopathies and polymyositis by distinct dermatologic findings, including Gottron’s
sign(nonpalpable macules over the extensor surface of joints). Patients may also have dilated nail-fold
capillaries and ragged, thickened cuticles. Distal onycholysis is most commonly associated with
onychomycosis, while hyperkeratotic plaques are not a feature of dermatomyositis. Polygonal papules
on the wrist flexor surfaces are seen in lichen planus.

176. A 60-year-old male presents with a 3-day history of abdominal pain and urinary urgency. He
also reports chills, fatigue, and decreased appetite. His temperature is 38.4°C (101.1°F), his
pulse rate is 96 beats/min, and his blood pressure is 126/84 mm Hg. On examination he is tender
in the left lower quadrant, and a rectal examination reveals a normal-size, nontender prostate
gland. A urinalysis reveals 5–10 WBCs/hpf and 5–10 RBCs/hpf. A CBC reveals a normal
hemoglobin and hematocrit, with a WBC count of 15,000/mm3 (N 4300–10,800) and 85%
neutrophils. A CT scan of the abdomen is shown on the following page.

Which one of the following is the most likely diagnosis?


A) Acute prostatitis
B) Appendicitis
C) Colon cancer
D) Diverticulitis
E) Pyelonephritis

Item 176 Item #176


Answer : D

The clinical picture of diverticulitis can be confused with urinary tract infections because bladder
symptoms can accompany the gastrointestinal symptoms. The sigmoid colon lies close enough to the
bladder that the bladder can be irritated by sigmoid diverticulitis. Conversely, urinary infections do not
cause painful defecation. Tenderness in the left lower quadrant and above the rectum is typical for
sigmoid diverticulitis.

101
No single test is routinely diagnostic for diverticulitis, but a CT scan that shows edema in the sigmoid
mesentery has the best positive predictive value. Appendicitis is usually diagnosed clinically, but like
diverticulitis is increasingly confirmed by CT. The tenderness is generally in the right lower quadrant.
CT findings are noted around the cecum, rather than adjacent to the sigmoid. Colon cancer is usually
asymptomatic when discovered, but when symptoms are present they usually include a change in bowel
habits, hematochezia, anemia, or symptoms of obstruction. Pericolic abscess is a rare presentation of
colon cancer.

177. A 2-year-old male has a 3-day history of a runny nose and cough, and a 2-day history of fever
reaching 40.0°C (104.0°F). He woke up with a rash this morning. His appetite is good and his
activity level is normal. On examination the child is afebrile with normal vital signs, and has a
fine, maculopapular, erythematous rash on the trunk and extremities. The remainder of the
examination is normal.

Which one of the following is the most likely cause of this patient’s rash?
A) Atopic dermatitis
B) Erythema infectiosum
C) Molluscum contagiosum
D) Pityriasis rosea
E) Roseola infantum

Item 177
Answer : E

This patient has the classic presentation for roseola infantum, which is caused by human herpesvirus 6.
The typical history includes a high fever in a child with either mild upper respiratory symptoms or no
other symptoms. After the fever subsides, a rash will appear. The rash is self-limited and no treatment is
required. Pityriasis rosea typically presents with a single herald patch that is oval-shaped and scaly with
central clearing, followed by a symmetric rash on the trunk in a typical distribution along the Langer
lines. The rash may last up to 12 weeks and no treatment is required.
Erythema infectiosum is caused by parvovirus B19 and is also known as fifth disease. The child will
typically have mild symptoms then an erythematous facial rash that has a ―slapped cheek‖ appearance.
This is sometimes followed by pink patches and macules in a reticular pattern. Once the rash appears the
child is no longer contagious.
Molluscum contagiosum is caused by a poxvirus and is characterized by scattered flesh-colored papules
with umbilicated centers. Atopic dermatitis typically presents as scaly, erythematous plaques, commonly
on the flexor surfaces of the extremities.

178. A 47-year-old female is concerned about a change in her menstrual pattern. Her monthly periods
continue, but for the past several months they have been heavier than usual and have been lasting
a few days longer. Last month she also noted some spotting for several days prior to the onset
of her menses. Her pelvic examination is normal.

Which one of the following would be most appropriate at this time?


A) Observation only, and reexamination in 3 months
B) A serum FSH level
C) Transvaginal ultrasonography
D) Progestin-only therapy to normalize bleeding
102
E) Cyclic estrogen-progestin therapy to normalize bleeding

Item 178
Answer : C

Abnormal uterine bleeding can be a sign of endometrial cancer in premenopausal women, who account
for 20% of cases of endometrial cancer. The American College of Obstetricians and Gynecologists
recommends that women with abnormal uterine bleeding should be evaluated for endometrial cancer if
they are older than 45 years or if they have a history of unopposed estrogen exposure (SOR C). Most
guidelines recommend either transvaginal ultrasonography or endometrial biopsy as the initial study in
the evaluation of endometrial cancer. Transvaginal ultrasonography is often preferred as the initial study
because of its availability, cost-effectiveness, and high sensitivity. If bleeding persists despite normal
transvaginal ultrasonography a tissue biopsy should be performed. The listed hormonal treatment
options may be appropriate once cancer is ruled out. An FSH level can help determine whether someone
is menopausal or approaching menopause, in which case they will likely be missing periods. Continued
observation would only delay the diagnosis.

179. A 63-year-old male presents to your office because his COPD is worsening. He indicates that
his exercise tolerance is steadily decreasing but he can still walk approximately 100 yards on flat
ground. His medications include formoterol (Perforomist) twice daily and albuterol (Proventil,
Ventolin) as needed, which he requires only occasionally. He had an exacerbation requiring
prednisone 7 months ago. He quit smoking 6 years ago. His oxygen saturation is 93% on room
air and 89% after walking briskly for 8 minutes. His FEV1 is 1.91 L (62% of predicted) and his
FEV1/FVC ratio is 0.57.

Which one of the following is most likely to improve his progressive dyspnea?
A) Stop formoterol and start fluticasone/salmeterol (Advair)
B) Stop formoterol and start tiotropium
C) Add tiotropium
D) Start oxygen at 2 L/min with exertion
E) Refer for pulmonary rehabilitation

Item 179
Answer :E

Pulmonary rehabilitation has multidimensional benefits in COPD management, including improved


exercise tolerance, quality of life, and mood. Aerobic exercise improves muscle mass, with high-
intensity exercise proving more efficacious than low-intensity exercise. According to Global Initiative
for Obstructive Lung Disease (GOLD) guidelines, an inhaled corticosteroid should be added for frequent
exacerbations and an FEV1 <50%. Although it is possible that a combined long-acting -agonist
(LABA)/corticosteroid would improve dyspnea more than a LABA alone, the benefit would not likely
be more than a pulmonary rehabilitation program. Switching from formoterol to tiotropium would not be
expected to provide a significant benefit, and the benefits of adding a LABA to tiotropium have not been
studied. Third-party payers restrict payment for oxygen therapy to those with an SaO2 <89%, a PaO2
<55 mm Hg, a hematocrit >55%, or documented cor pulmonale.

103
180. A 45-year-old male comes to the urgent care center with left foot pain that began at a ballroom
dancing competition. He states that he put all of his weight on the lateral portion of his foot while on
tiptoe, and felt the pain immediately. He now cannot bear weight on the foot. On examination it appears
slightly edematous with a small amount of ecchymosis. A radiograph reveals a proximal fifth metatarsal
metadiaphyseal fracture. The patient is eager to return to competitive dancing as soon as possible.

Which one of the following would be the best treatment for this patient?
A) Full weight bearing with the use of a compression dressing as needed for pain and swelling
B) A posterior splint with no weight bearing for 4 weeks
C) A walking cast for 6 weeks
D) Surgical referral

Item 180
Answer : D

This patient has a Jones fracture. The treatment plan for this type of fracture needs to account for the
activity level of the patient. It has been shown that active patients have shorter healing times and return
to activity sooner with surgical management. A competitive dancer would be best managed with surgery.
If the nonsurgical option is chosen the patient is given an initial posterior splint and followed up in 3–5
days, then placed in a short non–weight-bearing cast for 6 weeks, at which time a repeat radiograph is
taken. If the radiograph shows healing, the patient can return to gradual weight bearing. If the radiograph
does not show proper healing, then the period of non–weight bearing is extended.

181. A 45-year-old female comes to your outpatient clinic for a routine health maintenance
examination. She has no complaints, is generally healthy, and has an unremarkable family
history. She has been having regular periods and is sexually active with her husband, who had
a vasectomy 9 years ago. As you perform a pelvic examination you notice a fullness in the left adnexal
region that is mildly tender to palpation. The remainder of the examination is normal. You order pelvic
ultrasonography, which demonstrates a 4×5-cm cystic structure on the left ovary. There are no
internal septations or echogenic internal structures. A pregnancy test is negative.

Which one of the following would be most appropriate at this point?

A) Reassurance only
B) Repeat pelvic ultrasonography in 8–10 weeks
C) A serum CA-125 level
D) An oral contraceptive and repeat pelvic ultrasonography in 6–8 months
E) Referral to a gynecologist for diagnostic cystectomy

Item 181
ANSWER: B

Functional ovarian cysts are estimated to be present in approximately 15% of menstruating women and
are often found incidentally on a pelvic examination or on diagnostic imaging performed for other
reasons. The majority of these lesions resolve spontaneously within two or three menstrual cycles and
management should consist of follow-up ultrasonography 8–12 weeks after the cyst is identified. Cysts
that are particularly symptomatic, have less than simple features, are >10 cm in size, or that persist
104
longer than 12 weeks should be referred for consideration of diagnostic removal (cystectomy or
oophorectomy depending on patient characteristics). Patients who carry a BRCA gene or who have a
family history of ovarian cancer should also be considered for invasive testing sooner. Serum CA-125
testing is difficult to interpret in premenopausal patients, who frequently have elevated levels without
evidence of a malignancy. In postmenopausal women with an ovarian cyst a CA-125 level >35 U/mL
should prompt referral for removal.

182. A 4-year-old female is brought in by her mother because of a 3-day history of left ear drainage
and a low-grade fever. The patient has bilateral ventilation tubes that were placed 2 months ago.
Which one of the following would be most effective for resolving this child’s condition?

A) Saline rinses of the affected ear canal


B) Oral amoxicillin/clavulanate (Augmentin)
C) Ciprofloxacin eardrops (Cetraxal)
D) Fluocinonide eardrops

Item 182
ANSWER: C

In patients who present with an ear discharge and have ventilation tubes in place, antibiotic eardrops
(with or without corticosteroids) can resolve the discharge and improve the illness-related quality of life
more quickly than oral antibiotics, corticosteroid eardrops, or saline rinses. The antibiotic eardrops of
choice are fluoroquinolones, which are not ototoxic.

183. Which one of the following is appropriate in the care of adolescent patients?

A) Recommending that parents accompany their children at all times during visits in order
to increase rapport and trust
B) Avoiding discussion of weight with teenagers who are obese, because of the negative
impact on self-image
C) Recommending education about sex and contraception, aimed at reducing the rate of
unintended pregnancy
D) Beginning cervical cancer screening at age 18, or earlier with sexually active patients

Item 183
ANSWER: C

Sex education programs have been associated with a reduction in the rates of sexually transmitted
infections in adolescents. Clinicians should meet with adolescents privately to provide confidentiality.
Adolescents should be screened for obesity and provided appropriate behavioral counseling. Cervical
cancer screening should begin at age 21.

184. Which one of the following populations should be screened for asymptomatic bacteriuria?

A) Patients with diabetes mellitus


B) Patients who are pregnant
105
C) Hospitalized patients who have an indwelling Foley catheter
D) Hospitalized patients at the time of removal of an indwelling Foley catheter
E) Residents of long-term care facilities

Item 184
ANSWER: B

The U.S. Preventive Services Task Force (USPSTF) recommends screening for asymptomatic
bacteriuria with a urine culture for pregnant women at 12–16 weeks gestation or at the first prenatal visit
if it occurs later (A recommendation). The USPSTF recommends against screening for asymptomatic
bacteriuria in men and nonpregnant women (D recommendation). The Infectious Diseases Society of
America (IDSA) guidelines for asymptomatic bacteriuria recommend that pregnant women should be
screened for bacteriuria by urine culture at least once in early pregnancy, and they should be treated if
the results are positive (A-I recommendation). The IDSA also recommends screening for and treatment
of asymptomatic bacteriuria before transurethral resection of the prostate (A-I recommendation) and als
before other urologic procedures for which mucosal bleeding is anticipated (A-III recommendation). The
IDSA’s recommendation for the Choosing Wisely campaign is to not treat asymptomatic bacteriuria
with antibiotics. The only exceptions to this recommendation include pregnant women, patients
undergoing prostate surgery or other invasive urologic surgery, and kidney or kidney pancreas organ
transplant patients within the first year of receiving the transplant. Screening for asymptomatic
bacteriuria is not recommended for long-term care residents or patients with indwelling bladder catheters
without symptoms of a UTI (catheter-associated asymptomatic bacteriuria).

185. A 35-year-old male amateur rugby player presents with right anterolateral hip pain of several
months’ duration that has progressed to the point of interfering with his athletic performance.
The pain is accentuated when he transitions from a seated to a standing position, and especially
when he pivots on the hip while running, but he cannot recall any significant trauma to the area.
Over-the-counter analgesics do not relieve the pain. On examination his gait is stable. The
affected hip appears normal and is neither tender to palpation nor excessively warm to touch.
Although he has a full range of passive motion, obvious discomfort is evident with internal
rotation of the flexed and adducted right hip.
Which one of the following is most strongly suggested by this clinical picture?

A) Osteoarthritis
B) Avascular necrosis
C) Bursitis
D) Impingement
E) Pathologic fracture

Item 185
ANSWER: D

Gradually worsening anterolateral hip joint pain that is sharply accentuated when pivoting laterally on
the affected hip or moving from a seated to a standing position is consistent with femoroacetabular
impingement. The most sensitive finding is reproduction of the pain on a range-of-motion examination
by manipulating the hip into a position of flexion, adduction, and internal rotation (FADIR test). Special
radiographic imaging of the flexed and adducted hip can emphasize the anatomic abnormalities
associated with impingement that may go unnoticed on standard radiographic series views.
106
Although the pain associated with avascular necrosis is similarly insidious and heightened when bearing
weight, tenderness is usually evident with hip motion in any direction. Osteoarthritis of the hip generally
occurs in individuals of more advanced age than this patient, and the pain produced is typically localized
to the groin area and can be elicited by flexion, abduction, and external rotation (FABER test) of the
affected hip. Bursitis manifests as soreness after exercise and tenderness over the affected bursa.

186. A 60-year-old male sees you for a routine health maintenance visit. One of the patient’s friends
recently underwent screening for an abdominal aortic aneurysm (AAA), and he asks if he should
be screened. He is asymptomatic, has never smoked, and has no other risk factors.
Which one of the following does the U.S. Preventive Services Task Force advise with regard
to AAA screening for this patient?

A) There is insufficient evidence to recommend for or against screening


B) He does not require screening now or in the future
C) He should not be screened now, but should have abdominal duplex ultrasonography in
5 years
D) He should have abdominal duplex ultrasonography now, with no future screening
E) He should have abdominal duplex ultrasonography now and in 5 years

Item 186
ANSWER: B

The U.S. Preventive Services Task Force (USPSTF) recommends screening for abdominal aortic
aneurysm in men between the ages of 65 and 75 who have ever smoked (B recommendation). This
should be performed one time, using abdominal duplex ultrasonography. There may be a small benefit
for screening men who have never smoked, and the USPSTF recommends selectively offering screening
to this group (C recommendation). The evidence is insufficient to assess the benefits and harms in
women who have ever smoked (I recommendation), and the USPSTF recommends against screening for
women who have never smoked (D recommendation).

187. A 55-year-old male presents with shortness of breath and abdominal pain. He has a history of
diabetes mellitus, coronary artery disease, and hepatitis C. He usually is very active and
continues to work at a steel plant. He smokes 1 pack of cigarettes each day and drinks alcohol
daily. A chest radiograph shows a moderate right-sided pleural effusion. Thoracentesis reveals
a pleural fluid protein to serum protein ratio of 0.3, and a pleural LDH to serum LDH ratio of
0.4. The pleural glucose level is 140 mg/dL and his serum glucose level is 150 mg/dL.

Which one of the following is the most likely cause of this pleural effusion?

A) Asbestosis
B) Cirrhosis with ascites
C) Malignancy
D) Pneumonia
E) Viral illness

107
Item 187
ANSWER: B

Evaluation of the pleural fluid is important to assist in determining the cause of the effusion. Protein,
glucose, LDH, and cell counts should be measured in the pleural fluid. This can help determine if the
effusion is exudative or transudative. A pleural protein to serum protein ratio >0.5 or a pleural fluid
LDH to serum LDH ratio >0.6 suggests an exudative effusion. Lower ratios suggest a transudative
process. With transudates, the pleural fluid pH is typically between 7.40 and 7.55, with fewer than 1000
WBCs, and the glucose level is similar to the serum glucose level. Cirrhosis with ascites is a cause of
transudative effusion. Pleural effusions associated with malignancy, pneumonia, viral illness, and
asbestosis tend to be exudative.

188. Opioid therapies provide the greatest analgesic relief for most patients with a terminal illness.
However, concerns about which one of the following can inappropriately limit the use of opioids
in these patients?

A) Angina
B) Dementia
C) Gastritis
D) Renal failure
E) Respiratory depression

Item 188
ANSWER: E

Concerns about addiction and respiratory depression often limit the use of opioids or lead to inadequate
dosages in patients with a terminal illness who are experiencing pain at the end of life (SOR C).
Sedation (ranging from full consciousness to complete loss of consciousness) usually precedes
respiratory depression. Opioid use and dosages can therefore be effectively managed with close
monitoring for sedation, allowing patients to receive adequate medication to control pain. Close
monitoring allows clinicians to identify advancing sedation before it is compounded by continued opioid
administration that could lead to clinically significant respiratory depression (SOR C).

189. You see a 37-year-old asymptomatic male for the first time for a health maintenance visit. He
underwent a splenectomy 6 years ago following a motor vehicle accident.
Which one of the following would be appropriate in the care of this patient?

A) Routine health care only, given that he became asplenic as an adult


B) Instructing him to report any fever >101.5°F that lasts more than 72 hours
C) Providing oral amoxicillin or levofloxacin (Levaquin) for initial empiric treatment of a
fever if he is unable to obtain medical care within a few hours
D) Forgoing influenza vaccine if inactivated vaccine is not available
E) Administering 23-valent pneumococcal polysaccharide vaccine (PPSV23, Pneumovax
23) every 5 years indefinitely after the initial vaccination

108
Item 189
ANSWER: C

Amoxicillin, levofloxacin, and moxifloxacin should be taken by asplenic patients with a new onset of
fever if they cannot get to a medical facility within 2 hours for evaluation. Fever should be reported
immediately due to the lifelong significant risk of sepsis. Unless otherwise contraindicated, asplenic
patients should receive annual influenza immunization. Pneumococcal polysaccharide vaccine (PPSV23)
should be given twice, with the second dose given 5 years after the first.

190. A 15-year-old female comes to your office for treatment of acne vulgaris. Her complete history
and physical examination are unremarkable other than a moderate amount of closed comedones
and inflamed papules on her nose, forehead, and upper back. She has not previously tried any
topical or oral therapies, including over-the-counter medications.

Which one of the following would be indicated for this patient as monotherapy?

A) Topical benzoyl peroxide


B) Topical clindamycin (Cleocin T)
C) Oral isotretinoin (Absorica)
D) Oral minocycline (Minocin)
E) Oral spironolactone (Aldactone)

Item 190
ANSWER: A

Oral antibiotics are recommended for acne that is resistant to topical treatments. Oral isotretinoin is
indicated for severe nodular acne or moderate acne resistant to other treatments. Since this patient has
not tried any therapies, these two options would not yet be appropriate. Topical antibiotics are
recommended only in combination with benzoyl peroxide. Appropriate treatments would be topical
benzoyl peroxide, a topical retinoid, and oral contraceptives. Antiandrogen therapies such as
spironolactone are not indicated solely for acne vulgaris, although they may be appropriate for
concomitant conditions such as polycystic ovary syndrome.

191. A 38-year-old female who recently underwent a laparoscopic sleeve gastrectomy for weight loss
presents to your office for a follow-up visit. She has had no complications with her recent
postoperative course. Her medical history includes diabetes mellitus, hypertension, and
hyperlipidemia. She has also had problems with bilateral knee pain from osteoarthritis.
Patient education should include advising the patient to

A) drink extra fluids with meals


B) increase her intake of fibrous vegetables
C) avoid pregnancy for 3 years
D) take ibuprofen as needed for pain
E) have a bone density test in 2 years

Item 191
ANSWER: E

109
Treatment of adult obesity with bariatric surgery is becoming more common. In addition to counseling
patients about surgical options and the risks and benefits of surgery, the family physician is in a position
to provide both long-term support and postsurgical medical management. Bariatric surgery does result in
greater weight loss than nonsurgical interventions and is highly effective in treating comorbidities of
obesity, particularly diabetes mellitus. Bariatric surgery also reduces obesity-related mortality.
After bariatric surgery the patient’s postoperative medications may require adjustments and NSAIDs
should be avoided. Patients should be encouraged to eat three meals and one or two snacks daily. Very
dry foods, bread, and fibrous vegetables are most likely to cause problems. Fluids should be avoided
during meals and for 15–30 minutes before and after meals.
Those desiring pregnancy should wait 12–18 months after surgery. Recommended laboratory studies
include a CBC, a metabolic profile, a folic acid level, iron studies, a parathyroid hormone level, a lipid
profile, vitamin B12 levels, 24-hour urinary calcium excretion, and 25-hydroxyvitamin D levels. It is
recommended that bone density measurements be done every 2 years.

192. Which one of the following is a cause of pseudohyponatremia?

A) Hypercalcemia
B) Hyperkalemia
C) Hypermagnesemia
D) Hyperphosphatemia
E) Hypertriglyceridemia

Item 192
ANSWER: E

Plasma sodium concentration measurements can be unreliable in patients with severe hyperlipidemia or
hyperproteinemia (pseudohyponatremia). The other electrolyte abnormalities do not cause
hyponatremia.

193. A 13-year-old female with asthma sees you for the first time in a year. The patient has a cough
and you discover that she has not been taking her medications since you last saw her. She has
been having symptoms 3–4 days a week and has been awakening at night about 3 times a month.
Her FEV1 in the office is 90% of predicted.

This patient’s asthma should be classified as

A) intermittent
B) mild persistent
C) moderate persistent
D) severe persistent

Item 193
ANSWER: B

110
week,
FEV1 is >80% of predicted. Intermittent asthma does not interfere with normal activities and inhaled
short-acting 2-agonists during symptomatic periods are usually sufficient treatment.
Mild persistent asthma is defined as symptoms present >2 days per week but not daily, nighttime
awakenings 3–4 times per month, and inhaler use >2 days per week but not daily and not more than once
on any day. The FEV1 is >80% of predicted. Mild persistent asthma can cause minor limitations during
normal activities and should be treated with low-dose inhaled corticosteroids (ICs).
With moderate persistent asthma, symptoms are present daily, nighttime awakenings occur >1 time per
week but not nightly, and an inhaler is required daily. A patient with moderate persistent asthma has an
FEV1 that is 60%–80% of predicted and can experience some limitations during normal activities.
Moderate persistent asthma is treated with a combination of low-dose ICs and long-acting 2-agonists
(LABA) or medium-dose ICs as monotherapy.
Severe asthma is defined as symptoms present throughout the day, nighttime awakenings up to 7 times
per week, inhaler use several times per day, and an FEV1 <60% of predicted. Normal activities are
extremely limited by severe asthma. Treatment includes medium- to high-dose ICs with a LABA, and
consultation with an asthma specialist is recommended. Omalizumab is also indicated for patients who
have allergies.

194. A 37-year-old female who smokes 1 pack of cigarettes per day has just had her third child and
requests contraception. She does not intend to have any more children.
Which one of the following is the safest option for this patient?

A) Traditional combined oral contraceptive pills


B) Extended-cycle combined oral contraceptive pills
C) The contraceptive patch (Ortho Evra)
D) The contraceptive vaginal ring (NuvaRing)
E) The etonogestrel implant (Nexplanon)

Item 194
ANSWER: E

Cigarette smoking, increasing age, and exogenous estrogen, particularly at the supraphysiologic doses
used in contraceptives, all increase risk for vascular events such as venous thromboembolism and stroke.
The use of estrogen- cated because of this
risk. All of the contraceptive options listed contain estrogen except for the etonogestrel implant. Other
progestin-only contraceptive options that could be considered include depot medroxyprogesterone
acetate, the levonorgestrel-releasing IUD, and progestin-only oral contraceptive pills.

195. A 42-year-old male comes to your office with acute right flank pain that awakened him from
sleep. The pain is colicky and he says it is the most intense pain that he has ever felt. The
findings on a physical examination, in addition to blood on his urinalysis, make you suspect a
urinary tract stone.
Which one of the following imaging modalities would be most appropriate for confirming your
suspicion?
A) Abdominal radiography (KUB)
B) Standard CT of the abdomen and pelvis with intravenous contrast
C) Low-dose helical (spiral) noncontrast CT of the abdomen and pelvis
111
D) MRI of the abdomen and pelvis without contrast
E) Ultrasonography of the kidneys and bladder

Item 195
ANSWER: C

Because many radiologic options are available for detecting suspected urolithiasis, and many contexts
might affect their use, the American College of Radiology established appropriateness criteria to aid the
selection process. These guidelines rate the appropriateness of CT, ultrasonography, radiography, and
MRI for three categories of patients: (1) those presenting with a suspected stone, (2) those with recurrent
stone symptoms, and (3) those with abdominal pain in pregnancy. Each modality is rated on a scale from
9 (most appropriate) to 1 (least appropriate). For suspected stone disease in the case presented, the most
appropriate imaging modality is helical (spiral) noncontrast CT of the abdomen and pelvis with a rating
of 8 (usually appropriate). Standard CT of the abdomen and pelvis with intravenous contrast has an
appropriateness rating of 2 (usually not appropriate). Abdominal radiography (KUB) has an
appropriateness rating of 3 (usually not appropriate). MRI of the abdomen and pelvis without contrast
has a rating of 4 (may be appropriate). Ultrasonography of the kidneys and bladder is appropriate in
pregnancy, but in the case presented it is given an appropriateness rating of 6 (may be appropriate).

196. A 45-year-old premenopausal female with a BMI of 34.0 kg/m2 presents to your office with
increasing dyspnea on exertion. She has a recent history of iron deficiency anemia and wonders
if her iron level is low again. Her history is negative for heavy menses, bloody or melanotic
stools, abdominal pain, and unusual bleeding or bruising. Her evaluation for anemia has included
upper and lower endoscopy with normal findings. Last year the patient underwent gastric bypass
surgery and has been on a fairly restricted diet since that time. She uses a levonorgestrel IUD
(Mirena) for contraception.
A physical examination is unremarkable. Her serum ferritin level is 6 ng/mL (N 10–120) and
her hemoglobin level is 8.0 g/dL (N 12.0–16.0) with microcytic indices. You determine that she
has iron deficiency anemia.
Which one of the following would be the most appropriate management?

A) Oral iron replacement


B) Intravenous iron replacement
C) Blood transfusion
D) Urgent referral to a hematologist

Item 196
ANSWER: B

This patient has symptomatic iron deficiency anemia. Because she has had a gastric bypass, she is not
able to absorb iron adequately and therefore needs intravenous iron replacement. A blood transfusion
may help temporarily but will not restore her iron stores, so it is not indicated. She will not absorb oral
iron adequately. If she does not respond to intravenous iron, consultation with a hematologist may be
indicated.
197. A patient returns to your office for a refill of oxycodone (Roxicodone), which he has been taking
for 6 months for pain secondary to chronic osteomyelitis of his knee. His pain relief is adequate
at a dosage of 10 mg every 6–8 hours. He took his prescribed dose approximately 4 hours prior
112
to his visit. A urine drug screen using an enzyme-linked test is negative.
Which one of the following would be most appropriate at this point?

A) Stop prescribing pain medication for this patient


B) Order chromatography
C) Switch the patient to a codeine product and retest him
D) Refer the patient to a pain management specialist.

Item 197
ANSWER: B
Oxycodone often is not detected by an immunoassay test and unexpected results require follow-up with
a more accurate test such as gas chromatography/mass spectrometry or high-performance liquid
chromatography. Codeine can be detected more accurately, but substituting codeine for oxycodone
would be inappropriate. Pain management is reasonable if the patient is requiring large amounts of
opiates, has failed treatment, or has a history of drug abuse. A pain management agreement should be
initiated at the beginning of treatment with an opiate.

198. A 75-year-old male presents with weight loss, fatigue, and sleep disturbance. He also has a
6-week history of feeling depressed and not enjoying life anymore. He is not suicidal. After an
appropriate evaluation to rule out other causes, you diagnose major depressive disorder.
Which one of the following agents is indicated as first-line pharmacotherapy for this patient?

A) Aripiprazole (Abilify)
B) Dextroamphetamine/amphetamine (Adderall)
C) Nortriptyline (Pamelor)
D) Sertraline (Zoloft)
E) Venlafaxine (Effexor XR)

Item 198
ANSWER: D

Because of their favorable side-effect profile and low cost, SSRIs are the first-line agents for late-life
depression. SNRIs may be used as second-line agents when remission is not obtained with an SSRI.
Tricyclic antidepressants work as well as SSRIs and may be considered in recalcitrant cases, but side
effects may be troublesome, especially in this age group. The use of stimulants in depressed older adults
has not been well studied. Second-generation antipsychotic agents may be used as an add-on to an SSRI
or SNRI medication when the depression is resistant. Side effects and long-term safety may be issues
with this approach.

199. A 24-year-old female who has been treated for panic attacks presents for follow-up and is noted
to have a heart rate of 170 beats/min. Her blood pressure is 125/82 mm Hg and the remainder
of her vital signs are normal. She states that she feels anxious at this time but does not have
dizziness, chest pain, or other symptoms. She says this is similar to how she felt during panic
attacks in the past. An EKG is shown on the following page.
Which one of the following would be the best initial management?

A) Valsalva maneuvers
B) A short-acting benzodiazepine to use as needed for panic attacks
113
C) Intravenous adenosine (Adenocard)
D) Synchronized cardioversion
E) Urgent referral to a cardiologist

Item #199

Item 199
ANSWER: A

Supraventricular tachycardia (SVT) can be mistaken for panic disorder, particularly in women.
Atrioventricular nodal reentrant tachycardia is more common in women than men. For hemodynamically
stable SVT, Valsalva maneuvers are indicated as the first treatment because the maneuver is simple and
may resolve the episode. If this is ineffective, medications can be used. For hemodynamically unstable
SVT, synchronized cardioversion should be performed. In this case the patient is hemodynamically
stable and minimally symptomatic, so a Valsalva maneuver should be attempted. If this is unsuccessful
she may need medication.
Cardiology referral should be considered for SVT when the diagnosis is unclear, underlying cardiac
disease is suspected, the patient has syncope or works in or participates in high-risk activities, SVT is
not controlled with medications, a wide QRS complex is noted, or the patient desires ablation. Anxiety
treatment may be needed in this patient, but that should be evaluated further. At this time her most
urgent issue is her supraventricular tachycardia.

200. A 55-year-old male sees you for follow-up. His medical problems include morbid obesity, type
2 diabetes mellitus, hypertension, hyperlipidemia, and major depressive disorder. His
medications include metformin (Glucophage), glipizide (Glucotrol XL), lisinopril (Prinivil,
Zestril), aspirin, simvastatin (Zocor), and fluoxetine (Prozac). His BMI is 52.4 kg/m2 and he
is struggling to lose weight.
Which one of the following medication replacements could help promote weight loss?

A) Atorvastatin (Lipitor) instead of simvastatin


B) Canagliflozin (Invokana) instead of glipizide
C) Carvedilol (Coreg) instead of lisinopril
D) Paroxetine (Paxil) instead of fluoxetine
E) Pioglitazone (Actos) instead of glipizide

Item 200
ANSWER: B

114
Given the obesity epidemic in the United States, an awareness of therapies that affect weight is
imperative for family physicians. This patient is taking medications that help with weight loss
(metformin) and medications that are weight neutral (lisinopril, simvastatin, and fluoxetine). Glipizide,
however, causes weight gain, and switching to an SLGT2 inhibitor such as canagliflozin can help
promote weight loss.
Likewise, the patient could use a GLP-1 receptor agonist such as exenatide or an amylin mimetic
(pramlintide) for weight loss benefits. Sulfonylureas, thiazolidinediones, and insulins all promote weight
gain (SOR A). Fluoxetine and sertraline are weight neutral, whereas paroxetine can cause weight gain
(SOR B). The statins are weight neutral in general, and switching to atorvastatin should not affect
weight. ACE inhibitors, angiotensin receptor blockers, calcium channel blockers, thiazides, and -
adrenergic blockers are all weight neutral. In this patient with diabetes mellitus, an ACE inhibitor would
be preferable to carvedilol in terms of renal protection (SOR A).

201. A 45-year-old white female presents to your office after recent routine laboratory tests revealed a
platelet count of 100,000/mm3 (N 150,000–350,000). Six months ago the patient’s platelet count was
283,000/mm3. All other indices are within normal limits. The history is negative for easy bruising,
bleeding, fever, rash, or arthralgias. She has no family history of blood disorders or recent illness. Her
medications include ranitidine (Zantac), fluticasone (Flonase), and inhaled albuterol (Proventil,
Ventolin). She is otherwise healthy. A peripheral smear confirms thrombocytopenia.

Which one of the following would be most appropriate at this point?

A) Stopping ranitidine and repeating the CBC in 2 weeks


B) Oral prednisone, 50 mg daily for 7 days, and a repeat CBC in 1 week
C) Outpatient transfusion of 1 unit of platelets
D) Referral to a hematologist for further evaluation
E) Referral for a bone marrow biopsy

Item 201
ANSWER: A

Thrombocytopenia is a relatively common dyscrasia often discovered through routine laboratory studies.
There are many causes of thrombocytopenia but medication-induced thrombocytopenia should always
be considered. In this case the patient is taking an H2-blocker that may cause blood dyscrasias. The
offending agent should be stopped and a repeat level should be obtained in 2–4 weeks for patients with
mild asymptomatic thrombocytopenia (platelet count 100,000–150,000/mm3) and in 1–2 weeks for
moderate thrombocytopenia (platelet count 50,000–100,000/mm3) (SOR C). Prednisone is the first-line
treatment for immune thrombocytopenic purpura (SOR C); however in this situation, it is reasonable to
look for other causes first. If the thrombocytopenia is worse on repeat testing (<100,000/mm3) it is
reasonable to consider consultation for further evaluation and to consider a bone marrow biopsy. If the
platelet count improves, continued monitoring is indicated until it returns to normal. Platelet transfusions
are not indicated in stable, nonbleeding patients unless the platelet count drops below 10,000/mm3.

115
202. An 82-year-old white male consults you following several syncopal episodes that are clearly
orthostatic in nature. During the course of your evaluation you find that he has a large tongue, mild
cardiomegaly, and findings that suggest bilateral carpal tunnel syndrome.

The most likely diagnosis at this time is

A) pernicious anemia
B) cervical spondylosis
C) amyloidosis
D) polymyalgia rheumatica

Item 202
ANSWER: C

Amyloidosis is defined as the extracellular deposition of the fibrous protein amyloid at one or more
sites. It may remain undiagnosed for years. Features that should alert the clinician to the diagnosis of
primary amyloidosis include unexplained proteinuria, peripheral neuropathy, enlargement of the tongue,
cardiomegaly, intestinal malabsorption, bilateral carpal tunnel syndrome, and orthostatic hypotension.
Amyloidosis occurs both as a primary idiopathic disorder and in association with other diseases such as
multiple myeloma.

203. A 60-year-old male returns for a reevaluation of his asthma that you have been working to
control. In the past he has been adequately maintained with daily use of inhaled fluticasone/salmeterol
(Advair), along with montelukast (Singulair). Last month he experienced an exacerbation of his asthma
that responded well to oral prednisone. However, each time you have attempted to wean him off the
prednisone his asthma symptoms have returned.

Which one of the following would be most appropriate at this time?

a. Adding oral azithromycin (Zithromax)


b. Adding oral methotrexate
c. Adding oral theophylline
d. Adding long-term oral prednisone
e. Increasing the dosage of the corticosteroid

Item 203
ANSWER: E
This patient has severe asthma that is not responding to a moderate dose of an inhaled corticosteroid, a
leukotriene inhibitor, and a long-acting -agonist. The next appropriate step is to add a stronger dose of
inhaled corticosteroid. Methotrexate and azithromycin are considered inappropriate therapies.
Theophylline and low-dose oral prednisone are considered appropriate steps if the patient does not
respond to high doses of an inhaled corticosteroid. Other reasonable options for the treatment of severe
asthma would be a muscarinic antagonist such as tiotropium, or assessing for the presence of IgE-
dependent allergic asthma that may respond to omalizumab.

116
204. A 41-year-old healthy female presents for a health maintenance examination. Her last preventive
care visit was 3 years ago, when she had negative cervical cytology and HPV co-testing and a normal
lipid panel. She does not use tobacco, alcohol, or illicit drugs. She has not been sexually active since her
last visit, and her menstrual periods are regular. A complete physical examination is normal, including
blood pressure.

Which one of the following is recommended by the U.S. Preventive Services Task Force for this patient
at this time?

a. Daily folic acid supplementation


b. A manual breast examination
c. Chlamydia screening
d. A fasting lipid profile
e. A Papanicolaou test

Item 204
ANSWER: A

The U.S. Preventive Services Task Force (USPSTF) recommends that all women planning or capable of
pregnancy take a daily folic acid supplement containing 0.4–0.8 mg (400–800 g) (A recommendation).
A manual breast examination by the clinician or patient is not recommended by the USPSTF. Cervical
cytology (a Papanicolaou test) is recommended every 5 years if results are normal and it is combined
with negative high-risk HPV testing (A recommendation). Chlamydia screening is recommended yearly
for all sexually active females under age 25 and older individuals at higher risk. For women who are
over 25 and not at increased risk the USPSTF makes no recommendation for or against screening (C
recommendation). This patient was screened for lipid disorders 3 years ago. While the best screening
interval for lipids is not clearly defined, there is no clear recommendation to repeat screening lipids at
this time. The USPSTF makes no recommendation for or against screening for lipid disorders in women
who are not at increased risk for coronary heart disease (C recommendation).

205. An 82-year-old male presents to the emergency department with severe generalized abdominal
pain. He has a history of paroxysmal atrial fibrillation and stopped taking rivaroxaban (Xarelto) 2
months ago because of the cost. The physical examination reveals generalized abdominal tenderness and
an epigastric bruit. You suspect mesenteric ischemia.

Which one of the following is the recommended imaging study?

a. Ultrasonography
b. Endoscopy
c. Catheter angiography
d. CT angiography
e. MR angiography

117
Item 205
ANSWER: D

CT angiography (CTA) is the recommended imaging modality for the diagnosis of visceral ischemic
syndromes because of its 95%–100% accuracy. Images of the origins and length of the vessels can be
obtained rapidly, characterize the extent of stenosis or occlusion and the relationship to branch vessels,
and aid in the assessment of options for revascularization. Endoscopy is most useful for diagnosing
conditions other than mesenteric ischemia. The value of ultrasonography is extremely dependent on the
skills of the technologist. In addition, adequate imaging can be difficult to obtain in patients with
obesity, bowel gas, and heavy calcification in the vessels. Angiography with selective catheterization of
mesenteric vessels is now used after a plan for revascularization has been chosen. MR angiography takes
longer to perform than CTA, lacks the necessary resolution, and can overestimate the degree of stenosis.

206. A 21-year-old male presents to an acute care center with pain in his left shoulder after a bicycle
accident. His left arm is externally rotated and slightly abducted. A neurovascular examination is
normal. Plain radiographs show an anterior shoulder dislocation. Reduction is successful, which is
confirmed by a plain radiograph.

Which one of the following complications would be reduced by gentle range-of-motion exercises during
immobilization?

a. Acromioclavicular joint injury


b. Adhesive capsulitis (frozen shoulder)
c. Recurrent dislocation
d. Rotator cuff injury
e. Shoulder impingement syndrome

Item 206
ANSWER: B

After a shoulder dislocation, normal activity can resume when motion and strength in both arms is equal.
Immobilization of the shoulder after a dislocation is recommended for at least 1 week. Recurrent
shoulder dislocations are more common in younger patients and should be immobilized for 3 weeks in
patients under 30 years of age. In patients over 30 years of age, 1 week of immobilization will limit the
amount of joint stiffness. Prolonged immobilization is a risk factor for developing adhesive capsulitis
(frozen shoulder). Gentle range-of-motion exercises should be performed during the immobilization
period to limit the risk of adhesive capsulitis. Recurrent dislocations, rotator cuff injuries, shoulder
impingement syndrome, and acromioclavicular joint injuries are not reduced by gentle range-of-motion
exercises.

207. A 14-year-old female with mild depression sees you for follow-up. After her last visit she began a
trial of psychotherapy but her mother asks about additional treatment options. The daughter has no other
significant past medical history.

Which one of the following is recommended as first-line pharmacotherapy for this patient?

a. Citalopram (Celexa)
b. Fluoxetine (Prozac)
118
c. Mirtazapine (Remeron)
d. Paroxetine (Paxil)
e. Venlafaxine (Effexor XR)

Item 207
ANSWER: B

Expert panel guideline recommendations suggest fluoxetine should be the first-line pharmacotherapy
option for adolescents after a trial of psychotherapy. The patient should be monitored weekly for side
effects for a month after starting fluoxetine. If fluoxetine is ineffective, sertraline and citalopram are
recommended as alternatives. Venlafaxine should be avoided in adolescents because it is associated with
a statistically increased risk of suicidal behavior or ideation.

208. A 38-year-old previously healthy Ethiopian female presents with a 4-day history of sharp chest
pain that improves when she sits up. She entered the United States 1 week ago to begin a postdoctoral
research position. She had a recent negative HIV-1/HIV-2 test. She has no history of allergies and is not
pregnant. Her only medication is ibuprofen, 400 mg every 8 hours as needed for menstrual cramps. The
remainder of her history and physical examination are notable only for a pericardial friction rub. After
further evaluation she is presumptively diagnosed with pericarditis.

Which one of the following is the most likely cause of this patient’s symptoms?

a. Ibuprofen hypersensitivity
b. Candida infection
c. Enterovirus infection
d. Myocardial infarction
e. Metastatic cancer

Item 208
ANSWER: C

Most pericarditis is presumed to be viral in origin. Enteroviruses, herpesviruses, adenovirus, and


parvovirus B19 are common agents. Tuberculosis infection is also possible in this patient, considering
the high prevalence of tuberculosis in sub-Saharan Africa. Candida infection is much less common,
especially in an HIV-negative patient. Post–myocardial infarction syndrome, secondary metastatic
tumor, and drug reaction are infrequent causes, especially in a previously healthy patient. Ibuprofen may
be used to treat pericarditis.

209. A 75-year-old male is noted to have palpitations. He has COPD from smoking for many years and
uses an albuterol (Proventil, Ventolin) inhaler and inhaled corticosteroids. On examination his blood
pressure is 130/70 mm Hg, his pulse rate is 110 beats/min, and his rhythm is irregularly irregular.
Auscultation of the lungs reveals a few scattered wheezes and rhonchi. An EKG shows irregular R-R
intervals with narrow QRS complexes and no P waves.

Which one of the following would be the best choice to control this patient’s heart rate?

119
A) Cardioversion
B) Amiodarone (Cordarone)
C) Digoxin (Lanoxin)
D) Diltiazem (Cardizem)
E) Propranolol

Item 209
ANSWER: D

Atrial fibrillation is the most common cardiac arrhythmia and can be a source of morbidity and
mortality. If this is suspected, a 12-lead EKG should be obtained to confirm the diagnosis. Patients need
evaluation for possible cardioversion, rate versus rhythm control, and anticoagulation. The first-line
agent to achieve a target heart rate of <80 beats/min would be either a -blocker or a nondihydropyridine
calcium channel blocker. This patient has significant COPD, which eliminates the use of a nonselective
-blocker such as propranolol. A nondihydropyridine calcium channel blocker such as verapamil or
diltiazem would be a better choice. Adding digoxin could be considered if the initial therapy is
unsuccessful in controlling the heart rate. Amiodarone has significant toxicity and is usually not
recommended unless the first-line options fail.

210. A 36-year-old female sees you for a 6-week postpartum visit. Her pregnancy was complicated by
gestational diabetes mellitus. Her BMI at this visit is 33.0 kg/m2 and she has a family history of
diabetes. Which one of the following is this patient’s greatest risk factor for developing type 2 diabetes
in the future?

A) Her age
B) Obesity
C) The history of a completed pregnancy
D) The history of gestational diabetes
E) The family history of diabetes

Item 210
ANSWER: D

A history of gestational diabetes mellitus (GDM) is the greatest risk factor for future development of
diabetes mellitus. It is thought that GDM unmasks an underlying propensity to diabetes. While a healthy
pregnancy is a diabetogenic state, it is not thought to lead to future diabetes. This patient’s age is not a
risk factor. Obesity and family history are risk factors for the development of diabetes, but having GDM
leads to a fourfold greater risk of developing diabetes, independent of other risk factors (SOR C). It is
thought that 5%–10% of women who have GDM will be diagnosed with type 2 diabetes within 6 months
of delivery. About 50% of women with a history of GDM will develop type 2 diabetes within 10 years
of the affected pregnancy.

120
211. The fee-for-service system of medical payments establishes a reimbursement for individual care
episodes provided by physicians and hospitals. Which one of the following is the term for when this
reimbursement is adjusted up or down based on quality and/or efficiency measures?

A) Modified fee-for-service
B) Bundled payments
C) Flexible payment strategies
D) Value-based reimbursement
E) Accountable Care Organizations

Item 211
ANSWER: D

Value is often defined as quality divided by cost. As such, the value that a health care system provides
increases as the quality of that care increases, the costs of the care decrease, or both. Value-based
reimbursement is a system of health care reimbursement that seeks to motivate health care systems
and/or providers to increase the value of their services rather than just seeking to increase the quantity of
their services. An example of this type of payment incentive is the Center for Medicare and Medicaid
Services’s move to establish a program called value-based purchasing. This program increases
reimbursement rates for high-value hospitals and decreases reimbursement rates for hospitals that
provide lower-value services.

212. A 70-year-old male presents with lower extremity pain. Increased pain with which one of the
following would be most consistent with lumbar spinal stenosis?

A) Lumbar spine extension


B) Lumbar spine flexion
C) Internal hip rotation
D) Pressure against the lateral hip and trochanter

Item 212
ANSWER: A

Spinal extension that increases lumbar lordosis decreases the cross-sectional area of the spinal canal,
thereby compressing the spinal cord further. Walking downhill can cause this. Spinal flexion that
decreases lordosis has the opposite effect and will usually improve the pain, as will sitting.
Pain with internal hip rotation is characteristic of hip arthritis and is often felt in the groin. Pain in the
lateral hip is more typical of trochanteric bursitis.

213. A 60-year-old female sees you because she has recently lost 20 lb without trying and is having
trouble swallowing. Her other medical problems include obesity, tobacco abuse, and GERD.

Which one of the following is the most appropriate diagnostic test in this situation?

a. A barium swallow
b. Endoscopic esophageal ultrasonography
c. Chest CT
d. Upper endoscopy
121
Item 213
ANSWER: D

Dysphagia alone or with unintentional weight loss is the most common presenting symptom of
esophageal cancer. Adenocarcinoma is the most common esophageal cancer in developed nations, and
risk factors include GERD, obesity, and tobacco abuse. Upper endoscopy is the recommended diagnostic
tool (SOR B). If cancer is confirmed, CT and PET scanning are useful for staging.

214. A 60-year-old male presents for evaluation of mild pitting edema of both lower extremities for
several months. He has daytime fatigue and drowsiness but no orthopnea or paroxysmal nocturnal
dyspnea. His only medications are hydrochlorothiazide and lisinopril (Prinivil, Zestril).

A physical examination reveals normal vital signs but a BMI of 30.3 kg/m2. His lungs are clear and his
heart sounds are normal. There is no organomegaly. The patient has mild pitting edema of both lower
extremities up to the midcalf but no associated skin changes, ulcerations, or decrease in pulses. A CBC,
a comprehensive metabolic panel, a prealbumin level, and a chest radiograph are all normal.

In addition to a sleep study, which one of the following would be the most appropriate next step in the
evaluation of this patient?

a. An ankle-brachial index
b. A D-dimer assay
c. Lymphoscintigraphy
d. Echocardiography
e. Magnetic resonance venography

Item 214
ANSWER: D

When evaluating bilateral lower extremity edema, one should first look for systemic etiologies that
would result in edema, such as hepatic, renal, or cardiac failure. In patients with obesity or a history of
loud snoring, daytime drowsiness, or unrestful sleep, obstructive sleep apnea is likely. These patients can
be diagnosed through polysomnography. Echocardiography is also recommended to detect pulmonary
hypertension.
Chronic venous insufficiency would be associated with skin changes such as hemosiderin deposits or
venous ulcerations. If these findings are present, duplex ultrasonography should be ordered. If there is
suspected arterial insufficiency an ankle-brachial index can be determined. For those with a low
likelihood of deep vein thrombosis (DVT), a D-dimer assay can be ordered, but duplex ultrasonography
is a more definitive test. For those with negative ultrasonography, magnetic resonance venography may
be needed to rule out a pelvic or thigh DVT. Patients with suspected lymphedema can usually be
diagnosed clinically, although lymphoscintigraphy may be required.

122
215. A 22-year-old female who was diagnosed with bronchitis at an urgent care clinic 3 days ago sees
you because her cough is still present. She is very annoyed by the cough and is concerned because she
read online that she could have pneumonia. She asks if she should have a chest radiograph.

Which one of the following would be an indication for a chest radiograph in this patient?

a. A cough lasting more than 14 days


b. A respiratory rate >24/min
c. A temperature >37.5°C (99.5°F)
d. Wheezing on the lung examination
e. Cigarette smoking

Item 215
ANSWER: B

Adult patients with acute bronchitis rarely require a chest radiograph to rule out pneumonia. Indications
for a chest radiograph include dyspnea, tachypnea, tachycardia, temperature >100.0°F, bloody sputum,
or signs of focal consolidation on lung auscultation. In patients with bronchitis the cough lasts an
average of 18 days, so a chest radiograph would not be indicated after only 14 days. Smoking does not
influence the need for a chest radiograph, and wheezing is common in uncomplicated acute bronchitis.

216. A 66-year-old asymptomatic male presents with a 6-month history of ongoing pruritus. A physical
examination is normal. Laboratory studies are also normal except for an LDL-cholesterol level of 150
mg/dL, a free T4 level of 1.4 ng/dL (N 0.9–2.3), and a TSH level of 6.22 U/mL (N 0.4–4.5).

Which one of the following would be most appropriate at this point?


a. No treatment at this time and a repeat TSH level in 3 months
b. Testing for thyroid antibodies and treatment with levothyroxine if the test is negative
c. A 131I uptake and scan with no treatment pending results
d. Liothyronine (Cytomel)
e. Levothyroxine (Synthroid)

Item 216
ANSWER: A

This patient has subclinical hypothyroidism and should have a TSH level repeated in 1–3 months, as
TSH may fluctuate in patients without thyroid disease and return to normal on subsequent testing. In a
patient with a normal free T4 the TSH level must be >10 U/mL for a diagnosis of hypothyroidism. Mild
TSH elevations may be a normal manifestation of aging.

217. An otherwise healthy 64-year-old male presents with mild shortness of breath and lightheadedness
with exertion. He has a grade 3/6 systolic ejection murmur at the right second intercostal space radiating
to the neck. A transthoracic echocardiogram and cardiac catheterization with normal coronary arteries
reveals severe aortic stenosis.

Which one of the following would be the most appropriate management of this patient?

123
a. No treatment, and monitoring for increasing symptoms every 6 months
b. No treatment, and monitoring with echocardiography every 6 months
c. Medical management with an ACE inhibitor and continued monitoring
d. Medical management with a -blocker and continued monitoring
e. Prompt aortic valve replacement

Item 217
ANSWER: E

This patient should have his aortic valve replaced. He meets criteria for severe aortic stenosis with a
transthoracic velocity 4.0 m/sec and an aortic valve area <1.0 cm2. Symptomatic patients with severe
aortic stenosis have 2-year mortality rates of more than 50%. After valve replacement the 10-year
survival rate is almost identical to that of patients without aortic stenosis. Watchful waiting with
monitoring for symptoms and periodic echocardiograms is indicated for asymptomatic patients with
moderate to severe aortic stenosis who have a normal ejection fraction. There is no medical treatment
that delays the progression of aortic valve disease or improves survival. Measures to reduce
cardiovascular risk, including treatment of hypertension, are indicated. Rate-slowing calcium channel
blockers and -blockers that depress left ventricular function should be avoided if possible. ACE
inhibitors may improve symptoms in patients with aortic stenosis who are not surgical candidates.

218. A 38-year-old female with diabetes mellitus controlled by diet has a sodium level of 130 mEq/L
(N 136–145) on a routine basic metabolic panel. She does not use any dietary supplements and is not
taking a diuretic. You consider a diagnosis of syndrome of inappropriate secretion of antidiuretic
hormone (SIADH) and take additional history and order additional laboratory studies.

Which one of the following would be most consistent with SIADH?

a. A history of excessive beer drinking


b. A history of polydipsia
c. A serum glucose level of 350 mg/dL (N 70–100)
d. A urine sodium level of 40 mEq/L (N 20)
e. A urine osmolality of 90 mOsm/kg (N 300–900)

Item 218
ANSWER: D

A diagnosis of the syndrome of inappropriate secretion of antidiuretic hormone (SIADH) generally starts
with the discovery of hyponatremia and is confirmed after all other possible causes are excluded. The
root cause is the pathologic secretion of antidiuretic hormone (ADH), which can occur in response to
some drugs and a variety of conditions, including infections, tumors, and dysregulation in the nervous
system involving sympathetic tone and baroreflex response. Inappropriate release of ADH increases free
water reabsorption, which increases circulating blood volume, dilutes sodium, and lowers hematocrit
and hemoglobin. Urine output is often lowered because of this reabsorption, and the urine is more
concentrated (urine osmolality > plasma osmolality) with sodium levels >20 mEq/L. Modest weight gain
may be noted as a result of the increased blood volume.

124
Polydipsia also causes dilution of serum sodium and hemodilution but results in dilution of urine.
Excessive beer drinking may result in hyponatremia and hypokalemia (beer potomania) as a result of
overhydration with a fluid containing inadequate solute. Laboratory testing for sodium can be falsely
reduced in a hyperglycemic state. A simple calculation can correct for this: Corrected (Na+) = Measured
(Na+) + (2.4 × glucose (mg/dL) – 100 mg/dL)/100 mg/dL.

219. You see a 4-month-old male in your office with a 2-day history of cough, runny nose, fever, poor
feeding, and difficulty breathing. He was born at 38 weeks gestation via a normal spontaneous vaginal
delivery after an uncomplicated pregnancy. He did well after birth and went home with his mother after
a 48-hour hospital stay. He is breastfed and had been doing well until now. He has breastfed much less
than usual today and has had no wet diapers in the last 8 hours.

On examination you note a temperature of 38.9°C (102.0°F), a pulse rate of 176 beats/min, a respiratory
rate of 66/min, and an oxygen saturation of 92% on room air. The patient generally appears tachypneic
and clingy, is fussy during the examination, and has notable subcostal retractions and nasal flaring. A
nasal examination reveals crusted mucus at the nares bilaterally. Examination of the mouth reveals no
oral lesions. A cardiovascular examination reveals tachycardia with a regular rhythm and no murmur.
Auscultation of the lungs reveals diffuse crackles and wheezes without focal findings. His extremities
are warm, with a capillary refill time of <3 sec.

In addition to oral or intravenous rehydration, which one of the following treatment plans is most
appropriate for this patient at this time?

a. Send the child home and follow up tomorrow


b. Admit to the hospital for supportive care only
c. Admit to the hospital for inhaled bronchodilators
d. Admit to the hospital for inhaled bronchodilators and oral dexamethasone
e. Admit to the hospital for inhaled bronchodilators, oral dexamethasone, and intravenous antibiotics

Item 219
ANSWER: B

This patient has classic signs and symptoms of viral bronchiolitis, likely due to respiratory syncytial
virus (RSV). A chest radiograph is not indicated in a patient with a classic presentation and no focal
findings on examination. Most concerning is his history of low urine output, suggesting inadequate oral
intake. This is often related to a high respiratory rate and copious nasal secretions. The patient requires
hospitalization for monitoring of his respiratory status and supportive care, including intravenous or
nasogastric rehydration. At this time the infant does not require supplemental oxygen, as his oxygen
saturation is above 90%. Many medications have been studied for the treatment of bronchiolitis in
children and most have been found to not provide benefit with regard to the need for hospitalization,
length of hospitalization, or disease resolution. Medications that are NOT recommended include inhaled
bronchodilators, inhaled epinephrine, inhaled or systemic corticosteroids, and antibiotics.

220. A 13-year-old male is brought to your office because of pain in his foot. Two days ago he stepped
on a nail that went through his sneaker and caused a puncture wound to the base of his foot. On
examination today he has tenderness and erythema surrounding the wound, and you can express pus
from the wound. He is afebrile.

125
Which one of the following would be best to treat this patient’s cellulitis?

a. Amoxicillin/clavulanate (Augmentin)
b. Cephalexin (Keflex)
c. Ciprofloxacin (Cipro)
d. Doxycycline
e. Trimethoprim/sulfamethoxazole (Bactrim)

Item 220
ANSWER: C

Puncture wounds to the foot commonly get infected. Most soft-tissue infections from puncture wounds
are caused by gram-positive organisms. Staphylococcus aureus is the most common, followed by other
staphylococcal and streptococcal species. When the puncture wound is through the rubber sole of an
athletic shoe, Pseudomonas is the most frequent pathogen. Ciprofloxacin is the only oral antibiotic that
has antipseudomonal activity, and would be the most appropriate choice.

221. You are called by the parents of a 6-year-old male because he has a 2-week history of awakening
at night with severe back pain. You request an immediate evaluation in your office.
A likely cause of this pain is
A) rheumatoid arthritis
B) lumbar sprain
C) compression fracture
D) discitis
E) scoliosis

Item 221
ANSWER: D

Back pain that regularly occurs at night and awakens a child is usually associated with tumors or
infections, such as osteomyelitis, diskitis, osteoid osteoma, osteoblastoma, and spinal cord tumors. Other
possible symptoms associated with nighttime back pain include fever, malaise, and weight loss. Back
pain that occurs at night is an indication for immediate medical evaluation.

222. A 55-year-old male comes to your office for a routine visit due to an insurance change. He has no
health complaints and the physical examination is unremarkable except for some seborrheic keratoses,
onychomycosis, and a blood pressure of 171/90 mm Hg. On two subsequent visits his blood pressure is
168/92 mm Hg and 171/91 mm Hg and you recommend treatment. The patient states that he will not
take any prescription medications, as he believes they are harmful.

Which one of the following nonpharmacologic measures would be appropriate to recommend for this
patient?
A) Limiting sodium intake to 4 g per day
B) Limiting alcohol to no more than 3 drinks per day
C) Daily coenzyme Q10
D) Daily magnesium supplements
E) Moderate physical activity for 150 min or more per week

126
Item 222
ANSWER: E

Although pharmacologic therapy is the mainstay of treatment for hypertension in adults, there are
several nonpharmacologic options that have been shown to lower blood pressure. Moderate exercise 3–4
times per week for 40 minutes or more has been shown to lower high blood pressure, with the greatest
effect seen when patients exercise 150 minutes or more per week. Limiting sodium intake to 2400
mg/day decreases blood pressure, and further effects are seen when it is limited to 1500 mg/day. Alcohol
should be limited to no more than two drinks per day in men, and one drink per day in women.
Magnesium and coenzyme Q10 do not lower blood pressure.

223. A 12-year-old male is brought to your office with a 3-day history of nausea, vomiting, and fever.
His fever has resolved but he continues to have vomiting and diarrhea. He has had large-volume, loose,
nonbloody stools, as well as abdominal cramping. A stool culture is positive for Salmonella.
Which one of the following would be the most appropriate treatment?
A) Supportive care only
B) Ampicillin
C) Ceftriaxone (Rocephin)
D) Ciprofloxacin (Cipro)
E) Trimethoprim/sulfamethoxazole (Bactrim)

Item 223
ANSWER: A

Salmonella is a common cause of gastroenteritis. Transmission is most often associated with eggs,
poultry, undercooked ground meat, and dairy products from contaminated animals, or produce
contaminated by their waste. Salmonella infection is usually associated with nausea, vomiting, diarrhea,
and fever starting 6–48 hours after ingestion of contaminated water or food. Stools are usually moderate-
volume, loose, and nonbloody, although they can be large-volume watery stools with blood. While
Salmonella can cause severe infection, it is usually self-limited. Antibiotics should not be routinely used
to treat uncomplicated Salmonella gastroenteritis and may prolong the duration of Salmonella excretion
in stool. Antibiotic treatment should be reserved for patients who are severely ill or suspected of being
bacteremic. The threshold for treatment should also be decreased in those who are considered to be at
higher risk for severe illness and invasive disease, such as infants, the elderly, patients with sickle cell
disease, and immunosuppressed patients. Chronic fecal carriers of Salmonella may also benefit from
treatment. If treatment is required, ciprofloxacin, ampicillin, ceftriaxone, and
trimethoprim/sulfamethoxazole are all treatment options.

224. A patient is evaluated and admitted through the emergency department with nausea and vomiting
after receiving chemotherapy 1 day earlier. The admission diagnosis is dehydration and acute kidney
injury. Over the next 2 days the patient’s condition progressively declines. The diagnosis is changed to
sepsis with multiple organ failure about 48 hours after admission. Antibiotics are not started in a timely
manner. Upon review, it is felt that the diagnosis of sepsis and initiation of antibiotics could have been
considered earlier in the hospital course.

Which one of the following cognitive processes most likely contributed to the diagnostic error?

127
A) Anchoring bias
B) Commission bias
C) Hindsight bias
D) Omission bias
E) Outcome bias

Item 224
ANSWER: A

Diagnostic error is important because it is very common. Anchoring bias, also known as premature
closure, is defined by the Agency for Healthcare Research and Quality as relying on an initial diagnostic
impression despite subsequent information to the contrary. This is the most frequent single cause of
diagnostic error. In this case, once the diagnosis of dehydration and acute kidney injury were made,
clinicians became ―anchored‖ to that diagnosis and did not consider other possibilities until much later.
Hindsight bias and outcome bias occur when looking back at a case while knowing the result and
outcome. Commission and omission bias relate to the tendency toward action rather than inaction and
the tendency toward inaction rather than action, respectively.

225. A 45-year-old male sees you for a hypertension follow-up visit and points out a nonpainful bump
that he noted on his neck 3–4 months ago. He does not use tobacco products and is otherwise healthy.
He has not had any fever or pain. His blood pressure is controlled with lisinopril (Prinivil, Zestril). On
further evaluation a firm, mobile, nontender 1.5-cm right anterior cervical lymph node is noted.
Which one of the following would be most appropriate at this point?
A) Watchful waiting
B) Discontinuation of lisinopril
C) Antibiotics
D) Corticosteroids
E) Fine-needle aspiration

Item 220
ANSWER: E

This patient is at high risk for malignancy based on his age, sex, and lymphadenopathy for more than 4–
6 weeks. Fine-needle aspiration is an acceptable first-line test to evaluate for a reactive node versus
malignancy. Further testing may be necessary to confirm the diagnosis. Watchful waiting could delay
the diagnosis of a malignancy in a patient at high risk and would not be appropriate. Due to the duration
of his symptoms and presentation, lymphadenitis is unlikely and antibiotics would not be appropriate.
Corticosteroids are not recommended until a diagnosis is confirmed, as they may interfere with the
cytology. While medications can cause lymphadenopathy, lisinopril has not been associated with this
problem.

226. During a routine health maintenance visit a 38-year-old female expresses concern about her riskof
breast cancer because her mother and another relative have had breast cancer. She is asymptomatic and
your clinical breast examination reveals no masses.

For this patient, the U.S. Preventive Services Task Force recommends which one of the
following?
128
A) Administering a familial risk stratification tool
B) BRCA mutation testing
C) Bilateral screening mammography
D) MRI of the breasts
E) Referral for genetic counseling

Item 226
ANSWER: A

Mammographic screening is not recommended at the age of 38. The U.S. Preventive Services Task
Force (USPSTF) recommends against routine mammographic screening for breast cancer between the
ages of 40 and 49, but promotes a policy of individualized shared decision making. Mammography
every 2 years is recommended for women between the ages of 50 and 74 (B recommendation). The
USPSTF recommends any of several familial risk stratification tools for use in women who have a
family member with breast, ovarian, tubal, or peritoneal cancer, to assess for an increased risk of a
BRCA1 or BRCA2 mutation. If the screen is positive, a referral for genetic counseling is recommended
to determine if BRCA testing is indicated (B recommendation). MRI of the breasts is not recommended
for screening by the USPSTF but is recommended by some organizations as an adjunct to
mammography for women determined to be at high risk of breast cancer, such as those with a BRCA
mutation.

227. A 2-year-old female is brought to your office with an occasional barking cough that began late last
night. The child has mild intercostal indrawing and no stridor at rest.

Which one of the following would be most appropriate to help improve this child’s symptoms?
A) Humidification
B) A helium-oxygen mixture (heliox)
C) Oral dexamethasone
D) Nebulized albuterol
E) Nebulized budesonide (Pulmicort Respules)

Item 227
ANSWER: C

A single dose of oral dexamethasone improves symptoms in children with mild croup when compared
with placebo. It is as effective for reducing croup symptoms as nebulized budesonide and is less
distressing for the child. There is currently no evidence from randomized, controlled trials to support the
use of humidification or a helium-oxygen mixture to reduce the symptoms of croup.

228. At a routine visit, a 40-year-old female asks about beginning an exercise regimen. She has a family
history of heart disease and hypertension. She currently has no medical problems, but she is sedentary.
Which one of the following would be the most appropriate recommendation for this patient?
A) A baseline EKG and rhythm strip
B) An exercise stress test prior to beginning exercise
C) Jogging for 30 minutes twice a week
D) Fast walking for 30 minutes 5 or more days per week
129
Item 228
ANSWER: D

This patient would benefit from exercise to prevent or delay the onset of heart disease and hypertension,
and to manage her weight. Exercise stress testing is not specifically indicated for this patient. Current
recommendations are for healthy adults to engage in 30 minutes of accumulated moderate-intensity
physical activity on 5 or more days per week.

229. A 56-year-old male presents with a 2-day history of a fever and productive cough. He has mild
dyspnea with exertion and has pain in his right side when he takes a deep breath. On examination
his temperature is 38.4°C (101.1°F), his respiratory rate is 24/min, his pulse rate is 92
beats/min, and his oxygen saturation is 92% on room air. He has crackles in the right lower lung
posteriorly. The remainder of the examination is normal.

The most likely diagnosis is


A) upper respiratory infection
B) community-acquired pneumonia
C) heart failure
D) pulmonary embolus
E) acute leukemia

Item 229
ANSWER: B

This patient has pneumonia based on the clinical presentation and the physical findings of fever, cough,
and abnormal lung findings. A fever would not be a typical finding in pulmonary embolus or heart
failure. An upper respiratory infection is unlikely given the abnormal lung findings that suggest a lower
respiratory tract infection. This would not be a typical presentation for acute leukemia.

230. A 17-year-old seasonal farm worker presents with a 7-day history of left-sided facial weakness that
he first noted upon awakening. He has no facial pain. Approximately 2 weeks prior to the onset of this
problem he removed a tick from the left side of his neck, but he is uncertain how long it had been
present. The redness at the site resolved spontaneously and he had no additional symptoms until the
onset of facial weakness. He has not had any rash, fever, swollen glands, or neck stiffness. On
examination he has weakness of the muscles on the left side of his face, including the forehead, and he
can only partially close his left eye. The remainder of the physical examination is normal, including the
absence of rashes and lymphadenopathy.

In addition to corticosteroids, which one of the following would be indicated at this time?
A) Amoxicillin as a single dose
B) Ceftriaxone (Rocephin) for 7 days
C) Doxycycline for 14 days
D) Trimethoprim/sulfamethoxazole (Bactrim) for 10 days
E) No antibiotics

130
Item 230
ANSWER: C

Early systemic Lyme disease may manifest with facial nerve palsy, and treatment should
includecorticosteroids. The evidence for efficacy of antivirals for facial nerve palsy is lacking, especially
beyond 3–4 days after onset. But in this case, specific treatment to eradicate the Lyme disease is also
indicated,in order to prevent later, more severe systemic complications.For facial nerve palsy, treatment
with doxycycline or amoxicillin for 14 days is effective. Patients withmore severe neurologic
manifestations of Lyme disease, such as altered mental status,meningoencephalitis, or other cranial
nerve palsies, require longer courses of antibiotics, usually intravenously. A single-dose treatment with
doxycycline or amoxicillin is recommended as prophylaxis in asymptomaticpatients after a known tick
bite. This is only recommended for tick attachment longer than 36 hours, or of unknown duration.
231. A 23-year-old male presents with a lump in his left testicle that he found while showering last
week. He has a history of orchiopexy for cryptorchidism at age 17. He is otherwise healthy.Testicular
ultrasonography reveals a hypoechoic mass in his left testicle.
Which one of the following would be most appropriate at this time?
A) Watchful waiting
B) Serum -hCG, -fetoprotein, and LDH levels
C) CT of the abdomen and pelvis
D) Referral to a urologist

Item 231
ANSWER: D

Patients with a history of cryptorchidism are at high risk for the development of testicular cancer,
especially if orchiopexy is performed after puberty. If sonography shows a hypoechoic mass, a testicular
biopsy is contraindicated, since it may contaminate the scrotum or alter the lymphatic drainage. Radical
inguinal orchiectomy is both diagnostic and therapeutic. Watchful waiting would not be an option in this
high-risk patient. CT of the chest, abdomen, and pelvis, and measurement of the tumor markers are
useful for staging and as an indication of tumor burden, but they are not diagnostic.

232. A 40-year-old female presents with a 4-week history of a persistent sore throat despite supportive
treatment for a viral upper respiratory infection provided by an urgent care facility. She reports
palpitations, weight loss, frequent bowel movements, and anxiety with insomnia for the past month.
On examination she has a mildly enlarged thyroid gland. Laboratory evaluation is notable for a
suppressed TSH level along with elevated free T4 and total T3 levels. A radioactive iodine uptake scan
shows low uptake.
Which one of the following is the most likely diagnosis?
A) Factitious thyrotoxicosis
B) Graves disease
C) Multinodular goiter
D) Subacute thyroiditis
E) TSH-secreting pituitary adenoma

Item 232
ANSWER: D

131
The initial ―destructive‖ phase of subacute thyroiditis presents with signs, symptoms, and laboratory
findings of overt hyperthyroidism; however, a radioactive iodine uptake scan is negative in this phase.
Graves disease and toxic multinodular goiter also present with overt hyperthyroidism, but radioactive
iodine uptake is high. Factitious thyrotoxicosis is associated with low TSH and elevated or normal free
T4 and total T3, but a goiter is not present. A TSH-secreting pituitary adenoma results in elevated TSH,
free T4, and total T3.

233. A 22-year-old white female comes to your office for a routine health maintenance examination. She
has a BMI of 27.0 kg/m2 and a blood pressure of 113/78 mm Hg. She has no significant past medical
history or family history and the examination is normal.

According to U.S. Preventive Services Task Force guidelines, this patient should be screened
for diabetes mellitus beginning at age
A) 25
B) 30
C) 35
D) 40
E) 45

Item 233
ANSWER: D

According to U.S. Preventive Services Task Force guidelines, adults age 40–70 should be screened for
diabetes mellitus if they are overweight (BMI >25.0 kg/m2) or ob
should not be screened until age 40. Screening for diabetes mellitus should be considered for any adult
who has a risk factor such as a family history of diabetes, a personal history of gestational diabetes,
polycystic ovary syndrome, or being a member of a high-risk ethnic group (African-American, Hispanic,
American Indian, Alaskan Native, or Native Hawaiian).

234. A 26-year-old male presents with hand pain. He tells you he was out drinking with friends last night
and does not remember sustaining any injuries. On examination there is diffuse swelling and tenderness
across the dorsal and ulnar aspects of the hand. Radiographs are shown on the following page.
Which one of the following would be the most appropriate treatment?
A) A wrist extension splint
B) A molded finger splint
C) An ulnar gutter splint
D) A short arm cast
E) Surgical pin fixation

Item #234

132
Item 234
ANSWER: C

The radiograph shows a fracture of the fifth metacarpal head, commonly known as a boxer’s fracture.
There is only slight volar angulation and no displacement. The proper treatment for this fracture is an
ulnar gutter splint, which immobilizes the wrist, hand, and fourth and fifth digits. The wrist should be
positioned in slight extension with the metacarpophalangeal joint in 70°–90° of flexion and the proximal
interphalangeal joint in 5°–10° of flexion. Generally, 3 or 4 weeks of continuous splinting is adequate
for healing..
Surgical pinning is indicated in cases of significant angulation (35°–40° or more of volar angulation) or
in fractures with significant rotational deformity or displacement. The other options listed are
notappropriate treatments for this injury. This injury most commonly results from ―man-versus-wall‖
pugilistics, but other mechanisms of injury are possible.

235. A 16-year-old female who plays competitive soccer develops anterior knee pain that is worse with
downhill running and after prolonged sitting. An examination shows no effusion or instability, no joint
line tenderness, an increased Q-angle, and a negative McMurray’s test. A knee radiograph is negative.
Which one of the following is the most likely diagnosis?
A) Osgood-Schlatter syndrome
B) Patellofemoral pain syndrome
C) Pes anserine bursitis
D) Prepatellar bursitis
E) A torn medial meniscus

Item 235
ANSWER: B

Patellofemoral pain syndrome is a common cause of anterior knee pain, especially in women. It is
worsewith running downhill or going down stairs. It is not associated with a knee effusion. The
examination isoften positive for an apprehension test over the patella. A torn meniscus can cause medial
joint linetenderness as well as a positive McMurray’s test, defined as a click and/or pain when moving
the kneefrom flexion to extension with valgus stress. Prepatellar bursitis causes anterior knee pain,
usuallyassociated with tenderness, swelling, and redness over the prepatellar bursa. Osgood-Schlatter
syndromecauses anterior knee pain over the tibial tuberosity. Pes anserine bursitis causes medial knee
pain just distaland slightly posterior to the joint space.

236. You recently diagnosed diabetes mellitus in a 49-year-old male who also has chronic kidney
disease and New York Heart Association class III heart failure. Laboratory studies are
remarkable for a serum creatinine level of 2.0 mg/dL (N 0.6–1.2) and an estimated glomerular
filtration rate of 40 mL/min/1.73 m2.

Which one of the following classes of agents would be most appropriate for this patient?
A) A biguanide such as metformin (Glucophage)
B) A GLP-1 agonist such as liraglutide (Victoza)
C) An SGLT2 inhibitor such as canagliflozin (Invokana)
D) A thiazolidinedione such as rosiglitazone (Avandia)

133
Item 236
ANSWER: B

Of the options given, only a GLP-1 agonist such as liraglutide could be used for this patient because of
his comorbidities of chronic kidney disease and heart failure. Metformin is contraindicated in males with
a creatinine level >1.5 mg/dL and in females with a creatinine level >1.4 mg/dL. SGLT2 inhibitors are
not as safe or effective if the patient’s estimated glomerular filtration rate (eGFR) is <50
mL/min/1.73m2, and it is not recommended in patients with an eGFR <30 mL/min/1.73 m2. The
initiation of rosiglitazone is contraindicated in patients with established New York Heart Association
class III or classIV heart failure.

237. A 76-year-old female presents with a 3-month history of watery diarrhea with up to 12 episodes per
day. She has no hematochezia and no travel history. You suspect microscopic colitis.

Which one of the following is the test of choice to confirm the diagnosis?
A) A barium enema
B) A stool test for calprotectin
C) A celiac panel
D) A biopsy of the colon
E) A jejunal biopsy

Item 237
ANSWER: D

Microscopic colitis is characterized by intermittent secretory diarrhea in older patients, although all ages
can be affected. The cause is unknown, but there is some evidence that more than 6 months of NSAID
use increases the risk. Only a biopsy from the transverse colon can confirm the diagnosis. Two
histologic patterns are found: lymphocytic colitis and collagenous colitis. The other diagnostic studies
listed do not confirm the diagnosis.

238. A 2-year-old female is brought to your office with a 3-day history of rhinorrhea, fever, cough, and
increasing dyspnea. Her past medical history is unremarkable and she is up to date on her
immunizations. She has a respiratory rate of 40/min, a pulse rate of 120 beats/min, a temperature of
37.8°C (100.0°F), and an oxygen saturation of 93% on room air. She is alert and irritable, and has clear
rhinorrhea and expiratory wheezing, but good airflow overall. The remainder of the examination is
normal.

Which one of the following would be most appropriate at this time?


A) Supportive treatment only
B) Nebulized racemic epinephrine (Asthmanefrin)
C) Nebulized albuterol
D) A single dose of dexamethasone
E) A 5-day course of methylprednisolone (Medrol)

134
Item 238
ANSWER: A

No pharmacologic treatment shortens the course of viral bronchiolitis in a young child.


Supplementaloxygen is indicated if the oxygen saturation falls below 90%, but otherwise the most
effective treatment is simply supportive care (fluids, antipyretics, nasal bulb suction, etc.). None of the
pharmacologic options listed are recommended in the treatment of bronchiolitis in this scenario.

239. A 47-year-old female with a 10-year history of type 2 diabetes mellitus is concerned about the
recent onset of swelling in her legs accompanied by a sudden weight gain of 8 lb. She is also
experiencing increased fatigue and shortness of breath with mild exertion.

On examination she has a blood pressure of 150/95 mm Hg, which is above her baseline of 130/85 mm
Hg. Her lungs are clear to auscultation and a cardiac examination is also normal.
She has no hepatosplenomegaly, but her legs are swollen to the level of the midtibia bilaterally.You are
concerned that her symptoms and examination findings may be related to an underlying renal pathology.

To confirm your suspicion, the most appropriate diagnostic test at this time would be

A) a spot urine protein to creatinine ratio


B) a 24-hour urine creatinine determination
C) renal ultrasonography
D) renal enhanced MRI
E) a renal biopsy

Item 239
ANSWER: A

This patient has type 2 diabetes mellitus and presents with new-onset edema in her lower extremities, the
most common presenting symptom of nephrotic syndrome (NS). Patients with NS may also report
foamy urine, exertional dyspnea or fatigue, and significant fluid-associated weight gain. A 24-hour urine
collection for protein (not creatinine) can be used to diagnose proteinuria, but the collection process is
cumbersome and the specimen is often collected incorrectly. The protein-to-creatinine ratio from a
single urine sample is commonly used to diagnose nephrotic-range proteinuria. The role of a renal
biopsy in patients with NS is controversial and there are no evidence-based guidelines regarding
indications for a biopsy. Renal ultrasonography may be appropriate to assess for underlying conditions
and/or disease complications if the glomerular filtration rate is reduced. There is no data to support using
MRI in the diagnosis and management of nephrotic syndrome.

135
240. A mother brings in her 11-year-old daughter, stating that the child has not had a bowel movement
in 5 days. Although she is very embarrassed to talk to you, the daughter confirms that this is the case,
and that it has happened several times since she started middle school earlier this year, where the
bathrooms are very unpleasant. Both the mother and the daughter state that thishas not been a problem in
the past. The mother reports that the daughter is otherwise healthy, takes no medications, and has no past
surgical history. The patient has no discomfort with urination but does have some abdominal pain that
has grown worse over the last day or so. She is not having any trouble breathing. She reports a decreased
appetite over the last 3 days. The physical examination reveals normal vital signs, a normal BMI, and no
abdominal distention or tenderness to palpation.

Which one of the following would you advise at this point?


A) Observation only, as this problem will resolve on its own
B) Dedicated ―toilet time‖ before and after school and nightly before bed
C) A phosphate soda (Fleet) enema nightly
D) Polyethylene glycol (MiraLAX)
E) An abdominal radiograph

Item 240
ANSWER: D

Childhood constipation is a very common problem encountered in outpatient practice. Most childhood
constipation is functional, which appears to be the case with this patient. She has no evidence of organic
disease, and she had a change in lifestyle that caused her to retain stool, resulting in a cycle of
constipation. Polyethylene glycol is easily tolerated, safe, and easy to use. For this reason, it is the first-
line therapy. A daily dose should resolve this issue, although it is likely that the child will need another
course of treatment, as functional constipation is likely to recur. It is not reasonable in this case
to provide no treatment, as the child is uncomfortable. An enema would be unnecessary since oral
therapies are very likely to be effective. Behavioral therapies have not been shown to be as effective as
laxatives for functional constipation, and stringent therapy that is not related to food intake is likely to
increase the stress surrounding going to the toilet. There is no role for imaging in a case where the
suspicion of an organic cause is so low.

Good Luck

END..

136
American Board of Family Medicine

2018

2018 questions and answers

Special thanks to everyone who helped and contributed

Abdullah Al-Anazi
Asya Al-Ghanim
Faisal Al-Dossary
Hanan Al-Anazi
Maryam Al-Dhefeeri
Shaima Al-Amer
Sukainah Al-Dhahi
Yomna Al-Sayagh
Zaidani Al-Zaidani
1. A 68-year-old male presents to your office with a 2-day history of headache,
muscle aches, and chills. His wife adds that his temperature has been up to
104.1°F and he seems confused sometimes. His symptoms have not improved
with usual care, including ibuprofen and increased fluid intake. He and his wife
returned from a cruise 10 days ago but don’t recall anyone having a similar
illness on the ship. This morning he started to cough and his wife was concerned
because she saw some blood in his sputum. He also states that he experiences
intermittent shortness of breath and feels nauseated. His blood pressure is
100/70 mm Hg, heart rate 98/min, temperature 39.4°C (102.9°F), and oxygen
saturation 95% on room air.
Which one of the following would be the preferred method to confirm your
suspected diagnosis of Legionnaires’ disease?
A) Initiating azithromycin (Zithromax) to see if symptoms improve
B) A chest radiograph
C) Legionella polymerase chain reaction (PCR) testing
D) A sputum culture for Legionella
E) Urine testing for Legionella pneumophila antigen

ANSWER: E
A urine test for Legionella pneumophila antigen is the preferred method to
confirm Legionnaires’ disease. This test is rapid and will only detect Legionella
pneumophila antigen. A sputum culture is the gold standard for the diagnosis of
Legionnaires’ disease but it requires 48–72 hours. A chest radiograph does not
confirm the diagnosis but may show the extent of disease. Responding to
antibiotic treatment does not confirm a specific diagnosis.

Ref: Mercante JW, Winchell JM: Current and emerging Legionella diagnostics for laboratory and outbreak
investigations. Clin Microbiol Rev 2015;28(1):95-133. 2) National Center for Immunization and Respiratory
Diseases: Legionella (Legionnaires’ disease and Pontiac fever): Diagnosis, treatment, and prevention.
Centers for Disease Control and Prevention, 2017.

2. Which one of the following factors related to pregnancy and delivery increases
the risk of developmental dysplasia of the hip in infants?
A) A large-for-gestational age infant
B) Twin birth
C) Breech presentation
D) Cesarean delivery
E) Premature birth

ANSWER: C
Risk factors for developmental dysplasia of the hip in infants include a breech
presentation in the third trimester, regardless of whether the delivery was
cesarean or vaginal. Other indications to evaluate an infant for this condition
include a positive family history, a history of previous clinical instability, parental
concern, a history of improper swaddling, and a suspicious or inconclusive
physical examination. Twin birth, a large-for-gestational age infant, and
prematurity are not considered risk factors.

Ref: Shaw BA, Segal LS; Section on Orthopaedics: Evaluation and referral for developmental dysplasia of
the hip in infants. Pediatrics 2016;138(6):pii:e20163107. 2) Hauk L: Developmental dysplasia of the hip in
infants: A clinical report from the AAP on evaluation and referral. Am Fam Physician 2017;96(3):196-197.

3. A healthy 2-month-old female is brought to your office for a routine well baby
examination by both of her parents, who have no concerns. The parents refuse
routine recommended vaccines for their daughter because of their personal
beliefs.
You want to incorporate patient-centeredness and are also concerned about
improving the health of the population. You decide to follow the CDC
recommendations by
A) accepting their decision without further action
B) not offering vaccines at future visits to preserve a positive doctor-patient
relationship
C) having the parents sign a refusal to vaccinate form
D) dismissing the family from the practice
E) pursuing a court order for vaccine administration since the child has no
medical exemptions

ANSWER: C
Experts recommend that a refusal to vaccinate form be signed by patients or
parents who refuse a recommended vaccine. This form should document that the
patient/parents were provided the vaccine information statement (SOR C). The
CDC recommends against dismissing a patient or family from a practice if they
refuse vaccination. Physicians should continue to discuss the benefits of
immunizations at subsequent visits, because some patients/parents may
reconsider their decision not to vaccinate.
Ref: Spencer JP, Trondsen Pawlowski RH, Thomas S: Vaccine adverse events: Separating myth from
reality. Am Fam Physician 2017;95(12):786-794.

4. A 50-year-old male carpet layer presents with swelling of his right knee
proximal to the patella. He does not have any history of direct trauma, fever,
chills, or changes in the overlying skin. On examination the site is swollen but
minimally tender, with no warmth or erythema.
Which one of the following would be most appropriate at this point?
A) Rest, ice, and compression
B) Aspiration of fluid for analysis
C) Injection of a corticosteroid
D) An oral corticosteroid taper
E) Referral to an orthopedic surgeon for resection

ANSWER: A
Prepatellar bursitis is a common superficial bursitis caused by microtrauma from
repeated kneeling and crawling. Other terms for this include housemaid’s knee,
coal miner’s knee, and carpet layer’s knee. It is usually associated with minimal
to no pain. This differs from inflammatory processes such as acute gouty
superficial bursitis, which presents as an acutely swollen, red, inflamed bursa
and, in rare cases, progresses to chronic tophaceous gout with minimal or no
pain.
The proper management of prepatellar bursitis is conservative and includes ice,
compression wraps, padding, elevation, analgesics, and modification of activity.
There is little evidence that a corticosteroid injection is beneficial, even though it
is often done. If inflammatory bursitis is suspected, a corticosteroid injection may
be helpful. Fluid aspiration is indicated if septic bursitis is suspected. Surgery can
be considered for significant enlargement of a bursa if it interferes with function.

Ref: Khodaee M: Common superficial bursitis. Am Fam Physician 2017;95(4):224-231.

5. An 85-year-old female with a previous history of diabetes mellitus,


hypertension, dementia, and peptic ulcer disease has been in a skilled nursing
facility for 4 weeks for rehabilitation after a hip fracture repair secondary to a fall
during an ischemic stroke. She is transported to the emergency department
today when she develops confusion, shortness of breath, and diaphoresis. Her
blood pressure is 172/98 mm Hg, her heart rate is 122 beats/min with an irregular
rhythm, and her respiratory rate is 22/min. An EKG demonstrates atrial fibrillation
and 0.2 mV ST-segment elevation compared to previous EKGs. Her first troponin
level is elevated.
Which one of the following conditions in this patient is considered an ABSOLUTE
contraindication to fibrinolytic therapy?
A) Poorly controlled hypertension
B) Peptic ulcer disease
C) Alzheimer’s dementia
D) Hip fracture repair
E) Ischemic stroke

ANSWER: E
A history of an ischemic stroke within the past 3 months is an absolute
contraindication to fibrinolytic therapy in patients with an ST-elevation myocardial
infarction (STEMI), unless the stroke is diagnosed within 41⁄2 hours. Poorly
controlled hypertension, dementia, peptic ulcer disease, and major surgery less
than 3 weeks before the STEMI are relative contraindications that should be
considered on an individual basis.

Ref: O’Gara PT, Kushner FG, Ascheim DD, et al: 2013 ACCF/AHA guideline for the management of ST-
elevation myocardial infarction: A report of the American College of Cardiology Foundation/American Heart
Association Task Force on Practice Guidelines. Circulation 2013;127(4):e362-e425.

6. An otherwise healthy 57-year-old female presents with a sudden onset of


hearing loss. She awoke this morning unable to hear out of her left ear. There
was no preceding illness and she currently feels well otherwise. She does not
have ear pain, headache, runny nose, congestion, or fever, and she does not
take any daily medications.
On examination you note normal vital signs and find a normal ear, with no
obstructing cerumen and with normal tympanic membrane motion on pneumatic
otoscopy. You perform a Weber test by placing a tuning fork over her central
forehead. She finds that the sound lateralizes to her right ear. The Rinne test
shows sounds are heard better with bone conduction on the left and with air
conduction on the right.
You refer her to an otolaryngologist for further evaluation including audiometry.
You should also consider initiating which one of the following medications at this
visit in order to optimize the likelihood of recovery?
A) Acyclovir (Zovirax)
B) Amoxicillin/clavulanate (Augmentin)
C) Aspirin
D) Nifedipine (Procardia)
E) Prednisone

ANSWER: E
This patient has sudden sensorineural hearing loss (SSNHL) of the left ear
without any accompanying features to suggest a clear underlying cause. An
appropriate evaluation will fail to identify a cause in 85%–90% of cases.
Idiopathic SSNHL can be diagnosed if a patient is found to have a 30-dB hearing
loss at three consecutive frequencies and an underlying condition is not identified
by the history and physical examination.
The most recent guideline from the American Academy of Otolaryngology–Head
and Neck Surgery recommends that oral corticosteroids be considered as first-
line therapy for patients who do not have a contraindication. While there is
equivocal evidence of benefit, for most patients the risk of a short-term course of
corticosteroids is thought to be outweighed by the potential benefit, especially
when considering the serious consequences of long-term profound hearing loss.
Because the greatest improvement in hearing tends to occur in the first 2 weeks,
corticosteroid treatment should be started immediately. The recommended
dosage is 1 mg/kg/day with a maximum dosage of 60 mg daily for 10–14 days.
Antiviral medications, antiplatelet agents, and vasodilators such as nifedipine
have no evidence of benefit. Antibiotics also have no evidence of benefit in the
absence of signs of infection.

Ref: Stachler RJ, Chandrasekhar SS, Archer SM, et al: Clinical practice guideline: Sudden hearing loss.
Otolaryngol Head Neck Surg 2012;146(3 Suppl):S1-S35.

7. You are the team physician for the local high school track team. During a meet
one of the athletes inadvertently steps off the edge of the track and inverts her
right foot forcefully. She is able to bear weight but with significant pain. She
reports pain across her right midfoot. An examination reveals edema over the
lateral malleolus and diffuse tenderness, but she does not have any pain with
palpation of the navicular, the base of the fifth metatarsal, or the posterior distal
lateral and medial malleoli.
Which one of the following would be most appropriate at this time?
A) Radiographs of the right ankle only
B) Radiographs of the right foot only
C) Radiographs of the right foot and ankle
D) Lace-up ankle support, ice, compression, and clinical follow-up
E) Crutches and no weight bearing for 2 weeks, followed by a slow return to
weight bearing

ANSWER: D
The Ottawa foot and ankle rules should be used to determine the need for
radiographs in foot and ankle injuries. A radiograph of the ankle is recommended
if there is pain in the malleolar zone along with the inability to bear weight for at
least four steps immediately after the injury and in the physician’s office or
emergency department (ED), or tenderness at the tip of the posterior medial or
lateral malleolus. A radiograph of the foot is recommended if there is pain in the
midfoot zone along with the inability to bear weight for four steps immediately
after the injury and in the physician’s office or ED, or tenderness at the base of
the fifth metatarsal or over the navicular bone. The Ottawa foot and ankle rules
are up to 99% sensitive for detecting fractures, although they are not highly
specific. In this case there are no findings that would require radiographs, so
treatment for the ankle sprain would be recommended. Compression combined
with lace-up ankle support or an air cast, along with cryotherapy, is
recommended and can increase mobility. Early mobilization, including weight
bearing as tolerated for daily activities, is associated with better long-term
outcomes than prolonged rest.

Ref: Tiemstra JD: Update on acute ankle sprains. Am Fam Physician 2012;85(12):1170-1176. 2) Bica D,
Sprouse RA, Armen J: Diagnosis and management of common foot fractures. Am Fam Physician
2016;93(3):183-191.

8. A 65-year-old male with type 2 diabetes mellitus, hypertension, and obstructive


sleep apnea sees you for follow-up. He does not use tobacco or other drugs, and
his alcohol consumption consists of two drinks per day. His BMI is 31.0 kg/m2,
and he just started a fitness program. The patient tells you that his brother was
recently diagnosed with atrial fibrillation and he asks you if this increases his own
risk.
Which one of the following factors would increase the risk of atrial fibrillation in
this patient?
A) Alcohol use
B) Treatment with lisinopril (Prinivil, Zestril)
C) Treatment with pioglitazone (Actos)
D) Use of a continuous positive airway pressure (CPAP) device
E) Physical stress

ANSWER: A
Alcohol consumption greater than one drink/day has been associated with atrial
fibrillation. While not recommended to prevent atrial fibrillation, pioglitazone and
lisinopril have both been associated with lower rates of atrial fibrillation compared
to alternative therapies. Treatment of obstructive sleep apnea, along with a
regular fitness regimen, has been associated with a decrease in the recurrence
of atrial fibrillation.

Ref: Morin DP, Bernard ML, Madias C, et al: The state of the art: Atrial fibrillation epidemiology, prevention,
and treatment. Mayo Clin Proc 2016;91(12):1778-1810.

9. You have diagnosed a 32-year-old female with moderate iron deficiency


anemia, presumed to be due to chronic menstrual blood loss. She has no
gastrointestinal or genitourinary symptoms, and no bruising or bleeding other
than menstrual bleeding. Her vital signs are normal and a physical examination is
unremarkable. You initiate a trial of oral iron therapy.
Which one of the following would be the best way to assess the patient’s
response to oral iron?
A) A reticulocyte count in 1–2 weeks
B) A repeat hematocrit in 2 weeks
C) A peripheral smear to look for new RBCs in 4 weeks
D) A serum total iron binding capacity and ferritin level in 6 weeks

ANSWER: A
The reticulocyte count is the first and best indicator of iron absorption and bone
marrow response to oral iron therapy in the treatment of iron deficiency anemia.
An increase in reticulocytes is seen as early as 4 days, peaking at 7–10 days.
The rate of production of new RBCs slows thereafter due to a compensatory
decrease in erythropoietin as more iron becomes available. It typically takes 4–6
weeks before seeing recovery in the hematocrit, and for the RBC count and
indices to normalize. However it is usually 4–6 months before iron stores are fully
restored to normal levels, so treatment should continue for at least that long.

Ref: Killip S, Bennett JM, Chambers MD: Iron deficiency anemia. Am Fam Physician 2007;75(5):671-678. 2)
Kasper DL, Fauci AS, Hauser SL, et al (eds): Harrison’s Principles of Internal Medicine, ed 19. McGraw-Hill,
2015, pp 628-629.

10. You see an adult patient who has chronic urticaria and no other known
chronic conditions. He continues to experience hives after a 3-month course of
daily loratadine (Claritin).
Which one of the following would be the most appropriate addition to his
treatment regimen at this time?
A) A short course of oral corticosteroids
B) Cyclosporine
C) Ranitidine (Zantac)
D) Narrow-band UV light treatment

ANSWER: C
First- and second-generation H1 antihistamine receptor antagonists are generally
considered first-line treatment for chronic urticaria, and approximately 60% of
patients experience a satisfactory result. Second-generation options such as
loratadine have the added benefit of a lower likelihood of side effects such as
drowsiness. For those who fail to achieve the desired result with monotherapy
using an H1 antihistamine receptor antagonist, the addition of an H2
antihistamine receptor antagonist such as cimetidine or ranitidine is often
beneficial. The tricyclic antidepressant doxepin has strong H1 and H2
antihistamine receptor antagonist effects and has been used as an off-label
treatment option in some studies. A short course of oral corticosteroids, narrow-
band UV light treatment, or cyclosporine can be used in the management of
recalcitrant chronic urticaria, but these are considered second- or third-line
adjunctive options.

Ref: Perera E, Sinclair R: Evaluation, diagnosis and management of chronic urticaria. Aust Fam Physician
2014;43(9):621-625.

11. A 10-year-old male has an 8-mm induration 2 days after a tuberculin skin
test. He shares a bedroom with his 18-year-old brother who was recently
diagnosed with tuberculosis. There are no other historical or physical
examination findings to suggest active tuberculosis infection and a chest
radiograph is normal.
Which one of the following would be most appropriate at this point?
A) Monitoring with annual tuberculin skin testing
B) Observation and repeat tuberculin skin testing in 3 weeks
C) Rifampin (Rifadin) daily for 4 months
D) Isoniazid daily for 9 months
E) Once-weekly isoniazid and rifampin for 3 months

ANSWER: D
This patient’s close contact with a person known to be infected with tuberculosis
(TB) places him at risk for infection, so screening for TB is indicated. For this
patient, testing with either a tuberculin skin test or an interferon-gamma release
assay is appropriate. Based on CDC guidelines an induration 5 mm at 48–72
hours following an intradermal injection of tuberculin is a positive test in
individuals who have been in recent contact with a person with infectious TB,
those with radiographic evidence of prior TB, HIV-infected persons, and
immunosuppressed patients. For other individuals at increased risk for TB, the
threshold for a positive test is an induration 10 mm at 48–72 hours. For those
with no known risks for TB infection, the induration must exceed 15 mm in size to
be considered positive. Once positive, there is no indication for additional skin
tests.
A positive screening test along with a review of systems, a physical examination,
and a chest radiograph that do not show evidence of active infection confirms the
diagnosis of latent TB. For children age 2–11 years, treatment with isoniazid, 10–
20 mg/kg daily or 20–40 mg/kg twice weekly for 9 months, is the preferred and
most efficacious treatment regimen. The shorter 6-month treatment course is
considered an acceptable option for adults, but it is not recommended for
children. The use of rifampin alone or in combination with isoniazid is also an
acceptable option for adults but not for children under the age of 12.

Ref: National Center for HIV/AIDS, Viral Hepatitis, STD, and TB Prevention: Latent Tuberculosis Infection: A
Guide for Primary Health Care Providers. Centers for Disease Control and Prevention, 2013. 2) Lewinsohn
DM, Leonard MK, LoBue PA, et al: Official American Thoracic Society/Infectious Diseases Society of
America/Centers for Disease Control and Prevention clinical practice guidelines: Diagnosis of tuberculosis in
adults and children. Clin Infect Dis 2017;64(2):111-115.

12. A 62-year-old female with diabetes mellitus presents to your office with left
lower quadrant pain and guarding. She has a previous history of a shellfish
allergy that caused hives and swelling.
Further evaluation of this patient should include which one of the following?
A) Ultrasonography of the abdomen
B) CT of the abdomen and pelvis with oral and intravenous (IV) contrast
C) Oral corticosteroids and antihistamines, then CT of the abdomen and pelvis
with oral and IV contrast
D) Intravenous corticosteroids and antihistamines, then CT of the abdomen and
pelvis with oral and IV contrast
E) Laparotomy

ANSWER: B
Evaluation of this patient should include CT of the abdomen and pelvis with oral
and intravenous (IV) contrast. There is no reason to inquire about shellfish
allergies prior to CT with IV contrast, because premedication is not needed.
There is no correlation between shellfish allergies and allergic reactions to
contrast. Patients with moderately severe to severe reactions to IV contrast in the
past would need pretreatment with corticosteroids.

Ref: Narayan AK, Durand DJ, Feldman LS: Avoiding contrast-enhanced computed tomography scans in
patients with shellfish allergies. J Hosp Med 2016;11(6):435-437.

13. A 45-year-old female who works as a house cleaner presents with left
shoulder pain. On examination she has pain and relative weakness when
pushing toward the midline against resistance while the shoulder is adducted and
the elbow is bent to 90°. With the elbow still at 90° she is unable to keep her left
hand away from her body when you position her hand behind her back.
This presentation is most consistent with an injury of which one of the following
tendons?
A) Deltoid
B) Infraspinatus
C) Subscapularis
D) Supraspinatus
E) Teres minor

ANSWER: C
This patient’s pain and weakness while pushing against resistance reveals
weakness on internal rotation of the shoulder, which suggests a possible tear of
the subscapularis tendon. The inability to keep her hand away from her body
when it is placed behind her back describes a positive internal lag test, also
suggesting involvement of the subscapularis tendon. The infraspinatus and teres
minor are involved in external rotation rather than internal rotation. The
supraspinatus and deltoid are involved in abduction of the shoulder.

Ref: Woodward TW, Best TM: The painful shoulder: Part I. Clinical evaluation. Am Fam Physician
2000;61(10):3079-3088. 2) Verry C, Fernando S: Rotator cuff disease: Diagnostic tests. Am Fam Physician
2016;94(11):925-926.

14. An 8-year-old male is brought to your office because of acute lower


abdominal pain. He is not constipated and has never had abdominal surgery.
You suspect acute appendicitis.
Which one of the following would be most appropriate at this point?
A) Plain radiography
B) Ultrasonography
C) CT without contrast
D) CT with contrast
E) MRI

ANSWER: B
Ultrasonography is recommended as the first imaging modality to evaluate acute
abdominal pain in children. It avoids radiation exposure and is useful for
detecting many causes of abdominal pain, including appendicitis. After
ultrasonography, CT or MRI can be used if necessary to diagnose appendicitis.
Abdominal radiography is helpful in patients with constipation, possible bowel
obstruction, or a history of previous abdominal surgery.
The American Academy of Pediatrics Choosing Wisely recommendation on the
evaluation of abdominal pain states that CT is not always necessary. The
American College of Surgeons Choosing Wisely recommendation on the
evaluation of suspected appendicitis in children says that CT should be avoided
until after ultrasonography has been considered as an option.
Ref: American Academy of Pediatrics: Computed tomography (CT) scans are not necessary in the routine
evaluation of abdominal pain. ABIM Foundation Choosing Wisely campaign, 2013. 2) Reust CE, Williams A:
Acute abdominal pain in children. Am Fam Physician 2016;93(10):830-836. 3) ACR appropriateness criteria.
American College of Radiology, 2018.

15. You see a patient with a serum sodium level of 122 mEq/L (N 135–145) and
a serum osmolality of 255 mOsm/kg H2O (N 280–295). Which one of the
following would best correlate with a diagnosis of syndrome of inappropriate
antidiuresis?
A) A fractional excretion of sodium below 1%
B) Elevated urine osmolality
C) Elevated serum glucose
D) Elevated BUN
E) Low plasma arginine vasopressin

ANSWER: B
The syndrome of inappropriate antidiuresis (SIAD, formerly SIADH) is related to
a variety of pulmonary and central nervous system disorders in which
hyponatremia and hypo-osmolality are paradoxically associated with an
inappropriately concentrated urine. Most cases are associated with increased
levels of the antidiuretic hormone arginine vasopressin (AVP). Making a
diagnosis of SIAD requires that the patient be euvolemic and has not taken
diuretics within the past 24–48 hours, and the urine osmolality must be high in
conjunction with both low serum sodium and low osmolality. The BUN should be
normal or low and the fractional excretion of sodium >1%.
Fluid restriction (<800 cc/24 hrs) over several days will correct the
hyponatremia/hypo-osmolality, but definitive treatment requires eliminating the
underlying cause, if possible. In the case of severe, acute hyponatremia with
symptoms such as confusion, obtundation, or seizures, hypertonic (3%) saline
can be slowly infused intravenously but might have dangerous neurologic side
effects.
Elevated serum glucose levels may cause a factitious hyponatremia, but not
SIAD.

Ref: Ellison DH, Berl T: The syndrome of inappropriate antidiuresis. N Engl J Med 2007;356(2):2064-2072.
2) Verbalis JG, Goldsmith SR, Greenberg A, et al: Diagnosis, evaluation, and treatment of hyponatremia:
Expert panel recommendations. Am J Med 2013;126(10 Suppl 1):S1-S42.

16. A 45-year-old African-American male returns to your clinic to evaluate his


progress after 6 months of dedicated adherence to a diet and exercise plan you
prescribed to manage his blood pressure. His blood pressure today is 148/96
mm Hg. He is not overweight and he does not have other known medical
conditions or drug allergies.
Which one of the following would be the most appropriate initial antihypertensive
treatment option for this patient?
A) Chlorthalidone
B) Hydralazine
C) Lisinopril (Prinivil, Zestril)
D) Losartan (Cozaar)
E) Metoprolol

ANSWER: A
Lifestyle modifications addressing diet, physical activity, and weight are important
in the treatment of hypertension, particularly for African-American and Hispanic
patients. When antihypertensive drugs are also required, the best options may
vary according to the racial and ethnic background of the patient. The presence
or absence of comorbid conditions is also important to consider. For African-
Americans, thiazide diuretics and calcium channel blockers, both as
monotherapy and as a component in multidrug regimens, have been shown to be
more effective in lowering blood pressure than ACE inhibitors, angiotensin II
receptor blockers, or -blockers, and should be considered as first-line options
over the other classes of antihypertensive drugs unless a comorbid condition is
present that would be better addressed with a different class of drugs. Racial or
ethnic background should not be the basis for the exclusion of any drug class
when multidrug regimens are required to reach treatment goals.
Ref: James PA, Oparil S, Carter BL, et al: 2014 evidence-based guideline for the management of high blood
pressure in adults: Report from the panel members appointed to the Eighth Joint National Committee (JNC
8). JAMA 2014;311(5):507-520. 2) Whelton PK, Carey RM, Aronow WS, et al: 2017
ACC/AHA/AAPA/ABC/ACPM/AGS/APhA/ASH/ASPC/ NMA/PCNA guideline for the prevention, detection,
evaluation, and management of high blood pressure in adults: A report of the American College of
Cardiology/American Heart Association Task Force on clinical practice guidelines. J Am Coll Cardiol
2017;pii:S0735-1097(17)41519-1.

17. An 85-year-old female with metastatic breast cancer requests hospice care.
She has type 2 diabetes mellitus, stage 3 renal failure, and heart disease.
The patient’s eligibility for hospice care will be based on her
A) age
B) cancer diagnosis
C) comorbid conditions
D) life expectancy
E) Medicare Part B plan

ANSWER: D
Eligibility for hospice care is based on a life expectancy of 6 months or less in the
natural course of an illness. A majority of hospice patients have cancer but it is
not a requirement to qualify for hospice care. Age is not relevant. Comorbid
conditions may affect longevity but are not required. For those insured by
Medicare, Medicare Part A provides hospice care but Medicare Part B does not.

Ref: Medicare Hospice Benefits. Centers for Medicare & Medicaid Services, 2018, p 7.

18. A 15-year-old female presents with a 3-month history of intermittent


abdominal pain and headaches. She does not have any associated weight loss,
fever, nausea, change in bowel habits, or other worrisome features. An
examination is unremarkable. She does report being stressed at school and has
a PHQ-2 score of 4.
Which one of the following would be most appropriate at this point?
A) Further evaluation for depression
B) Laboratory studies
C) Abdominal imaging
D) Citalopram (Celexa)
E) Fluoxetine (Prozac)

ANSWER: A
The U.S. Preventive Services Task Force recommends depression screening for
all adolescents 12–18 years of age. Although this patient has abdominal pain, the
history and physical examination suggest that depression may be playing a role
in her somatic complaints. She had a positive initial depression screen on her
PHQ-2. This is a brief screening tool, and a positive result merits further
evaluation. The evaluation should include a full PHQ-A or a discussion with a
qualified clinician. If the patient meets the criteria for major depressive disorder
then she should receive treatment for her depression, which could include
medication. Both fluoxetine and citalopram have been approved by the FDA to
treat depression in this age group. She could also be referred for psychotherapy.
Further laboratory studies and imaging may be appropriate at some point, but the
most urgent need is to evaluate her positive depression screen.

Ref: Richardson LP, Rockhill C, Russo JE, et al: Evaluation of the PHQ-2 as a brief screen for detecting
major depression among adolescents. Pediatrics 2010;125(5):e1097-e1103. 2) Final Recommendation
Statement: Depression in Children and Adolescents: Screening. US Preventive Services Task Force, 2016.

19. A 69-year-old male with type 2 diabetes mellitus, obesity, and a history of
coronary artery disease sees you for follow-up of his diabetes. His hemoglobin
A1c has increased to 8.7% despite therapy with metformin (Glucophage), 1000
mg twice daily, and insulin glargine (Lantus).
Which one of the following additional medications would be most effective for
reducing his blood glucose level and lowering his risk of cardiovascular events?
A) Exenatide (Byetta)
B) Glipizide (Glucotrol)
C) Liraglutide (Victoza)
D) Rosiglitazone (Avandia)
E) Sitagliptin (Januvia)

ANSWER: C
Liraglutide, exenatide, and dulaglutide are all GLP-1 receptor agonists. Of these,
only liraglutide has been shown to lower the risk of recurrent cardiovascular
events and has received FDA approval for this indication. Glipizide (a
sulfonylurea), rosiglitazone, and sitagliptin have not been associated with
improved cardiovascular outcomes. Empagliflozin, an SGLT2 inhibitor, has also
been associated with secondary prevention of cardiovascular disease.

Ref: Marso SP, Daniels GH, Brown-Frandsen K, et al: Liraglutide and cardiovascular outcomes in type 2
diabetes. N Engl J Med 2016;375(4):311-322. 2) Sattar N, Petrie MC, Zinman B, Januzzi JL Jr: Novel
diabetes drugs and the cardiovascular specialist. J Am Coll Cardiol 2017;69(21):2646-2656.

20. A 2-year-old African-American male with a history of sickle cell disease is


brought to your office for a well child check. Which one of the following would be
most appropriate for screening at this time?
A) A chest radiograph
B) A DXA scan
C) Abdominal ultrasonography
D) Renal Doppler ultrasonography
E) Transcranial Doppler ultrasonography

ANSWER: E
Individuals with sickle cell disease are at increased risk for vascular disease,
especially stroke. All sickle cell patients 2–16 years of age should be screened
with transcranial Doppler ultrasonography (SOR A). A chest radiograph,
abdominal ultrasonography, a DXA scan, and renal Doppler ultrasonography are
not recommended for screening patients with sickle cell disease.
Ref: Yawn BP, John-Sowah J: Management of sickle cell disease: Recommendations from the 2014 Expert
Panel report. Am Fam Physician 2015;92(12):1069-1076.

21. You perform the initial newborn examination on a male on his first day of life,
following an uncomplicated vaginal delivery at an estimated gestational age of 37 weeks
and 6 days. The prenatal course was significant for the initial presentation for prenatal
care at 22 weeks gestation. You note that the infant’s upper lip is thin and the philtrum is
somewhat flat.
Which additional finding would increase your concern for fetal alcohol syndrome?
A) Curvature of the fifth digit of the hand (clinodactyly)
B) A supernumerary digit of the hand
C) Flattening of the head (plagiocephaly)
D) Metatarsus adductus in one foot
E) Syndactyly of the toes (webbed feet)

ANSWER: A
In addition to clinodactyly, fetal alcohol syndrome is associated with camptodactyly
(flexion deformity of the fingers), other flexion contractures, radioulnar synostosis,
scoliosis, and spinal malformations. It is also associated with many neurologic,
behavioral, and cardiovascular abnormalities, as well as other types of abnormalities.
Plagiocephaly, supernumerary digits, syndactyly, and metatarsus adductus are common
in newborns but are not related to fetal alcohol spectrum disorders.

Ref: Denny L, Coles S, Blitz R: Fetal alcohol syndrome and fetal alcohol spectrum disorders. Am Fam
Physician 2017;96(8):515-522.

22. An otherwise healthy 3-year-old child with no allergies is found to have otitis media
with effusion in the right ear. Which one of the following would you recommend?
A) No treatment, and follow-up in 3 months
B) Amoxicillin
C) Oral antihistamines
D) Nasal corticosteroids
E) Tympanostomy tube placement

ANSWER: A
This child has otitis media with effusion, and the recommended course of action is to
follow up in 3 months. Medications, including decongestants, antihistamines, antibiotics,
and corticosteroids, are not recommended.

Ref: Lambert M: AAO-HNS releases updated guideline on management of otitis media with effusion. Am
Fam Physician 2016;94(9):747-749.

23. A 32-year-old male presents with a 4-week history of persistent low back pain. He
started feeling tightness in his low back after helping a friend move into a new
apartment. The pain does not radiate, there is no associated paresthesia or numbness,
and he has not had any bowel or bladder incontinence. The pain is constant and
worsens with prolonged sitting. He rates the pain as 6 on a scale of 10. Ibuprofen has
provided minimal relief. Examination of the lumbar area over the paraspinous muscles
reveals minimal tenderness. A neurovascular examination and a straight leg raise are
normal in both lower extremities.
Which one of the following would be most appropriate at this point?
A) Imaging studies of the lumbar spine
B) A short course of an oral corticosteroid
C) Gabapentin (Neurontin) started at a low dose and titrated to effect
D) A skeletal muscle relaxant and an NSAID
E) A short-acting opioid and an NSAID

ANSWER: D
This patient has acute to subacute nonspecific low back pain. Combination treatment
with an NSAID and a skeletal muscle relaxant is recommended as second-line therapy
when an NSAID is ineffective as monotherapy. Opioids have not been shown to have
significant benefit when added to an NSAID and would not be recommended as a
second-line treatment. Systemic corticosteroids do not have evidence to support their
use in the treatment of acute nonspecific back pain. Gabapentin does not have evidence
to support its use in treating acute back pain and has been shown to produce only
minimal improvement in chronic back pain. This patient has no red-flag symptoms so
imaging studies are not recommended at this
time.
Ref: Qaseem A, Wilt TJ, McLean RM, et al: Noninvasive treatments for acute, subacute, and chronic low
back pain: A clinical practice guideline from the American College of Physicians. Ann Intern Med
2017;166(7):514-530

24. A 48-year-old female with GERD treated with a proton pump inhibitor for the past 2
years sees you for a routine visit. She reports that she has paresthesia and numbness in
both feet. Her hemoglobin A1c is 5.8%, her hemoglobin level is 10.4 g/dL (N 12.0–16.0),
and her mean corpuscular volume is 102 m3 (N 81–99). Microfilament testing shows
decreased sensation in both feet.
Which one of the following is the most likely cause of her peripheral neuropathy?
A) Charcot-Marie-Tooth disease
B) Diabetic peripheral neuropathy
C) Hyperthyroidism
D) Tarsal tunnel syndrome
E) Vitamin B12 deficiency

ANSWER: E
This patient has polyneuropathy, macrocytic anemia, and a history of chronic proton
pump inhibitor use. The most likely cause is vitamin B12 deficiency and a serum level is
indicated. Her hemoglobin A1c is 5.8%, which puts her at risk of developing diabetes
mellitus but is not indicative of diabetes. Charcot-Marie-Tooth disease is a rare cause of
polyneuropathy and unlikely in this case. Hypothyroidism, and not hyperthyroidism, is
associated with polyneuropathy. Tarsal tunnel syndrome causes a
mononeuropathy.
Ref: Langan RC, Goodbred AJ: Vitamin B12 deficiency: Recognition and management. Am Fam Physician
2017;96(6):384-389.
25. A 60-year-old male presents with a several-month history of a dry cough and
progressive shortness of breath with exertion. On examination he has tachypnea and
bibasilar end-inspiratory dry crackles, and a chest radiograph reveals interstitial
opacities.
Which one of the following patient occupations would most likely support a diagnosis of
silicosis?
A) Baker
B) Firefighter
C) Stone cutter
D) Goat dairy farmer
E) High-tech electronics fabricator

ANSWER: C
Family physicians should be aware of the environmental exposures associated with
pulmonary disease. Stone cutting, sand blasting, mining, and quarrying expose patients
to silica, which is an inorganic dust that causes pulmonary fibrosis (silicosis).
Occupational exposure to beryllium, which is also an inorganic dust, occurs in the high-
tech electronics manufacturing industry and results in chronic beryllium lung disease.
Exposure to organic agricultural dusts (fungal spores, vegetable products, insect
fragments, animal dander, animal feces, microorganisms, and pollens) can result in
“farmer’s lung,” a hypersensitivity pneumonitis. Other organic dust exposures, such as
exposures to grain dust in bakers, can lead to asthma, chronic bronchitis, and COPD.
Firefighters are at risk of smoke inhalation and are exposed to toxic chemicals that can
cause many acute and chronic respiratory symptoms.
Ref: Kasper DL, Fauci AS, Hauser SL, et al (eds): Harrison’s Principles of Internal Medicine, ed 19.
McGraw-Hill, 2015, pp 1687-1694, 1708-1716.

26. A 28-year-old female presents with a 3-month history of fatigue and postural
lightheadedness. On examination she is diffusely hyperpigmented, especially her skin
creases and areolae. A CBC and basic metabolic panel are normal except for an
elevated potassium level. You order a corticotropin stimulation test.
Prior to the corticotropin injection, you should order which one of the following tests to
confirm that this patient has a primary insufficiency and not a secondary (pituitary)
disorder?
A) ACTH
B) Aldosterone
C) Melanocyte-stimulating hormone
D) Renin
E) TSH

ANSWER: A
A plasma ACTH level is recommended to establish primary adrenal insufficiency. The
sample can be obtained at the same time as the baseline sample in the corticotropin
test. A plasma ACTH greater than twice the upper limit of the reference range is
consistent with primary adrenal insufficiency. Aldosterone and renin levels should be
obtained to establish the presence of adrenocortical insufficiency, but these do not
differentiate primary from secondary adrenal insufficiency. The hyperpigmentation of
Addison’s disease is caused by the melanocyte-stimulating hormone (MSH)–like effect
of the elevated plasma levels of ACTH. ACTH shares some amino acids with MSH and
also produces an increase in MSH in the blood. TSH is not part of the feedback loop of
adrenal insufficiency.

Ref: Bornstein SR, Allolio B, Arlt W, et al: Diagnosis and treatment of primary adrenal insufficiency: An
Endocrine Society clinical practice guideline. J Clin Endocrinol Metab 2016;101(2):364-389.

27. You see a 3-year-old female with a 2-day history of intermittent abdominal cramps,
two episodes of emesis yesterday, and about five watery, nonbloody stools each day.
She does not have a fever, her other vital signs are normal, and she has not traveled
recently. Today she has tolerated sips of fluid but still has mild fatigue and thirst. An
examination is normal except for mildly dry lips. A friend at preschool had a similar
illness recently.
Which one of the following would be the most appropriate initial management of this
patient?
A) A sports drink and food on demand
B) Half-strength apple juice and food on demand
C) Ginger ale and no food yet
D) Water and no food yet
E) A bolus of intravenous normal saline and no food yet

ANSWER: B
Family physicians often see patients with diarrheal illnesses and most of these are viral.
Patients sometimes have misconceptions about preferred fluid and feeding
recommendations during these illnesses. The World Health Organization recommends
oral rehydration with low osmolarity drinks (oral rehydration solution) and early
refeeding. Half-strength apple juice has been shown to be effective, and it approximates
an oral rehydration solution. Its use prevents patient measurement errors and the
purchase of beverages with an inappropriate osmolarity. Low osmolarity solutions
contain glucose and water, which decrease stool frequency, emesis, and the need for
intravenous fluids compared to higher osmolarity solutions like soda and most sports
drinks. Water increases the risk of hyponatremia in children. This patient is not ill enough
to need intravenous fluids. Early refeeding has been shown to decrease the duration of
illness.
Ref: The treatment of diarrhoea: A manual for physicians and other senior health workers. World Health
Organization, 2005. 2) Barr W, Smith A: Acute diarrhea. Am Fam Physician 2014;89(3):180-189. 3) Chang
JG: Oral rehydration solutions for the treatment of acute watery diarrhea. Am Fam Physician
2017;96(11):700-701.

28. A 32-year-old female requests a physical examination prior to participating in an


adult soccer league. Her blood pressure is 118/70 mm Hg and her pulse rate is 68
beats/min. The examination is otherwise normal except for a systolic murmur that
intensifies with Valsalva maneuvers. She says that she has recently been experiencing
mild exertional dyspnea and moderate chest pain. The chest pain has been atypical and
is not necessarily related to exertion. Echocardiography reveals hypertrophic
cardiomyopathy.
In addition to referring the patient to a cardiologist, you recommended initiating therapy
with
A) amiodarone (Cordarone)
B) amlodipine (Norvasc)
C) furosemide (Lasix)
D) lisinopril (Prinivil, Zestril)
E) metoprolol

ANSWER: E
Hypertrophic cardiomyopathy is the most common primary cardiomyopathy, with a
prevalence of 1:500 persons. Many patients with hypertrophic cardiomyopathy are
asymptomatic and are diagnosed during family screening, by auscultation of a heart
murmur, or incidentally after an abnormal result on electrocardiography. On examination
physicians may hear a systolic murmur that increases in intensity during Valsalva
maneuvers. The main goals of therapy are to decrease exertional dyspnea and chest
pain and prevent sudden cardiac death. -Blockers are the initial therapy for patients with
symptomatic hypertrophic cardiomyopathy. Nondihydropyridine calcium channel
blockers such as verapamil can be used if -blockers are not well tolerated.
Ref: Brieler J, Breeden MA, Tucker J: Cardiomyopathy: An overview. Am Fam Physician 2017;96(10):640-
646

29. An 85-year-old female with advanced Alzheimer’s disease is brought to your office
for treatment of agitation, aggressive behavior, and delusions. Behavioral and
psychological interventions have had little success and the family is willing to try
medications because they prefer to keep the patient at home.
Which one of the following would most likely help control this patient’s symptoms?
A) Alprazolam (Xanax)
B) Aripiprazole (Abilify)
C) Clozapine (Clozaril)
D) Donepezil (Aricept)
E) Haloperidol

ANSWER: B
Nonpharmacologic interventions are the first-line treatment for patients with behavioral
and psychological symptoms of dementia. Antipsychotic medications can be prescribed
for refractory cases but this is an off-label use. Both the patient and family should be
aware that the use of atypical antipsychotics for behavioral symptoms of dementia is
associated with increased mortality. Patients should be monitored for side effects and
the medication should be discontinued if there is no evidence of symptom improvement
after a month. Typical antipsychotics such as haloperidol have significant side effects
and would not be a good choice. Donepezil is initiated early in the course of Alzheimer’s
disease to delay progression of the disease. Benzodiazepines are likely to cause
significant side effects including sedation, increased confusion, and
falls. Several of the antipsychotics, such as ziprasidone and clozapine, are ineffective.
Results with olanzapine, quetiapine, and risperidone are inconsistent. Aripiprazole
produces small reductions in behavioral and psychological symptoms of dementia, and it
has the least adverse effects of the atypical antipsychotics.
Ref: Reese TR, Thiel DJ, Cocker KE: Behavioral disorders in dementia: Appropriate nondrug interventions
and antipsychotic use. Am Fam Physician 2016;94(4):276-282.
30. A 30-year-old female with anovulatory uterine bleeding asks about treatment options.
An examination is normal and blood testing is negative. She is unmarried and is
undecided about having children.
Which one of the following would be the most appropriate treatment for this patient?
A) Oral progestin during the luteal phase
B) A levonorgestrel-releasing IUD
C) Endometrial ablation
D) Hysterectomy

ANSWER: B
Few treatments for dysfunctional uterine bleeding have been studied. NSAIDs, oral
contraceptive pills, and danazol have not been shown to have sufficient evidence of
effect for the treatment of dysfunctional uterine bleeding. Progestin is effective when
used on a 21-day cycle, but not if used only during the luteal phase. Hysterectomy and
ablation are very effective, but both eliminate fertility. In a young woman unsure
about having children, the levonorgestrel-releasing IUD is the most effective treatment
that preserves fertility (SOR A).
Ref: Sweet MG, Schmidt-Dalton TA, Weiss PM, Madsen KP: Evaluation and management of abnormal
uterine bleeding in premenopausal women. Am Fam Physician 2012;85(1):35-43. 2) Kellerman RD, Bope
ET (eds): Conn’s Current Therapy 2018. Elsevier Saunders, 2018, pp 1073-1074.

31. A 73-year-old male with advanced degenerative arthritis of the knees asks what you
would recommend for relief. He does not wish to have a total knee replacement. He says
that NSAIDs have not been effective.
Which one of the following would be the best recommendation?
A) Acetaminophen
B) Intra-articular corticosteroids
C) Intra-articular hylan GF 20 (Synvisc)
D) Physical therapy for quadriceps strengthening
E) Tramadol (Ultram)

ANSWER: D
Quadriceps-strengthening exercises have been shown in good studies to stabilize the
knee and reduce pain for patients with degenerative arthritis. Acetaminophen has not
been shown to produce clinically significant improvement from baseline pain. Intra-
articular corticosteroids can acutely relieve pain and effusions but do not affect
moderate-term outcomes. Hylan GF 20 products are minimally effective. Opiates and
other similar drugs are addictive and should be avoided.
Ref: Cohen D: Exercise for osteoarthritis of the knee. Am Fam Physician 2015;92(9):774-776. 2) Bannuru
RR, Schmid CH, Kent DM, et al: Comparative effectiveness of pharmacologic interventions for knee
osteoarthritis: A systematic review and network meta-analysis. Ann Intern Med 2015;162(1):46-54.

32. A 66-year-old male recently underwent percutaneous angioplasty for persistent


angina with exertion. He does not have any symptoms now. His LDL-cholesterol level is
90 mg/dL.
Which one of the following would be most appropriate for secondary prevention of this
patient’s coronary artery disease?
A) No drug treatment
B) Evolocumab (Repatha), 140 mg subcutaneously every 2 weeks
C) Ezetimibe (Zetia), 10 mg daily
D) Rosuvastatin (Crestor), 20 mg daily
E) Simvastatin (Zocor), 40 mg daily

ANSWER: D
Patients <75 years of age with established coronary artery disease should be on high-
intensity statin regimens if tolerated. These regimens include atorvastatin, 40–80
mg/day, and rosuvastatin, 20–40 mg/day. Moderate-intensity regimens include
simvastatin, 40 mg/day. Monotherapy with non-statin medications (bile acid
sequestrants, niacin, ezetimibe, and fibrates) does not reduce cardiovascular
morbidity or mortality. The PCSK9 inhibitors evolocumab and alirocumab are second-
line or add-on therapies at this time.
Ref: Braun MM, Stevens WA, Barstow CH: Stable coronary artery disease: Treatment. Am Fam Physician
2018;97(6):376-384.

33. A 62-year-old female who is a new patient requests a thyroid evaluation because
she has a history of abnormal thyroid test results. You obtain a copy of her records,
which include a TSH level of 0.2 U/mL (N 0.4–4.2) and a free T4 level of 2.0 ng/dL (N
0.8–2.7) from 3 years ago. She reports feeling well and has no other health conditions.
She does not take any medications.
A physical examination reveals normal vital signs, a BMI of 23.0 kg/m2, no neck masses,
a normal thyroid size, and normal heart sounds. Laboratory studies reveal a TSH level of
0.1 U/mL, a free T4 level of 2.5 ng/dL, and a free T3 level of 3.1 pg/mL (N 2.3–4.2).
Treatment for this condition would be indicated if the patient has an abnormal
A) calcium level
B) DXA scan
C) glucose level
D) lipid level
E) thyroid ultrasonography study

ANSWER: B
This patient has subclinical hyperthyroidism as evidenced by her low TSH level with
normal free T4 and free T3 levels. Common causes of subclinical hyperthyroidism include
Graves disease, autonomous functioning thyroid adenoma, and multinodular toxic goiter.
Subclinical hyperthyroidism may progress to overt hyperthyroidism; this is more likely in
patients with TSH levels <0.1 U/mL. Even in the absence of overt hyperthyroidism these
patients are at higher risk for several health conditions, including atrial fibrillation, heart
failure, and osteoporosis. For this reason it is important to assess for these conditions
and consider treating the underlying thyroid condition, as well as the complication. The
American Thyroid Association recommends treating patients with complications who are
either over age 65 or have a TSH level <0.1 U/mL. Lipid and glucose abnormalities are
not known to be related to subclinical hyperthyroidism. Calcium levels
may be abnormal in hyperparathyroidism but not hyperthyroidism. Thyroid
ultrasonography may be helpful to determine the cause of hyperthyroidism but is not
used to help decide when to treat subclinical hyperthyroidism.
Ref: Donangelo I, Suh SY: Subclinical hyperthyroidism: When to consider treatment. Am Fam Physician
2017;95(11):710-716.

34. A 43-year-old male who works in a warehouse sees you because of dizziness. He
first noticed mild dizziness when he rolled over and got out of bed this morning. He had
several more severe episodes that were accompanied by nausea, and on one occasion
vomiting occurred after he tilted his head upward to look for items on the higher shelves
at work. You suspect benign paroxysmal positional vertigo, so you perform the Dix-
Hallpike maneuver as part of the examination.
Which one of the following findings during the examination would confirm the diagnosis?
A) Nystagmus when vertigo is elicited
B) Vertigo that occurs immediately following the test-related head movement
C) Persistence of vertigo for 5 minutes following the test-related head movement
D) A drop in systolic blood pressure of >10 mm Hg when supine

ANSWER: A
Benign paroxysmal positional vertigo (BPPV) originates in the posterior semicircular
canal in the majority of patients (85%–95% range reported). The Dix-Hallpike maneuver,
which involves moving the patient from an upright to a supine position with the head
turned 45° to one side and the neck extended 20° with the affected ear down, will elicit a
specific series of responses in these patients. Following a latency period that typically
lasts 5–20 seconds but sometimes as long as 60 seconds, the patient will experience the
onset of rotational vertigo. The objective finding of a torsional, upbeating nystagmus will
be associated with the vertigo. The vertigo and nystagmus typically increase in intensity
and then resolve within 1 minute from onset.
Ref: Bhattacharyya N, Gubbels SP, Schwartz SR, et al: Clinical practice guideline: Benign paroxysmal
positional vertigo (update). Otolaryngol Head Neck Surg 2017;156(3 Suppl):S1–S47.

35. You are initiating pharmacologic therapy for a 75-year-old patient with depression.
Which one of the following would be most appropriate for this patient?
A) Amitriptyline
B) Escitalopram (Lexapro)
C) Imipramine (Tofranil)
D) Paroxetine (Paxil)

ANSWER: B
Escitalopram is a preferred antidepressant for older patients (SOR C). Paroxetine should
generally be avoided in older patients due to a higher likelihood of adverse effects (SOR
C). Amitriptyline, imipramine, and paroxetine are highly anticholinergic and sedating, and
according to the Beers Criteria, they can cause orthostatic hypotension. They have an
“avoid” recommendation (SOR A).

Ref: Kovich H, DeJong A: Common questions about the pharmacologic management of depression in
adults. Am Fam Physician 2015;92(2):94-100. 2) CCSMH national guidelines for seniors’ mental health.
Canadian Coalition for Seniors’ Mental Health. 3) By the American Geriatrics Society 2015 Beers Criteria
Update Expert Panel: American Geriatrics Society 2015 updated Beers Criteria for potentially inappropriate
medication use in older adults. J Am Geriatr Soc
2015;63(11):2227-2246.

36. A 58-year-old male with a 30-pack-year smoking history comes to your office to
discuss screening for COPD. His older brother and sister have both recently been
diagnosed with COPD and he wants to be screened for this soon. He continues to
smoke and does not express a desire to quit. He does not have shortness of breath,
cough, orthopnea, paroxysmal nocturnal dyspnea, or dyspnea on exertion. His only
medication is aspirin, 81 mg daily. He has never used inhaled medications such as
albuterol (Proventil, Ventolin). His family history is otherwise negative. You counsel him
on tobacco cessation today.
Which one of the following is recommended with regard to COPD screening for this
patient?
A) No screening
B) Spirometry with pre- and postbronchodilator testing
C) Posteroanterior and lateral chest radiographs
D) Noncontrast CT of the chest
E) 1-Antitrypsin deficiency gene testing

ANSWER: A
All patients with a smoking history and symptoms of COPD such as a chronic cough with
sputum production and/or chronic and progressive dyspnea should be screened for
COPD with spirometry. However, asymptomatic individuals such as this patient should
not be screened with spirometry regardless of risk factors. Neither chest radiography nor
chest CT has a role in screening for COPD. Screening for 1-antitrypsin deficiency in the
absence of a family history is not recommended.
Ref: Final Recommendation Statement: Chronic Obstructive Pulmonary Disease: Screening . US
Preventive Services Task Force, 2016. 2) Press VG, Cifu AS, White SR: Screening for chronic obstructive
pulmonary disease. JAMA 2017;318(17):1702-1703.

37. The dietary herbal supplement with the highest risk for drug interactions is
A) black cohosh
B) ginseng
C) St. John’s wort (Hypericum perforatum)
D) saw palmetto
E) valerian

ANSWER: C
St. John’s wort can reduce the effectiveness of multiple medications because it is an
inducer of CYP3A4 and P-glycoprotein synthesis. Concurrent use of St. John’s wort with
drugs that are metabolized with these systems should be avoided. These include
cyclosporine, warfarin, theophylline, and oral contraceptives. St. John’s wort should be
avoided in patients taking either over-the-counter or prescription medications.
Ref: Asher GN, Corbett AH, Hawke RL: Common herbal dietary supplement—Drug interactions. Am Fam
Physician 2017;96(2):101-107.

38. A 32-year-old female sees you for a health maintenance visit. She reports that she
experiences severe anxiety when involved in social situations. She lives with her mother
and dreads meeting unfamiliar people. At work she remains in her cubicle throughout the
day and avoids staff parties. She has a history of alcoholism in remission. She has
otherwise been in good health and a physical examination is normal.
Which one of the following would be first-line treatment for this patient?
A) Amitriptyline
B) Bupropion (Wellbutrin)
C) Escitalopram (Lexapro)
D) Lorazepam (Ativan)
E) Pregabalin (Lyrica)

ANSWER: C
Social anxiety disorder can be treated with psychotherapy, pharmacotherapy, or both.
Several medications have been used for the treatment of social anxiety disorder. SSRIs
are considered to be the first-line pharmacologic treatment. Response rates reported for
the SNRI venlafaxine have been similar to those reported for SSRIs. Randomized trials
have also supported the efficacy of benzodiazepines for social anxiety disorder, but they
carry a risk of physiologic dependence and withdrawal symptoms and are not
recommended for patients with coexisting depression or a history of substance abuse.
Response rates to pregabalin have been lower than with SSRIs. Tricyclic
antidepressants and bupropion are not considered to be useful in the treatment of social
anxiety disorder.
Ref: Leichsenring F, Leweke F: Social anxiety disorder. N Engl J Med 2017;376(23):2255-2264.

39. A 67-year-old female presents with a swollen wrist after falling on her outstretched
hand. A radiograph of the affected wrist is shown below.
Prior to surgical intervention, you recommend application of a
A) radial gutter splint
B) sugar tong splint
C) thumb spica splint
D) forearm circumferential cast
ANSWER: B
Fractures involving the distal end of the radius are the most common upper extremity
fractures and are most common in elderly women. The mechanism of injury is usually
from falling on an outstretched hand (FOOSH). Prompt surgical intervention is
recommended in patients with neurovascular compromise, open fractures, or evidence
of compartment syndrome. In general, circumferential casts should be avoided, as
the underlying swelling can compromise distal circulation. The splint of choice in patients
with these fractures is a sugar tong splint. Radial gutter splints are indicated for
uncomplicated fractures of the second and third metacarpals. Thumb spica splints are
often used in patients with suspected scaphoid fractures (SOR B).
Ref: Eiff MP, Hatch R: Fracture Management for Primary Care, ed 3. Elsevier Saunders, 2018, p 367.

40. A 7-year-old female with asthma is brought to your office because of her fourth
episode of wheezing in the last 3 months. She has also had to use her short-acting -
agonist rescue inhaler more frequently.
Which one of the following should be added to reduce the frequency of asthma
exacerbations?
A) A leukotriene receptor antagonist
B) A long-acting -agonist
C) An inhaled corticosteroid
D) Inhaled cromolyn via nebulizer

ANSWER: C
Pediatric asthma is the most commonly encountered chronic illness, occurring in nearly
one out of seven individuals. Short-acting -agonists in the form of metered-dose inhalers
are clearly favored for acute exacerbations, as well as for intermittent asthma. Treatment
for persistent asthma requires the use of inhaled corticosteroids, with short-acting -
agonists used for exacerbations. For patients not well controlled with those options,
either a long-acting -agonist or a leukotriene receptor antagonist may be added. While
both cromolyn and nedocromil are fairly devoid of adverse effects, their use is limited
because of a lack of efficacy in the prevention of acute asthma exacerbations.
Ref: Dunn NA, Neff LA, Maurer DM: A stepwise approach to pediatric asthma. J Fam Pract 2017;66(5):280-
286. 2) Global strategy for asthma management and prevention. Global Initiative for Asthma, 2018.

41- A 55-year-old female presents with the new onset of palpitations. An


underlying cardiac cause should be suspected if the patient's palpitations

A) affect her sleep

B) are associated with dry mouth

C) are worse in public places

D) last less than 5 minutes


ANSWER: A

Palpitations are a common symptom in ambulatory care. Cardiac causes are the most
worrisome so it is important to distinguish cardiac from noncardiac causes. Patients with a
history of cardiovascular disease, palpitations that affect their sleep, or palpitations that
occur at work have an increased risk of an underlying cardiac cause (positive likelihood
ratio 2.0-2.3) (SOR C). Psychiatric illness, adverse effects of medications, and substance
.abuse are other common causes

Palpitations that are worse in public places and those of very short duration (<5 minutes),
especially if there is a history of anxiety, are often related to panic disorder. However, even
a known behavioral issue should not be presumed to be the cause of palpitations, as
nonpsychiatric causes are found in up to 13% of such cases. The use of illicit substances
such as cocaine and methamphetamine can cause palpitations that are associated with dry
mouth, pupillary dilation, sweating, and aberrant behavior. Excessive caffeine can also
.cause palpitations

Ref: Wexler RK, Pleister A, Raman SV: Palpitations: Evaluation in the primary care setting. Am Fam Physician
.2017;96(12):784-789

42- A 69-year-old female presents with scaling, redness, and irritation under her
breasts for the past several months. She has tried several over-the-counter antifungal
creams without any improvement. On examination you note erythematous, well
demarcated patches with some scale under both breasts. You examine the rash with a
Wood's lamp to confirm your suspected diagnosis. ?This rash is most likely to fluoresce

A) bright yellow

B) coral pink

C) lime green

D) pale blue

E) totally white

ANSWER: B

A Wood's lamp may assist with the diagnosis of certain skin conditions. This patient's
presentation is consistent with erythrasma caused by a Corynebacterium minutissimum
infection, and use of an ultraviolet light would reveal a coral pink color. Pale blue
fluorescence occurs with Pseudomonas infections, yellow with tinea infections, and totally
white with vitiligo. A lime green fluorescence is not characteristic of a particular skin
.condition
Ref: Kasper DL, Fauci AS, Hauser SL, et al (eds): Harrison's Principles of Internal Medicine, ed 19. McGraw-Hill,
2015, pp 339-344. 2) Habif TP: Clinical Dermatology: A Color Guide to Diagnosis and Therapy, ed 6. Elsevier,
.2016

43- A 25-year-old female who recently moved to the area comes in for a well
woman visit. She reports that she has had yearly Papanicolaou (Pap) tests and sexually
transmitted infection (STI) screening since age 21 with no abnormal results. She has had a
total of six sexual partners. She is asymptomatic and does not have any history of STIs or
new partners in the past year. Your nurse informs her that STI screening can be done, but a
Pap test is not necessary at this time. ?The patient is concerned about not having a Pap test
this year and asks you why it is not recommended. You explain that the most important
reason is that

A) she has no history of STIs

B) she has had several normal Pap tests in a row

C) she is in a low-risk group for HPV infection

D) Pap test abnormalities would require no further evaluation in a patient her age

E) the risk of harm from unnecessary procedures and treatment exceeds the potential
benefit at her age

ANSWER: E

Annual HPV screening in patients age 21-29 years has very little effect on cancer prevention
and leads to an increase in procedures and treatments without significant benefit. In this
age group there is a high prevalence of high-risk HPV infections but a low incidence of
cervical cancer. If this patient were due for a Papanicolaou (Pap) test and results were ASC-
US with a positive high-risk HPV or a higher grade abnormality, colposcopy would be
recommended. Current recommendations are for a Pap test with cytology every 3 years for
women age 21-29 years with normal results, and the frequency does not change with an
increased number of normal screens. HPV is the most common sexually transmitted
infection (STI) and up to 79% of sexually active women contract HPV infection in their
.lifetime, so the lack of other STIs does not preclude the possibility of an HPV infection

Ref: Juckett G, Hartman-Adams H: Human papillomavirus: Clinical manifestations and prevention. Am Fam
Physician 2010;82(10):1209-1213. 2) Massad LS, Einstein MH, Huh WK, et al: 2012 updated consensus
guidelines for the management of abnormal cervical cancer screening tests and cancer precursors. Obstet
Gynecol 2013;121(4):829-846. 3) ACOG releases guideline on cervical cancer screening. Am Fam Physician
.2013;88(11):776-777

44- A 42-year-old male with hypertension and hyperlipidemia sees you for a routin
health maintenance examination. His blood pressure is 185/105 mm Hg. He does not have
any current symptoms, including headache, chest pain, edema, or shortness of breath. He is
adherent to his current medication regimen, which includes lisinopril (Prinivil, Zestril), 10
mg daily, and simvastatin (Zocor), 20 mg at night. A thorough history and physical
examination are both unremarkable. ?Which one of the following would be the most
appropriate next step

A) A 30-minute rest period followed by a repeat blood pressure reading

B) Clonidine (Catapres), 0.2 mg given in the office

C) A comprehensive metabolic panel, fasting lipid profile, and TSH level

D) A stress test

E) Hospital admission for blood pressure reduction

ANSWER: A

The first step in the management of severe hypertension is determining whether a


hypertensive emergency is present. A thorough history and physical examination are crucial
(SOR C). Severe hypertension (blood pressure >180 mm Hg systolic or >110 mm Hg
diastolic) with end-organ damage constitutes a hypertensive emergency. A physical
examination should center on evaluating for papilledema, neurologic deficits, respiratory
compromise, and chest pain. If end-organ damage is present the patient should be
hospitalized for monitored blood pressure reduction and further diagnostic workup. If end-
organ damage is not present and the physical examination is otherwise normal, a 30-minute
rest with reevaluation is indicated. Approximately 30% of patients will improve to an
acceptable blood pressure without treatment (SOR C). Home medications should then be
adjusted with outpatient follow-up and home blood pressure monitoring (SOR A). Short-
acting antihypertensives are indicated if mild symptoms are noted such as headache,
lightheadedness, nausea, shortness of breath, palpitations, anxiety, or epistaxis. Diagnostic
testing is not immediately indicated for asymptomatic patients (SOR C). A basic metabolic
panel or other testing should be considered if mild symptoms are present. Aggressive
lowering of blood pressure can be detrimental and a gradual reduction over days to weeks
.)is preferred (SOR C

Ref: Oza R, Garcellano M: Nonpharmacologic management of hypertension: What works? Am Fam Physician
2015;91(11):772-776. 2) Gauer R: Severe asymptomatic hypertension: Evaluation and treatment. Am Fam
.Physician 2017;95(8):492-500

45 - You see a 53-year-old female with diabetes mellitus, hypertension, mixed


hyperlipidemia, and GERD. Recent laboratory studies include an incidental finding of
thrombocytopenia. The patient has no other significant past medical history, and she does
not use tobacco or drink alcohol. Her current medications include metformin (Glucophage),
lisinopril (Prinivil, Zestril), omeprazole (Prilosec), calcium citrate, and pravastatin
(Pravachol). A physical examination is notable for a BMI of 31.3 kg/m2. Her skin, heart,
lungs, abdomen, and extremities are normal. Results of a CBC and a comprehensive
metabolic panel are normal with the following exceptions
)Platelets 70,000 (N 150,000-379,000

Glucose 108mg/dL

)Bilirubin 0.4mg/dL(N0.0-0.4

)Alkaline phosphatase 175U/L(N38-126

)ALT(SGPT) 52 U/L (N 10-28

A peripheral smear is normal except for reduced platelets. Tests for hepatitis B, hepatitis C,
.and HIV are negative

The most likely etiology of this patient's thrombocytopenia is

A) a hematologic malignancy

B) chronic liver disease

C) drug-induced thrombocytopenia

)D) immune thrombocytopenic purpura (ITP

E) primary bone marrow failure

ANSWER: B

This patient presents with a typical example of nonalcoholic steatohepatitis (NASH)


progressing toward cirrhosis, with multiple risk factors including diabetes mellitus,
hyperlipidemia, obesity, and mildly elevated hepatic transaminases. Abnormalities of other
cell lines would likely occur if a hematologic malignancy or bone marrow failure were
present. While immune thrombocytopenic purpura is a diagnostic consideration, it is much
less common than NASH and requires other causes to be ruled out. This patient is not taking
any medications that have been frequently reported to cause drug-induced
.thrombocytopenia

.Ref: Gauer RL, Braun MM: Thrombocytopenia. Am Fam Physician 2012;85(6):612-622

46- patient has a past medical history that includes a sleeve gastrectomy for weight loss. .
Which one of the following medications should be AVOIDED in this patient

A) Acetaminophen

)B) Gabapentin (Neurontin

C) Hydrocodone

D) Ibuprofen

)E) Tramadol (Ultram


ANSWER: D

NSAIDs such as ibuprofen are thought to increase the risk of anastomotic ulcerations or
perforations in patients who have had bariatric surgery and should be completely avoided
after such surgery if possible (C Recommendation, Level of evidence 3). It is also
recommended that alternative pain medications that can be used are identified prior to the
surgery (D Recommendation). Options such as acetaminophen, gabapentin, hydrocodone,
and tramadol can be considered in patients who have had bariatric surgery if the
.medications are clinically appropriate otherwise

Ref: Mechanick JI, Youdim A, Jones DB, et al: Clinical practice guidelines for the perioperative nutritional,
metabolic, and nonsurgical support of the bariatric surgery patient-2013 update: Cosponsored by American
Association of Clinical Endocrinologists, the Obesity Society, and American Society for Metabolic & Bariatric
Surgery. Obesity (Silver Spring) 2013;21(Suppl 1):S1-S27. 2) Schroeder R, Harrison TD, McGraw SL: Treatment
.of adult obesity with bariatric surgery. Am Fam Physician 2016;93(1):31-37

47 - An 11-year-old female is referred to you after a sports physical examination


because 2+ protein was found on a random dipstick urinalysis. She feels well and does not
have any health concerns. She plays soccer an average of 5 days a week. ?The patient's
medical history is unremarkable and she takes no medications. Menarche has not occurred.
She does not report any urinary or back symptoms, recent illness, edema, or weight change.
A physical examination is normal. A dipstick urinalysis in your office shows 1+ protein but
is otherwise normal. ?Which one of the following would you recommend next

A) Withdrawing from all physical activity for 24 hours and a 24-hour urine for protein

B) A spot protein/creatinine ratio performed on first morning urine

C) Serum BUN, creatinine, electrolyte, and albumin levels

D) Ultrasonography of the kidneys and bladder

E) Referral to a pediatric nephrologist

ANSWER: B

It is important to distinguish serious illness from benign causes of proteinuria, which are
the most common etiology in children. Confirming the presence of proteinuria is the next
step in this case because functional (exercise/stress-induced) and orthostatic proteinuria
are common types of proteinuria and are transient. A 24-hour urine for protein is a possible
option, but would be impractical and burdensome for a healthy-acting 11-year-old. The
pediatric nephrology panel of the National Kidney Foundation reported that a spot
protein/creatinine ratio is a reliable test for ruling out proteinuria. A specialist referral,
blood analysis, and ultrasonography are unnecessary unless persistent proteinuria is
.identified
Ref: Hogg RJ, Portman RJ, Milliner D, et al: Evaluation and management of proteinuria and nephrotic syndrome
in children: Recommendations from a pediatric nephrology panel established at the National Kidney Foundation
conference on proteinuria, albuminuria, risk, assessment, detection, and elimination (PARADE). Pediatrics
2000;105(6):1242-1249. 2) Leung AK, Wong AH, Barg SS: Proteinuria in children: Evaluation and differential
.diagnosis. Am Fam Physician 2017;95(4):248-254

48 - Intensive behavioral intervention has more benefit than other treatment


modalities in treating children who have been diagnosed with

A) attention-deficit/hyperactivity disorder

B) autism

C) depression

D) obsessive-compulsive disorder

E) posttraumatic stress disorder

ANSWER: B

The only evidence-based treatment that confers significant benefits to children with autism
is intensive behavioral interventions, which should be initiated before 3 years of age.
Attention-deficit/hyperactivity disorder can be treated with cognitive-behavioral therapy
(CBT) but medication is often required. CBT is as effective, if not more effective, than
.medication for treating anxiety, depression, and trauma-related disorders

Ref: Coffey SF, Banducci AN, Vinci C: Common questions about cognitive behavior therapy for psychiatric
disorders. Am Fam Physician 2015;92(9):807-812

49- A patient asks which shingles vaccine he should receive. Which one of the .
following is an advantage of the recombinant zoster vaccine (Shingrix) compared to the live
)zoster vaccine (Zostavax

A) Improved efficacy

B) Lower cost

C) Subcutaneous administration

D) Proven safety for immunocompromised patients

E) Administration as a single dose

ANSWER: A

The recombinant zoster vaccine is preferred over the live zoster vaccine due to its increased
efficacy. The recombinant vaccine is estimated to be about 97% effective for preventing
shingles, compared to 51% with the live vaccine. It requires two intramuscular doses
separated by 2-6 months, compared to only one subcutaneous dose with the live vaccine.
It is also slightly more expensive than the live vaccine. Although the recombinant vaccine is
not a live vaccine, studies are still ongoing as to whether it is safe to give to
.immunocompromised patients

Ref: Le P, Sabella C, Rothberg MB: Preventing herpes zoster through vaccination: New developments. Cleve Clin J
.)Med 2017;84(5):359-366. 2) Shingrix: A new herpes zoster vaccine. Prescriber's Letter 2017;24(12

50 - A 45-year-old female sees you because of an increase in fibromyalgia pain. On


examination she has a BMI of 35.6 kg/m2 and normal vital signs except for a blood pressure
of 156/91 mm Hg. Her other medical problems include obstructive sleep apnea, type 2
diabetes mellitus, hypertension, and generalized anxiety disorder. She smokes one pack of
cigarettes daily and does not drink alcohol. She is currently taking metformin (Glucophage),
500 mg twice daily; lisinopril (Prinivil, Zestril), 10 mg daily; gabapentin (Neurontin), 300
mg 3 times daily; oxycodone (OxyContin), 10 mg every 6 hours; and lorazepam (Ativan), 1
mg 3 times daily. ?Which one of the following findings in this patient's history greatly
increases her risk of an accidental overdose

A) Tobacco use

B) Morbid obesity

C) Use of oxycodone

D) Use of oxycodone and lorazepam

E) Use of lorazepam and gabapentin

ANSWER: D

The increase in opiate-related accidental overdoses has become a significant concern in


recent years, prompting the CDC to release updated guidelines for the use of narcotic
medications for chronic noncancer pain. There are several concerning issues in this
patient's care. Her obstructive sleep apnea, psychiatric ailments, and concurrent use of
opiates and benzodiazepines all increase the risk of an accidental overdose. The CDC also
warns against using opiates in patients with heart failure, chronic pulmonary diseases, and
.a personal history of drug or alcohol abuse

These risks are so great that the CDC recommends that chronic noncancer pain be primarily
treated with nonpharmacologic and nonopiate medications. The use of opioids should be
reserved for recalcitrant cases under close supervision at the lowest effective dose for the
shortest time possible. The CDC also recommends against using opiates in fibromyalgia and
neuropathy due to limited efficacy and side-effect profiles (SOR B). The concurrent use of
opiates and benzodiazepines should be avoided in nearly all situations (SOR C). Safety
.should never be compromised for reduced pain and increased functionality

Ref: Hudson S, Wimsatt LA: How to monitor opioid use for your patients with chronic pain. Fam Pract Manag
2014;21(6):6-11. 2) Raleigh MF, Dunn AM: Controlled-release oxycodone for neuropathic pain and fibromyalgia
in adults. Am Fam Physician 2015;91(5):286-287. 3) Bredemeyer M: CDC develops guideline for opioid
prescribing. Am Fam Physician 2016;93(12):1042-1043. 4) Wingrove P, Park B, Bazemore A: Rural opioid use
disorder treatment depends on family physicians. Am Fam Physician 2016;94(7):546. 5) Lembke A, Humphreys
K, Newmark J: Weighing the risks and benefits of chronic opioid therapy. Am Fam Physician 2016;93(12):982-
.990

51 - A 34-year-old female presents with a 3-month history of a minimally


productive cough. She has never smoked. She does not have any fever, weight loss,
rhinorrhea, congestion, or heartburn. She does not have a known history of allergies or
asthma and has tried over-the-counter cold remedies, cough syrups, and cough drops
without significant relief. She is otherwise healthy and takes no medications. On
examination her vital signs are normal. An ear, nose, and throat examination is remarkable
for swollen nasal turbinates. A lung examination is normal. Given the duration of the cough,
you order a chest radiograph, which is normal as well. ?Which one of the following would be
most appropriate at this point

A) A trial of an intranasal corticosteroid

B) A trial of an inhaled bronchodilator

C) A trial of a proton pump inhibitor

D) A sinus radiograph

E) Referral for allergy testing

ANSWER: A

According to the CDC, cough is the most common symptom resulting in primary care visits.
Chronic cough in adults is defined as one that lasts 8 weeks or more. The workup should
include a history focusing on potential triggers, as well as the identification of any red flags.
If the physical examination is normal and the patient's history does not indicate the cause of
.the cough, a chest radiograph is appropriate

The most common cause of chronic cough in adults is upper airway cough syndrome.
Patients might have nasal symptoms such as rhinorrhea or congestion. Physical findings can
include swollen turbinates and posterior pharyngeal cobblestoning, or they can be
unremarkable. Initial treatment may include the use of decongestants, oral or intranasal
antihistamines, intranasal corticosteroids, or saline nasal rinses (SOR C). Symptoms should
resolve within a few weeks, and referral for allergy testing can be considered if they are not
resolved within 2 months. CT of the sinuses can be considered as well, but sinus
.radiographs are more specific

Other common causes of chronic cough include asthma, nonasthmatic eosinophilic


bronchitis, and GERD. If asthma is suspected, spirometry is indicated. If spirometry is
positive for asthma, a trial of an inhaled bronchodilator is indicated. If there are other
indications of GERD such as heartburn, globus sensation, or hoarseness, an antacid or a trial
. .of a proton pump inhibitor is indicated
52 - A 68-year-old female presents with a history of episodic severe lower
abdominal pain relieved by defecation. She has had a long history of constipation with
normal to very firm stools. Her history and a physical examination are otherwise normal. A
colonoscopy 3 years ago was normal. You diagnose constipation-predominant irritable
bowel syndrome. ?Which one of the following agents would be the most appropriate
treatment for this patient

A) Lactulose

B) Magnesium citrate

C) Milk of magnesia

D) Polyethylene glycol

E) Sodium phosphate

ANSWER: D

Hypertonic osmotic laxatives such as milk of magnesia, magnesium citrate, and sodium
phosphate draw water into the bowel and should be used with caution in older adults and
those with renal impairment because of the risk of electrolyte abnormalities and
dehydration in patients with irritable bowel syndrome (IBS). Lactulose, also an osmotic
laxative, should be avoided in patients with IBS because it is broken down by colonic flora
and produces excessive gas. Polyethylene glycol, a long-chain polymer of ethylene oxide, is a
large molecule that causes water to be retained in the colon, which softens the stool and
increases the number of bowel movements. It is approved by the FDA for short-term
treatment in adults and children with occasional constipation and is commonly prescribed
for patients with IBS. It is considered safe and effective for moderate to severe constipation
.when used either daily or as needed

.Ref: Sultan S, Malhotra A: Irritable bowel syndrome. Ann Intern Med 2017;166(11):ITC81-ITC96

53 - A 48-year-old male presents with pain in the right antecubital fossa after lifting
a trailer in his garage. On examination you note ecchymosis and tenderness in the
antecubital fossa. You suspect a possible distal biceps tendon rupture. ?Which one of the
following would be most appropriate at this point

A) A Speed's test

B) Plain radiographs of the elbow

C) MRI of the elbow

D) A local corticosteroid injection

E) Referral for physical therapy


ANSWER: C

Distal biceps tendon ruptures are relatively uncommon, accounting for about 3% of tendon
ruptures. In a patient with a suspected distal biceps tendon rupture, clinical signs can be
unreliable and MRI imaging is the test of choice. Bony abnormalities do not contribute to
the evaluation of this tendon. A Speed's test is used to evaluate pain related to the long head
of the biceps tendon. Surgical repair is the treatment of choice when the tendon is ruptured.
.Physical therapy and local corticosteroid injections are not beneficial

Ref: Churgay CA: Diagnosis and treatment of biceps tendinitis and tendinosis. Am Fam Physician
2009;80(5):470-476. 2) Kane SF, Lynch JH, Taylor JC: Evaluation of elbow pain in adults. Am Fam Physician
.2014;89(8):649-657

54 - A 72-year-old male with type 2 diabetes mellitus sees you for routine follow-
up. He takes metformin (Glucophage), 1000 mg twice daily. He is sedentary and does not
adhere to his diet. His BMI is 32.0 kg/m2. The examination is otherwise within normal
limits. His hemoglobin A1c is 9.5%. ?Which one of the following is recommended by the
American Diabetes Association to better control his blood glucose

A) Start an intensive diet and exercise program for weight loss

B) Start home monitoring of blood glucose with close follow-up

C) Start basal insulin at 10 units/day

D) Stop metformin and start a sulfonylurea

E) Stop metformin and start a basal and bolus insulin regimen

ANSWER: C

According to the American Diabetes Association's 2018 guidelines for the management of
diabetes, a healthy person with a reasonable life expectancy should have a hemoglobin A1c
goal of <7%. Metformin is recommended as first-line therapy as long as there are no
contraindications. If the hemoglobin A1c is not at the goal or is ?9%, then adding another
agent to metformin is recommended. Basal insulin at 10 units/day is an acceptable choice
for additional therapy to improve blood glucose control. Diet, exercise, and home
monitoring of blood glucose are recommended in addition to starting another agent for
.blood glucose control

Ref: American Diabetes Association: 6. Glycemic targets: Standards of medical care in diabetes-2018. Diabetes
Care 2018;41(Suppl 1):S55-S64. 2) American Diabetes Association: 8. Pharmacologic approaches to glycemic
.treatment: Standards of medical care in diabetes-2018. Diabetes Care 2018;41(Suppl 1):S73-S85
55 - A 47-year-old male presents with bilateral lower extremity edema of
undetermined etiology extending to the proximal lower extremities, associated with fatigue.
His lipid levels were also very high on recent testing. He does not take any daily medications
and his thyroid function is normal. The only significant findings on examination are lower
extremity edema and some periorbital edema. ?Which one of the following urine tests could
help confirm the most likely diagnosis

A) Crystals

B) Ketones

C) pH

D) Protein

E) Specific gravity

ANSWER: D

Nephrotic syndrome includes peripheral edema, heavy proteinuria, and hypoalbuminemia.


Hyperlipidemia also occurs frequently and can be significant. Nephrotic-range proteinuria
is a spot urine showing a protein/creatinine ratio >3.0-3.5 mg protein/mg creatinine or a
24-hour urine collection showing >3.0-3.5 g of protein. Testing urine for ketones, pH,
.specific gravity, or crystals does not help to diagnose nephrotic syndrome

.Ref: Kodner C: Diagnosis and management of nephrotic syndrome in adults. Am Fam Physician 2016;93(6):479

56 - You are notified by the nurse that a 66-year-old female who was admitted for
pain control for her bone metastases is still having breakthrough pain. You gave her 10 mg
of immediate-release oxycodone (Roxicodone) 15 minutes ago. ?You are hoping to optimize
pain control and minimize sedation, so you advise the nurse that the last dose will have its
peak effect

A) now

B) 1 hour after it was given

C) 2 hours after it was given

D) 4 hours after it was given

ANSWER: B

Most orally administered immediate-release opioids such as morphine, oxycodone, and


hydromorphone reach their peak effect at about 1 hour, at which time additional
medication can be given if the patient is still in pain. Intravenous opioids reach their peak
effect at about 10 minutes and intramuscular and subcutaneous opioids at about 20-30
minutes. Additional medication may therefore be given at those intervals if additional pain
.relief is required

Ref: Groninger H, Vijayan J: Pharmacologic management of pain at the end of life. Am Fam Physician
.2014;90(1):26-32

57 - A 62-year-old female has a history of COPD graded as moderate on pulmonary


function testing, with an FEV1 of 65% of predicted and a PaO2 of 57 mm Hg. Because her
symptoms of dyspnea on exertion and fatigue seem out of proportion to her pulmonary
function tests, you order echocardiography, which shows a pulmonary artery systolic
pressure of 50 mm Hg, indicating pulmonary hypertension. ?Which one of the following
would be most effective for decreasing mortality in this situation

A) Supplemental oxygen

)B) An endothelin receptor antagonist such as bosentan (Tracleer

)C) A calcium channel blocker such as nifedipine (Procardia

)D) A phosphodiesterase 5 inhibitor such as sildenafil (Revatio

E) Referral for pulmonary artery endarterectomy

ANSWER: A

The only proven therapy for pulmonary hypertension related to COPD is supplemental
oxygen. Supplemental oxygen should be recommended when the PaO2 is <60 mm Hg,
because it has been shown to improve mortality by lowering pulmonary arterial pressures.
Treatments effective for pulmonary artery hypertension should not be used. Pulmonary
vasodilators such as nifedipine, sildenafil, and bosentan may cause a ventilation-perfusion
mismatch. Pulmonary endarterectomy may be indicated for pulmonary hypertension
.caused by chronic thromboembolic disease

Ref: Mandel J, Poch D: Pulmonary hypertension. Ann Intern Med 2013;158(9):ITC5-1-ITC5-16. 2) Dunlap B,
.Weyer G: Pulmonary hypertension: Diagnosis and treatment. Am Fam Physician 2016;94(6):463-469

58 - Which one of the following antihypertensive drugs may reduce the severity of
sleep apnea

)A) Amlodipine (Norvasc

B) Hydralazine

)C) Lisinopril (Prinivil, Zestril

D) Metoprolol

)E) Spironolactone (Aldactone


ANSWER: E

Diuretics lessen the severity of obstructive sleep apnea and reduce blood pressure.
Aldosterone antagonists offer further benefit beyond that of traditional diuretics. Resistant
hypertension is common in patients with obstructive sleep apnea. Resistant hypertension is
also associated with higher levels of aldosterone, which can lead to secondary pharyngeal
.edema, increasing upper airway obstruction

Ref: Torres G, S?nchez-de-la-Torre M, Barbé F: Relationship between OSA and hypertension. Chest
.2015;148(3):824-832

59 - The U.S. Preventive Services Task Force recommends screening all adults for
obesity and offering intensive, multicomponent behavioral interventions to patients with a
BMI 30 kg/m2. This recommendation is based on trials that show that behavioral weight-
loss interventions for overweight and obese patients with elevated plasma glucose levels
reduce the incidence of diabetes mellitus by 30%-50% over 2-3 years and the number
needed to treat is 7. ?What is the absolute risk reduction for developing diabetes, based on
these trials

A) 1/7

B) 1/5

C) 1/0.7

D) 1/0.2

E) 1/0.02

ANSWER: A

The number needed to treat (NNT) is defined as the number of people who would need to
receive an intervention in order for one person to benefit. It is the inverse of the absolute
risk reduction (ARR). The ARR is the difference in risk for a disease without and with an
.intervention. The correct formula for calculating NNT is 1/ARR

Ref: Final Recommendation Statement: Obesity in Adults: Screening and Management. US Preventive Services
.Task Force, 2016. 2) EBM glossaries. American Family Physician website, 2018

60 - A 25-year-old female sees you because of irregular menses, hirsutism, and


moderate acne. She is sexually active in a monogamous relationship with a male, has never
been pregnant, and prefers not to become pregnant at this time. ?Which one of the following
is considered first-line therapy

)A) Clomiphene (Clomid

)B) Letrozole (Femara


C) Levonorgestrel/ethinyl estradiol

)D) Metformin (Glucophage

)E) Spironolactone (Aldactone

ANSWER: C

The Endocrine Society recommends hormonal contraception as the first-line medication for
women diagnosed with polycystic ovary syndrome (PCOS) who are experiencing irregular
menses, acne, and hirsutism and do not desire pregnancy (SOR A). Metformin may help
regulate menses but has not been shown to be as effective as oral hormone therapy. In a
2015 Cochrane review, oral contraceptives were recommended as the most effective
treatment for hirsutism. Either letrozole or clomiphene is appropriate for women diagnosed
.with PCOS who want to become pregnant

Ref: Williams T, Mortada R, Porter S: Diagnosis and treatment of polycystic ovary syndrome. Am Fam Physician
.2016;94(2):106-113

61. A 68-year-old male presents with chronic right knee pain from osteoarthritis that
inhibits his activity and is associated with stiffness throughout the day. He has tried
acetaminophen and NSAIDs with limited effect. He has consulted an integrative medicine
specialist who recommended multiple modalities to reduce pain and increase function, and
he asks whether you think they would be helpful.

Which one of the following measures recommended by the other physician has the
STRONGEST evidence of benefit?

A) A low-impact aerobic exercise program

B) Lateral wedge insoles

C) Oral glucosamine and chondroitin

D) A platelet-rich plasma injection

E) Needle lavage of the knee

ANSWER: A

Despite the prevalence of osteoarthritis of the knee and a myriad of treatment modalities
available for those with symptomatic disease, there is very limited evidence to suggest that
many of these treatments are effective. There is strong evidence to suggest that self-
management programs, strengthening exercises, low-impact aerobic exercises, and
neuromuscular education have some benefit. Moderate evidence recommends against the
use of needle lavage of the knee; the two main studies of this modality showed little or no
benefit. In 15 studies, 14 outcomes were not statistically significant, including three pain
and three functional outcomes. There is also moderate evidence to recommend against the
use of lateral wedge insoles. Four studies of lateral wedge insoles showed no significant
change in pain or function of the knee when compared to neutral insoles. The evidence is
inconclusive for platelet-rich plasma injections. A few studies have shown decreased pain in
patients after injection, but there was no placebo control, so the effectiveness cannot be
adequately assessed. Glucosamine and chondroitin have been shown with strong evidence
to be ineffective when compared to placebo.

62. A 52-year-old female nurse sees you for the first time. She was previously a patient of a
recently retired physician in your practice. Her history is significant for a Roux-en-Y gastric
bypass, degenerative joint disease of both knees and shoulders, and chronic low back pain.
She takes oxycodone (Roxicodone), 5–10 mg every 4 hours. She tells you that she has been
taking this for almost 10 years as treatment for various pains. She says that acetaminophen
just “does not touch the pain” and that physical therapy has not worked. She asks you to
continue this medication.

Which one of the following would be the most appropriate management of this patient?

A) Add an NSAID to the current regimen

B) Initiate weekly urine drug screens

C) Taper oxycodone by 5%–10% every 1–4 weeks

D) Discontinue oxycodone

ANSWER: C

According to the Choosing Wisely recommendations from the American Society of


Anesthesiologists, opioids should not be used as first-line therapy for chronic noncancer
pain. However, more than one-half of patients who receive continuous opioids for 90 days
are still receiving them after 4 years. Chronic opioids should not be abruptly discontinued.
When discontinuing chronic opioid therapy, the best practice is to reduce the dosage by
5%–10% every 1–4 weeks, but even this may be too fast for some patients.

While controlled substance prescribing plans are considered good practice for long-term
opioid use, continuing opioids for this patient would not be good practice given the
indication of chronic noncancer pain and the need for safety in her work. Because her use of
opioids should be tapered, weekly urine drug screens would continue to be positive and
therefore would not be an appropriate management strategy for this patient. NSAIDs are
not indicated for this patient due to her history of gastric bypass.
63. A 13-year-old male sees you because of pain in his throwing arm. He is a very dedicated
football quarterback and has been practicing throws and playing games every day for 2
months. The pain started gradually over the season, and there is no history of acute injury.
The patient is right-hand dominant, and on examination he has pain when he raises his right
arm above his shoulder. There is also tenderness to palpation of the proximal and lateral
humerus.

Which one of the following would be most appropriate at this point?

A) Injection of 10 mL of lidocaine into the subacromial space B) Plain radiographs of the


shoulder

C) Ultrasonography of the supraspinatus muscle

D) MRI of the shoulder

E) A bone scan of the shoulder

ANSWER: B

Pain in the shoulder of a young athlete can be caused by many problems, including
acromioclavicular strain, biceps tendinitis, glenohumeral instability, and rotator cuff
pathology. Although rotator cuff pathologies are the most frequent cause of shoulder pain in
adults, they are uncommon in children. Unique to children, however, is a repetitive use
injury causing disruption at the proximal growth plate of the humerus. This condition is
referred to as Little League shoulder and can be seen on plain radiographs as widening,
demineralization, or sclerosis at the growth plate. If the radiograph is normal but suspicion
for this condition is high, a bone scan or MRI can be ordered.

64. A 30-year-old female presents with an episode of recurrent, painful vesicular lesions on
the labia. She noted a tingling, burning sensation a few days before the lesions appeared. A
few years ago she had a similar outbreak just before the birth of her second child.

Which one of the following is indicated for this patient?

A) Doxycycline

B) Fluconazole (Diflucan)

C) Metronidazole

D) Penicillin G benzathine (Bicillin L-A) E) Valacyclovir (Valtrex)

ANSWER: E

This patient has a recurrent outbreak of genital herpes, and valacyclovir is the preferred
treatment. Penicillin G benzathine is a treatment for syphilis, which usually begins as a
painless papule that transforms into the classic chancre. Fluconazole and metronidazole are
treatments for yeast vaginitis and bacterial vaginitis; these conditions present with itching
and a vaginal discharge but not vesicular lesions.

Doxycycline is a treatment for Chlamydia infection, which is often completely asymptomatic


and detected only with screening

65. A 62-year-old Asian female presents to your office with pain and redness in her left eye
that started last night. She does not wear contact lenses. The pain has become more severe
and she now has a headache, light sensitivity, and mild nausea. Examination of the eyes
reveals diffuse conjunctival injection on the left. Her pupils are 4 mm bilaterally but the left
one reacts poorly to light. Her visual acuity is 20/30 on the right and 20/100 on the left.

Which one of the following would be most appropriate at this time?

A) Polymyxin B/trimethoprim ophthalmic drops (Polytrim)

B) Prednisolone ophthalmic drops (Omnipred)

C) An erythrocyte sedimentation rate and C-reactive protein level

D) MRI of the brain with contrast

E) Emergent evaluation by an ophthalmologist

ANSWER: E

This patient has symptoms and examination findings that are concerning for acute angle-
closure glaucoma. Her risk factors include her age, sex, and Asian ancestry. The examination
findings include conjunctival redness, corneal edema, a poorly reactive mid-dilated pupil,
decreased vision, severe eye pain, headache, and nausea. This condition needs to be
evaluated and treated emergently to preserve vision. The examination is not consistent with
infectious conjunctivitis, which generally does not cause severe pain, headache, or
decreased pupillary response. Conditions such as scleritis or episcleritis may present with
similar features, but the pupillary response may help differentiate them from glaucoma.
Referral to an ophthalmologist would still be most prudent. This patient’s presentation is
not consistent with a vasculitis or multiple sclerosis.

66. A 24-year-old female with a history of bulimia nervosa sees you for treatment of
depression. She is currently receiving cognitive-behavioral therapy. You decide that she
requires medication to treat her depression.

Which one of the following medications has been associated with an increased risk of
seizures in patients with bulimia nervosa?
A) Bupropion (Wellbutrin)

B) Fluoxetine (Prozac)

C) Nortriptyline (Pamelor)

D) Sertraline (Zoloft)

E) Venlafaxine (Effexor XR)

ANSWER: A

Antidepressants in every class (SSRIs, SNRIs, tricyclic antidepressants, and monoamine


oxidase inhibitors) have been shown to reduce bulimic symptoms and can be used safely to
treat depression, with the exception of bupropion. Bupropion use has been associated with
an increased risk of seizures in patients with bulimia and an FDA warning limits its use.

67. At a routine well child visit the mother of a 3-year-old male expresses concern that his
toes turn in, causing a clumsy gait when he walks. You diagnose internal tibial torsion,
because his feet point inward when his patellae face forward. The examination is otherwise
normal.

Which one of the following is recommended at this time?

A) No intervention

B) Shoe modification with wedges to externally rotate the feet while walking C) Night
splinting with the feet externally rotated

D) Serial casting to gradually externally rotate the feet

E) Surgery to correct the deformity

ANSWER: A

Internal tibial torsion usually resolves spontaneously by age 5. Surgery may be considered
in patients older than 8 years of age who have a severe residual deformity, especially if it is
symptomatic or cosmetically unacceptable. Night splints, shoe modifications, other
orthotics, casting, and braces are not recommended for this condition.

68. A 24-year-old female seeks your advice regarding the recent onset of a cough when
running. She moved to the United States from Mexico last year and her symptoms first
became apparent during her first winter in the Midwest. The cough starts after she has been
running approximately 1 mile but no sputum is produced and no other symptoms occur.
She has no other health concerns.
A physical examination and office spirometry are consistent with a healthy young adult. You
ask her to run around the outside of the clinic several times and then you reexamine her.
The only change noted is an increase in her pulse rate and a 10% drop in her FEV1.

Which one of the following would be the most appropriate initial treatment for this patient?

A) An endurance conditioning program

B) An over-the-counter antihistamine as needed

C) An inhaled corticosteroid 2 hours before running

D) An inhaled short-acting 2-agonist 15 minutes before running E) Daily use of an inhaled


long-acting 2-agonist

ANSWER: D

This patient’s history and examination findings are typical for exercise-induced asthma. The
most appropriate initial treatment for this condition is an inhaled short-acting 2-agonist
(SABA) 15 minutes before exercise (SOR A). Daily use of an inhaled long-acting 2-agonist as
a single agent is not recommended even for those who continue to experience symptoms
when using an inhaled SABA (SOR B). The addition of a daily inhaled corticosteroid is an
appropriate consideration for patients who require more than a SABA to control symptoms
but these should not be used on an as-needed basis before exercise (SOR B). Use of an
antihistamine in an individual with exercise-induced asthma but no known allergies is not
recommended (SOR B). Other treatment considerations with weak recommendations
include a low-sodium diet, air humidification, and supplemental dietary fish oils.

69. Which one of the following malignancies is associated with hereditary


hemochromatosis?

A) Biliary carcinoma B) Chronic myeloid leukemia C) Hepatocellular carcinoma

D) Multiple myeloma

E) Pancreatic cancer

ANSWER: C

Hereditary hemochromatosis is a genetic disorder of iron regulation and subsequent iron


overload. Possible end-organ damage includes cardiomyopathy, cirrhosis of the liver, and
hepatocellular carcinoma. Symptoms are often nonspecific early on, but manifestations of
iron overload eventually occur. The diagnosis should be suspected in patients with liver
disease or abnormal iron studies indicative of iron overload. A liver biopsy can confirm the
diagnosis and the degree of fibrosis. Identification of such patients and proper ongoing
treatment with phlebotomy may prevent the development of hepatocellular carcinoma and
other complications of this disease. There is some data that suggests an association of
breast cancer with hereditary hemochromatosis but not with any of the other malignancies
listed.

70. You admit a previously healthy 62-year-old female to the hospital for intractable nausea
and vomiting with intravascular volume depletion and hypotension. She lives in rural
northern New Mexico. Prior to the onset of her symptoms she had been gardening and
cleaning out a chicken coop, where she encountered several rodents. She is febrile and you
obtain blood and urine cultures. Two out of four blood culture bottles are positive for gram-
negative rods.

Which one of the following is the most likely pathogen?

A) Brucella melitensis

B)Coxiella burnetii

C)Escherichia coli

D)Listeria monocytogenes

E)Yersinia pestis

ANSWER: E

Yersinia pestis is an aerobic fermentative gram-negative rod. It causes a zoonotic infection


with humans as the accidental host. The disease is spread by a bite from a flea vector, direct
contact with infected tissue, or inhalation of infectious aerosols from a person with
pulmonary plague. Plague occurs in two regions in the western United States. One region
includes northern New Mexico, northern Arizona, and southern Colorado, and the other
region includes California, southern Oregon, and far western Nevada.

Escherichia coli is also an aerobic fermentative gram-negative rod but it generally causes
symptoms of gastroenteritis, hemolytic-uremic syndrome, urinary tract infection, intra-
abdominal infection, and meningitis.

E. coli infection does not have a specific regional distribution.

Listeria monocytogenes is a gram-positive rod and causes an influenza-like illness with or


without gastroenteritis in adults. Infection occurs through ingestion of contaminated food
products such as milk, cheese, processed meats, and raw vegetables. Outbreaks can occur in
any geographic distribution.

Coxiella burnetii is a gram-negative intracellular bacterium that causes Q fever. Human


infections are associated with contact with infected cattle, sheep, goats, dogs, and cats.
Brucella melitensis is agram-negative coccobacilli that causes brucellosis. Humans are
accidental hosts who can develop the disease from contact with tissues rich in erythritol,
and from shedding of organisms in milk, urine, and birth products from goats and sheep.

71. A 21-year-old female is being evaluated for secondary causes of refractory hypertension.
Which one of the following would be most specific for fibromuscular dysplasia?

A) A serum creatinine level

B) An aldosterone:renin ratio

C) 24-hour urine for metanephrines

D) Renal ultrasonography

E) Magnetic resonance angiography of the renal arteries

ANSWER: E

In young adults diagnosed with secondary hypertension, evaluation for fibromuscular


dysplasia of the renal arteries with MR angiography or CT angiography is indicated (SOR C).
The aldosterone/renin ratio is the most sensitive test to diagnose primary
hyperaldosteronism. Renal ultrasonography is an indirect test that is not as sensitive or
specific for fibromuscular dysplasia. Serum creatinine elevation shows renal involvement
but does not identify the cause. Testing for metanephrines is indicated only if a
pheochromocytoma is suspected.

72. Of the following, which one is the greatest risk factor for developing knee osteoarthritis
as an older adult?

A) A sedentary lifestyle

B) Cigarette smoking

C) Low socioeconomic status

D) Male sex

E) Obesity

ANSWER: E

Because debilitating knee osteoarthritis is a frequent health concern in older adults,


physicians should try to identify and possibly modify factors that increase the risk for this
condition. Pooled data from many large studies has been sufficient to clearly identify
several major risk factors for the development and progression of osteoarthritis of the
knees. Overweight and obesity have consistently been found to approximately double the
risk for developing knee osteoarthritis. Other factors that have been identified as risk
factors include female sex, advancing age (50–75 years of age), and previous trauma.
Smoking, inactivity, moderate physical activity, and socioeconomic status have not been
shown to affect one’s risk for developing knee osteoarthritis. However, any of these factors
in the extreme may be detrimental to joint health in general.

73. A staff member at a local assisted living facility calls you about an 88-year-old female
who has chronic urinary incontinence and well controlled hypertension. A urinalysis was
obtained after the patient reported some dizziness and malaise. She does not have dysuria
and has had no change to her incontinence. The patient is afebrile and other vital signs are
normal. The

urine culture reveals >100,000 colony-forming units of Escherichia coli, with sensitivities
pending. In addition to supportive care and hydration, which one of the following would be
indicated at this time?

A) Ciprofloxacin (Cipro)

B) Fosfomycin (Monurol)

C) Nitrofurantoin (Macrodantin)

D) Trimethoprim/sulfamethoxazole (Bactrim) E) No antibiotics

ANSWER: E

This patient has asymptomatic bacteriuria and does not require antibiotic therapy at this
time. In women age 70 and older the incidence of asymptomatic bacteriuria is 16%–18%,
and in chronically incontinent and disabled older adults rates may reach 43%. Symptoms
that raise concern for a urinary tract infection (UTI) include acute dysuria, new or
worsening urinary urgency or frequency, new incontinence, gross hematuria, and
suprapubic or costovertebral angle tenderness. General malaise in the absence of these
symptoms is unlikely to represent a UTI and unlikely to improve with antibiotic therapy.

When antibiotic therapy is indicated for a UTI, trimethoprim/sulfamethoxazole remains the


first-line agent. Nitrofurantoin may be used for those with a creatinine clearance >40
mL/min/1.73 m2. Ciprofloxacin is recommended as a first-line agent only in communities
with trimethoprim/sulfamethoxazole resistance rates above 10%–20%. Fosfomycin may be
used for more highly resistant organisms. The choice of antibiotic should be guided by
bacterial pathogens if they are known.
74. A 61-year-old white male with type 2 diabetes mellitus sees you for a follow-up visit. His
blood pressure is 156/94 mm Hg. At a visit 1 week ago his blood pressure was 150/92 mm
Hg. Laboratory studies obtained prior to this visit show a BUN of 16 mg/dL (N 6–20), a
serum creatinine level of 0.9 mg/dL (N 0.7–1.3), and microalbuminuria on a urinalysis. His
diabetes is well controlled with metformin (Glucophage) and he is taking aspirin.

Which one of the following would you recommend?

A) Observation only

B) An ACE inhibitor

C) A -blocker

D) A calcium channel blocker E) A diuretic

ANSWER: B

The panel members of the Eighth Joint National Committee for the management of blood
pressure recommended that ACE inhibitors should be initiated for renal protection in adults
with diabetes mellitus, hypertension, and microalbuminuria. This patient appears to be in
an early stage of nephropathy, and ACE inhibitors will reduce the decline in renal function. -
Blockers are no longer recommended for first-line treatment. In white patients who do not
have diabetes, therapy may be started with ACE inhibitors, thiazide diuretics, or calcium
channel blockers.

75. A 66-year-old female with a previous history of hypertension, stable angina, and carotid
endarterectomy presents with acute upper abdominal pain, which has developed over the
past 3 hours. A physical examination reveals epigastric tenderness without guarding or
rebound, but does not reveal a cause for the level of pain reported by the patient. Initial
laboratory findings are within normal limits, including a CBC, glucose, lactic acid, amylase,
lipase, liver enzymes, and kidney function tests. You suspect acute mesenteric ischemia.

Which one of the following diagnostic imaging tests is the preferred initial evaluation for
this problem?

A) Duplex ultrasonography

B) CT angiography

C) Catheter angiography

D) Magnetic resonance angiography (MRA) E) Upper and lower GI endoscopy


ANSWER: B

CT angiography (CTA) is the recommended imaging procedure for the diagnosis of acute
mesenteric vascular disease. The procedure can also identify other possible intra-
abdominal causes of pain. Duplex ultrasonography is also accurate, especially for proximal
lesions, but can be difficult to perform in patients with obesity, bowel gas, and marked
calcification of the vessels, and may be problematic in patients presenting acutely, due to
the length of the study and the abdominal pressure required. It is more useful in cases of
suspected chronic mesenteric ischemia. Endoscopy is often normal in acute ischemia and
may not reach the ischemic section of bowel. MR angiography may be useful, but it takes
longer to perform than CTA and lacks the necessary resolution. Catheter angiography is
required for endovascular therapies such as thrombolysis or angioplasty with or without
stenting, but is usually not performed for making the initial diagnosis in the acute setting.

76. A 38-year-old patient wishes to start contraception. She currently takes lisinopril
(Prinivil, Zestril) for hypertension and also takes sumatriptan (Imitrex) occasionally for
migraines at the first sign of flashing lights or zigzagging lines in her vision. Her medical,
family, and social histories are otherwise unremarkable. An examination is notable only for
a blood pressure of 130/80 mm Hg and a BMI of 36.0 kg/m2.

The patient is interested in using either the vaginal ring or the contraceptive patch. Which
one of the following would you recommend?

A) Transdermal norelgestromin/ethinyl estradiol (Ortho Evra) B) The etonogestrel/ethinyl


estradiol vaginal ring (NuvaRing)

C) Neither method due to her migraines

D) Neither method due to her age

E) Losing weight before starting either method

ANSWER: C

Family physicians are often asked to provide contraception and need to be familiar with the
current methods and contraindications. Estrogen-containing products, including the
contraceptive patch and the vaginal ring, are contraindicated in smokers >35 years of age
and in patients with migraine with aura.

77. A 45-year-old male sees you for follow-up of several chronic medical problems including
hypertension, diabetes mellitus, and obesity. He is a truck driver, smokes one pack of
cigarettes per day, and does not exercise. His blood pressure is 166/94 mm Hg and his
hemoglobin A1c is 9.7%. His medical conditions have been difficult to control with
medications and he has been resistant to making lifestyle changes.

Which one of the following strategies would be most effective for inducing significant
behavioral change?

A) Counsel the patient on the complications of smoking and uncontrolled diabetes

B) Utilize motivational interviewing to explore the patient’s level of desire to change C)


Treat the patient with an SSRI and refer him to a counselor

D) Transfer the patient to another family physician in your community

ANSWER: B

Patients who are resistant to change require skillful management. Motivational


interviewing is a technique that has been shown to improve the therapeutic physician-
patient alliance and help to engage patients in their own care. The other options listed are
not helpful and may damage the therapeutic relationship.

78. A 47-year-old male who lives at sea level attempts to climb Mt. Rainier. On the first day
he ascends to 3400 m (11,000 ft). The next morning he has a headache, nausea, dizziness,
and fatigue, but as he continues the climb to the summit he becomes ataxic and confused.

Which one of the following is the treatment of choice?

A) Administration of oxygen and immediate descent B) Dexamethasone, 8 mg


intramuscularly C) Acetazolamide, 250 mg twice a day

D) Nifedipine (Procardia), 10 mg immediately, followed by 30 mg in 12 hours E) Helicopter


delivery of a portable hyperbaric chamber

ANSWER: A

This patient initially showed signs of acute mountain sickness. These include headache in an
unacclimated person who recently arrived at an elevation >2500 m (8200 ft), plus one or
more of the following: anorexia, nausea, vomiting, insomnia, dizziness, or fatigue. The
patient’s condition then deteriorated to high-altitude cerebral edema, defined as the onset
of ataxia and/or altered consciousness in someone with acute mountain sickness. The
management of choice is a combination of descent and supplemental oxygen. Often, a
descent of only 500–1000 m (1600–3300 ft) will lead to resolution of acute mountain
sickness. Simulated descent with a portable hyperbaric chamber also is effective, but
descent should not be delayed while awaiting helicopter delivery. If descent and/or
administration of oxygen is not possible, medical therapy with dexamethasone and/or
acetazolamide may reduce the severity of symptoms. Nifedipine has also been shown to be
helpful in cases of high-altitude pulmonary edema where descent and/or supplemental
oxygen is unavailable.

79. A 60-year-old male presents with the lesion shown below. It has grown over the last few
months. His past medical history includes well controlled hypertension. He takes lisinopril
(Prinivil, Zestril), 10 mg daily, and aspirin, 81 mg daily.

After the diagnosis is established with a biopsy, which one of the following has the highest
cure rate for this problem?

A) Standard wide excision

B) Electrodesiccation and curettage

C) Mohs surgery

D) Photodynamic therapy

E) Radiation therapy

ANSWER: C

This patient most likely has a basal cell carcinoma, which can be proven by a shave biopsy.
Given its size and location, Mohs surgery would be the preferred treatment. It also has the
highest cure rate of any of the options listed, including a standard wide excision,
electrodesiccation and curettage, photodynamic therapy, and radiation therapy. It has a
99% cure rate for primary basal cell cancers, compared with just over 91% for other
methods. Photodynamic therapy and radiation therapy should be used for lesions such as
this only if surgery is not an option due to medical comorbidities and/or patient preference.

80. A 55-year-old female sees you for a preoperative evaluation prior to having cataract
surgery. The patient has a previous history of type 1 diabetes mellitus. She reports that she
takes a brisk daily walk and has no angina or other cardiac symptoms. The cardiovascular
and pulmonary examinations are unremarkable.

Which one of the following would be most appropriate for the preoperative cardiac
evaluation of this patient?

A) No further evaluation

B) An EKG

C) A treadmill stress test

D) Pharmacologic stress testing E) A chest radiograph


ANSWER: A

This 55-year-old patient is undergoing a low-risk procedure. While her diabetes mellitus is
a cardiovascular risk factor, she is asymptomatic, her age lowers her risk, and her functional
status is good. She should be allowed to undergo cataract surgery with no further
evaluation. Guidelines from the American College of Cardiology and the American Heart
Association recommend that the patient be allowed to undergo surgery with no further
testing.

81. The novel anticoagulants (NOACs) include apixaban (Eliquis), dabigatran (Pradaxa),
edoxaban (Savaysa), and rivaroxaban (Xarelto). Which one of the following should be
considered when starting or adjusting the dosage of a NOAC?
A) Serum albumin
B) INR
C) Liver enzymes
D) Partial thromboplastin time
E) Renal function
ANSWER: E
The novel anticoagulants (NOACs) require dosage adjustments based on renal function.
There are no dosing recommendations for NOACs based on liver function or albumin
level. The INR is used to adjust warfarin dosing and the partial thromboplastin time is
used to adjust heparin dosing.

82. A 42-year-old male with alcohol use disorder tells you that his last drink was 7 days
ago and asks if there are any medications available to help him maintain abstinence
from alcohol. He has no other medical or psychological problems.
Which one of the following pharmacologic agents could help reduce this patient’s alcohol
consumption and increase abstinence?
A) Acamprosate
B) Amitriptyline
C) Paroxetine (Paxil)
D) Promethazine
E) Venlafaxine (Effexor XR)

ANSWER: A
For this patient, acamprosate is the most effective medication to help maintain alcohol
abstinence.
Antidepressants may be beneficial in patients with coexisting depression. The antiemetic
ondansetron may also help decrease alcohol consumption in patients with alcohol use
disorder.
83. A 68-year-old male with a 40-pack-year history of smoking presents with a 2-month
history of dyspepsia and difficulty swallowing. He also reports a 20-lb unintentional
weight loss. He takes omeprazole (Prilosec), 20 mg daily.
Which one of the following would be most appropriate at this point?
A) Increasing omeprazole to 40 mg twice daily
B) Abdominal CT
C) Barium esophagography
D) Esophageal manometry
E) Upper endoscopy

ANSWER: E
This patient has risk factors and symptoms that suggest esophageal cancer. According
to the Society of Thoracic Surgeons and the National Comprehensive Cancer Network,
upper endoscopy with a biopsy of suspicious lesions is the recommended initial
evaluation for symptoms of esophageal cancer (SOR C). Esophagography would be
appropriate in patients unable to undergo endoscopy but would not be the preferred test.
CT of the abdomen is not indicated in the initial evaluation for esophageal cancer but
can be integrated with a PET scan for staging. Esophageal manometry is reserved for
patients with dysphagia if upper endoscopy does not establish a diagnosis and a motility
disorder is suspected. Increasing the dosage of the proton pump inhibitor would not be
an appropriate treatment for this patient’s condition and may delay the diagnosis and
treatment of suspected cancer if the patient isnot referred promptly for upper endoscopy.
84. A 16-year-old white male sees you for a sports preparticipation examination. His
height is 193 cm (76 in), his weight is 69 kg (152 lb), and he appears to have long arms.
A physical examination reveals a high arched palate, kyphosis, myopia, and pectus
excavatum.
Which one of the following valvular abnormalities is most likely in this patient?
A) Mitral stenosis
B) Pulmonic stenosis
C) Aortic stenosis
D) Aortic insufficiency
E) Bicuspid aortic valve

ANSWER: D
This adolescent has findings of Marfan syndrome. It is associated with arachnodactyly,
an arm span greater than height, a high arched palate, kyphosis, lenticular dislocation,
mitral valve prolapse, myopia, and pectus excavatum. The cardiac examination may
reveal an aortic insufficiency murmur, or a murmur associated with mitral valve prolapse.
Cardiovascular defects are progressive, and aortic root dilation occurs in 80%–100% of
affected individuals. Aortic regurgitation becomes more common with increasing age.

85. A 46-year-old male with a 30-pack-year smoking history has had multiple episodes
of coughing up blood that he describes as a “quarter size” amount. This has happened
over the last couple of days. He has not had any chronic cough and has not been ill. A
chest radiograph is negative.
Which one of the following would be the most appropriate management at this point?
A) Observation with no further workup unless the cough persists for >1 month or the
quantity of hemoptysis increases
B) CT of the chest
C) Referral for bronchoscopy
D) Referral for nasolaryngoscopy

ANSWER: B
While a plain chest radiograph should come first in the workup for hemoptysis, patients
with normal radiographs who have a higher risk of malignancy (age 40 and a smoking
history of 30 years) should undergo CT, usually with contrast. If CT is negative,
pulmonary consultation and possible bronchoscopy should be pursued.
Nasolaryngoscopy is not indicated if the initial history and examination do not indicate
an upper airway source. Observation alone is not appropriate in patients with risk factors
for malignancy.

86. A 68-year-old female presents for evaluation of shortness of breath with activity for
the past several weeks. She used to walk 2 miles daily for exercise but can no longer do
so because of dyspnea and chest tightness. She also reports mild lower extremity
edema. She has a history of a bicuspid aortic valve and aortic stenosis.
Echocardiography 1 year ago showed moderately severe aortic stenosis with a mean
valve area of 1.1 cm2.
Echocardiography today shows aortic stenosis with an aortic valve area of 0.9 cm2, a
mean pressure gradient of 42 mm Hg, and a transaortic velocity of 4.3 m/sec. The
ejection fraction is estimated to be 50%.
Which one of the following is indicated at this time?
A) Atorvastatin (Lipitor)
B) Furosemide (Lasix)
C) Lisinopril (Prinivil, Zestril)
D) Metoprolol succinate (Toprol-XL)
E) Referral for aortic valve replacement

ANSWER: E
This patient has severe symptomatic aortic stenosis. The only therapy shown to improve
symptoms and mortality in such patients is an aortic valve replacement. In patients with
asymptomatic disease, watchful waiting is usually the recommended course of action.
No medications or other therapies have been shown to prevent disease progression or
alleviate symptoms. Patients with coexisting hypertension should be managed medically
according to accepted guidelines. Diuretics should be used with caution due to their
potential to reduce left ventricular filling and cardiac output, which leads to an increase in
symptoms.

87. You suspect a 45-year-old female may have irritable bowel syndrome. She has a 6-
month history of crampy, diffuse abdominal pain associated with defecation. Her
symptoms occur several days per week.
According to the Rome IV criteria, an associated symptom that would help in making this
diagnosis is
A) a change in stool frequency
B) increased gas and bloating
C) pain brought on by eating
D) waking up at night to defecate
E) weight loss of 5 lb (2 kg)

ANSWER: A
The Rome IV criteria are widely used as guidelines to diagnose suspected irritable bowel
syndrome. These criteria specify that there should be recurrent abdominal pain
associated with two or more additional symptoms at least 1 day per week in the last 3
months. These symptoms include pain related to defecation, a change in stool
frequency, or a change in stool form. Pain brought on by eating and increased gas and
bloating are observed in irritable bowel syndrome but are not included in the Rome IV
criteria. Weight loss and waking at night to defecate are not typically seen in this
disorder.

88. The U.S. Preventive Services Task Force recommends routine screening for
gestational diabetes mellitus no sooner than
A) 16 weeks gestation
B) 20 weeks gestation
C) 24 weeks gestation
D) 32 weeks gestation

ANSWER: C
The U.S. Preventive Services Task Force recommends screening for gestational
diabetes mellitus after 24 weeks gestation with a fasting blood glucose level, a 50-g oral
glucose challenge, or an assessment of risk factors (A recommendation). Screening at
an earlier date receives a rating of insufficient evidence, and screening at later dates is
not recommended (SOR C).

89. A 20-year-old football player presents with pain in the proximal fifth metatarsal. The
pain was initially present only after practices, but now it causes push-off pain during
practice. There is tenderness to palpation. Plain films show no signs of fracture.
Which one of the following would be most appropriate at this point?
A) Start NSAIDs and allow him to continue practicing as tolerated
B) Place him at non–weight bearing for 2 weeks and repeat the plain films
C) Place him in a hard shoe for 3 weeks and then reexamine
D) Order MRI of the foot
E) Order a bone scan of the foot

ANSWER: D
A stress fracture in the proximal fifth metatarsal is particularly prone to nonunion and
completion of the fracture. Because complete non–weight bearing or surgical
intervention may be necessary with this high-risk fracture, MRI is indicated as the most
sensitive test. Bone scans are sensitive but nonspecific. Most stress fractures of the
metatarsals occur distally and can be managed with a hard shoe initially, with
progressive activity as tolerated. NSAIDs are discouraged because of possible effects
on fracture healing.

90. An 84-year-old female with severe dementia due to Alzheimer’s disease is a resident
of a long-term care facility. She has been hitting the staff while receiving personal care
and recently had an altercation with another resident. Behavioral interventions have
been unsuccessful in managing her symptoms and you suggest to the patient’s family
that she be started on low-dose risperidone (Risperdal). They ask about appropriate use
of the drug and the potential for side effects.
Which one of the following would be appropriate advice?
A) Extrapyramidal side effects are more common compared to typical antipsychotics
B) Dementia-related psychosis is an FDA-approved indication
C) No monitoring will be necessary
D) The risk of diabetes mellitus is decreased
E) The risk of mortality is increased
ANSWER: E
Both typical and atypical antipsychotics increase the risk of mortality in patients with
dementia. The FDA has a black box warning on these medications, including
risperidone, about the increased risk of mortality in patients with dementia. Risperidone
is not approved by the FDA for dementia-related psychosis. The typical antipsychotics
are more commonly associated with extrapyramidal side effects. Diabetes mellitus
and agranulocytosis are associated with the atypical antipsychotics, including
risperidone. Periodic monitoring of serum glucose levels and CBCs is recommended.

91. Which one of the following diabetes mellitus medications is MOST likely to cause
weight gain?
A) Empagliflozin (Jardiance)
B) Glimepiride (Amaryl)
C) Liraglutide (Victoza)
D) Metformin (Glucophage)
E) Sitagliptin (Januvia)

ANSWER: B
Since many patients with diabetes mellitus are obese, the impact of medications on the
patient’s weight is important to consider. Treatment with sulfonylureas, including
glimepiride, is associated with weight gain.
Empagliflozin, liraglutide, metformin, and sitagliptin are not associated with weight gain.
In particular,the SGLT2 inhibitors such as empagliflozin and the GLP1 agonists such as
liraglutide are associated with clinically significant weight loss.

92. A previously healthy 34-year-old female presents with a 1-hour history of


palpitations. She does not have a cough, shortness of breath, wheezing, or chest pain.
An EKG is shown below.
Which one of the following laboratory tests is most likely to demonstrate the cause of the
patient’s underlying problem?
A) BNP
B) D-dimer
C) Lactic acid
D) Troponin
E) TSH
ANSWER: E
This patient’s EKG shows atrial fibrillation with a rapid ventricular response. A TSH level
should be obtained in all patients presenting with acute atrial fibrillation, because
patients with subclinical hyperthyroidism have a threefold increased risk of developing
atrial fibrillation. D-dimer has a negative predictive value in the diagnosis of pulmonary
embolism. Elevated troponin is a diagnostic marker of acute myocardial infarction and a
troponin level should be obtained when acute coronary syndrome is being considered as
a cause of acute atrial fibrillation. Elevated lactic acid is associated with sepsis. BNP
levels should be ordered if heart failure is suspected (SOR C).
93. A 14-year-old male is brought to your office with a 2-month history of a lump in his
left chest. An examination reveals a slightly tender 2-cm area of concentric firm mobile
tissue under the left areola. He has no skin changes, nipple discharge, or associated
adenopathy. The right side is unremarkable. A genital examination reveals Tanner 3
development but is otherwise unremarkable. Growth curves are appropriate for the
patient’s age, with a BMI of 19.1 kg/m2.
Which one of the following would be most appropriate at this point?
A) Follow-up in 6–12 months
B) A prolactin level
C) Ultrasonography of the left breast
D) Tamoxifen (Soltamox), 10 mg/day for 3 months
E) A biopsy
ANSWER: A
This patient’s history and the examination support the diagnosis of adolescent
physiologic gynecomastia. The most appropriate next step is follow-up with this patient
in 6–12 months. One-half of all adolescent males will experience some form of
gynecomastia. This condition is often bilateral, but it is more common on the left side if it
is unilateral. It will typically resolve 6–24 months after onset. Patients should be asked
about medications and supplements, because these may be a cause of nonphysiologic
breast enlargement. Concerning factors include persistence for longer than 2 years;
hard, immobile, nontender masses; masses >5 cm; nipple discharge; testicular masses;
and systemic symptoms such as weight loss. Evaluation for persistent gynecomastia can
include laboratory studies to exclude hepatic, renal, and thyroid disorders, and
can progress to include tests to detect gonadotropin and hormone-related tumors and
disorders. Imaging and/or a biopsy would be indicated if signs of a carcinoma were
noted. The additional options listed are not indicated at this point, although they are a
part of the recommended algorithm for further evaluation and treatment considerations.

94. A 52-year-old male presents for evaluation of a long-standing facial rash. He reports
that the rash is itchy, with flaking and scaling around his mustache and nasolabial folds.
Which one of the following is most likely to be beneficial?
A) Topical antibacterial agents
B) Topical antifungal agents
C) Topical vitamin D analogues
D) Oral zinc supplementation

ANSWER: B
Seborrheic dermatitis is commonly seen in the office setting and affects the scalp,
eyebrows, nasolabial folds, and anterior chest. The affected skin appears as
erythematous patches with white to yellow greasy scales. The etiology is not exactly
known, but it is likely that the yeast Malassezia plays a role. Topical antifungals are
effective and recommended as first-line agents. Topical low-potency corticosteroids are
also effective alone or when used in combination with topical antifungals, but they should
be used sparingly due to their adverse effects. The other agents listed have no role in
the management of seborrheic dermatitis (SOR A).

95. A 58-year-old male sees you for a routine health maintenance visit. He has a 20-
pack-year smoking history and proudly tells you that he quit “for good” 1 year ago. You
congratulate him on this accomplishment and encourage him to continue to abstain from
tobacco. He has not seen a physician for 20 years.
U.S. Preventive Services Task Force recommendations for this patient include which
one of the following?
A) Abdominal aortic aneurysm screening
B) Fall prevention screening
C) Hepatitis C screening
D) Lung cancer screening with low-dose CT

ANSWER: C
The U.S. Preventive Services Task Force (USPSTF) recommends one-time screening
for hepatitis C virus infection for adults born between 1945 and 1965. Abdominal aortic
aneurysm screening with ultrasonography is recommended for men 65–75 years of age
who have any history of smoking. The USPSTF recommends annual screening for lung
cancer with low-dose CT in adults 55–80 years of age who have a 30-pack-year
smoking history and currently smoke or have quit within the past 15 years. Fall
risk screening is recommended in community-dwelling adults 65 years of age or older.

96. A 30-year-old female who gave birth to a healthy infant 3 months ago has had mildly
depressed moods almost daily for the last 7 weeks. She takes very little joy in daily
activities and interacting with her baby. She is exclusively breastfeeding and has
difficulty sleeping. She says that she felt fine during the first month after the delivery, and
has not experienced any homicidal or suicidal ideations. You rule out postpartum
psychosis and bipolar disorder. Which one of the following would be most appropriate at
this point?
A) Reassurance only
B) A home health visit
C) Oral contraceptives
D) Trazodone (Oleptro)
E) Referral for psychotherapy

ANSWER: E
This patient has peripartum depression. All women should be screened for depression
during pregnancy and the postpartum period (SOR B). Reassurance may be appropriate
for the baby blues, which usually start 2–3 days after birth and last less than 10 days.
First-time mothers, adolescent mothers, and mothers who have experienced a traumatic
delivery may benefit from home health visits or peer support to prevent but not treat
peripartum depression. Mild to moderate peripartum depression can be treated with
psychotherapy or SSRIs, with consideration of medications with the lowest serum
medication levels in breastfed infants. Tricyclic antidepressants such as trazodone are
not considered first-line treatment for peripartum depression.

97. Which one of the following is the preferred first-line agent in the treatment of
rheumatoid arthritis?
A) Adalimumab (Humira)
B) Etanercept (Enbrel)
C) Hydroxychloroquine (Plaquenil)
D) Methotrexate (Trexall)
E) Prednisone
ANSWER: D
The American College of Rheumatology recommends methotrexate, a nonbiologic
disease-modifying antirheumatic drug (DMARD), as a first-line agent in the treatment of
rheumatoid arthritis in the absence of contraindications, such as underlying liver disease.
Starting DMARDs within 3 months of the onset of rheumatoid arthritis symptoms is more
likely to result in sustained remissions. The addition of short-term prednisone is indicated
in select cases when disease activity is high. The use of biological agents such as
adalimumab, etanercept, and others is indicated only in refractory cases and in patients
who cannot tolerate nonbiologic DMARDs.

98. A 68-year-old female sees you for a routine health maintenance visit. She feels well
and says she has been eating more carefully and exercising for 45 minutes 4 days a
week for the past 6 months. Her past medical history includes controlled hypertension
and osteoarthritis of the knee. Her family history is notable for a myocardial infarction in
her mother at 48 years of age. Her only medication is lisinopril (Prinivil, Zestril).
The physical examination is notable only for a BMI of 36.0 kg/m2. Laboratory findings are
notable for significant hyperlipidemia and you recommend starting a statin. She reports
that she will undergo an elective total knee replacement next month and asks about the
safety of starting a new medication before this surgery.
You recommend that she
A) start a statin immediately to decrease her risk of cardiovascular disease and
perioperative mortality
B) start a statin immediately to decrease her risk of cardiovascular disease, although her
risk of perioperative mortality will not be affected
C) start a statin immediately to decrease her risk of cardiovascular disease, stop the
statin 1 week before surgery, and resume taking it after the surgery, to decrease her
risk of perioperative mortality
D) start a statin immediately after the surgery to decrease her risk of cardiovascular
disease and perioperative mortality
E) start a statin after she is released postoperatively by her surgeon to decrease her risk
of cardiovascular disease and perioperative mortality

ANSWER: A
Family physicians are often consulted for perioperative medical management. Studies
have shown decreased perioperative mortality in patients who continue statins and in
patients with clinical indications for statin therapy who start statins prior to undergoing
vascular or high-risk surgeries such as joint replacement. A meta-analysis of 223,000
patients showed a significant reduction in perioperative mortality in patients receiving
statin therapy versus placebo who underwent noncardiac surgical procedures. This
patient has a clinical indication (multiple risk factors) to start statin therapy now.
99. A 40-year-old male presents to your office for follow-up of an abnormal clean-catch
urine test performed at his employee health clinic during a preemployment screening
examination.
He had a positive urine dipstick for hemoglobin and 5 RBCs/hpf on microscopy. The
urine was negative for protein, WBCs, and casts. A basic metabolic panel was notable
for a creatinine level of 0.8 mg/dL (N 0.6–1.2) and a BUN of 15 mg/dL (N 8–23). He
reports that he has been healthy and has not sought medical care in the last 5 years. He
quit smoking 6 months ago and walks the dog daily for 30 minutes. A physical
examination today is normal.
According to the guidelines of the American Urological Association, which one of the
following would be the most appropriate next step in the workup?
A) Repeat urine microscopy
B) Urine cytology
C) Cystoscopy
D) Renal ultrasonography
E) Retrograde pyelography

ANSWER: C
Asymptomatic microhematuria is defined as 3 or more RBCs/hpf on a properly collected
urine specimen in the absence of an obvious benign cause. Vigorous exercise, viral
illness, trauma, and infection have been ruled out as a cause of hematuria in this patient.
His renal function is normal. The most appropriate next step in evaluating a patient 35
years of age is to perform a urologic evaluation with cystoscopy. Cystoscopy is also
recommended for patients of any age who have risk factors for urinary tract
malignancy. The initial examination should also include CT urography with and without
contrast. When CT with contrast is contraindicated, an alternative is retrograde
pyelography in conjunction with noncontrast CT, MR urography, or ultrasonography.
Obtaining urine cytology and urine markers is not recommended as part of the routine
evaluation of asymptomatic microhematuria. A repeat urinalysis with microscopy is not
needed to confirm asymptomatic microhematuria. According to the American Urological
Association, one positive urine sample is sufficient to prompt an evaluation.

100. A 68-year-old female presents with a 3-month history of low back pain and fatigue.
She has unintentionally lost 15 lb. A physical examination is positive for vertebral point
tenderness over the third and fourth lumbar vertebrae. Initial laboratory testing reveals a
normocytic anemia, elevated total protein, and a mild decrease in renal function.
You order a lumbar spine radiograph and additional diagnostic testing. Which one of the
following would be most appropriate at this point?
A) A serum ferritin level and iron studies
B) TSH and vitamin B12 levels
C) Serum protein electrophoresis
D) MRI of the lumbar spine
E) A bone marrow biopsy
ANSWER: C
This patient’s presentation is concerning for hematologic malignancy, in particular
multiple myeloma.Along with radiography, the next appropriate step is serum protein
electrophoresis. If laboratory work shows a monoclonal spike or if a skeletal survey
indicates lytic lesions, referral to an oncologist is indicated for a bone marrow biopsy.
MRI of the lumbar spine would be premature and obtaining iron studies, a TSH level, or
a vitamin B12 level would not adequately address the initial abnormal laboratory studies
or facilitate making the diagnosis of multiple myeloma.

101. A 48-year-old female with type 2 diabetes mellitus has been unable to achieve
optimal glycemic control with lifestyle modifications alone. You recommend that she start
medication. Which one of the following medications is generally recommended as the
first-line medication for initiating treatment for type 2 diabetes mellitus?

A) Alogliptin (Nesina)
B) Empagliflozin (Jardiance)
C) Glipizide (Glucotrol)
D) Metformin (Glucophage)
E) Pioglitazone (Actos)

ANSWER: D
Metformin should be the first medication prescribed for diabetes mellitus when an oral
agent is required (SOR A). Metformin can efficiently lower glycemic levels and is linked
to weight loss and fewer occurrences of hypoglycemia. It is also less expensive than
most other options. If more than one agent is required, continuing metformin is
recommended along with the addition of one or more of the following:
a sulfonylurea such as glipizide, a thiazolidinedione such as pioglitazone, an SGLT2
inhibitor such as empagliflozin, or a DPP-4 inhibitor such as alogliptin.

Ref: Hauk L: Type 2 diabetes mellitus: ACP releases updated recommendations for oral pharmacologic
treatment. Am Fam Physician 2017;96(7):472-473. 2) Qaseem A, Barry MJ, Humphrey LL, et al: Oral
pharmacologic treatment of type 2 diabetes mellitus: A clinical practice guideline update from the American
College of Physicians. Ann Intern Med 2017;166(4):279-290. 3) Armstrong C: ADA updates standards of
medical care for patients with diabetes mellitus. Am Fam Physician 2017;95(1):40-43.

102. A 22-year-old female presents to your office for evaluation of nasal and sinus
congestion,frequent sneezing, and itchy red eyes. These symptoms have been present
5–7 days per week for the past 6 months. She has had similar symptoms in the past but
they have never lasted this long. She moved into a new home 2 months ago. There are
no animals in the house. She has tried over-the-counter fexofenadine (Allegra) with only
partial relief of symptoms.
Which one of the following would be the most appropriate recommendation at this time?

A) Use of a mite-proof impermeable pillow cover


B) Intranasal saline irrigation
C) Intranasal azelastine (Astepro)
D) Intranasal budesonide (Rhinocort)
E) CT of the sinuses
ANSWER: D
This patient has symptoms consistent with allergic rhinitis, and the presence of
symptoms more than 4 days per week and for more than 4 weeks places her into the
persistent symptoms category. In addition to allergen avoidance and patient education,
an intranasal corticosteroid should be the first-line treatment for
allergic rhinitis with persistent symptoms (SOR A). The Choosing Wisely
recommendations from the American Academy of Otolaryngology-Head and Neck
Surgery Foundations include avoiding sinonasal imaging in patients with symptoms
limited to a primary diagnosis of allergic rhinitis. Impermeable pillow or mattress covers
are often recommended but there is no evidence of any benefit (SOR A). Intranasal
saline irrigation is beneficial and can be used as monotherapy for mild intermittent
symptoms, but intranasal corticosteroids are likely to provide more
benefit for more persistent symptoms. Intranasal antihistamines such as azelastine are
more expensive, less effective, and more likely to produce adverse effects than
intranasal corticosteroids, so they are not recommended as first-line therapy (SOR B).
Ref: Sur DK, Plesa ML: Treatment of allergic rhinitis. Am Fam Physician 2015;92(11):985-992.

103. An elderly male presents with a shallow, irregularly shaped ulceration over the
medial aspect of his right lower leg between the lower calf and medial malleolus. There
is some surrounding edema with pigment deposition over the lower leg. He reports
aching and burning pain in the lower leg with daytime swelling. His symptoms improve
with leg elevation. You make a diagnosis of venous stasis ulcer. Which one of the
following would be the most appropriate management?

A) The use of foam dressings rather than other standard dressings


B) The use of silver-based antiseptic products even if there is no infection
C) Compression therapy
D) A 3-week course of systemic antibiotics

ANSWER: C
This patient likely has a venous stasis ulceration. The use of compression therapy with a
pressure of 30–40 mm Hg is the mainstay of treatment. There is no evidence for the use
of systemic antibiotics for lower-extremity ulcerations. Likewise, there is no evidence to
support the use of either silver-based or honey-based preparations in ulcerations with no
infection. Foam dressings are no more effective than other standard dressings.
Ref: Singer AJ, Tassiopoulos A, Kirsner RS: Evaluation and management of lower-extremity ulcers. N Engl
J Med 2017;377(16):1559-1567.

104. Which one of the following is the most reliable measure to protect children from
lead toxicity in the United States?

A) Anticipatory guidance for parents and caregivers during well child visits
B) Checking the serum lead level after a known exposure
C) Eliminating the sources of lead in the community
D) Iron and calcium supplementation to reduce lead absorption
E) Providing appropriate cleaning equipment to families with known lead in the home

ANSWER: C
Although lead poisoning in children has decreased over the past few decades it is still a
problem in the pediatric population. The most reliable and cost-effective way to protect
U.S. children from lead toxicity is primary prevention, which includes reducing or
eliminating the sources of lead in the community. Checking serum lead levels after
exposures, anticipatory guidance regarding hand washing or dust control, iron and
calcium supplementation, and providing cleaning equipment have been shown to have
either little or no effect, or they address high lead levels only after the lead poisoning has
occurred.
Ref: Council on Environmental Health: Prevention of childhood lead toxicity. Pediatrics
2016;138(1):pii:e20161493.

105. A 64-year-old female with hypertension, diabetes mellitus, hyperlipidemia, and


chronic kidney disease has had headaches that have been escalating over the past 6
months and are associated with double vision and ataxia. Her medications include
lisinopril (Prinivil, Zestril) and atorvastatin (Lipitor). She weighs 61 kg (135 lb) and her
blood pressure is 144/64 mmHg. A basic metabolic panel is normal except for a
creatinine level of 2.1 mg/dL (N 0.6–1.1) and an estimated glomerular filtration rate of 26
mL/min/1.73 m2.You decide to order MRI of the brain. Which one of the following would
be most appropriate with regard to the use of gadolinium contrast in this patient?

A) Use of gadolinium if the patient’s blood pressure is controlled to a goal systolic


pressure of <130 mm Hg
B) Use of gadolinium if the patient is pretreated with n-acetylcysteine and intravenous
normal saline
C) Use of gadolinium if lisinopril is stopped 48 hours before the MRI
D) Avoiding the use of gadolinium contrast

ANSWER: D
The use of gadolinium contrast has been associated with acute kidney injury and also
with the development of nephrogenic systemic sclerosis in patients with stage 4 or 5
chronic kidney disease. Because of these risks, the FDA recommends avoiding
gadolinium contrast in patients with a glomerular filtration rate <30 mL/min/1.73 m2, as
well as in patients with acute renal failure. The risk of nephrogenic systemic sclerosis is
not affected by blood pressure, medications, intravenous hydration, or pretreatment with
n-acetylcysteine.

Ref: Rivera JA, O’Hare AM, Harper GM: Update on the management of chronic kidney disease. Am Fam
Physician 2012;86(8):749-754.

106. A 29-year-old gravida 2 para 1 comes to the hospital for scheduled induction of
labor. Her last delivery was a spontaneous vaginal delivery without complications. Her
pregnancy has been uneventful. Oxytocin (Pitocin) is used during induction according to
the hospital protocol and her labor progresses without difficulty.
Which one of the following should be AVOIDED to minimize the risk of postpartum
hemorrhage in this patient?

A) Administration of oxytocin with delivery of the anterior shoulder


B) Controlled cord traction
C) Active management of the third stage of labor
D) Routine episiotomy
E) Manual removal of a retained placenta

ANSWER: D
Postpartum hemorrhage (PPH) is the cause of one-fourth of maternal deaths worldwide
and 12% in the United States. It is defined as the loss of 1000 mL of blood or the loss of
blood with coinciding signs and symptoms of hypovolemia within 24 hours after delivery.
Twenty percent of PPH occurs in patients without risk factors, so methods to prevent this
common problem should be in place with every delivery.
Active management of the third stage of labor (AMTSL) is crucial in the prevention of
PPH. Administering oxytocin with or soon after the delivery of the anterior shoulder is the
most important step of this process (SOR A). Even if oxytocin is used for induction, or as
a part of AMTSL, it is still the most effective treatment for PPH (SOR A). Controlled cord
traction is part of AMTSL and is necessary for the delivery
of the placenta. If a retained placenta occurs it may be necessary to manually remove
the placenta with necessary anesthesia. Trauma such as lacerations and episiotomies
increases the risk of postpartum hemorrhage, so routine episiotomy should be avoided
(SOR A).
Ref: American College of Obstetricians and Gynecologists: ACOG Practice Bulletin: Clinical management
guidelines for obstetrician-gynecologists number 183, October 2017: Postpartum hemorrhage. Obstet
Gynecol 2017;130(4):e168-e186. 2) Evensen A, Anderson JM, Fontaine P: Postpartum hemorrhage:
Prevention and treatment. Am Fam Physician 2017;95(7):442-449.

107. A 64-year-old male with midsternal chest pain is brought to the emergency
department by ambulance. He is on oxygen and an intravenous line is in place. Shortly
after arrival he loses consciousness and becomes pulseless and apneic, and CPR is
begun. Cardiac monitoring shows ventricular tachycardia with a rate of 160
beats/min.Which one of the following would be most appropriate at this point?

A) Amiodarone, intravenous infusion, followed by synchronized cardioversion


B) Adenosine (Adenocard), rapid intravenous push, repeated in 1–2 minutes if needed
C) Epinephrine, intravenous push, followed by synchronized cardioversion
D) Lidocaine (Xylocaine), intravenous push, repeated in 5 minutes if needed
E) Defibrillation

ANSWER: E
Pulseless ventricular tachycardia (VT) should be treated the same as ventricular
fibrillation. The first step is defibrillation. If that is unsuccessful, epinephrine is
administered and defibrillation is reattempted. Lidocaine, adenosine, and procainamide
may be used for the initial treatment of a wide-complex
tachycardia of uncertain type, but should not be used for the initial treatment of pulseless
VT. Synchronized cardioversion alone would be indicated for the initial treatment of rapid
unstable tachycardia
with a pulse.
Ref: Link MS, Berkow LC, Kudenchuk PJ, et al: Part 7: Adult advanced cardiovascular life support: 2015
American Heart Association guidelines update for cardiopulmonary resuscitation and emergency
cardiovascular care. Circulation 2015;132(18 Suppl 2):S444-S464. 2) Walls RM, Hockberger RS, Gausche-
Hill M, et al (eds): Rosen’s Emergency Medicine: Concepts and Clinical Practice , ed 9. Elsevier Inc,
2018, p 90.
108. In patients with COPD, which one of the following inhaled medications has been
shown to reduce exacerbations and exacerbation-related hospitalizations?

A) Albuterol (Proventil, Ventolin)


B) Fluticasone (Flovent)
C) Ipratropium (Atrovent)
D) Salmeterol (Serevent)
E) Tiotropium (Spiriva)

ANSWER: E
A Cochrane review found that the long-acting antimuscarinic agent tiotropium improved
quality of life and reduced exacerbations and exacerbation-related hospitalizations in
patients with underlying COPD. Tiotropium was noted to be superior to long-acting -
agonists such as salmeterol. Albuterol, fluticasone,
and ipratropium have not been shown to have these effects (SOR A).
Ref: Chong J, Karner C, Poole P: Tiotropium versus long-acting -agonists for stable chronic obstructive
pulmonary disease. Cochrane Database Syst Rev 2012;(9):CD009157. 2) Karner C, Chong J, Poole P:
Tiotropium versus placebo for chronic obstructive pulmonary disease. Cochrane Database Syst Rev
2014;(7):CD009285.

109. A 52-year-old male sees you for a routine health maintenance examination. He
does not take any medications, does not drink alcohol, and is feeling well. A physical
examination is normal with the exception of a BMI of 33.2 kg/m2. Routine laboratory
studies reveal mild elevations of ALT (SGPT) and AST (SGOT), which remain elevated
on repeat testing 2 months later. Hepatitis B and hepatitis C testing are negative. In
addition to ultrasonography of the liver, which one of the following laboratory studies
should be ordered to further evaluate this patient?

A) Serum ferritin
B) Serum phosphorus
C) -Fetoprotein
D) Carcinoembryonic antigen (CEA)
E) Serum protein electrophoresis

ANSWER: A
Mild asymptomatic elevations (<5 times the upper limit of normal) of ALT and AST are
common in primary care. It is estimated that approximately 10% of the U.S. population
has elevated transaminase levels. The most common causes of elevated transaminase
levels are nonalcoholic fatty liver disease and alcoholic liver disease. The initial
evaluation should include assessment for metabolic syndrome and insulin
resistance. Waist circumference, blood pressure, a fasting lipid level, and a fasting
glucose level or hemoglobin A1c should be obtained. A CBC with platelets and
measurement of serum albumin, iron, total iron-binding capacity, and ferritin levels would
also be indicated. Iron studies should be ordered to rule out hereditary
hemochromatosis, which is an autosomal recessive disease that causes increased iron
absorption in the intestines and release by tissue macrophages.

Ref: Oh RC, Hustead TR, Ali SM, Pantsari MW: Mildly elevated liver transaminase levels: Causes and
evaluation. Am Fam Physician 2017;96(11):709-715.
110. A 62-year-old white male with a 3-month history of diabetes mellitus has a
hemoglobin A1c of 7.8%. Which one of the following is the best parameter for determining
if he can safely take metformin (Glucophage)?

A) 24-hour urine for creatinine clearance


B) BUN/creatinine ratio
C) Estimated glomerular filtration rate
D) Serum creatinine
E) Urine microalbumin

ANSWER: C
Until recently metformin was contraindicated for patients with renal dysfunction
suggested by a creatinine level of 1.5 mg/dL for men and 1.4 mg/dL for women.
However, available evidence now supports the use of metformin in individuals with mild
to moderate chronic renal disease, defined by the estimated glomerular filtration rate
(eGFR). Patients with an eGFR between 45 and 60 mL/min/1.73 m2 (chronic
mild kidney disease) are now permitted to take metformin. Metformin should not be used
in patients with an eGFR <45 mL/min/1.73 m2 (moderate kidney disease), as lactic
acidosis is more likely to occur. The eGFR is used instead of the serum creatinine level
because the equation includes age, sex, race, and other
parameters.

Ref: Inzucchi SE, Lipska KJ, Mayo H, et al: Metformin in patients with type 2 diabetes and kidney disease: A
systematic review. JAMA 2014;312(24):2668-2675. 2) Lowes R, Nainggolan L: FDA: Metformin safe for
some patients with renal problems. Medscape, 2016. 3) Choby B: Diabetes update: New pharmacotherapy
for type 2 diabetes. FP Essent 2017;456:27-35.

111. A 43-year-old male presents with a 6-week history of right ankle pain. The pain
worsens with walking or running for a moderate distance and fails to improve with heat
application or reduction of activity. He has been following a moderate cardiovascular
exercise program for several years without problems and did not increase his physical
activity before the onset of the pain. He does not recall any injury to the ankle. On
examination the area of pain is localized in the right Achilles tendon proximal to its
insertion. No swelling, redness, or deformity is apparent but tenderness is elicited with
application of moderate fingertip pressure to the tendon.
Which one of the following would be the most appropriate initial treatment?

A) Use of a heel cup in the right shoe


B) A 1-month course of daily NSAIDs at a prescription dosage
C) An eccentric gastrocnemius-strengthening program
D) A corticosteroid injection into the right Achilles tendon sheath
E) Immobilization of the right ankle for 3 weeks with a boot

ANSWER: C
Pain located between the myotendinous junction and the insertion of the Achilles tendon
that occurs during prolonged walking or running is typical for midsubstance Achilles
tendinopathy. The mechanisms resulting in pain are complex and not fully understood
but inflammation is believed to contribute little to the process. This is evidenced in part
by the ineffectiveness of treatments typically used to reduce inflammation such as
NSAIDs and corticosteroids, which are not recommended in the treatment of this
condition (SOR A). Other commonly used musculoskeletal therapeutic modalities such
as immobilization, ultrasonography, orthotics, massage, and stretching exercises have
not been shown to consistently offer significant benefits and are not considered to be
first-line therapy for Achilles tendinopathy.
A gastrocnemius-strengthening eccentric exercise program performed in sets of
controlled, slow, active release from weight-bearing full extension to full flexion of the
foot at the ankle has been shown to reduce pain and improve function in the 60%–90%
range, making this the logical first-line treatment for Achilles tendinopathy (SOR A). The
less common insertional Achilles tendinopathy localized to the enthesis is typically more
recalcitrant, and immobilization in a walking boot for a period of time may be necessary
before eccentric exercise can be tolerated.

Ref: Childress MA, Beutler A: Management of chronic tendon injuries. Am Fam Physician 2013;87(7):486-
490.

112. A 45-year-old male presents to your office with a 2-month history of a


nonproductive cough, mild shortness of breath, fatigue, and a 5-lb weight loss. On
examination his lungs are clear. A PPD skin test is negative. A chest radiograph shows
bilateral hilar adenopathy and his angiotensin converting enzyme level is elevated. A
biopsy of the lymph node shows a noncaseating granuloma.
Which one of the following would be the most appropriate initial treatment?

A) Azathioprine (Imuran)
B) Fluconazole (Diflucan)
C) Isoniazid
D) Levofloxacin (Levaquin)
E) Prednisone

ANSWER: E
This patient has sarcoidosis that has been confirmed by a biopsy. He is symptomatic so
treatment would be indicated. The recommended initial treatment for sarcoidosis is oral
corticosteroids. Anti-infective agents are not appropriate treatment for sarcoidosis.
Immunosuppressants are second- and third-line therapy for sarcoidosis and would not
be recommended as first-line treatment.
Ref: Soto-Gomez N, Peters JI, Nambiar AM: Diagnosis and management of sarcoidosis. Am Fam
Physician2016;93(10):840-848.

113. A 40-year-old white female sees you for the first time. When providing a history she
describes several problems, including anxiety, insomnia, fatigue, persistent depressed
mood, and low libido. These symptoms have been present for several years and are
worse prior to menses, although they also occur to some degree during menses and
throughout the month. Her menstrual periods are regular for the most part. Based on this
history, the most likely diagnosis is

A) premenstrual dysphoric disorder


B) menopause
C) dysthymia
D) anorexia nervosa
E) dementia
ANSWER: C
Psychological disorders, including anxiety, depression, and dysthymia, are frequently
confused with premenstrual dysphoric disorder, and must be ruled out before initiating
therapy. Symptoms are cyclic in true premenstrual dysphoric disorder. The most
accurate way to make the diagnosis is to have the patient carefully record daily
symptoms on a menstrual calendar for at least two cycles. Dysthymia consists of a
pattern of ongoing, mild depressive symptoms that have been present for at least 2
years and are less severe than those of major depression, which is consistent with the
findings in this case.
Ref: American Psychiatric Association: Diagnostic and Statistical Manual of Mental Disorders , ed 5.
American PsychiatricAssociation, 2013, pp 372-379. 2) Hofmeister S, Bodden S: Premenstrual syndrome
and premenstrual dysphoric disorder.Am Fam Physician 2016;94(3):236-240

114. A 33-year-old gravida 3 para 2 presents for prenatal care 8 weeks after her last
menstrual period. She asks if she will need any immunizations during this pregnancy.
Which one of the following vaccines is recommended for all women with each
pregnancy?

A) 13-valent pneumococcal conjugate vaccine (PCV13, Prevnar 13)


B) Hepatitis B
C) MMR
D) Tdap
E) Varicella

ANSWER: D

Tdap is recommended for all women with each pregnancy, preferably between 27 and
36 weeks gestation. Live vaccines such as varicella and MMR are contraindicated during
pregnancy. There is inadequate data to recommend vaccination against pneumococcal
disease during pregnancy. Hepatitis B vaccine is recommended during pregnancy only
for women at high risk for infection.
Ref: Omer SB: Maternal immunization. N Engl J Med 2017;376(13):1256-1267.

115. A 26-year-old male presents with a rash on his anterior neck in the area of his
beard that has been present for over a year. On examination he has dark, curly facial
hair, and you find slightly tender, red, hyperpigmented papules on the superior anterior
neck. Which one of the following would you recommend to improve this patient’s rash?

A) Shaving with a multi-blade razor


B) Shaving with electric clippers
C) Pulling the skin taut while shaving
D) Plucking hairs rather than shaving
E) Oral cephalexin (Keflex)

ANSWER: B

This patient has pseudofolliculitis barbae, which is a common condition affecting the face
and neck in people with tightly curled hair. The condition occurs when hairs are cut at an
angle and curl in on themselves, creating a foreign body reaction. The condition may
progress to scarring and keloid formation. Cessation of hair removal improves the
condition. If this is not desired, less aggressive hair trimming is recommended. Clippers
generally result in a less close shave and contribute less to pseudofolliculitis barbae.
Multi-blade razors, pulling the skin taut, and plucking hairs all result in shorter hair and
are likely to exacerbate the problem. The description of the rash is not consistent with
secondary infection, so oral cephalexin would not be indicated at this time. Treatment is
similar to the treatment of acne, with benzoyl peroxide, topical retinoids, and topical
antibiotics having a role, along with topical corticosteroids.

Ref: Kundu RV, Patterson S: Dermatologic conditions in skin of color: Part II. Disorders occurring
predominately in skin of color. Am Fam Physician 2013;87(12):859-865.

116. A 54-year-old male comes to your office to establish care. He has a past history of
hypertension treated with lisinopril (Prinivil, Zestril) and hydrochlorothiazide but has not
taken his medications for over a year. He does not have any symptoms, including chest
pain, shortness of breath, or headache. On examination his blood pressure is 200/115
mm Hg on two separate readings taken 5 minutes apart. The remainder of the physical
examination is normal. Which one of the following management options would be most
appropriate?

A) Institute out-of-office monitoring with an ambulatory device and follow up in 2


weeks
B) Restart the patient’s previous antihypertensive medications and follow up within 1
week
C) Administer a short-acting antihypertensive medication in the office to lower his
blood pressure to <160/100 mm Hg
D) Hospitalize for hypertensive emergency

ANSWER: B
This patient has severe asymptomatic hypertension (systolic blood pressure 180 mm
Hg or diastolic blood pressure 110 mm Hg). If there were signs or symptoms of acute
target organ injury, such as neurologic deficits, altered mental status, chest pain,
shortness of breath, or oliguria, hospitalization for a hypertensive emergency would be
indicated. Because this patient was asymptomatic and has a known history of
hypertension, restarting his prior antihypertensive regimen and following up in 2 weeks
would be the most appropriate management option. If he had no past history of
hypertension it would be reasonable to consider out-of-office monitoring with an
ambulatory device for 2 weeks before initiating treatment. In the absence of acute target
organ injury, blood pressure should be gradually lowered to less than 160/100 mmHg
over several days to weeks. Aggressively lowering blood pressure can lead to adverse
events such as myocardial infarction, cerebrovascular accident, or syncope, so
administering a short-acting antihypertensive medication in the office should be reserved
for the management of hypertensive emergencies.
Ref: Gauer R: Severe asymptomatic hypertension: Evaluation and treatment. Am Fam Physician
2017;95(8):492-500.

117. A 17-year-old female comes to your office with an 8-month history of amenorrhea.
Menarche occurred at age 12 and her menses were regular until the past year. On
examination the patient’s vital signs are in the normal range for her age but she has a
BMI of 16.1 kg/m2, which is below the third percentile for her age. She is a high school
senior who dances with the local ballet company. She practices dance several hours a
day and works out regularly. She tells you that she follows a strict 800-calorie/day diet to
keep in shape for ballet. You order a CBC, a comprehensive metabolic panel, a urine -
hCG level, FSH and LH levels, and a TSH level. Which one of the following is also
recommended as part of the workup?

A) An EKG
B) Pelvic ultrasonography
C) Abdominal/pelvic CT
D) A DXA scan
E) A nuclear bone scan

ANSWER: D
Relative energy deficiency in sport (RED-S), formerly known as the female athlete triad,
is a relatively common condition in female athletes, and is characterized by amenorrhea,
disordered eating, and osteoporosis. It is more common in sports that promote lean body
mass. Female athletes should be screened for the disorder during their preparticipation
evaluations. Individuals who present with one or more components of RED-S should be
evaluated for the other components. This patient has a low BMI for her age, which
indicates an eating disorder, and secondary amenorrhea, and should be screened for
osteoporosis using a DXA scan. The International Society for Clinical Densitometry
recommends using the Z-score, rather than the T-score, when screening children or
premenopausal women. The T-score is based on a comparison to a young adult at peak
bone density, whereas the Z-score uses a comparison to persons of the same age as
the patient. A Z-score less than –2.0 indicates osteoporosis. The American College of
Sports Medicine defines low bone density as a Z score of –1.0 to –2.0. An EKG is not
required in this patient since she has normal vital signs. Pelvic ultrasonography is not
necessary unless an abnormal finding is identified on a pelvic examination.
Abdominopelvic CT would be inappropriate given the patient’s age and lack of
abdominopelvic symptoms such as pain or a mass. A nuclear bone scan likewise is not
recommended, as it is not used to diagnose osteoporosis (SOR C).
Ref: Joy EA, Van Hala S, Cooper L: Health-related concerns of the female athlete: A lifespan approach. Am
Fam Physician 2009;79(6):489-495. 2) Mendelsohn FA, Warren MP: Anorexia, bulimia, and the female
athlete triad: Evaluation and management. Endocrinol Metab Clin North Am 2010;39(1):155-167. 3) Javed
A, Tebben PJ, Fischer PR, Lteif AN: Female athlete triad and its components: Toward improved screening
and management. Mayo Clin Proc 2013;88(9):996-1009.

118. A 45-year-old electrician presents to your office with concerns about a bump on his
left elbow. He does not recall any injury. The bump is painful to touch but causes no
other symptoms. He is worried since it has been present for at least a month and never
goes away. On examination the patient is afebrile. He has a 4-cm movable fluctuant
growth at the tip of his left olecranon that is slightly tender to touch. There is no warmth
or erythema and he has full range of motion of his elbow. There is no other joint
involvement. Which one of the following would you recommend?

A) No further evaluation
B) Laboratory testing, including a CBC with differential
C) Plain radiography
D) Ultrasonography
E) Aspiration
ANSWER: A
This patient presents with chronic olecranon bursitis. The diagnosis can be made from
his history and the physical examination, and no additional workup is indicated at this
time. Chronic bursitis is due to repetitive microtrauma, and the olecranon is the most
common location. Patients typically have minimal pain, no history of injury, no systemic
symptoms, and no signs of acute infection or inflammation.Treatment initially consists of
avoiding recurrent trauma by protecting the area with an elbow pad and not leaning on it,
as well as cryotherapy, compression of the affected area, and over-the-counter
analgesics.If the lesion is inflamed or appears septic then laboratory testing should be
performed, including a CBC with differential, a glucose level, an erythrocyte
sedimentation rate, and a C-reactive protein level. Joint aspiration and/or
ultrasonography may be indicated if the diagnosis is not apparent. A plain radiograph
would be indicated to rule out a fracture in a patient with traumatic bursitis
.
Ref: Khodaee M: Common superficial bursitis. Am Fam Physician 2017;95(4):224-231.

119. A 25-year-old gravida 1 para 0 at 24 weeks gestation comes to your office with right
lower extremity swelling and pain. Her pregnancy has been uncomplicated so far and
her only medication is a prenatal vitamin. She does not have chest pain, shortness of
breath, or fever. She recently started feeling the baby move, and an anatomy scan at 20
weeks gestation was normal. Lower extremity Doppler ultrasonography confirms a right
lower extremity deep vein thrombosis (DVT). Laboratory studies including a CBC,
coagulation studies, and renal function are normal. Which one of the following would be
the most appropriate initial treatment of her DVT?

A) Oral apixaban (Eliquis)


B) Oral aspirin
C) Oral warfarin (Coumadin)
D) Subcutaneous enoxaparin (Lovenox)
E) Subcutaneous heparin

ANSWER: D
Enoxaparin is the most appropriate pharmacologic therapy for anticoagulation in patients
who are pregnant. Aspirin is not used as treatment for deep vein thrombosis. Apixaban,
warfarin, and heparin either have not been studied for use in pregnancy or there is data
indicating potential fetal harm.
Ref: Kearon C, Akl EA, Ornelas J, et al: Antithrombotic therapy for VTE disease: CHEST guideline and
Expert Panel report. Chest 2016;149(2):315-352.

120. A 46-year-old female with a past medical history of polycystic ovary syndrome and
migraine headaches presents with bilateral, hyperpigmented patches along her
mandible. The patches are asymptomatic but bother her cosmetically and seem to be
darkening. Which one of her medications would be most likely to contribute to her
melasma?

A) B-complex vitamins
B) Metformin (Glucophage)
C) Oral contraceptives
D) Spironolactone (Aldactone)
E) Sumatriptan (Imitrex)

ANSWER: C

Melasma is a progressive, macular, nonscaling hypermelanosis of skin exposed to the


sun, typically involving the face and dorsal forearms. It is often associated with
pregnancy and the use of oral contraceptives or anticonvulsants (SOR C). Some
melasma is idiopathic. Women are nine times more likely to be affected than men, and
darker-skinned individuals are also at greater risk. There are three common
patterns of melasma: centrofacial, malar, and mandibular.
Ref: Plensdorf S, Livieratos M, Dada N: Pigmentation disorders: Diagnosis and management. Am Fam
Physician 2017;96(12):797-804.

121. A 73-year-old female is brought to your office by her daughter, who is concerned
that there may be some memory changes in her mother. She has noticed that her
mother frequently repeats herself and has made several medication errors lately.
The patient has type 2 diabetes mellitus, hypertension, depression, and hypothyroidism.
Her current medications include glyburide (DiaBeta), aspirin, lisinopril (Prinivil, Zestril),
hydrochlorothiazide, atorvastatin (Lipitor), and sertraline (Zoloft).
A physical examination reveals a blood pressure of 136/72 mm Hg. She scores 26/30 on
a Saint Louis University Mental Status (SLUMS) examination, which suggests mild
cognitive impairment. A cardiac examination and a foot examination are normal.
Laboratory studies reveal a hemoglobin A1c of 7.0% and a TSH level of 3.8 U/mL (N
0.4–4.2). A basic metabolic panel is normal and her glucose level is 93 mg/dL. A CBC is
normal.

Which one of the following medications should be stopped in this patient?


A) Atorvastatin
B) Glyburide
C) Hydrochlorothiazide
D) Lisinopril
E) Sertraline

ANSWER: B

This patient has signs of mild neurocognitive impairment. In this case one possible
contributor to this condition is hypoglycemia. While it is unknown whether minor
hypoglycemic events can contribute to dementia, major events have been associated
with a greater risk of dementia. The sulfonylurea glyburide carries a risk of significant
hypoglycemia, especially in elderly patients. Her hemoglobin A1c of 6.1% correlates with
an estimated average glucose of 128 mg/dL, corroborating this concern. Glyburide in
particular is listed on the Beers Criteria because of its potential to cause prolonged
hypoglycemia.

122. A 75-year-old white male presents to your office following hospitalization for an
episode of heart failure. His edema has resolved but he still becomes symptomatic with
minor exertion such as walking less than a block. A recent chest radiograph shows
cardiomegaly, and echocardiography reveals an ejection fraction of 25%. He is currently
taking furosemide (Lasix), 20 mg daily; carvedilol (Coreg), 25 mg twice daily; and
lisinopril (Prinivil, Zestril), 20 mg daily. His vital signs include a pulse rate of 60
beats/min, a blood pressure of 110/70 mm Hg, a respiratory rate of 18/min, and a
temperature of 37.0°C (98.6°F). No crackles or hepatojugular reflux are noted on
auscultation.

Which one of the following would improve this patient’s symptoms and decrease his
mortality risk?
A) Digoxin
B) Hydralazine and isosorbide dinitrate (BiDil)
C) Hydrochlorothiazide
D) Spironolactone (Aldactone)

ANSWER: D

For patients with left ventricular systolic dysfunction, clinical trials have demonstrated
that ACE inhibitors,-blockers, angiotensin receptor blockers, and aldosterone
antagonists decrease hospitalizations and all-cause mortality. In African-American
patients, all-cause mortality and hospitalizations have been reduced by hydralazine and
isosorbide dinitrate. Aldosterone antagonists such as spironolactone, as well as -
blockers, decrease mortality in patients with symptomatic heart failure (SOR A). Digoxin
improves symptoms of heart failure but does not improve mortality.

123. A 67-year-old female who was recently diagnosed with colon cancer presented to
the emergency department 2 days ago with acute shortness of breath and was
diagnosed with a pulmonary embolism. She was started on enoxaparin (Lovenox) and
was hemodynamically stable during her stay in the hospital. Her shortness of breath has
improved and her oxygen saturation is currently 95% on room air.

Which one of the following would be most appropriate for this patient?
A) Continue enoxaparin upon discharge
B) Discontinue enoxaparin and start rivaroxaban (Xarelto)
C) Discontinue enoxaparin and start warfarin (Coumadin)
D) Start warfarin and continue enoxaparin until the INR is 2.0

ANSWER: A

Enoxaparin and other low molecular weight heparins are effective and are the preferred
agents for acute and long-term anticoagulation in patients with an active malignancy
(SOR B). Warfarin has been shown to be less effective in cancer patients and is not
recommended to treat venous thromboembolic disease in this setting (SOR B). The
novel oral anticoagulants including rivaroxaban have not been studied in the setting of
malignancy and are not recommended.

124. A 32-year-old Yazidi female from Iraq is brought to your office to establish care.
She is a refugee who was relocated 2 weeks ago.
Which one of the following would be appropriate at this visit?
A) Having a family member who speaks English serve as an interpreter
B) Screening for posttraumatic stress disorder
C) Hepatitis B vaccine
D) Varicella vaccine

ANSWER: B

A full history and physical examination are indicated for all refugees within 30 days of
arrival in the United States, with a professional medical interpreter if needed (SOR C). In
addition to addressing medical needs, the focus should be on emotional support and
barriers to health care access (SOR C). All refugees should be screened for depression,
anxiety, and posttraumatic stress disorder (SOR C). They should also be screened for
anemia, hypertension, impaired fasting glucose, nutritional deficiencies, tuberculosis,
and COPD (SOR C). If there is no vaccination documentation, routine vaccines should
be provided except for varicella and hepatitis B. Serology should be performed before
these vaccines are administered (SOR C).

125. A 35-year-old male has a negative past medical history and a normal physical
examination. He reports that he smokes half a pack of cigarettes per day and has 3–4
beers per week. A comprehensive metabolic panel reveals an ALT (SGPT) of 30 U/L (N
10–40) and an AST (SGOT) of 84 U/L (N 10–30). The remaining laboratory studies are
negative. There is no family history of liver disease.

The laboratory findings suggest which one of the following?


A) Hepatitis C
B) Hemochromatosis
C) Gilbert syndrome
D) Alcoholic liver disease
E) Nonalcoholic liver disease

ANSWER: D

An AST (SGOT) to ALT (SGPT) ratio greater than 2:1 suggests alcoholic liver disease,
and a ratio of 3:1 or higher is highly suggestive of alcoholic liver disease. With most
hepatocellular disorders, including nonalcoholic fatty liver disease, viral hepatitis, and
iron overload disorder, the patient will have an AST to ALT ratio <1.

126. A 48-year-old female presents with dyspnea with exertion. She has never smoked.
A physical examination is normal, including vital signs and pulse oximetry. A chest
radiograph reveals mild hyperexpansion of the chest, and pulmonary function testing
reveals an FEV1/FVC ratio of 0.67, unchanged after bronchodilator use. An EKG and
stress echocardiogram are normal. You suspect COPD.
Which one of the following is the most likely underlying cause of this patient’s pulmonary
disease?
A) Allergic bronchopulmonary aspergillosis
B) 1-Antitrypsin deficiency
C) Hemochromatosis
D) Primary pulmonary hypertension
E) Hypertrophic obstructive cardiomyopathy
ANSWER: B

This patient is a nonsmoker but has typical symptoms and findings of COPD. alfa1-
Antitrypsin deficiency should be considered in patients with very premature COPD or in
patients without risk factors for COPD such as smoking, secondhand smoke exposure,
or other smoke exposure. Dyspnea would be present and lung function would be normal
in patients with primary pulmonary hypertension or hypertrophic obstructive
cardiomyopathy. Hemochromatosis may cause liver function abnormalities but not
abnormal lung function. Allergic bronchopulmonary aspergillosis is associated with
asthma, not COPD

127. A 52-year-old pianist is concerned that she may have carpal tunnel syndrome.
Which one of the following would be consistent with this problem?
A) Weakness of thumb adduction
B) Decreased sensation over the thenar eminence
C) Decreased sensation over the dorsal aspect of the fourth finger
D) Decreased sensation over the dorsal aspect of the fifth finger
E) Decreased sensation over the palmar aspect of the thumb, index, and middle finger

ANSWER: E

Carpal tunnel syndrome is the most common entrapment neuropathy of the upper
extremity. It is caused by compression of the median nerve as it travels through the
carpal tunnel. Classically, patients with this condition experience pain and paresthesias
in the distribution of the median nerve, which includes the palmar aspect of the thumb,
index, and middle fingers, and the radial half of the ring finger. In more severe cases
motor fibers are affected, leading to weakness of thumb abduction and opposition.
Sensation over the thenar eminence should be normal in patients with carpal tunnel
syndrome because it is in the distribution of the palmar cutaneous branch of the median
nerve, which branches off proximal to the carpal tunnel.

128. A 67-year-old female with hypertension and atrial fibrillation has been taking
warfarin (Coumadin) for the past 10 years. She has been hemodynamically stable for
many years with no complications from her atrial fibrillation. She is scheduled to undergo
elective bladder sling surgery for urinary incontinence. She does not have any other
significant past medical history.

Which one of the following would be the most appropriate perioperative management of
her warfarin?

A) Continue warfarin without interruption


B) Discontinue warfarin the day prior to surgery and provide bridge therapy with low
molecular weight heparin
C) Discontinue warfarin 2 days prior to surgery and restart it 2 days postoperatively
unless there is a bleeding complication
D) Discontinue warfarin 2 days prior to surgery and restart it 5 days postoperatively
unless there is a bleeding complication
E) Discontinue warfarin 5 days prior to surgery and restart it 12–24 hours
postoperatively unless there is a bleeding complication
ANSWER: E

Perioperative management of chronic anticoagulation requires an assessment of the


patient’s risk for thromboembolism and the risk of bleeding from the surgical procedure.
High-risk patients include those with mechanical heart valves, a stroke or TIA within the
past 3 months, venous thromboembolism within the past 3 months, or coronary stenting
within the previous 12 months. High-risk patients require bridging therapy with low
molecular weight heparin, while patients at low risk do not require bridging
anticoagulation. For low-risk patients, it is recommended that warfarin be discontinued 5
days prior to surgery and restarted 12–24 hours postoperatively. This patient is at low
risk for thromboembolism because her CHA2DS2-VASc score is 3. A patient with atrial
fibrillation should receive bridging therapy with a CHA2DS2-VASc score 6.
This patient’s surgery is associated with a high risk for bleeding, so it is preferable to
stop her warfarin 5 days before the operation.

129. A 48-year-old female smoker presents with solid, but not liquid, dysphagia that
causes her to feel as if food is “getting stuck.” She sometimes regurgitates this food.
When you ask her where it feels like the food is sticking she points to a location below
the suprasternal notch.

The most appropriate next step is


A) a fluoroscopic swallowing study
B) barium radiography
C) CT of the chest
D) endoscopy
E) esophageal manometry

ANSWER: D

Solid but not liquid dysphagia suggests a structural lesion. A location in the chest
indicates esophageal dysphagia. Endoscopy is the single most useful test for
esophageal dysphagia and can visualize mucosal lesions better than barium
radiography. Therapy can also be performed during the procedure. A fluoroscopic
swallowing study would be indicated if the patient’s history pointed to oral or pharyngeal
dysphagia. Even if it is thought that the dysphagia is caused by a motility disorder,
endoscopy is still preferred, because neoplastic and inflammatory conditions can
produce spasm and motility symptoms.Manometry can be performed if endoscopy does
not adequately explain the symptoms.
130. A 7-year-old female is brought to your office by her mother for follow-up of an
urgent care visit. The child has a 5-day history of abdominal pain and low-grade fevers
to 100.1°F. Her mother took her to an urgent care clinic last night when the patient
developed the rash shown below. The rash is not pruritic or painful. She does not have
any sick contacts, urinary symptoms, or changes in bowel habits.
A physical examination is normal except for the rash and minimal diffuse abdominal
tenderness. A CBC and basic metabolic panel are normal and a urinalysis is notable
only for microhematuria (30–40 RBCs/hpf) and mild proteinuria (30 mg/dL).
The following laboratory studies were obtained at the urgent care clinic.
Basic metabolic panel : normal
Urinalysis
Color : yellow/clear
Leukocyte esterase : negative
Nitrite :negative
Protein: 30 mg/dL (normal negative)
Glucose : negative (normal)
Bilirubin : negative (normal)
RBCs : 34/hpf (N <4)
WBCs : 4/hpf (N <5)
Bacteria : none
Squamous epithelial cells : <1 (normal)
Ketones :negative (normal)
Blood large (normal negative)
Urine Gram stain : no bacteria, no PMNs
Urine culture : negative × 24 hours
In addition to close follow-up, which one of the following is the next appropriate step in
the management of this child?
A) Supportive care only
B) Amoxicillin for 10 days
C) Prednisone tapered over 10 days
D) A biopsy of a skin lesion
E) Referral to a nephrologist for consideration of a renal biopsy

ANSWER: A

This patient most likely has Henoch-Schönlein purpura. In addition to close observation,
the only treatment is supportive care, including adequate oral hydration. There is no
indication for antibiotics, and oral corticosteroids have not been shown to be beneficial.
In patients with progressive renal impairment, referral to a nephrologist is warranted, but
given this patient’s normal renal function at this time it is not indicated. A skin biopsy of
the purpura would most likely show a leukocytoclastic vasculitis and would not help in
the diagnosis.

Ref: Trnka P: Henoch-Schönlein purpura in children. J Paediatr Child Health 2013;49(12):995-1003.

131. An 18-month-old female is brought to your office in January for evaluation of a


cough and fever. She has no chronic medical conditions. She abruptly developed a
barking cough and hoarseness with a low-grade fever 2 days ago. The cough is worse at
night. She has been drinking normally but is not interested in eating. On examination she
is alert and resists the examination. Her respiratory rate and effort are normal. She has
no stridor or wheezing.

Which one of the following would be most appropriate at this point?


A) A nasal swab for influenza testing
B) A chest radiograph
C) A single dose of oral dexamethasone
D) Azithromycin (Zithromax)
E) Oseltamivir (Tamiflu)

ANSWER: C

This patient has symptoms consistent with croup, a lower respiratory infection that is
common in the winter months in children ages 6 months to 3 years. The diagnosis is
clinical and should be suspected in children with a history of a sudden onset of a deep
cough, hoarseness, and a low-grade fever. Randomized studies have shown that even
with mild croup (an occasional barking cough with no stridor at rest), oral corticosteroids
provide some benefit.

132. You are reviewing the home health care progress report of a 68-year-old female
who was hospitalized with pneumonia 2 months ago. The patient moved to the area to
live with her daughter following treatment for breast cancer 5 years earlier. Before the
hospitalization her only medical needs had been for preventive services, treatment for
hypertension, and surveillance for problems related to her chemotherapy and for return
of her cancer. During the recent hospitalization oxygen supplementation was required to
maintain healthy oxygen saturation levels, and after failing several attempts at weaning,
home oxygen service was arranged.
You ask the home health nurse to test the patient’s oxygen saturation after 1 hour on
room air and the nurse reports that the patient’s oxygen saturation is now consistently
above 90% on room air. The care plan provided by the home health service includes a
recommendation for the continuation of supplemental oxygen.

Which one of the following would be most appropriate for this patient?
A) Order arterial blood gas studies to confirm her oxygenation status
B) Discontinue oxygen supplementation
C) Discontinue daytime use of oxygen and continue nighttime oxygen
D) Continue oxygen use, but only as needed when short of breath
E) Continue oxygen use to obtain a saturation >92% on room air

ANSWER: B

Hypoxemia following an acute illness is often short-lived and as many as half the
patients prescribed home oxygen on discharge from the hospital will not meet criteria
supporting continuation after 3 months. For this group of patients there is no apparent
benefit derived from supplemental oxygen once their oxygen saturation is 88% or
greater on room air. Potential harmful effects of continuing unnecessary home oxygen
include decreased mobility, falls, house fires, and mucosal irritation, and oxygen toxicity
must be considered as well. Continuing home oxygen beyond what is needed also
results in a misallocation of resources. According to the American Thoracic Society and
the American College of Chest Physicians, prescriptions for supplemental home oxygen
should not be renewed for patients who have recently been hospitalized for acute
illnesses without assessing them for ongoing hypoxemia.

133. A 27-year-old female with a past medical history of polycystic ovary syndrome
(PCOS) would like to become pregnant. Which one of the following treatments for PCOS
is associated with greater live-birth and ovulation rates?
A) Finasteride (Proscar)
B) Letrozole (Femara)
C) Metformin (Glucophage)
D) Spironolactone (Aldactone)

ANSWER: B

In a double-blind randomized trial, letrozole was associated with greater live-birth and
ovulation rates compared to clomiphene (SOR A). A Cochrane review indicated that
metformin does not increase fertility in patients diagnosed with polycystic ovary
syndrome (PCOS). Spironolactone and finasteride are both used to treat PCOS in
women who do not desire pregnancy.

134. A 58-year-old male with a history of tobacco and alcohol abuse presents with the
sudden onset of many well circumscribed brown, oval, rough papules with a “stuck-on”
appearance on his trunk and proximal extremities. On examination you also note an
unintentional 6-kg (13-lb) weight loss over the last 3 months and conjunctival pallor. A
review of systems is positive for more frequent stomachaches, decreased appetite, and
mild fatigue. You order a laboratory workup.
Which one of the following would be most appropriate at this point?
A) Reassurance that the skin lesions are benign
B) A skin biopsy
C) Referral to a dermatologist
D) CT of the abdomen and pelvis
E) Upper and lower endoscopy

ANSWER: E

This patient’s age, risk factors, red-flag symptoms, and other clinical findings indicate the
need for endoscopy. The Leser-Trélat sign may be defined as the abrupt onset of
multiple seborrheic keratoses, which is an unusual finding that often indicates an
underlying malignancy, most commonly an adenocarcinoma of the stomach. CT is not
an initial approach for diagnosing a suspected malignancy of the stomach or colon.
Further skin evaluation and lifestyle changes, which are indicated, will not address the
need for evaluation of weight loss and other abnormal symptoms and findings.

135. A previously healthy 6-year-old male is brought to your office because he has a
fever. After a complete history and physical examination you are concerned that the
child has Rocky Mountain spotted fever.
Which one of the following would be the most appropriate management?
A) Supportive care only
B) Amoxicillin
C) Doxycycline
D) Rifampin (Rifadin)

ANSWER: C

The treatment of Rocky Mountain spotted fever (RMSF) must be started as soon as the
diagnosis is suspected in order to decrease mortality. Doxycycline is the only approved
therapy for RMSF for individuals of all ages, including children <8 years of age. Of the
other options listed, only rifampin and chloramphenicol have been used for the treatment
of RMSF, but they are not FDA approved.
Providing supportive care or waiting for confirmation of the diagnosis would not be
appropriate. Laboratory tests such as a CBC and chemistries can be helpful in looking
for other causes of a patient’s symptoms but findings will not be specific for RMSF.
Serologies may be helpful but are not available immediately and may be negative early
in the disease process.

136. A 38-year-old female presents for ongoing management of type 2 diabetes mellitus,
obesity, and chronic abdominal pain related to her history of recurrent pancreatitis. She
says that her self-monitored blood glucose has been running in the range of 200–300
mg/dL on most occasions. She is not currently taking any medications but has tried
metformin (Glucophage) and extended-release metformin (Glucophage XR)
unsuccessfully in the past. On both occasions she experienced worsening abdominal
pain and diarrhea. She does not feel she can manage insulin and requests an oral
medication. Her hemoglobin A1c in your office today is 9.0%.
In addition to lifestyle and nutrition counseling,
which one of the following would be the best treatment at this time?
A) Restart metformin
B) Start empagliflozin (Jardiance)
C) Start liraglutide (Victoza)
D) Start sitagliptin (Januvia)

ANSWER: B
Metformin should be used as first-line therapy in type 2 diabetes to reduce microvascular
complications, assist in weight management, reduce the risk of cardiovascular events,
and reduce the risk of mortality in patients (SOR A). Patients who are intolerant of
metformin are unlikely to be successful with a third trial of that agent. Empagliflozin, an
SGLT2 inhibitor, is considered a second-line choice for patients who are intolerant of
metformin. Both sitagliptin, a DPP-4 inhibitor, and liraglutide, a GLP-1 receptor agonist,
should be avoided or used with caution in patients with a history of pancreatitis.

137. According to the American Academy of Pediatrics guidelines, when school


personnel suspect that a child has head lice, which one of the following is the most
appropriate management strategy?

A) The child should be sent home until treated, and a notice should be sent to the
parents of the child’s classmates that a case of lice has occurred at the school
B) The child should be sent home and may return to school after an over-the-counter
treatment has been started
C) The child should be sent home and may return to school after treatment prescribed
by a licensed clinician has been started
D) The child should be sent home and may return to school once treatment has been
completed and the child is free of all nits and lice
E) The child should remain in class and should not be treated unless there is a clear
diagnosis and live lice are seen

ANSWER: B

Metformin should be used as first-line therapy in type 2 diabetes to reduce microvascular


complications, assist in weight management, reduce the risk of cardiovascular events,
and reduce the risk of mortality in patients (SOR A). Patients who are intolerant of
metformin are unlikely to be successful with a third trial of that agent. Empagliflozin, an
SGLT2 inhibitor, is considered a second-line choice for patients who are
intolerant of metformin. Both sitagliptin, a DPP-4 inhibitor, and liraglutide, a GLP-1
receptor agonist, should be avoided or used with caution in patients with a history of
pancreatitis.

138. Which one of the following comorbidities would falsely lower the hemoglobin A1c
level in a patient with type 2 diabetes mellitus?
A) Vitamin B12 deficiency
B) Iron deficiency anemia
C) Hemolytic anemia
D) Chronic kidney disease
E) A history of splenectomy
ANSWER: C

Several factors can alter the hemoglobin A1c value, including variability and erythrocyte
lifespan. When the mean erythrocyte lifespan is increased by a condition such as
asplenia, hemoglobin A1c increases because of increased RBC exposure time for
glycation. Conversely, when the mean erythrocyte lifespan is decreased by conditions
such as hemolytic anemia, hemoglobin A1c is decreased because of reduced RBC
exposure time for glycation. Conditions that decrease erythropoiesis, such as iron
deficiency anemia, increase the mean age of the RBC, thereby increasing hemoglobin
A1c. Severe chronic kidney disease may increase RBC glycation through lipid peroxidase
of hemoglobin and by extending the erythrocyte lifespan due to decreased erythropoietin
levels, causing a false elevation of hemoglobin A1c. Vitamin B12 deficiency also
decreases erythropoiesis and leads to falsely elevated hemoglobin A1c.

139. A 52-year-old male with a long-standing history of hypertension, COPD, type 2


diabetes mellitus, and bipolar disorder is brought to your office by his daughter because
of a new onset of tremors. He is currently taking aspirin, hydrochlorothiazide, atenolol
(Tenormin), glyburide (DiaBeta), lithium, inhaled albuterol, and inhaled tiotropium
(Spiriva). Except for a recent episode of dehydration, his medication has worked well
and no medication changes have been made within the past 2 years. On examination
his heart rate is 52 beats/min and a neurologic examination reveals mild ataxia and
coarse tremors. The remainder of the physical examination is normal.

Which one of the following is the most likely cause of his clinical findings?
A) Albuterol
B) Atenolol
C) Lithium
D) Tiotropium

ANSWER: C

Lithium is a drug with a narrow therapeutic index and a low volume of distribution.
Elderly patients are more likely to develop lithium toxicity due to their lower muscle mass
and age-related decreased glomerular filtration rate (GFR). Chronic toxicity is more
common than acute toxicity and is often precipitated by events causing volume
depletion, such as vomiting, diarrhea, and acute gastroenteritis.
Drugs that impact renal function or volume status, such as ACE inhibitors, NSAIDs, and
diuretics, can also precipitate toxicity. Chronic toxicity often presents with signs and
symptoms related to the gastrointestinal tract (nausea, vomiting, and diarrhea, which can
further worsen toxicity), heart (arrhythmias and conduction delays), and central nervous
system (coarse tremors, ataxia, agitation, and confusion). Albuterol and tiotropium both
cause transient tremors and tachycardia, but are not associated with ataxia. Atenolol is
associated with bradycardia, but not tremors or ataxia (SOR B).

140. A previously healthy 57-year-old patient who smokes is hospitalized and treated
with a fluoroquinolone for community-acquired pneumonia. Which one of the following
could be expected with a 5-day course of antibiotics compared to a longer course in
patients such as this?
A) Slower clinical improvement
B) Higher hospital readmission rates
C) Higher mortality rates
D) Slower resumption of normal activity
E) No difference in clinical outcome
ANSWER: E

A 5-day course of antibiotics for community-acquired pneumonia produces the same


clinical success rates as longer treatment programs. There is no difference in the rate of
clinical improvement, hospital readmissions, or mortality between longer or shorter
treatment courses. Patients are often discharged from the hospital before significant
clinical improvement occurs, leading both patients and physicians to believe that longer
antibiotic courses must be prescribed. Physicians must educate their patients about the
benefit of shorter antibiotic courses, including fewer adverse effects, lower cost, and
lower rates of bacterial resistance.

141. A 45-year-old female has a history of intermittent asthma and her only medication is
an albuterol (Proventil, Ventolin) inhaler. Over the past 2 months her asthma has limited
her activities. She is using her inhaler daily and waking up at night once or twice a week
with a cough.
Which one of the following would be the preferred medication to control her asthma?

A) Fluticasone (Flovent)
B) Salmeterol (Serevent Diskus)
C) Fluticasone/salmeterol (Advair)
D) Montelukast (Singulair)

ANSWER: C
This patient has intermittent asthma that has become at least moderate persistent as
defined by the frequency of her symptoms. The National Asthma Education and
Prevention guidelines recommend a moderate-dose inhaled corticosteroid (ICS) with a
long-acting bronchodilator as the preferred treatment in moderate persistent asthma.
Fluticasone/salmeterol at a dosage of 250/50 g is the only option that fits this category.
Montelukast alone is an alternative treatment for mild persistent asthma (SOR A).

Ref: McCracken JL, Veeranki SP, Ameredes BT, Calhoun WJ: Diagnosis and management of asthma in
adults: A review. JAMA 2017;318(3):279-290.

142. In addition to group B Streptococcus (GBS), which one of the following is the most
common cause of neonatal sepsis?

A) Escherichia coli
B) Group A Streptococcus
C) Listeria monocytogenes
D) Staphylococcus aureus
E) Streptococcus pneumonia
Item 142

ANSWER: A
Newborns with sepsis may have focal signs of infection such as pneumonia or
respiratory distress syndrome, but they also may have nonfocal signs and symptoms. In
the newborn period the two most common causes of neonatal sepsis are group B
Streptococcus and Escherichia coli. Listeria monocytogenes was once a more
common cause but it is now uncommon. Streptococcal pneumonia is an uncommon
cause of sepsis in neonates. Staphylococcus aureus and group A Streptococcus are
not as common but should be
considered in newborns with cellulitis.

Ref: Polin RA, Watterberg K, Benitz W, Eichenwald E: The conundrum of early-onset sepsis. Pediatrics
2014;133(6):1122-1123. 2) Hay WW Jr, Levin MJ, Deterding RR, Abzug MJ: Current Diagnosis &
Treatment: Pediatrics ,ed 23. McGraw-Hill Education, 2016, pp 1150-1169.

143. The U.S. Preventive Services Task Force recommends which one of the following
screening options for major depressive disorder (MDD) in adolescents 12–18 years of
age?
A) Do not screen because the harms outweigh the benefits
B) Do not screen because valid screening tools are not available for this population
C) Do not screen because reliable treatment options are not effective unless MDD is
clinically apparent
D) Screen if systems are in place for diagnosis, treatment, and follow-up
E) The evidence is currently insufficient to recommend for or against screening

ANSWER: D

The U.S. Preventive Services Task Force (USPSTF) recommends screening for major
depressive disorder (MDD) in adolescents 12–18 years of age. Screening should be
implemented with systems in place to ensure adequate diagnosis, effective treatment,
and appropriate follow-up (B recommendation). The USPSTF found adequate evidence
that screening instruments for depression in adolescents can accurately identify MDD in
this age group in primary care settings, and that the treatment of MDD detected through
screening in this age group is associated with moderate benefits. Based on current
evidence, the USPSTF also concluded that the evidence is insufficient to assess the
balance of benefits and harms from screening for MDD in children 11 years of age and
younger.

Ref: Final Recommendation Statement: Depression in Children and Adolescents: Screening . US


Preventive Services Task Force, 2016.

144. An otherwise healthy 64-year-old male comes to your office accompanied by his
wife because of tinnitus that has affected both ears for the last 3 years. It has been most
troublesome at bedtime. His wife says that he is becoming irritable and depressed
because he is bothered by the buzzing in his ears many times during the day. His only
medication is allopurinol (Zyloprim) for the prevention of gout. The most likely identifiable
cause of this patient’s tinnitus is:

A) medication
B) Meniere’s disease
C) temporomandibular joint dysfunction
D) sensorineural hearing loss
E) impacted cerumen

ANSWER: D

Although tinnitus is idiopathic, sensorineural hearing loss is the most common identified
cause. It can also be caused by other otologic, vascular, neoplastic, neurologic,
pharmacologic, dental, and psychological factors. Almost all patients with tinnitus should
undergo audiometry with tympanometry, and some patients require neuroimaging or
assessment of vestibular function with electronystagmography. Counseling may also
improve the chances of successful subsequent treatment. Several medications can
cause tinnitus, but allopurinol is not one of them.

Ref: Yew KS: Diagnostic approach to patients with tinnitus. Am Fam Physician 2014;89(2):106-113.

145. A 49-year-old male is concerned about lesions on his penis that he has noticed
over the past 6 months. He was circumcised as a child and has had the same female
sexual partner for 5 years. He does not have any pain, itching, or dysuria. On
examination you note multiple
reddish-blue papules on the scrotum and a few similar lesions on the shaft of the penis.
The most likely diagnosis is
A) pearly penile papules
B) lichen nitidus
C) lichen sclerosus
D) angiokeratomas
E) squamous cell carcinoma in situ (Bowen’s disease)

ANSWER: D

Penile lesions are usually easily diagnosed from clinical findings. Pearly penile papules
are common and benign, and present as small, skin-colored, dome-shaped papules in a
circular pattern around the coronal sulcus. Lichen nitidus is benign but uncommon. It
presents as discrete, pinhead-sized hypopigmented papules that are asymptomatic.
Papules are often found scattered all over the penis, as well as on the abdomen and
upper extremities.
Lichen sclerosus is more common and appears as hypopigmented lesions with the
texture of cellophane. The lesions are usually located on the glans or prepuce. Atrophy,
erosions, and bullae are common, and patients often present with itching, pain, bleeding,
and possibly phimosis or obstructed voiding. Lichen sclerosus is associated with
squamous cell cancer in a small percentage of cases. Carcinoma in situ is a
premalignant condition that is more common in uncircumcised males over age 60.
Lesions are typically beefy red, raised, irregular plaques and can be found on the glans,
meatus, frenulum, coronal sulcus, and prepuce. Lesions can be ulcerated or crusted.
Pruritus and pain are common. A biopsy is important for making the diagnosis.
Angiokeratomas are lesions that are usually asymptomatic, circumscribed, red or bluish
papules. They may appear solely on the glans of the penis, but are also found on the
scrotum, abdomen, thighs, groin, and extremities. They may be misdiagnosed as pearly
papules or carcinoma. Treatment is not necessary unless the lesions are bleeding or
extensive. It is important to realize that angiokeratomas on the shaft of the penis, the
suprapubic region, or the sacral region can be associated with Fabry disease. Patients
with this finding should be promptly referred.

Ref: Buechner SA: Common skin disorders of the penis. BJU Int 2002;90(5):498-506. 2) Teichman JM, Sea
J, Thompson IM, Elston DM: Noninfectious penile lesions. Am Fam Physician 2010;81(2):167-174. 3) Habif
TP: Clinical Dermatology: A Color Guide to Diagnosis and Therapy , ed 6. Elsevier, 2016, p 914.

146. A 36-year-old female presents with a 10-year history of daily headaches. The
headaches are bilateral, have a pressure and tightening quality, and are not aggravated
by activity. They tend to worsen as the day progresses. There is no associated
prodrome, nausea, or sensitivity to light or noise. A neurologic examination is normal.
Which one of the following has been shown to reduce the severity and duration of this
type of headache?

A) Amitriptyline
B) OnabotulinumtoxinA (Botox)
C) Propranolol
D) Sertraline (Zoloft)
E) Topiramate (Topamax)

ANSWER: A

Amitriptyline may reduce headache duration and severity compared with placebo for
chronic tension-type headaches (SOR B). SSRIs have no proven benefit for headache
prophylaxis over placebo or tricyclic antidepressants in patients with chronic daily
headaches. Propranolol reduces the frequency of migraine headaches, although its
effectiveness for chronic migraine is unclear. Propranolol is not effective for tension
headaches. Topiramate can reduce the frequency of chronic migraine headaches by
50% but is not effective for tension-type headaches. OnabotulinumtoxinA has been
shown to reduce headache frequency in chronic migraine, but evidence of its
effectiveness is lacking for chronic tension-type headaches.

Ref: Yancey JR, Sheridan R, Koren KG: Chronic daily headache: Diagnosis and management. Am Fam
Physician 2014;89(8):642-648.

147. A 38-year-old female with a 5-year history of diabetes mellitus has developed a
“pins and needles” sensation in her feet. Which one of the following is considered first-
line therapy for her condition?

A) Acupuncture
B) Lidocaine 5% spray
C) Oxycodone (Roxicodone)
D) Pregabalin (Lyrica)
E) Venlafaxine (Effexor XR)
ANSWER: D

Pregabalin is considered first-line therapy for painful diabetic peripheral neuropathy


(SOR A). Based on a meta-analysis, the American Academy of Neurology recommends
pregabalin as first-line medication and gabapentin as a first-line alternative. While
opioids such as oxycodone may provide a possible benefit in the treatment of
neuropathy, the risk of dependency and adverse effects limits their use to patients with
pain not relieved by first-line therapies. Acupuncture is not recommended as a first-line
therapy due to the lack of high-quality, randomized, controlled trials. Venlafaxine and
lidocaine 5% spray are considered second-line therapies.

Ref: Snyder MJ, Gibbs LM, Lindsay TJ: Treating painful diabetic peripheral neuropathy: An update. Am Fam
Physician 2016;94(3):227-234.

148. A 32-year-old female sees you for evaluation of hair loss. On examination she has
a smooth, circular area of complete hair loss on her scalp with no other skin changes.
Which one of the following would you recommend?
A) An oral antifungal agent
B) Topical minoxidil (Rogaine)
C) Topical immunotherapy
D) Topical corticosteroids
E) Intralesional corticosteroids

ANSWER: E

This patient has alopecia areata, which is a chronic, relapsing, immune-mediated


inflammatory disorder affecting hair follicles that results in patchy hair loss. The
treatment of choice is intralesional corticosteroid injections. Topical immunotherapy is
reserved for patients with extensive disease, such as >50% scalp involvement. Topical
corticosteroids are less effective and are usually reserved for children and adults who
cannot tolerate intralesional injections. Minoxidil is used for androgenetic alopecia and is
less effective for alopecia areata. Oral antifungal drugs are used to treat tinea capitis.

Ref: Phillips TG, Slomiany WP, Allison R: Hair loss: Common causes and treatment. Am Fam Physician
2017;96(6):371-378.

149. You are evaluating a 64-year-old female in the emergency department for
pyelonephritis. Her past medical history is negative and she has previously been in good
health. The patient appears acutely ill but is oriented. On examination her weight is 100
kg (220 lb), her temperature is 38.9°C (102.0°F), her pulse rate is 110 beats/min, her
respiratory rate is 24/min, her blood pressure is 136/72 mm Hg, and her oxygen
saturation is 94% on room air. Initial laboratory findings include a venous lactate level of
4.0 mmol/L (N 0.6–1.7).
You decide to start normal saline intravenously. Which one of the following would be the
most appropriate initial rate?
A) 100 mL/hr
B) 150 mL/hr
C) 200 mL/hr
D) 3000 mL over 30 minutes
E) 3000 mL over 3 hours
ANSWER: E

The Surviving Sepsis Campaign recommends that patients with elevated serum lactate
or hypotension receive isotonic intravenous fluids such as normal saline or lactated
Ringer’s solution at an initial rate of 30 mL/kg in the first 3 hours using small boluses of
approximately 500 mL. A serum lactate value >36 g/dL (4 mmol/L) is correlated with
increased severity of illness and poorer outcomes even if hypotension is not yet present.
Patients who are hypotensive or whose serum lactate level is >36 g/dL require
intravenous fluids or colloid to expand their circulating volume and effectively restore
perfusion pressure. The administration of 30 mL/kg of fluid is recommended as a fluid
challenge, which should be started as early as possible in the course of septic shock.

Ref: Rhodes A, Evans LE, Alhazzani W, et al: Surviving Sepsis Campaign: International guidelines for
management of sepsis and septic shock: 2016. Crit Care Med 2017;45(3):486-552.

150. A 54-year-old female sees you for a wellness examination. Her last screening
mammography 10 years ago revealed dense breasts but was otherwise normal.
A past history of which one of the following would indicate the need for MRI of the
breasts?
A) Very dense breasts
B) Morbid obesity
C) Combination estrogen/progesterone therapy for the last 3 years
D) Chest radiation for Hodgkin’s disease
E) Radioiodine treatment for Graves disease

ANSWER: D

MRI of the breasts should be reserved for women at very high risk for breast cancer
such as those with genetic mutations, a history of breast irradiation, or a very high-risk
family history. Women who had chest radiation therapy during childhood or adolescence,
generally for Hodgkin’s disease, are at an extremely high risk for breast cancer.

Ref: Nattinger AB, Mitchell JL: Breast cancer screening and prevention. Ann Intern Med
2016;164(11):ITC81-ITC96.

151. You prescribe amoxicillin suspension, 480 mg twice daily for 10 days, for a child
who weighs 12 kg (26 lb). To decrease the risk of a dosing error with this prescription,
you ask your nurse to provide the parents with appropriate education and
A) a written copy of the prescription
B) a disposable teaspoon
C) a medication cup
D) an oral syringe

ANSWER: D

A recent office-based, randomized, controlled trial demonstrated that over 40% of


parents made dosing errors with medicine cups compared to a 17% error rate with an
oral syringe. Oral syringes are marked with milliliters, not cubic centimeters, so dosages
should use milliliters. Tableware varies in volume and should not be used. Medication
bottles are not marked for measuring dosing volumes. Discharge instructions are
important, but a written copy of this prescription will not help the guardian measure to the
nearest milliliter.

Ref: Yin HS, Parker RM, Sanders LM, et al: Liquid medication errors and dosing tools: A randomized
controlled experiment. Pediatrics 2016;138(4):e20160357. 2) Ebell MH, Grad R: Top 20 research studies of
2016 for primary care physicians. Am Fam Physician 2017;95(9):572-579.

152. A 26-year-old female presents with a skin rash and chronic diarrhea. She reports
being previously diagnosed with eczema, and while the rash has responded well to
topical corticosteroids it flares when they are stopped. The skin rash is very itchy and
appears as mildly erythematous papules and vesicles clustered on the elbows and
knees, as well as the posterior neck and scalp. A comprehensive metabolic panel is
normal, and a CBC reveals a mild microcytic, hypochromic anemia. Antinuclear
antibodies are negative, a TSH level is normal, and a tissue transglutaminase antibody
test is positive.
Which one of the following is the most likely diagnosis?
A) Cutaneous lupus erythematosus
B) Dermatitis herpetiformis
C) Eczema
D) Eczema herpeticum
E) Lichen simplex chronicus

ANSWER: B

This patient has celiac disease with both intestinal and extraintestinal manifestations
(diarrhea and dermatitis herpetiformis, respectively). Iron deficiency anemia due to
chronic blood loss is a common finding in patients with celiac disease. Eczema
herpeticum is the appearance of a herpetic infection complicating eczema. This is a
serious acute problem that can be life-threatening in severe cases. Lichen simplex
chronicus is a chronic skin condition that is perpetuated by scratching. Lesions are
commonly thickened and excoriated. Diarrhea and anemia are not associated with this
disorder. While eczema is possible based on the description of the rash, the intestinal
manifestations and positive tissue transglutaminase antibody in this patient point to
celiac disease. This patient’s rash does not suggest cutaneous lupus, which is also
unlikely given the negative antinuclear antibody test.

Ref: Pelkowski TD, Viera AJ: Celiac disease: Diagnosis and management. Am Fam Physician
2014;89(2):99-105.

153. You see a 26-year-old male for the first time. He has a history of major depression
over the past 4–5 years. He currently does not take any medications. His psychiatric
history reveals at least two episodes of mania, most recently 1 year ago when he was
hospitalized during the episode.
Which one of the following is CONTRAINDICATED as monotherapy in treating this
patient’s depression at this time?
A) Divalproex (Depakote)
B) Fluoxetine (Prozac)
C) Lamotrigine (Lamictal)
D) Lithium
E) Quetiapine (Seroquel)
ANSWER: B

With a history of major depression and mania, this patient would be classified as having
bipolar I disorder. SSRIs can be used to treat major depression in these patients, but
they are insufficient as monotherapy for controlling or preventing mania. They may even
precipitate a manic episode in patients with bipolar I disorder and are contraindicated as
monotherapy for these patients (SOR B). Patients with bipolar I disorder are sometimes
treated with a combination of antidepressants such as an SSRI, plus a mood stabilizer,
which includes anticonvulsants (lamotrigine, divalproex), atypical antipsychotics
(quetiapine), or lithium, but this is considered second-line therapy. All of these mood
stabilizers are effective in the treatment and prevention of mania. The use of these
medications in combination with psychotherapy generally provides the best results in
patients with bipolar I disorder.

Ref: Bobo WV: The diagnosis and management of bipolar I and II disorders: Clinical practice update. Mayo
Clin Proc 2017;92(10):1532-1551.

154. A 35-year-old white female presents with recurrent wheezing and coughing over the
past few weeks, and recent production of brown sputum plugs. She is a regular patient
of yours and has a long history of asthma and multiple allergies. She has been treated
four times in the last 3 months for asthma exacerbations and generally feels better the
first day she takes her corticosteroid, but any attempt at tapering leads to a recurrence of
symptoms. She previously had good control of her asthma, although she has required
regular use of a high-dose inhaled corticosteroid and a long-acting -agonist. In spite of
just completing a course of levofloxacin (Levaquin) for suspected pneumonia she returns
today with a recurrence of the same symptoms.
A physical examination is unremarkable with the exception of diffuse expiratory
wheezing. She has no fever or other abnormal vital signs. A chest radiograph shows
opacities in the upper and middle lobes and a CBC is concerning for eosinophilia.

Which one of the following is the most likely diagnosis?


A) Allergic bronchopulmonary aspergillosis
B) Community-acquired pneumonia
C) Pulmonary embolism
D) Medication nonadherence

ANSWER: A

Allergic bronchopulmonary aspergillosis (ABPA) affects 1%–12% of immunocompetent


patients with asthma and is important to consider in patients with recurrent
exacerbations because it can cause permanent lung damage if it is undetected and
untreated. The symptoms alone are insufficient for a diagnosis, but this clinical
presentation should prompt consideration of the diagnosis, and some of the symptoms
and findings noted are included in the diagnostic criteria. The major diagnostic criteria
include the presence of asthma or cystic fibrosis and immediate skin reactivity to
Aspergillus antigens, peripheral eosinophilia, transient pulmonary infiltrates or opacities,
central bronchiectasis on a chest radiograph or CT, serum precipitating antibodies to
Aspergillus fumigatus , and elevated Aspergillus IgE- and/or IgG-specific antibodies.
Minor criteria that support the diagnosis include production of brownish mucus plugs,
identification of Aspergillus in the sputum, and delayed skin sensitivity to Aspergillus.
Pneumonia is unlikely in this case given recent treatment with a respiratory
fluoroquinolone and a lack of common symptoms such as fever, chills, tachycardia,
tachypnea, and pleuritic chest pain, along with a cough productive of mucopurulent
sputum. The most common symptoms of pulmonary embolism include dyspnea, chest
pain, syncope, tachypnea, and a cough. While medication nonadherence may increase
asthma exacerbations and wheezing, it would be unlikely to be related to the new brown
mucus production.

Ref: Watkins RR, Lemonovich TL: Diagnosis and management of community-acquired pneumonia in adults.
Am Fam Physician 2011;83(11):1299-1306. 2) Wilbur J, Shian B: Diagnosis of deep venous thrombosis
and pulmonary embolism. Am Fam Physician 2012;86(10):913-919. 3) Greenberger PA, Bush RK, Demain
JG, et al: Allergic bronchopulmonary aspergillosis. J Allergy Clin Immunol Pract 2014;2(6):703-708. 4)
Mouthon L, Dunogue B, Guillevin L: Diagnosis and classification of eosinophilic granulomatosis with
polyangiitis (formerly named Churg-Strauss syndrome). J Autoimmun 2014;48-49:99-103.

155. An 89-year-old female with a history of hypertension and glaucoma is brought to


the emergency department by her family with shortness of breath. She has been trying
to get her home ready for sale prior to moving into an assisted living facility. She says
that she has not been sleeping well for weeks because she is worried about the move.
On admission the patient has a blood pressure of 140/92 mm Hg, a pulse rate of 86
beats/min, a respiratory rate of 26/min, a temperature of 36.6°C (97.9°F), and an oxygen
saturation of 95% on room air. A physical examination is normal other than faint basilar
crackles. A chest radiograph shows a slightly prominent cardiac silhouette, peribronchial
cuffing, and coarse perihilar lung markings. An EKG reveals a normal sinus rhythm with
global T-wave inversion of the precordial and limb leads. Her troponin I peaks at 0.953
ng/mL (N 0.000–0.780). Echocardiography reveals a normal size right ventricle with
moderate right ventricular hypokinesis, left ventricular apical ballooning, a left ventricular
ejection fraction estimated at 30%, and a moderately increased pulmonary artery
pressure estimated at 43 mm Hg. A radionuclide myocardial perfusion imaging study is
normal.
Which one of the following is the most likely diagnosis?
A) Acute coronary syndrome
B) Acute pericarditis
C) Cardiac amyloidosis
D) Takotsubo cardiomyopathy
E) Viral myocarditis

ANSWER: D

Takotsubo cardiomyopathy (TTC) is also known as apical ballooning syndrome and


stress-induced cardiomyopathy. It generally occurs in postmenopausal women with a
mean age of 62–76 years. The clinical presentation is similar to that of acute coronary
syndrome. Evaluation with an EKG, cardiac biomarkers, and imaging is needed to
differentiate between these two conditions. This patient presents with classic apical and
midsegment left ventricular hypokinesis, or apical ballooning, and a new T-wave
inversion with modest elevations in cardiac troponin. While she has an identifiable
characteristic emotional stressor, up to one-third of patients with TTC do not have an
identifiable stressor.
In this scenario, a negative myocardial perfusion scan makes coronary artery disease or
acute coronary syndrome unlikely. Patients with viral myocarditis typically present with
fever, myalgia, and signs and symptoms of heart failure following a viral syndrome.
Cardiac amyloidosis is a restrictive cardiomyopathy that is typically associated with
thickened walls of both ventricles and markedly dilated atria. Patients with acute
pericarditis present with chest pain, a pericardial friction rub on examination, an ST-
segment elevation on EKG, and a pericardial effusion on echocardiography.

Ref: Scantlebury DC, Prasad A: Diagnosis of Takotsubo cardiomyopathy. Circ J 2014;78(9):2129-2139. 2)


Kasper DL, Fauci AS, Hauser SL, et al (eds): Harrison’s Principles of Internal Medicine, ed 19. McGraw-
Hill, 2015, pp 1553-1577.

156. Which one of the following is the leading cause of cancer death in men in the
United States?
A) Colorectal cancer
B) Liver cancer
C) Lung cancer
D) Non-melanoma skin cancer
E) Prostate cancer

ANSWER: C

According to the CDC, the leading causes of cancer death in men from 2011–2015 were
lung cancer (53.8 deaths per 100,000 per year), prostate cancer (19.5 deaths per
100,000 per year), colorectal cancer (17.3 deaths per 100,000 per year), and pancreatic
cancer (12.6 deaths per 100,000 per year).

Ref: US Cancer Statistics Working Group: US cancer statistics data visualization tool, based on November
2017 submission data (1999–2015). Centers for Disease Control and Prevention and National Cancer
Institute, 2018.

157. You see a patient who is being treated for opioid use disorder with buprenorphine.
Which one of the following can be used as adjuvant treatment to reduce stress-related
opioid cravings and increase abstinence?
A) Clonidine (Catapres)
B) Methadone
C) Naloxone
D) Naltrexone (Vivitrol)
E) Nifedipine (Procardia)

ANSWER: A

Clonidine (0.1–0.3 mg every 6–8 hours) is a useful adjunct to buprenorphine in the


treatment of opioid use disorder to help increase the rates of abstinence and decrease
stress-related opioid cravings (SOR C). Naloxone is an opioid antagonist used to treat
overdoses. Nifedipine is a common antihypertensive like clonidine but it has no role in
the treatment of opioid use disorder. Methadone and naltrexone are used to treat opioid
use disorder but neither of these agents would be used simultaneously with
buprenorphine.
Ref: Zoorob R, Kowalchuk A, Mejia de Grubb M: Buprenorphine therapy for opioid use disorder. Am Fam
Physician 2018;97(5):313-320.

158. A 5-month-old female is brought to your office for evaluation of a fever to 103.1°F
over the past 2 days. Her immunizations are up to date. On examination she does not
appear ill. Her vital signs include a rectal temperature of 39.0°C (102.2°F), a heart rate of
90 beats/min, a respiratory rate of 40/min, and an oxygen saturation of 98% on room air.
The child is alert and responsive, appears well hydrated, and has no rash or petechiae.
The HEENT examination, including fontanelles, is normal, the chest is clear, there are
no murmurs, and the abdomen is soft.

Which one of the following would be most appropriate at this time?


A) No further testing or treatment at this visit, with close outpatient follow-up
B) A urinalysis and culture, with close outpatient follow-up
C) A CBC with differential, a urinalysis and culture, and close outpatient follow-up
D) A CBC, a urinalysis and culture, a chest radiograph, a lumbar puncture, and
consideration of inpatient monitoring
E) A CBC with differential, blood cultures, a urinalysis and culture, a chest radiograph, a
lumbar puncture, and empiric inpatient antibiotic treatment

ANSWER: B

For children with a fever without localizing signs, management depends on the child’s
age and findings on examination. For children 3–36 months of age with a fever 39°C
(102°F), reassurance that this is likely a self-limited viral infection is appropriate, with
instructions to return if there are new signs or symptoms. If the temperature is >39° and
the child has received appropriate vaccines on schedule, then a urinalysis and culture
should be performed for all children <6 months of age and for uncircumcised boys <2
years of age. A more extensive workup would be appropriate for children <3 months of
age.

Ref: Kliegman RM, Stanton BF, Geme JW III, et al (eds): Nelson Textbook of Pediatrics, ed 20. Elsevier
Saunders, 2016, pp 1280-1283.

159. As the newly appointed medical director in your primary care community health
center, you identify antibiotic stewardship as a priority. You communicate with all clinical
and administrative staff members to express a consistent message to patients about
appropriate indications for antibiotic use.

Which one of the following is an example of an action aimed at improving antibiotic


prescribing practices?
A) Avoiding the use of in-house rapid strep testing for pediatric patients
B) Considering patient expectations and satisfaction when prescribing antibiotic therapy
C) Ensuring that patients with common cold symptoms are seen for evaluation
D) Reducing the use of clinical decision support tools in the electronic health record
E) Writing a delayed antibiotic prescription when appropriate

ANSWER: E
The core elements of antibiotic stewardship are commitment, action for policy and
practice, tracking and reporting, and education and expertise. Delayed prescription
strategies for appropriate conditions such as upper respiratory infections and otitis media
are effective in reducing antibiotic use. Using evidence-based guidelines, clinical support
tools, and triage systems also results in decreased antibiotic use. Using call centers or
nurse triage reduces unnecessary visits, and nurses can effectively manage upper
respiratory infection symptoms via phone consultations.

Ref: Sanchez GV, Fleming-Dutra KE, Roberts RM, Hicks LA: Core elements of outpatient antibiotic
stewardship. MMWR Recomm Rep 2016;65(6):1-12.

160. The presence of RBC casts on microscopic examination of a spun urine sediment
is pathognomonic for which one of the following conditions?
A) Acute glomerulonephritis
B) Acute papillary necrosis
C) Acute pyelonephritis
D) Acute tubular necrosis
E) Nephrotic syndrome

ANSWER: A

The presence of RBC casts on microscopic examination of fresh spun urine sediment
indicates acute glomerulonephritis, which may be due to a variety of immunologic,
infectious, or postinfectious causes, with the classic example being poststreptococcal
glomerulonephritis. Many cases of acute glomerulonephritis require renal biopsies for
definitive diagnosis and treatment. Acute pyelonephritis causes bacteriuria, pyuria, and
possibly WBC casts. Acute tubular necrosis is usually associated with hypotension,
acute blood loss, sepsis, or rhabdomyolysis, and there may be granular or epithelial
casts in the urine sediment, but not RBC casts. Acute papillary necrosis is the result of
medullary (not cortical) renal injury, and although gross hematuria may be present, there
are no RBC casts. Nephrotic syndrome is not associated with hematuria but rather with
massive proteinuria (>3.5 g/24 hr).

Ref: Kasper DL, Fauci AS, Hauser SL, et al (eds): Harrison’s Principles of Internal Medicine, ed 19.
McGraw-Hill, 2015, pp 289, 294, 1834-1836.

161. A 15-year-old male sees you after injuring his right index finger while playing
volleyball. He has pain and a flexion deformity at the distal interphalangeal (DIP) joint.
Which one of the following would be an indication for further evaluation before splinting?
A) The patient wants to continue athletic activities
B) The patient first presented for treatment 3 weeks after the injury
C) The patient is unable to passively fully extend the joint
D) The patient is unable to actively fully extend the joint
E) A radiograph shows a bony avulsion of 10% of the joint space

ANSWER: C
Mallet finger, an injury to the distal extensor tendon of the finger at the distal
interphalangeal (DIP) joint,is usually caused by forceful flexion of an extended DIP joint.
This is frequently the result of being struck by an object such as a ball. The inability to
actively extend the DIP joint is a hallmark of mallet finger.
The inability to passively extend the DIP joint completely may be an indication of trapped
soft tissue or bone that may require surgery. Up to one-third of distal extensor tendon
injuries are associated with an avulsion fracture, and if the avulsion is greater than 30%
of the joint space, referral to an orthopedist is recommended. Splinting with strict use of
the splint and avoidance of any flexion of the DIP joint is the recommended treatment,
and is beneficial even with a delayed presentation. Athletic activities may be continued
with the splint in place.
Ref: Leggit JC, Meko CJ: Acute finger injuries: Part I. Tendons and ligaments. Am Fam Physician
2006;73(5):810-816. 2) Daniels JM, DeCastro A, Stanton RN: Finger injuries: 5 cases to test your skills. J
Fam Pract 2013;62(6):300-304.

162. A 42-year-old male presents with a fever, cough, and chest pain. A rapid influenza
test is positive.

Which one of the following would be most appropriate for the management of this
patient’s pleuritic chest pain?
A) Colchicine (Colcrys)
B) Hydrocodone
C) Ibuprofen
D) Prednisone
E) Tramadol (Ultram)

ANSWER: C
NSAIDs such as ibuprofen should be used as first-line treatment for the control of
pleuritic pain (SOR B). While NSAIDs do not have the analgesic potency of narcotics,
they do not cause respiratory suppression or change the patient’s sensorium.
Corticosteroids should be reserved for patients who cannot take NSAIDs.
Ref: Reamy BV, Williams PM, Odom MR: Pleuritic chest pain: Sorting through the differential diagnosis. Am
Fam Physician 2017;96(5):306-312.

163. A 55-year-old male sees you for an annual health maintenance visit. He is a former
smoker and has a history of type 2 diabetes mellitus, hypertension, and hyperlipidemia.
He had a normal colonoscopy at age 50, and had an ST-elevation myocardial infarction
2 years ago treated with a drug-eluting stent. He is currently asymptomatic and does not
have any chest pain, hypoglycemia, dyspepsia, melena, or rectal bleeding. His
medications include metformin (Glucophage), 2000 mg daily; glimepiride (Amaryl), 2 mg
daily; bisoprolol (Zebeta), 5 mg daily; losartan/hydrochlorothiazide (Hyzaar), 50 mg/12.5
mg daily; rosuvastatin (Crestor), 20 mg daily; clopidogrel (Plavix), 75 mg daily; and
aspirin, 81 mg daily. His blood pressure is 128/76 mm Hg and his heart rate is 63
beats/min. A physical examination is unremarkable. His hemoglobin A1c is 6.4%.
You recommend that the patient stop taking
A) aspirin
B) clopidogrel
C) aspirin and clopidogrel
D) metformin

ANSWER: B
Patients with drug-eluting stents should be on dual antiplatelet therapy with aspirin plus
a thienopyridine such as clopidogrel for a minimum of 1 year. At the time of this patient’s
visit, 2 years after the stent placement, there is no indication to continue clopidogrel, but
aspirin therapy should be continued indefinitely. All of the patient’s other medications
have current active indications and should be continued, although if the patient
experiences hypoglycemia, the sulfonylurea could be decreased or discontinued.
Ref: Levine GN, Bates ER, Bittl JA, et al: 2016 ACC/AHA guideline focused update on duration of dual
antiplatelet therapy in patients with coronary artery disease: A report of the American College of
Cardiology/American Heart Association Task Force on Clinical Practice Guidelines: An update of the 2011
ACCF/AHA/SCAI guideline for percutaneous coronary intervention, 2011 ACCF/AHA guideline for coronary
artery bypass graft surgery, 2012 ACC/AHA/ACP/AATS/PCNA/ SCAI/STS guideline for the diagnosis and
management of patients with stable ischemic heart disease, 2013 ACCF/AHA guideline for the management
of ST-elevation myocardial infarction, 2014 AHA/ACC guideline for the management of patients with non–
ST-elevation acute coronary syndromes, and 2014 ACC/AHA guideline on perioperative cardiovascular
evaluation and management of patients undergoing noncardiac surgery. Circulation 2016;134(10):e123-

164. In a 60-year-old patient who has not previously received pneumococcal vaccine,
which one of the following would be an indication for both 13-valent pneumococcal
conjugate vaccine (PCV13, Prevnar 13) and 23-valent pneumococcal polysaccharide
vaccine (PPSV23, Pneumovax 23)?
A) Alcoholism
B) Chronic renal failure
C) Cigarette smoking
D) COPD
E) Diabetes mellitus

ANSWER: B

Both 13-valent pneumococcal conjugate vaccine (PCV13) and 23-valent pneumococcal


polysaccharide vaccine (PPSV23) are recommended for patients with chronic renal
failure. Indications for PPSV23 alone in immunocompetent persons younger than 65
include chronic lung disease, diabetes mellitus, chronic heart disease, smoking, and
alcoholism.
Ref: Kobayashi M, Bennett NM, Gierke R, et al: Intervals between PCV13 and PPSV23 vaccines:
Recommendations of the Advisory Committee on Immunization Practices (ACIP). MMWR Morb Mortal
Wkly Rep 2015;64(34):944-947.

165. A 36-year-old male presents with a 2-day history of painless right-sided facial
droop. There are no associated symptoms and his medical history is otherwise
unremarkable. An examination is remarkable for an unfurrowed right brow, mouth droop,
a sagging right lower eyelid, and a complete inability to move the muscles of the right
face and forehead. No other weakness is elicited and no rash is seen.

Which one of the following would be the most appropriate management at this point?
A) Reassurance only
B) Valacyclovir (Valtrex) alone
C) A tapering dose of prednisone alone
D) Valacyclovir and a tapering dose of prednisone
E) Immediate transfer to the emergency department
ANSWER: D

Early recognition and effective treatment of acute Bell’s palsy (idiopathic facial paralysis)
has been shown to decrease the risk of chronic partial paralysis and pain.
Corticosteroids have been shown in a meta-analysis to decrease chronic symptoms, but
a Cochrane meta-analysis of 10 studies concluded that antiviral medication along with
corticosteroids is significantly more effective than corticosteroids alone.
The medications are most effective if started within 72 hours of symptom onset. The
same analysis showed that antiviral medications alone were less effective than
corticosteroids alone. This patient’s presentation is not consistent with stroke or another
emergency. Because supranuclear input to the facial nerves comes from both cerebral
hemispheres, strokes and other central pathologies affecting the facial nerves typically
spare the forehead, which is not the case in this patient.
Ref: Baugh RF, Basura GJ, Ishii LE, et al: Clinical practice guideline: Bell’s palsy executive summary.
Otolaryngol Head Neck Surg 2013;149(5):656-663. 2) Gagyor I, Madhok VB, Daly F, et al: Antiviral
treatment for Bell’s palsy (idiopathic facial paralysis). Cochrane Database Syst Rev 2015;(11):CD001869.

166. A 35-year-old female presents with fatigue. She has been falling asleep at work for
the past 6 weeks. She is married with two children and works as a nurse at the
community hospital. Since she returned to work 12 weeks ago after maternity leave, her
infant has had multiple respiratory infections and has not slept well through the night.
Her menstrual cycle has been irregular and heavy for the past several months. A CBC
and TSH level are normal.

Which one of the following laboratory tests would be appropriate at this visit?
A) 25-Hydroxyvitamin D
B) -hCG
C) D-dimer
D) A serum antibody test for Lyme disease

ANSWER: B

25-Hydroxyvitamin D levels should not be measured in patients presenting with fatigue


(SOR A). A serum antibody test for Lyme disease or a D-dimer would not be indicated
for this patient based on her history and symptoms. Because of the patient’s history of
irregular menses, a -hCG level would be indicated.
Ref: LeFevre ML, LeFevre NM: Vitamin D screening and supplementation in community-dwelling adults:
Common questions and answers. Am Fam Physician 2018;97(4):254-260.

167. In a patient presenting with truncal obesity, hypertension, type 2 diabetes mellitus,
hirsutism, osteopenia, and skin fragility, which one of the following tests is needed to
confirm the diagnosis of Cushing syndrome?
A) A dexamethasone suppression test
B) Inferior petrosal sinus sampling
C) Plasma corticotropin
D) Plasma free cortisol
E) Urinary free cortisol

ANSWER: E
In a patient presenting with obesity, hypertension, type 2 diabetes mellitus, and
hirsutism, who also has thin skin and osteopenia, an elevated 24-hour collection
showing high urinary free cortisol confirms the presence of Cushing syndrome. The
dexamethasone suppression test, though still commonly used, no longer has a place in
the diagnosis and treatment of patients with Cushing syndrome. Corticotropin-dependent
and corticotropin-independent causes of Cushing syndrome can be separated by
measuring plasma corticotropin. Plasma free cortisol measurements should be obtained
only to determine the success or failure of transsphenoidal microadenomectomy or
adrenalectomy. Inferior petrosal sinus sampling is used to confirm the source of
corticotropin secretion before surgical intervention.
Ref: Loriaux DL: Diagnosis and differential diagnosis of Cushing’s syndrome. N Engl J Med
2017;376(15):1451-1459.

168. A 42-year-old female presents with shortness of breath that has slowly worsened
over the past 6 months. She can now walk only 10 feet without becoming short of
breath. She does not have a cough or chest pain. Her history is significant only for
obesity. She smoked one pack of cigarettes per day for 20 years and quit smoking 6
years ago. Her blood pressure is 138/88 mm Hg, pulse rate 92 beats/min, respiratory
rate 18/min, and oxygen saturation 92% on room air. Her BMI is 42.4 kg/m2.
Her heart has a regular rate and rhythm with no murmurs and her lungs are clear to
auscultation. Her lower extremities have bilateral 1+ edema. A chest radiograph is
normal. Spirometry reveals a decreased FVC with a normal FEV1/FVC ratio. A CBC, a
TSH level, and a basic metabolic panel are all normal except for a serum bicarbonate
level of 35 mEq/L (N 22–29).
These findings are most consistent with
A) asthma
B) COPD
C) obstructive sleep apnea
D) obesity hypoventilation syndrome
E) pulmonary fibrosis

ANSWER: D

This patient has obesity hypoventilation syndrome (OHS), a disorder in which central
obesity leads to chronic hypoventilation due at least in part to restricted diaphragm
excursion. Current criteria for this condition include hypoventilation leading to carbon
dioxide retention (PaCO2 >45 mm Hg) in an individual with a BMI > 30 kg/m2 when other
causes of chronic alveolar hypoventilation have been ruled out. These patients retain
bicarbonate to compensate for the respiratory acidosis. It has been suggested that an
increased serum bicarbonate level (>29 mEq/L) in the absence of another cause for
metabolic alkalosis should be included in the definition of OHS.
OHS leads to a restrictive pattern on spirometry, which this patient has. Asthma and
COPD are obstructive lung diseases and can therefore be ruled out in this patient who
has no signs of airway obstruction on spirometry. Obstructive sleep apnea is often
present in patients with OHS, but sleep apnea alone does not lead to daytime
hypoventilation and carbon dioxide retention. Pulmonary fibrosis is a cause of restrictive
lung disease and has not yet been completely ruled out in this patient, but a normal
chest radiograph makes this less likely. Comprehensive pulmonary function testing,
including the diffusion capacity of the lung for carbon monoxide (DLCO), would help rule
this out. Pulmonary fibrosis leads to a decreased DLCO while OHS does not.
Ref: Johnson JD, Theurer WM: A stepwise approach to the interpretation of pulmonary function tests. Am
Fam Physician 2014;89(5):359-366. 2) Piper A: Obesity hypoventilation syndrome: Weighing in on therapy
options. Chest 2016;149(3):856-868.

169. You see a 58-year-old female who received a drug-eluting stent 10 days ago during
a hospitalization for acute coronary syndrome and coronary artery disease. She asks for
recommendations about anticoagulation. You determine that she is not at high risk for
bleeding.
Which one of the following would you recommend?
A) Long-term aspirin use
B) Clopidogrel (Plavix) and aspirin for 30 days and then aspirin alone
C) Clopidogrel alone for 1 year and then aspirin alone
D) Clopidogrel and aspirin for 1 year and then aspirin alone
E) Prasugrel (Effient) for 1 year with no anticoagulation after that

ANSWER: D

Coronary artery stenting is a common procedure, and stent restenosis carries a high
mortality rate. Current American College of Cardiology guidelines recommend dual
antiplatelet therapy (aspirin with a second agent such as clopidogrel) for at least 12
months following the placement of a drug-eluting stent. Dual antiplatelet therapy with
aspirin plus clopidogrel for more than 1 year gives no additional benefit and carries an
additional risk of bleeding. Aspirin has been shown to be effective for the secondary
prevention of heart disease and should be continued after 1 year.
Ref: Ebell MH: No benefit to prolonged dual antiplatelet therapy after drug-eluting stent placement. Am Fam
Physician 2014;90(7):502. 2) Smith JN, Negrelli JM, Manek MB, et al: Diagnosis and management of acute
coronary syndrome: An evidence-based update. J Am Board Fam Med 2015;28(2):283-293. 3) Mukherjee
D: ACC/AHA guideline update on duration of dual antiplatelet therapy in CAD patients. American College of
Cardiology, 2016. 4) Mikhail MA, Mohabbat AB, Ghosh AK: Perioperative cardiovascular medication
management in noncardiac surgery: Common questions. Am Fam Physician 2017;95(10):645-650.

170. A 45-year-old female presents with throbbing right-sided heel pain that started a
few weeks ago. She says the pain is worst in the morning and seems to improve during
the day but will return after a long day on her feet. She does not have a history of
trauma, change in exercise, unexplained fever, or unintended weight loss.
On examination the patient’s vital signs are normal. You note pain on palpation of the
right medial calcaneal tuberosity and along the plantar fascia, and pain with passive
dorsiflexion of the right foot. The skin over the foot reveals no sign of trauma, lesions, or
masses.
Which one of the following is the most likely cause of this patient’s heel pain?
A) The heel spur
B) A calcaneal stress fracture
C) Heel pad syndrome
D) Plantar fasciitis
E) Sever’s disease
ANSWER: D
Plantar fasciitis is the most common cause of heel pain, with a prevalence of 10% in the
general population. It often presents with throbbing heel pain that is worst in the morning
with the first step after rest. Palpation of the medial calcaneal tuberosity and dorsiflexion
of the affected foot will elicit sharp pain. Diagnostic imaging is not required. Heel spurs
are present in approximately 50% of patients with plantar fasciitis, but can also be found
in patients without plantar fasciitis. Calcaneal stress fractures are caused by repetitive
overuse and the pain usually begins after an increase in weight-bearing activities or a
change in activities. It usually occurs only with activity, but may eventually also occur at
rest. Heel pad syndrome causes pain with deep palpation of the middle of the heel
or walking barefoot on harder surfaces. Sever’s disease is the most common cause of
heel pain in children and adolescents 8–12 years of age.

Ref: Tu P: Heel pain: Diagnosis and management. Am Fam Physician 2018;97(2):86-93.

171. A 36-year-old female singer presents with a 10-day history of hoarseness. She has
never smoked and does not take any medications. Her vital signs are normal. An
oropharyngeal examination is normal, her chest is clear to auscultation, and there is no
cervical adenopathy and no masses. She is anxious to be able to sing again as soon as
possible.
Which one of the following would you advise at this time?
A) No talking, whispering, or throat clearing for 48 hours
B) No singing or loud talking for 5–7 days
C) Nebulized hypertonic saline treatments 3 times daily for 2–3 days
D) Nebulized ribavirin twice daily for 3 days
E) Inhaled corticosteroids twice daily for 5 days

ANSWER: A

Complete vocal rest, including no whispering or throat clearing, is the most effective and
quickest initial remedy for short-duration laryngitis, whether viral or due to vocal overuse
or abuse. Limiting voice use or whispering, as opposed to complete vocal rest, will likely
prolong and possibly worsen hoarseness. Clearing the throat of mucus should also be
avoided for the same reason. Inhaled corticosteroids and antibiotics are not effective
treatments for laryngitis. Hypertonic saline nebulization treatments would likely cause
violent coughing fits that would worsen the condition. Nebulized ribavirin is never
indicated for use in adults.

Ref: House SA, Fisher EL: Hoarseness in adults. Am Fam Physician 2017;96(11):720-728.

172. A 74-year-old female with a long-standing history of coronary artery disease is


hospitalized for pneumonia. The patient improves with treatment and is
hemodynamically stable. An EKG performed on the third day of hospitalization is shown
below. Which one of the following would be the most appropriate next step?
A) Cardiac rhythm monitoring with no additional treatment
B) Atropine
C) Transcutaneous pacing
D) Transvenous pacing
ANSWER: A
Second degree Mobitz type I (Wenckebach) heart block is characterized by an
intermittent blockade of electrical impulses from the atria to the ventricles at the level of
the atrioventricular node. This prevents generation of a QRS complex. It is characterized
by progressive prolongation of the PR interval until a P wave is not followed by a QRS
complex. P waves come at regular intervals so PP intervals are normal. Following the
missed QRS complex, the PR interval returns to its baseline duration. A pacemaker is
not recommended in patients with second degree Mobitz type I heart block who are
asymptomatic. It is recommended in symptomatic patients, however, and is guided by
electrophysiologic studies.
Ref: Kasper DL, Fauci AS, Hauser SL, et al (eds): Harrison’s Principles of Internal Medicine, ed 19.
McGraw-Hill, 2015, pp 1470-1476.

173. While sitting in the waiting room a patient develops the acute onset of diffuse hives,
itching, and flushing; swelling of the lips, tongue, and uvula; and bilateral wheezing. He
becomes weak and almost passes out.
Which one of the following would be the most appropriate immediate treatment?
A) Corticosteroids
B) Diphenhydramine (Benadryl)
C) Epinephrine
D) Glucagon
E) Normal saline

ANSWER: C

This patient has symptoms of anaphylaxis. Symptoms include an acute onset (minutes
to several hours); involvement of the skin, mucosal tissue, or both; plus one of the
following: respiratory compromise (dyspnea, wheezing, bronchospasm, stridor, reduced
peak expiratory flow, hypoxemia), reduced blood pressure, or associated symptoms of
end-organ dysfunction (hypotonia, collapse, syncope, incontinence). The first and most
important treatment in anaphylaxis is intramuscular epinephrine, 1:1000 dilution dosed
at 0.01 mg/kg (maximal dose of 0.3 mg in children and 0.5 mg in adults) (SOR B).
Management of the airway, breathing, and circulation is also essential (SOR B).
Other essential treatments include volume replacement with normal saline for the
treatment of hypotension that does not respond to epinephrine (SOR B). Histamine H1-
receptor antagonists such as diphenhydramine and corticosteroids may be considered
as second-line treatments in patients with anaphylaxis (SOR C). Glucagon can be
considered for patients who are taking -blockers.
Ref: Arnold JJ, Williams PM: Anaphylaxis: Recognition and management. Am Fam Physician
2011;84(10):1111-1118. 2) Commins SP: Outpatient emergencies: Anaphylaxis. Med Clin North Am
2017;101(3):521-536.

174. A 50-year-old male with hypertension who is not at increased risk for
gastrointestinal bleeding should begin low-dose aspirin at what 10-year risk level for
cardiovascular disease?
A) 1%
B) 7.5%
C) 10%
D) 15%
E) 20%

ANSWER: C

Low-dose aspirin therapy is recommended by the U.S. Preventive Services Task Force
for the primary prevention of cardiovascular disease (CVD) in patients 50–59 years of
age who have a risk of CVD 10% (USPSTF B recommendation). The recommendation
statement adds that the patient should have a life expectancy of at least 10 years,
should be willing to take daily aspirin for at least 10 years, and should not be at
increased risk for gastrointestinal bleeding. The decision to start aspirin therapy for
patients 60–69 years of age should be based on individual considerations (USPSTF C
recommendation). For adults younger than 50 or age 70 or older, the evidence is
insufficient to assess the balance of benefits and harms (C recommendation). The
recent Aspirin in Reducing Events in the Elderly (ASPREE) trial indicated that
daily aspirin use in those over age 70 did not significantly lower the risk of cardiovascular
disease, and did not increase disability-free survival.
Ref: Final Recommendation Statement: Aspirin Use to Prevent Cardiovascular Disease and Colorectal
Cancer: Preventive Medication. US Preventive Services Task Force, 2017. 2) McNeil JJ, Nelson MR,
Woods RL, et al: Effect of aspirin on all-cause mortality in the healthy elderly. N Engl J Med 2018;[Epub
ahead of print]. 3) McNeil JJ, Wolfe R, Woods RL, et al: Effect of aspirin on cardiovascular events and
bleeding in the healthy elderly. N Engl J Med 2018;[Epub ahead of print].

175. A U.S. hospital or birthing center seeking to be certified as “Baby-Friendly” by the


Baby-Friendly Hospital Initiative must satisfy which one of the following criteria in
addition to meeting other requirements?
A) Demonstrating proper use of an infant car seat to parents prior to discharge
B) Providing no other food or fluids to breastfeeding infants without a medical
indication
C) Providing a pacifier to each baby prior to discharge
D) Providing easy access to a variety of infant formulas
E) Providing on-site daycare facilities for staff

ANSWER: B

The Baby-Friendly Hospital Initiative is a global program established by UNICEF and


WHO to promote healthy infant feeding and mother-baby bonding. The primary objective
is to educate the public on the benefits of breastfeeding and encourage, promote, and
facilitate breastfeeding as outlined in the UNICEF/WHO Ten Steps to Successful
Breastfeeding chart. These steps promote breastfeeding to the public and provide
guidelines for hospitals and birthing centers for the successful initiation and continuation
of breastfeeding. Baby-friendly facilities must have a written breastfeeding policy that is
routinely communicated to all health care staff, and all health care staff must be trained
in the skills necessary to implement this policy. All pregnant women should be informed
about the benefits and management of breastfeeding. Mothers should be helped to
initiate breastfeeding within an hour after birth and shown how to breastfeed and to
maintain lactation, even if they are separated from their infants. Breastfeeding infants
should not be given food other than breast milk, unless medically indicated. If mothers
choose to give formula after appropriate education, they should be instructed in proper
preparation and use. Rooming in should be practiced, allowing mothers and infants to
remain together 24 hours a day. Mothers should be encouraged to breastfeed on
demand. Breastfeeding infants should not be given pacifiers or artificial nipples. Mothers
should be referred to breastfeeding support groups on discharge from the hospital. In
addition, the hospital must comply with the International Code of Marketing of Breast
Milk Substitutes, which requires that formula companies cannot give free gifts to staff or
mothers, that breast milk substitutes are not marketed in the maternity unit, and that
breast milk supplements and infant feeding supplies are purchased at fair market price.
Ref: Perrine CG, Galuska DA, Dohack JL, et al: Vital signs: Improvements in maternity care policies and
practices that support breastfeeding—United States, 2007–2013. MMWR Morb Mortal Wkly Rep
2015;64(39):1112-1117. 2) Guidelines and evaluation criteria for facilities seeking baby-friendly designation.
Baby-Friendly USA, 2016.

176. A 50-year-old gravida 2 para 2 who is 3 years post menopausal presents with
fatigue, headache, galactorrhea, and loss of libido. Your evaluation reveals elevated
serum prolactin and a pituitary adenoma of 5–6 mm. You recommend
A) bromocriptine (Parlodel)
B) estrogens
C) haloperidol
D) testosterone
E) neurosurgical consultation

ANSWER: A

This patient has a pituitary microadenoma. Microadenomas <10 mm in size that are
secreting prolactin may be treated with a dopaminergic agent such as bromocriptine.
This will lower the prolactin level and shrink the adenoma. Nonprolactin-secreting
adenomas, especially those >10 mm in size (macroadenomas), require neurosurgical
evaluation.
Ref: Molitch ME: Diagnosis and treatment of pituitary adenomas: A review. JAMA 2017;317(5):516-524.

177. Referral for bariatric surgical evaluation is indicated for patients with a BMI of
A) 35 kg/m2 and mild cognitive impairment
B) 36 kg/m2 and type 2 diabetes mellitus
C) 37 kg/m2 and no other medical problems
D) 40 kg/m2 and active alcohol abuse
E) 42 kg/m2 and uncontrolled schizophrenia

ANSWER: B

Inclusion criteria for bariatric surgery include a BMI 40 kg/m2 without coexisting
medical problems or a BMI 35 kg/m2 with one or more severe obesity-related
comorbidities such as diabetes mellitus. Exclusion criteria include active substance
abuse, uncontrolled severe psychiatric illness, severe cardiopulmonary disease that
makes the surgical risk prohibitive, and lack of cognitive function to comprehend the
associated risks, benefits, and required lifestyle changes.
Ref: Schroeder R, Harrison TD, McGraw SL: Treatment of adult obesity with bariatric surgery. Am Fam
Physician 2016;93(1):31-37.

178. A 75-year-old patient is admitted to the hospital. The Joint Commission National
Patient Safety Goals program requires medication reconciliation for this patient both on
admission and at the time of discharge.
The primary intent of this reconciliation is to detect
A) potentially inappropriate medication use in the elderly
B) high-risk medication use
C) medication discrepancies
D) polypharmacy
E) adverse drug effects

ANSWER: C

Maintaining and communicating accurate patient medication information is one of the


goals of the Joint Commission National Patient Safety Goals program. This includes
medication reconciliation, which is intended to identify and resolve discrepancies. In this
process, a clinician compares the medications a patient should be using and is actually
using with the new medications that are ordered. While adverse drug effects, potentially
inappropriate medication use in the elderly, high-risk medication use, or polypharmacy
might also occur and might be beneficial to address, these are not the primary focus of
medication reconciliation.
Ref: National Patient Safety Goals Effective January 1, 2015: Hospital Accreditation Program. The Joint
Commission, 2015. 2) AHRQ patient safety primer: Medication reconciliation. Agency for Healthcare
Research and Quality, updated 2017.

179. A 29-year-old male smoker presents with a 10-day history of a cough. He also had
a low-grade fever for 2 days that has resolved. He has had some mild rhinorrhea and
has noted that the cough has become productive of greenish sputum over the past 3–4
days. He has not tried any medication. An examination reveals some mild rhinorrhea but
his lungs are clear.
Which one of the following would be most appropriate at this point?
A) Supportive care only
B) A chest radiograph
C) Albuterol (Proventil, Ventolin)
D) Antibiotic therapy
E) An inhaled corticosteroid

ANSWER: A

The defining symptom of acute bronchitis is cough. Even in smokers the etiologic agent
is viral at least 90% of the time, so antibiotics are not indicated. Unless wheezing is
noted, albuterol is not helpful. Inhaled corticosteroids are used in maintenance therapy
for asthma. Indications for an adult patient with acute bronchitis to have a chest
radiograph include: bloody sputum, rusty-colored sputum, or dyspnea; a pulse rate >100
beats/min; a respiratory rate >24/min; or a temperature >37.8°C (100.0°F). A chest
radiograph is also indicated if there are abnormal findings on a chest examination such
as fremitus, egophony, or focal consolidation. Supportive care is made easier by
informing the patient that symptoms are likely to last 2–3 weeks. Symptoms may be
managed with measures such as dextromethorphan, guaifenesin, or honey.
Ref: Kinkade S, Long NA: Acute bronchitis. Am Fam Physician 2016;94(7):560-565.
180. According to the recommendations of the American Heart Association, which one of
the following patients requires endocarditis prophylaxis?
A) A 10-year-old female with a previous history of Kawasaki disease without valvular
dysfunction
B) A 22-year-old female who underwent surgical repair of a ventricular septal defect
1 year ago
C) A 28-year-old female with mitral valve prolapse without regurgitation
D) A 35-year-old female with a history of infectious endocarditis in her 20s that was
related to intravenous drug use
E) A 42-year-old female with a history of rheumatic fever with chorea who has normal
cardiovascular findings

ANSWER: D

The American Heart Association and the American College of Cardiology have
decreased the number of indications for antibiotic prophylaxis prior to dental procedures.
Currently antibiotics are indicated for prosthetic cardiac valves, previous infective
endocarditis, unrepaired cyanotic congenital heart disease or a repaired congenital
defect with a residual shunt, and a cardiac transplant with valve regurgitation due to
a structurally abnormal valve. Amoxicillin, 2 g, is the antibiotic prophylaxis of choice.

Ref: Wilson W, Taubert KA, Gewitz M et al: Prevention of infective endocarditis: Guidelines from the
American Heart Association: A guideline from the American Heart Association Rheumatic Fever,
Endocarditis, and Kawasaki Disease Committee, Council on Cardiovascular Disease in the Young, and the
Council on Clinical Cardiology, Council on Cardiovascular Surgery and Anesthesia, and the Quality of Care
and Outcomes Research Interdisciplinary Working Group. Circulation 2007;116(15):1736-1754. 2) Oral
health topics: Antibiotic prophylaxis prior to dental procedures. American Dental Association, updated 2018.

181. A 69-year-old male presents with acute right hip pain, which has been worsening
over thepast week and is now causing difficulty walking. He has had occasional hip pain
in the past but this is more severe than previous episodes. He has no history of trauma
and he feels well otherwise. His medical history includes hypertension, hyperlipidemia,
osteoarthritis, and psoriasis. His current medications include
lisinopril/hydrochlorothiazide (Zestoretic), aspirin, and adalimumab (Humira).
An examination reveals normal vital signs and a BMI of 29.3 kg/m2. The joint is not red
or swollen. There is no tenderness over the greater trochanter, groin, or buttock. Active
and passive range of motion of the hip is limited in all directions due to pain. A
radiograph shows mild degenerative changes of the hip joint. A C-reactive protein level
is mildly elevated.

Which one of the following would be indicated at this point to rule out a serious cause of
joint pain?
A) A radionuclide bone scan
B) Arthrocentesis
C) CT
D) MR arthrography
E) MRI
ANSWER: B
This patient has a history and physical examination concerning for septic arthritis, which
is a rheumatologic emergency due to the potential for joint destruction. Joint swelling,
redness, and warmth may accompany the pain but these are more difficult to detect at
the hip than the knee. Systemic symptoms such as fever may occur but are absent in
more than 40% of patients, particularly elderly patients and those who are
immunocompromised. Risk factors for septic arthritis include underlying joint disease
such as rheumatoid arthritis or osteoarthritis, and immunosuppressive states such as
HIV infection, diabetes mellitus, and taking immunosuppressive medications. This
patient has a history of osteoarthritis and is taking adalimumab, an immunosuppressive
agent. Although there may be clues to the diagnosis of septic arthritis on imaging and
laboratory assessment, the diagnostic test of choice is analysis of synovial fluid obtained
through arthrocentesis. A radionuclide bone scan, CT, MR arthrography, and MRI are
not sensitive enough to rule out septic arthritis.

182. Which one of the following is the most appropriate psychotherapy for patients with
obsessive-compulsive disorder?
A) Traditional psychotherapy
B) Cognitive-behavioral therapy
C) Psychoanalysis
D) Psychodynamic therapy
ANSWER: B
Cognitive-behavioral therapy, specifically exposure and response prevention, is
considered the most effective psychotherapy method (SOR A). There is no evidence for
psychodynamic or “talk” therapy.
Traditional psychotherapy and psychoanalysis are less effective than cognitive-
behavioral therapy.

183. A 63-year-old female is concerned about her long-term use of medication. She has
been taking omeprazole (Prilosec), 20 mg daily for the past 4 months, and tells you that
it works well to relieve her symptoms of heartburn and regurgitation. She notes,
however, that if she misses a dose her symptoms return.
You tell her that long-term proton pump inhibitor use is associated with which one of the
following complications?
A) Gastrointestinal malignancy
B) Hip fracture
C) Myocardial infarction
D) Nephrotic syndrome
E) Vitamin D deficiency
ANSWER: B

Proton pump inhibitors (PPIs) are safe and well tolerated for short-term use. It is
recommended that the lowest dosage and shortest duration of therapy be used to
control symptoms of GERD. Long-term PPI use is associated with fractures,
hypomagnesemia, vitamin B12 deficiency, iron deficiency, and acute interstitial nephritis
with progression to chronic kidney disease. Use of PPIs has also been associated with
community-acquired pneumonia and Clostridium difficile infection, although studies
have been conflicting.
Vitamin D deficiency, nephrotic syndrome, gastrointestinal malignancy, and myocardial
infarction are not proven complications of long-term PPI use.

184. A 4-year-old male is brought to your office for a well child examination. The patient
has no significant medical history. The mother has noted new skin lesions first appearing
on the back, with a new lesion behind the right knee. She has not used any new
detergents or skin or hair care products. She has not made any changes in the patient’s
diet. The child does not have pruritus.
The examination reveals a temperature of 37.2°C (99.0°F), a pulse rate of 80
beats/min,and a blood pressure within normal limits. The examination is unremarkable
except for Non erythematous flesh-colored, dome-shaped papules with a central
indentation, on the lower back and popliteal fossa.

Which one of the following would be most appropriate for the initial management of this
condition?
A) Observation only
B) Consistent use of emollients and avoiding frequent hot baths
C) Use of a topical low-dose corticosteroid cream once daily until resolved
D) Use of a topical antifungal cream until resolved
E) Paring, followed by topical salicylic acid or cryotherapy

ANSWER: A

Molluscum contagiosum is a common disease during childhood, but can also occur in
adolescents and adults. It is caused by a poxvirus and is characterized by flesh-colored,
dome-shaped papules with central umbilication, most commonly on the trunk, axilla,
popliteal or antecubital fossae, and crural folds. If lesions are asymptomatic and not
inflamed, the initial treatment is observation, with most lesions resolving spontaneously
within 2–12 months. If the lesions are inflamed or pruritic, then topical corticosteroid
treatment, chemical treatment with cantharidin, podofilox 0.5% solution, curettage, or
cryotherapy may be indicated.
Atopic dermatitis (eczema) is initially treated with emollients and by avoiding frequent hot
baths. Verruca (warts) are commonly treated with paring, followed by topical salicylic
acid or cryotherapy. Antifungal cream would not be appropriate

185. A 34-year-old male has a 3-day history of a runny nose, postnasal drainage, sinus
congestion, and left-sided facial pain. He also reports a mild cough and difficulty
sleeping due to the congestion. He is afebrile and the examination reveals inflammation
of the nasal mucosa, purulent rhinorrhea, and mild left maxillary sinus tenderness to
percussion.

Which one of the following would be the most appropriate pharmacotherapy?


A) Amoxicillin/clavulanate (Augmentin)
B) Levofloxacin (Levaquin)
C) Loratadine (Claritin)
D) Mometasone (Nasonex)
ANSWER: D
This patient presents with symptoms of acute rhinosinusitis. In the first 3–4 days, viral
and bacterial rhinosinusitis are indistinguishable. Guidelines from the American
Academy of Otolaryngology—Head and Neck Surgery suggest that antibiotics should
not be routinely prescribed for acute mild to moderate sinusitis unless symptoms persist
for 7 days or worsen after initial improvement. Watchful waiting without antibiotic
treatment is appropriate when follow-up is accessible (SOR A). In this scenario antibiotic
therapy is not indicated.
Amoxicillin with or without clavulanate is appropriate for symptoms lasting 7 or more
days without improvement and is the first-line antibiotic treatment for acute bacterial
rhinosinusitis (SOR A). Due to the risk of adverse effects and no benefit over -lactams,
respiratory fluoroquinolones are not considered first-line antibiotic therapy. Symptomatic
treatment is recommended within the first 10 days of the onset of symptoms and may be
continued if antibiotics are started. Intranasal corticosteroid use has a modest
therapeutic benefit for patients with acute rhinosinusitis. Decongestants and
antihistamines have not been proven effective for the treatment of acute rhinosinusitis.

186. At what age should a patient at average risk be switched from a universal screening
strategy for colon cancer to a more individualized strategy?
A) 45
B) 55
C) 65
D) 75
E) 85
ANSWER: D
Although national guidelines vary, it is generally advised to start routine colon cancer
screening between ages 45 and 50, and to screen with a more individualized approach
between ages 75 and 85. Factors to consider include life expectancy, the patient’s
overall health, whether the patient has been screened previously, and patient
preference. Most guidelines recommend stopping colon cancer screening in patients
older than 85 years or when their life expectancy falls below 10 years (SOR B).

187. An 18-month-old male is brought to your office for a well child check. He is walking
only with assistance. You and the parents are concerned about gross motor delay.
Which one of the following findings would be most suggestive of muscular dystrophy in
this patient?
A) A cross-legged “scissoring” posturing
B) Head lag when sitting up
C) Hyperreflexia in the legs
D) Partial hemiparesis of the lower extremities
E) Toe walking
ANSWER: B

Some abnormal gross motor developmental findings suggest muscular dystrophy. Signs
of increased muscular tone, such as cross-legged posturing, neck stiffness, and
hyperreflexia, suggest a central cause of motor delay such as cerebral palsy. Head lag
due to neck muscle weakness in infants is a classic early finding of muscular
dystrophies. Hemiparesis similarly suggests a central nervous system abnormality. Toe
walking can be seen with both central and peripheral neuromuscular abnormalities,
including muscular dystrophy, but is less specific, and therefore less helpful, in
differentiating the cause of motor delay. In muscular dystrophies it is a sign of
quadriceps weakness.

188. Which one of the following is most commonly associated with oligohydramnios?
A) Anencephaly
B) Esophageal atresia
C) Hydrops
D) Maternal -thalassemia
E) Posterior urethral valves
ANSWER: E
Amniotic fluid volume is regulated in part by fetal swallowing, inspiration, and urination.
Some malformations of the urinary tract, including renal agenesis and persistent
obstruction from posterior urethral valves, lead to oliguria or anuria, and are associated
with marked oligohydramnios.
Anencephaly, esophageal atresia, heart failure, and maternal -thalassemia are
associated with polyhydramnios. Anencephaly is probably the most common cause of
polyhydramnios, via transudation from the exposed meninges; swallowing difficulties and
excessive urination may also be contributing factors. Esophageal atresia is almost
always associated with polyhydramnios due to an inability to swallow. Intrauterine heart
failure, whether due to dysrhythmias, structural defects, or severe anemia, often leads to
fetal hydrops, which is associated with polyhydramnios. -Thalassemia, relatively
common in Asians, can also cause fetal hydrops and polyhydramnios.

189. You see a 47-year-old female for follow-up of a rash. She is a carpenter and was
seen 4 days ago for increasing redness and tenderness of her anterior shin after hitting
the area with a board 3 days earlier. She was afebrile during that visit and the area was
red but not fluctuant. She chose observation rather than treatment at that time. The
patient smokes 10 cigarettes daily. Past medical, surgical, and family histories are
otherwise negative.
Screening for diabetes mellitus was normal last year.
Today the patient’s anterior shin is still tender. She is afebrile and other vital signs are
unremarkable. The extent of the infection was drawn 4 days ago with an indelible marker
by your partner. Currently the area of redness extends beyond this border. There is no
fluctuance or drainage of the wound. The skin appears mildly indurated.

Which one of the following would be best to provide coverage against Streptococcus
pyogenes or methicillin-resistant Staphylococcus aureus (MRSA) in this patient?
A) Amoxicillin/clavulanate (Augmentin) and ciprofloxacin (Cipro)
B) Cephalexin and dicloxacillin
C) Dicloxacillin and fosfomycin (Monurol)
D) Doxycycline and trimethoprim/sulfamethoxazole (Bactrim)
E) Trimethoprim/sulfamethoxazole and cephalexin

ANSWER: E
Clindamycin or a combination of trimethoprim/sulfamethoxazole (or doxycycline or
minocycline) plus cephalexin (or dicloxacillin or amoxicillin/clavulanate) should provide
adequate coverage for Streptococcus and methicillin-resistant Staphylococcus aureus
(MRSA) for mild to moderate cellulitis. Doxycycline plus trimethoprim/sulfamethoxazole
would provide inadequate coverage for streptococcal bacteria. Cephalexin plus
dicloxacillin would provide inadequate coverage for MRSA. The primary indication for
ciprofloxacin is treatment of infections with gram-negative rods. Fosfomycin is indicated
only for urinary tract infections. Neither is typically used in the treatment of cellulitis.

190. A 65-year-old male brings in results from a health fair screening. He was advised to
see you because he had a hemoglobin level of 10.2 g/dL (N 14.0–18.0) and a mean
corpuscular volume of 80 m3 (N 80–94). A review of systems is unremarkable except for
recent fatigue, and a physical examination is also unremarkable. You order laboratory
testing, with the following results:
Ferritin ------------------------------15 ng/mL (N 20–150)
Vitamin B12 ------------------------- 420 pg/mL (N 200–900)
Folate -------------------------------- 12 ng/mL (N 2–20)
Reticulocyte index ------------------0.3% (N 0.5–1.0)
The most likely diagnosis is
A) iron deficiency anemia
B) vitamin B12 deficiency
C) anemia of chronic disease
D) hemolysis
E) myelodysplastic anemia

ANSWER: A
This patient most likely has iron deficiency anemia. The low normal mean corpuscular
volume, low serum ferritin, and low reticulocyte index are all consistent with iron
deficiency. Vitamin B12 deficiency would be indicated by low vitamin B12 and a macrocytic
anemia. Serum ferritin would be higher with anemia of chronic disease and
myelodysplastic anemia. The reticulocyte index would be high with hemolysis.

191. The mother of a newborn infant is concerned because her baby’s eyes are
sometimes crossed. Assuming the intermittent eye crossing persists, which one of the
following is the most appropriate age for ophthalmologic referral?
A) 10–14 days
B) 6 months
C) 12 months
D) 24 months
ANSWER: B

In many normally developing infants there may be imperfect coordination of eye


movements and alignment during the early days and weeks of life, but proper
coordination should be achieved by age 4–6 months. Persistent deviation of an eye in an
infant requires evaluation.
192. A 47-year-old male presents with a 3-day history of fever, chills, low back pain, and
urinary frequency. He does not have any nausea, vomiting, or abdominal pain. There is
no significant past medical history.
The patient’s vital signs include a temperature of 38.1°C (100.6°F), a pulse rate of 88
beats/min, and a respiratory rate of 14/min. The examination reveals a mildly tender
lower abdomen with no guarding or rebound tenderness; no costovertebral angle
tenderness; and an enlarged, homogeneous, exquisitely tender prostate.

Which one of the following is indicated to help guide this patient’s treatment?
A) A serum prostate-specific antigen level
B) A culture of prostate secretions after massage of the prostate
C) A culture of midstream voided urine
D) CT of the abdomen and pelvis with intravenous and oral contrast
E) An ultrasound-guided prostate biopsy

ANSWER: C
This patient has clinically diagnosable acute bacterial prostatitis, and no further testing,
including imaging, is required to establish the diagnosis. Culture of a midstream voided
urine may aid in identifying the pathogen, but prostate massage should be avoided
because it may increase the risk of bacteremia. A prostate biopsy is not indicated in the
presence of acute infection, and a prostate-specific antigen level is not indicated
because it is likely to be elevated in the presence of infection.

193. A 57-year-old female is admitted to the hospital with lower lobe pneumonia. She
has no history of diabetes mellitus. She has not met sepsis criteria but had a blood
glucose level of 172 mg/dL in the emergency department.
Insulin should be started if this patient has a persistent blood glucose level greater than
or equal to
A) 120 mg/dL
B) 140 mg/dL
C) 160 mg/dL
D) 180 mg/dL
ANSWER: D
Insulin therapy should be initiated in hospitalized patients with persistent hyperglycemia,
starting at a threshold of 180 mg/dL. Once insulin therapy is started, a target glucose
range of 140–180 mg/dL is recommended for the majority of hospitalized patients,
regardless of whether they have a critical illness.

194. A healthy 33-year-old male sees you for a pretravel consultation. He plans to attend
a 4-week intensive Spanish language course in Antigua, Guatemala. You discuss
immunizations, malaria prophylaxis, injury prevention, and traveler’s diarrhea.
Which one of the following interventions is most likely to prevent traveler’s diarrhea?
A) Avoiding food from street vendors
B) Avoiding the use of ice in beverages
C) Taking a probiotic
D) Taking a prophylactic antibiotic
E) Washing hands frequently
ANSWER: E
Traveler’s diarrhea is caused predominantly by bacterial pathogens (up to 80%–90%)
and is associated with hygiene practices. Handwashing has been shown to reduce the
risk of traveler’s diarrhea by 30%. The use of alcohol-based hand sanitizer is also
effective. Although it is considered traditional advice, avoiding street vendor foods, tap
water, ice, and raw foods has not been shown to reduce the risk of traveler’s diarrhea.
There is not sufficient evidence to recommend taking a probiotic to reduce the risk of
traveler’s diarrhea. Due to concerns about antimicrobial resistance and altering
protective bowel flora, taking prophylactic antibiotics is generally not recommended for
healthy travelers. However, using an antibiotic for as-needed treatment is appropriate.

195. A 52-year-old male smoker presents to your office in January with worsening
respiratory symptoms over the past 24 hours, along with a rapid onset of fever and chills,
nausea, myalgias, and sore throat. He has a history of mild chronic bronchitis and
hypertension, and his medications include tiotropium (Spiriva) inhaled daily;
lisinopril/hydrochlorothiazide (Zestoretic), 20/12.5 mg daily; and albuterol (Proventil,
Ventolin) as needed.
On examination the patient has a temperature of 38.8°C (101.8°F), a heart rate of 102
beats/min, a respiratory rate of 24/min, and an oxygen saturation of 94% on room air. He
is ill-appearing and pale. Examination of his throat reveals mild erythema, and chest
auscultation reveals bilateral bronchovesicular breath sounds with no crackles or
wheezing.
The examination is otherwise unremarkable. Laboratory and radiology services are not
available.

Which one of the following would be most appropriate at this point?


A) Observation only, with follow-up in a few days
B) Azithromycin (Zithromax)
C) Oseltamivir (Tamiflu)
D) Penicillin VK
E) Prednisone

ANSWER: C
This patient has findings consistent with influenza, including a rapid onset of fever,
nausea, and sore throat, and negative pulmonary findings. Influenza is considered a
clinical diagnosis and confirmation of the diagnosis with laboratory testing is not
required. Treatment of influenza is recommended for individuals at a high risk of
influenza-related complications. High-risk individuals include those with chronic lung
disease; cardiovascular (excluding hypertension), renal, hepatic, hematologic, or
neurologic disease; or age >65. Children on long-term aspirin therapy, and pregnant and
postpartum women are also considered high risk. This patient should be treated with
antiviral medication because of his chronic pulmonary disease. While pneumonia and
streptococcal pharyngitis should be considered in the differential diagnosis, these are
less likely given the examination findings, and antibiotics are not recommended.
Prednisone is not indicated for influenza-like illness and may cause harm.
196. A 33-year-old gravida 2 para 2 presents with a 1-year history of amenorrhea, hot
flashes, and vaginal dryness. She previously had normal menses and takes no
medications. Her past medical and surgical histories are negative. The patient is 178 cm
(70 in) tall and her BMI is 22.0 kg/m2. Her vital signs are normal. A physical examination
is normal except for vaginal dryness. Laboratory studies reveal a negative urine
pregnancy test, normal TSH and prolactin levels, and elevated LH and FSH levels.

The most likely diagnosis is


A) intrauterine synechiae (Asherman syndrome)
B) functional hypothalamic amenorrhea
C) polycystic ovary syndrome
D) primary ovarian insufficiency
E) Turner’s syndrome

ANSWER: D
This patient presents with secondary amenorrhea. The differential diagnosis includes
polycystic ovary syndrome (PCOS), intrauterine synechiae (Asherman syndrome),
functional hypothalamic amenorrhea, hypothyroidism, hyperprolactinemia, and primary
ovarian insufficiency (also known as premature ovarian failure). This patient’s
presentation and the laboratory findings are most consistent with a diagnosis of primary
ovarian insufficiency. This is defined as menopause before the age of 40 due to ovarian
folliculardepletion. Laboratory findings will usually reveal a low serum estradiol and
elevated FSH and LH levels.
This condition is different than menopause because of the age of presentation and the
unpredictability of long-term ovarian function (up to 10% of cases spontaneously remit
and patients have a temporary returnof fertility).
Patients with PCOS typically present with obesity, difficulty conceiving, and normal or
low FSH and LH levels. This patient’s normal weight and prior history of normal menses
make this diagnosis less likely.
Intrauterine synechiae is characterized by scar tissue inside the uterus. Risk factors
include intrauterine procedures, pregnancy, inflammation, and infection. Patients present
with abnormal uterine bleeding, recurrent pregnancy loss, dysmenorrhea, and infertility.
FSH and LH levels are usually normal.
Functional hypothalamic amenorrhea is characterized by suppression of the
hypothalamic-pituitary-ovarian axis, usually due to extreme stress, excessive exercise,
marked weight loss, and/or dysfunctional eating.
LH and FSH levels are usually low or low-normal. Turner’s syndrome is caused by the
45,X genotype, and patients have short stature, a webbed neck, a low hairline, and
cardiac abnormalities. This is unlikely in a patient who is 178 cm (70 in) tall and has a
normal examination

197. A 62-year-old female presents for a health maintenance visit. She is interested in
staying up to date on her preventive care recommendations. She smoked for 4 years
during college. Her BMI is 23.0 kg/m2. She feels well and does not have any specific
health concerns. She had a colonoscopy 4 years ago and no polyps were found. A
screening mammogram 6 months ago was normal.
Which one of the following screening measures is recommended by the U.S. Preventive
Services Task Force for this patient?
A) A urinalysis to detect asymptomatic bacteriuria
B) Measurement of hemoglobin A1c
C) Screening for hepatitis A and B viruses
D) Screening for hepatitis C virus
E) Lung cancer screening with low-dose CT of the lungs without contrast

ANSWER: D

The U.S. Preventive Services Task Force recommends screening for hepatitis C virus
(HCV) infection in persons at risk for infection and also one-time screenings for adults
born between 1945 and 1965 (Grade B recommendation). HCV is the most common
chronic bloodborne pathogen in the United States and a leading cause of complications
of chronic liver disease. The prevalence of the anti-HCV antibody in the United States is
approximately 1.6% in non-institutionalized persons. According to data from 1999 to
2008, approximately 75% of patients in the United States living with HCV infection were
born between1945 and 1965. This patient has a normal weight so diabetes screening is
not recommended. Screening for asymptomatic bacteriuria is not recommended in none
pregnant patients.

198. A 69-year-old male presents for follow-up of hypertension treated with


spironolactone (Aldactone) and amlodipine (Norvasc). His past medical history is
remarkable only for a kidney stone several years ago. A physical examination is
unremarkable. A comprehensive metabolic panel is unremarkable except for a calcium
level of 12.0 mg/dL (N 8.0–10.0).

Which one of the following is the most likely cause of his elevated calcium level?
A) Excessive ingestion of calcium supplements
B) His current medication regimen
C) Occult malignancy
D) Primary hyperparathyroidism
E) Vitamin D deficiency

ANSWER: D

The most common cause of hypercalcemia is hyperparathyroidism. This is seldom


symptomatic and is often discovered through routine blood testing. Hypercalcemia due
to cancer can be caused by secretion of the parathyroid hormone–related protein and by
osteoclastic bone resorption. Other causes of hypercalcemia include thiazide diuretics,
lithium, vitamin D intoxication, hyperthyroidism, milk alkali syndrome from excessive
calcium antacid ingestion, adrenal insufficiency, and lymphoma.
199. A 7-month-old male is admitted to the hospital for respiratory syncytial virus
bronchiolitis. His temperature is 37.9°C (100.2°F), pulse rate 160 beats/min, respiratory
rate 70/min, and oxygen saturation 92% on room air. Auscultation of the lungs reveals
diffuse wheezing and crackles accompanied by nasal flaring and retractions.

Which one of the following interventions would most likely be beneficial?


A) Bronchodilators
B) Corticosteroids
C) Epinephrine
D) Nasogastric fluids
E) Oxygen supplementation to maintain O2 saturation above 95%

ANSWER: D

The mainstay of therapy for acute respiratory syncytial virus bronchiolitis is supportive
care, and maintaining hydration is important. Infants with respiratory rates >60/min may
have poor feeding secondary to difficulty breathing and oral rehydration may increase
the risk of aspiration. In these cases, nasogastric or intravenous fluids should be
administered. Oxygen saturation of 90% or more on room air is sufficient for infants with
bronchiolitis, and using supplemental oxygen to maintain higher oxygen saturations only
prolongs hospitalization because of an assumed need for oxygen. Bronchodilators
should not be administered to infants with bronchiolitis, because they have not been
shown to have any effect on the need for hospitalization, oxygen saturation, or disease
resolution. In addition, there is no evidence to support the use of epinephrine or
corticosteroids in the inpatient setting.

200. A 30-year-old male presents with intermittent right upper quadrant pain after meals.
He has been in moderate pain for the past 3 hours. On examination the patient’s vital
signs are normal except for a temperature of 39.2°C (102.6°F). He appears toxic.
Examination of the abdomen reveals a positive Murphy’s sign.
Laboratory Findings
WBCs ----------------------------- 3000/mm3 (N 4300–10,800)
ALT (SGPT) --------------------- 132 U/L (N 10–55)
AST (SGOT) --------------------- 123 U/L (N 9–25)
Alkaline phosphatase -----------200 U/L (N 45–115)
Bilirubin ----------------------------- 2.6 mg/dL (N 0.0–1.0)
Lipase--------------------------------15 U/dL (N 3–19)
Ultrasonography reveals cholelithiasis. Other findings include an enlarged gallbladder,
thickening of the gallbladder wall, and a common bile duct diameter of 11 mm.

Which one of the following is the most likely cause of this patient’s symptoms?
A) Acute cholangitis
B) Acute viral hepatitis
C) Cholangiocarcinoma
D) Gallstone pancreatitis
ANSWER: A

Gallstones are often asymptomatic and found incidentally on imaging. However, they
may become symptomatic, which usually causes pain in the right upper quadrant or
epigastrium. Most patients with symptomatic gallstones present with chronic
cholecystitis, which causes recurrent attacks of pain. The pain is constant, increases in
severity at the beginning, and lasts from 1 to 5 hours. It often starts during the night after
a fatty meal and may be associated with nausea and vomiting. Abdominal
ultrasonography is the initial imaging method.
The two main complications of choledochal stones are cholangitis and pancreatitis.
Acute cholangitis is a bacterial infection. Bacterial growth is enhanced by obstruction of
the duct. It may present as a mild self-limited disease but can also lead to sepsis. Cases
typically present with fever, pain, and jaundice.
Laboratory findings include an elevated WBC count and elevated bilirubin,
transaminases, and alkaline phosphatase. Ultrasonography will show a dilated bile duct
in many cases, although it might not be dilated in acute obstruction.
Patients with pancreatitis present with pain, nausea, and vomiting. The pain is usually
epigastric and radiates to the back. It reaches its maximum intensity within 1 hour and
may last for days. The physical examination may reveal tachycardia, hypotension,
tachypnea, and fever. The abdomen may be distended and is typically tender to
palpation. The diagnosis requires two of three primary features: abdominal pain,
elevation of serum amylase or lipase, and findings on imaging studies that are consistent
with the diagnosis.
Ultrasonography can show pancreatic enlargement or edema, and visualization of
gallstones will suggest choledocholithiasis as the cause of the pancreatitis.

201. A 73-year-old female presents with a 3-day history of pain, numbness, and
weakness in her right arm and shoulder. She reports that the problem began when she
went out for a walk, tripped on an asphalt ledge, and fell to the ground. She was able to
get herself back up and walk home but has had upper extremity problems since then.
Her chronic medical conditions include early Alzheimer’s dementia and hyperlipidemia.
After a full examination you order radiographs (shown below).

Which one of the following would be most appropriate at this point?


A) Figure-of-eight bandaging
B) Physical therapy
C) Shoulder reduction
D) Sling immobilization
E) Surgical decompression

ANSWER: C

This patient’s radiographs show an anterior shoulder dislocation on anteroposterior and


scapular Y views. Treatment of this condition includes reduction of the dislocation, which
can be accomplished in this case with a local anesthetic or conscious sedation. An acute
shoulder dislocation reduction may be attempted without pain medication. Surgical
decompression and figure-of-eight bandaging are not indicated. Sling immobilization and
physical therapy may be appropriate after shoulder reduction.
Ref: Monica J, Vredenburgh Z, Korsh J, Gatt C: Acute shoulder injuries in adults. Am Fam Physician
2016;94(2):119-127.

202. The U.S. Preventive Services Task Force recommends which one of the following
for women who are planning on or are capable of becoming pregnant?
A) Testing for hepatitis C
B) Annual pelvic examinations
C) Folic acid, 0.4–0.8 mg daily
D) Vitamin B12, 1000 g daily
E) Vitamin D, 400–800 IU daily

ANSWER: C
To prevent neural tube defects in newborns, the U.S. Preventive Services Task Force
recommends folic acid, 0.4–0.8 mg daily, for all women who are planning on or are
capable of becoming pregnant (USPSTF A recommendation).

Ref: Final Recommendation Statement: Folic Acid for the Prevention of Neural Tube Defects:
Preventive Medication . US Preventive Services Task Force, 2017.

203. An 11-year-old female is brought to your office for a well child visit. The mother is
concerned because the patient’s back seems to have a curve. The patient is not aware
of this although she has frequent back pain.
An examination is notable for a BMI above the 95th percentile for her age, and breast
bud development. Menarche has not occurred. When she leans forward with her arms
outstretched there is a 12° curve in her spine with a rib hump. Radiography reveals a
measured Cobb angle of 20°.

Which one of the following indicates a need for referral to a specialist?


A) Back pain
B) A Cobb angle of 20°
C) Female sex
D) Obesity
E) Premenarchal status

ANSWER: B

There are three major risk factors for curve progression of idiopathic scoliosis: the
magnitude of the curve at presentation, the potential for future growth, and female sex.
Of these factors, curve progression has the most impact on the need for referral versus
observation. The Cobb angle is based on spine radiology that quantifies the magnitude
of the scoliosis curve. If the Cobb angle is more than or equal 20° there is a high risk
that the curve will progress and that the patient may need treatment. Age, sex,
menstrual status, pubertal status, and growth potential are less important factors.
Scoliosis typically does not cause pain and it is more likely that this patient’s weight is
contributing to her back pain.

Ref: Horne JP, Flannery R, Usman S: Adolescent idiopathic scoliosis: Diagnosis and management. Am
Fam Physician 2014;89(3):193-198.
204. A healthy 43-year-old executive presents with problems falling asleep and staying
asleep. Doxepin (Silenor) and extended-release melatonin have not helped.
In addition to behavioral interventions, which one of the following would be the most
appropriate pharmacologic therapy for this patient’s insomnia at this time?
A) Diphenhydramine (Benadryl)
B) Doxylamine (Unisom)
C) Eszopiclone (Lunesta)
D) Olanzapine (Zyprexa)
E) Zaleplon (Sonata)

ANSWER: C

Although behavioral interventions are the mainstay of treatment for insomnia, they often
need to be supplemented by pharmacologic therapy. When both doxepin and extended-
release melatonin fail to provide benefit, a member of the Z-drug class should be tried
next. Among the Z-drugs only eszopiclone provides an early peak onset and a long half-
life, with a 1-hour approximate time to peak and a 6-hour half-life. While zaleplon has an
equally short time to peak of 1 hour, it also has a 1 hour half-life. Antihistamines,
including diphenhydramine and doxylamine, as well as atypical antipsychotics such as
olanzapine, are not indicated unless used primarily to treat another condition.

Ref: Matheson E, Hainer BL: Insomnia: Pharmacologic therapy. Am Fam Physician 2017;96(1):29-35.

205. A 46-year-old male presents with a persistent cough that has been present for
several months and was not preceded by an upper respiratory infection. He does not
have a history of asthma, does not smoke, and takes no medications. His symptoms
consist of short bursts of coughing that produce a small amount of mucoid sputum
during the day. He does not have emesis or nausea. The cough sometimes wakes him
at night but does not seem to be specific to any particular posture. He does not have a
fever, shortness of breath, wheezing, heartburn, or nasal symptoms. A thorough physical
examination is normal and a chest radiograph appears normal.

Which one of the following would be the most appropriate next step in the management
of this patient?
A) Amoxicillin/clavulanate (Augmentin)
B) An empiric trial of a proton pump inhibitor
C) CT of the chest
D) CT of the sinuses
E) Referral for bronchoscopy

ANSWER: B

Chronic cough is defined as a cough lasting at least 8 weeks. If a thorough history (with
attention to ACE inhibitor use), a physical examination, and a plain-film chest radiograph
do not suggest an obvious cause for the cough, experts suggest that the three most
common etiologies are gastroesophageal reflux, persistent postnasal drip, and
unrecognized asthma. Treating a chronic cough empirically with a high-dose proton
pump inhibitor for 2–3 months is considered a reasonable choice before further
investigations are attempted. Ordering an esophageal pH probe or
esophagogastroduodenoscopy would also be considered appropriate. Postnasal drip is
often due to allergic rhinitis or another noninfectious condition and some guidelines
recommend empiric nasal corticosteroid sprays and/or first-generation oral antihistamine
use. CT of the chest and bronchoscopy may become necessary if the evaluation and
treatment for these three common conditions does not improve the patient’s symptoms.
Since there are no symptoms of bacterial sinusitis, the use of a broad-spectrum
antibiotic is not justified.

Ref: Irwin RS, Baumann MH, Bolser DC, et al: Diagnosis and management of cough executive summary:
ACCP evidence-based clinical practice guidelines. Chest 2006;129(1 Suppl):1S-23S. 2) Broaddus VC,
Mason RJ, Ernst JD, et al: Murray & Nadel’s Textbook of Respiratory Medicine, ed 6. Elsevier Saunders,
2016, pp 497-514. 3) Smith JA, Woodcock A: Chronic cough. N Engl J Med 2016;375(16):1544-1551.

206. A 25-year-old female comes to your office at 30 weeks gestation for a routine
obstetric follow-up. Her pregnancy has been uncomplicated except for a lack of
insurance. She is an immigrant from Guatemala and does not speak English. You have
used a telephone interpreting service for her previous visits, but today her 15-year-old
niece is with her and she says that she would be happy to interpret for you.

Which one of the following would be most appropriate?


A) Have the niece ask the patient for permission to act as interpreter
B) Proceed with the visit, since it is unlikely that you will need to discuss complicated
medical issues
C) Confirm the niece’s significant understanding of English, then have her interpret
D) Recommend the use of the telephone interpreting service as best medical practice
E) Tell her that it is illegal to use interpreters who are not certified

ANSWER: D

Physicians should use trained interpreters whenever possible to avoid mistakes and
pitfalls associated with using family members or untrained interpreters for medical
interviews. Simply being bilingual does not mean someone will be an appropriate
interpreter. Although it is technically legal to use a nontrained interpreter for this patient,
title VI of the Civil Rights Act requires interpreter services for all patients with limited
English capabilities who are receiving federal assistance (except Medicare Part B). This
patient does not have Medicaid, but ethically it would be appropriate to extend the same
courtesy to her. It is difficult to predict what may occur in any visit, and a physician is
open to serious medical mistakes by assuming a visit does not require an interpreter,
because the patient may bring up a serious medical issue. Confidentiality is a concern
when using family members to interpret, since they may not have an
understanding of the need for confidentiality. It is not appropriate to ask the patient
through her family member about her preferences for interpreting.

Ref: Juckett G, Unger K: Appropriate use of medical interpreters. Am Fam Physician 2014;90(7):476-480.

207. A 65-year-old female presents for follow-up of a DXA scan for osteoporosis
screening. Results of the scan reveal osteopenia, with a T-score of –2.0.
Treatment for osteopenia is indicated when the 10-year risk of a major fracture reaches
A) 5%
B) 10%
C) 20%
D) 35%

ANSWER: C
The National Osteoporosis Foundation recommends pharmacologic treatment when a
DXA scan reveals a T-score –2.5 (the cutoff for a diagnosis of osteoporosis), or when
the T-score falls between –1.0 and –2.5 (the diagnosis criterion for osteopenia) and the
10-year risk of a major fracture reaches 20%. The T-score of –2.0 places this patient in
the “osteopenic” range. A 10-year probability of a hip fracture 3% is also an indication
for treatment.

Ref: Cosman F, de Beur SJ, LeBoff MS, et al: Clinician’s guide to prevention and treatment of osteoporosis.
Osteoporosis Int 2014;25(10):2359-2381. 2) Jeremiah MP, Unwin BK, Greenawald MH, Casiano VE:
Diagnosis and management of osteoporosis. Am Fam Physician 2015;92(4):261-268.

208. A 21-year-old gravida 1 para 0 is diagnosed with overt hyperthyroidism early in the
first trimester. The most appropriate management at this time is
A) observation only
B) methimazole (Tapazole)
C) propylthiouracil
D) radioactive iodine
E) thyroidectomy

ANSWER: C

Overt hyperthyroidism during pregnancy is associated with adverse effects to the mother
and fetus, so treatment is required. Since methimazole is associated with birth defects
when used in the first trimester, propylthiouracil is preferred. Methimazole should be
considered after the first trimester because the risk of congenital anomalies is less than
the risk of liver failure associated with propylthiouracil. Surgery and radioactive iodine
should only be used if there is a clear indication, and radioactive iodine would not be
appropriate during pregnancy.

Ref: Carney LA, Quinlan JD, West JM: Thyroid disease in pregnancy. Am Fam Physician 2014;89(4):273-
278.

209. A 55-year-old male presents with severe pain, swelling, and erythema in his left first
metatarsophalangeal joint. His symptoms started yesterday and he has never had this
problem in the past. He has a history of hypertension, but normal renal function and no
diabetes mellitus. There is no overlying skin lesion or obvious source of infection.
Which one of the following would be the most appropriate treatment for this patient?
A) Allopurinol (Zyloprim)
B) Cephalexin (Keflex)
C) Colchicine (Colcrys)
D) Febuxostat (Uloric)
ANSWER: C

This patient has a classic presentation of podagra (acute metatarsophalangeal joint


gout). Without an overlying skin lesion as an indicator of infection, this patient can be
assumed to have gout in this classic presentation. Low-dose colchicine, 1.2 mg initially,
followed by 0.6 mg in 1 hour, is recommended over high-dose colchicine, 1.2 mg initially,
followed by 0.6 mg hourly for 6 hours. The high-dose regimen increases side effects but
the effectiveness is not improved. This case should not be assumed to represent a
septic joint and treated with cephalexin, given the typical podagra presentation.
Febuxostat and allopurinol are urate-lowering drugs used as treatment for intercritical
gout and not for acute treatment. Generally, treatment with urate-lowering therapy is not
necessary in patients having fewer than two attacks per year.

Ref: Qaseem A, McLean RM, Starkey M, et al: Diagnosis of acute gout: A clinical practice guideline from the
American College of Physicians. Ann Intern Med 2017;166(1):52-57. 2) Qaseem A, Harris RP, Forciea MA;
Clinical Guidelines Committee of the American College of Physicians: Management of acute and recurrent
gout: A clinical practice guideline from the American College of Physicians. Ann Intern Med 2017;166(1):58-
68.

210. A 30-year-old male is treated with topical medications for his papulopustular
rosacea with only partial improvement. The preferred antibiotic is
A) amoxicillin
B) cephalexin (Keflex)
C) doxycycline
D) erythromycin
E) trimethoprim/sulfamethoxazole (Bactrim)

ANSWER: C
Tetracycline and its derivatives have historically been used for the treatment of
papulopustular rosacea and there is data to support their use. A modified-release
doxycycline is FDA-approved for this indication. Amoxicillin, cephalexin, erythromycin,
and trimethoprim/sulfamethoxazole lack evidence to support their use in the treatment of
papulopustular rosacea.

Ref: Oge LK, Muncie HL, Phillips-Savoy AR: Rosacea: Diagnosis and treatment. Am Fam Physician
2015;92(3):187-196.

211. A 75-year-old female sees you because of a bulge at the vaginal opening. A pelvic
examination confirms descent of the vaginal wall to just beyond the hymen. This
protrusion is bothering her and interfering with her quality of life. She has had two
vaginal deliveries. She is sexually active and has not had any pelvic surgery.
Which one of the following would be the most appropriate initial treatment for this
problem?
A) Kegel exercises
B) A ring pessary
C) A space-occupying pessary
D) Hysteropexy
E) Hysterectomy
ANSWER: B

Pessaries are considered first-line treatment for pelvic organ prolapse (SOR C). Ring
pessaries provide support and are the initial choice in most circumstances. Sexual
intercourse can still occur with a ring pessary, which can be inserted and removed by the
patient. Space-occupying pessaries are associated with more vaginal discharge and
irritation and do not allow for sexual intercourse. While they can improve stress and urge
urinary incontinence, Kegel exercises do not treat pelvic organ prolapse. Surgery,
including hysterectomy or hysteropexy that conserves the uterus, can be considered
after first-line treatment with a pessary.

Ref: Iglesia CB, Smithling KR: Pelvic organ prolapse. Am Fam Physician 2017;96(3):179-185.

212. A mother brings in her 2-week-old infant for a well child check. She reports that she
is primarily breastfeeding him, with occasional formula supplementation.
Which one of the following should you advise her regarding vitamin D intake for her
baby?
A) Breastfed infants do not need supplemental vitamin D
B) He does not need supplemental vitamin D if he is taking at least 16 oz of formula
per day
C) Vitamin D supplementation should not be started until he is at least 6 months old
D) He should be given 400 IU of supplemental vitamin D daily
E) Intake of vitamin D in excess of 200 IU/day is potentially toxic

ANSWER: D

The American Academy of Pediatrics recommends a daily intake of 400 IU of vitamin D


in infants, children, and adolescents (SOR C). Breastfeeding does not provide adequate
levels of vitamin D. Exclusive formula feeding probably provides adequate levels of
vitamin D, but infants who consume less than 1 liter of formula per day need
supplementation with 400 IU of vitamin D daily. Vitamin D supplementation should be
started within the first 2 months of life.

Ref: Wagner CL, Greer FR: American Academy of Pediatrics Section on Breastfeeding; American Academy
of Pediatrics Committee on Nutrition: Prevention of rickets and vitamin D deficiency in infants, children, and
adolescents. Pediatrics 2008;122(5):1142-1152. 2) Casey CF, Slawson DC, Neal LR: Vitamin D
supplementation in infants, children, and adolescents. Am Fam Physician 2010;81(6):745-748. 3) Insititute
of Medicine: Dietary Reference Intakes for Calcium and Vitamin D. The National Academies Press, 2011.
4) Bly E, Huntington J, Harper AL: Clinical inquiry. What is the best age to start vitamin D supplementation to
prevent rickets in breastfed newborns? Breastfed infants who take vitamin D beginning at 3 to 5 days of life
don’t develop rickets. J Fam Pract 2013;62(12):755-763. 5) Munns CF, Shaw N, Kiely M, et al: Global
consensus recommendations on prevention and management of nutritional rickets. J Clin Endocrinol
Metab 2016;101(2):394-415.

213. A 70-year-old male comes to your office for preoperative clearance for a right total
hip replacement. He is a nonsmoker and has a history of mild hypertension controlled
with amlodipine (Norvasc). The history indicates that he is able to perform 7–8 METS of
activity without any concerning symptoms. A physical examination, including vital signs,
is normal.
Your evaluation should include which one of the following tests?
A) A chest radiograph
B) Coagulation testing
C) An EKG
D) Liver function studies
E) Renal function studies

ANSWER: E

The American Society of Anesthesiologists (ASA) has recently revised its physical status
classification system. A healthy patient would be classified as ASA I, while a patient with
mild systemic disease would be classified as ASA II. All patients who are having major
surgery should be offered preoperative laboratory testing, including a CBC and renal
function testing. For patients who are ASA III or IV and have chronic liver disease or take
anticoagulants, coagulation testing should be considered. There is no compelling
evidence to support either a chest radiograph or an EKG as part of a routine
preoperative evaluation.

Ref: Martin SK, Cifu AS: Routine preoperative laboratory tests for elective surgery. JAMA 2017;318(6):567-
568.

214. A 12-year-old female is brought to your office with a 3-week history of left groin pain
that is most bothersome after she participates in gym class at her middle school. She
does not recall a specific injury and does not participate in extracurricular sports. She
had an upper respiratory infection about a month ago but has otherwise been well.
An examination reveals a BMI at the 95th percentile for her age. Her vital signs are
within normal limits. A musculoskeletal examination is remarkable for limited internal
rotation of the hip.

Which one of the following is the most likely diagnosis?


A) Adductor muscle strain
B) Apophysitis of the anterior superior iliac spine
C) Legg-Calvé-Perthes disease
D) Slipped capital femoral epiphysis
E) Transient synovitis

ANSWER: D

The most common hip disorder in adolescents (ages 8–15) is slipped capital femoral
epiphysis (SCFE). Early diagnosis and treatment are critical in preventing disability
related to early-onset degenerative disease of the hip. In the past, SCFE has been more
common in boys than in girls but that prevalence is changing due to the rise in obesity.
SCFE should be suspected in an adolescent who has unexplained pain in the hip, groin,
thigh, or knee. It is rarely associated with trauma, overuse, or prior illness. On
examination the most indicative sign is limited internal rotation of the involved hip.
Bilateral radiographs of the hips, including frog-leg lateral views, should be obtained in
any adolescent who presents with a new limp and pain in the groin, hip, thigh, or knee
(SOR C).
Adductor muscle strain (groin strain) is very uncommon in adolescents. Patients
suspected of having a groin strain should also undergo radiography. Legg-Calvé-Perthes
disease and transient synovitis are more common in children under age 10. The
presenting symptoms of hip pain and a limp are similar to SCFE. Apophysitis of the
anterior superior iliac spine is common in adolescents but is caused by overuse. It is
mostly seen in runners, dancers, and ice hockey and soccer players ages 14–18.

Ref: Peck DM, Voss LM, Voss TT: Slipped capital femoral epiphysis: Diagnosis and management. Am Fam
Physician 2017;95(12):779-784.

215. A 36-year-old male presents with a 4-month history of persistent nasal congestion
and rhinorrhea. On examination he has clear nasal discharge and edema of the nasal
mucosa but no nasal polyps are noted. His current medications include intranasal
fluticasone (Flonase).
Which one of the following would be the most appropriate management of his chronic
symptoms?
A) Recommend neti pot nasal irrigation
B) Add oral amoxicillin
C) Add oral prednisone
D) Replace fluticasone with budesonide (Rhinocort) nasal spray

ANSWER: A

In addition to nasal corticosteroids, saline irrigation is a mainstay in the treatment of


chronic rhinosinusitis. Low-pressure, high-volume irrigation, such as with a neti pot, is
superior to nasal saline spray (SOR B). There is no evidence that one nasal
corticosteroid is better than another. The role of antibiotics in the treatment of chronic
rhinosinusitis is unclear. Antibiotics may be helpful in patients with signs of bacterial
infection, such as mucopurulent drainage or acute worsening of symptoms. Oral
corticosteroids are an option for the short-term improvement of severe symptoms in
patients with nasal polyps who are already on maintenance therapy with both nasal
saline irrigation and an intranasal corticosteroid spray.

Ref: Sedaghat AR: Chronic rhinosinusitis. Am Fam Physician 2017;96(8):500-506.

216. A 45-year-old male with a 30-pack-year smoking history reports a chronic cough
with a small amount of phlegm production and dyspnea with strenuous exercise. You
order spirometry, which shows a pre- and postbronchodilator FEV1/FVC ratio of 0.6 and
an FEV1 of 85% of predicted.

Which one of the following agents would be the best initial pharmacologic management?
A) An inhaled corticosteroid
B) A short-acting anticholinergic
C) A long-acting anticholinergic
D) A long-acting 2-agonist
E) Theophylline

ANSWER: B

This patient has COPD and is in a risk category of A (low risk, fewer symptoms) based
on the Global Initiative for Chronic Obstructive Lung Disease (GOLD) combined
assessment of COPD. As a result, either a short-acting anticholinergic or a short-acting
2-agonist should be selected as the initial pharmacologic management. Long-acting 2-
agonists or long-acting anticholinergics are indicated for patients with a GOLD combined
assessment category of B or worse. Long-acting inhaled corticosteroids are indicated for
patients with a GOLD combined assessment category of C or worse. Due to its narrow
therapeutic window, modest benefit, and need for monitoring, theophylline is not
recommended as an initial agent and should be considered as an alternative only for
patients with severe refractory symptoms.

Ref: Gentry S, Gentry B: Chronic obstructive pulmonary disease: Diagnosis and management. Am Fam
Physician 2017;95(7):433-441. 2) Pocket guide to COPD diagnosis, management, and prevention: A guide
for health care professionals. Global Initiative for Chronic Obstructive Lung Disease, 2017.

217. A 50-year-old female sees you for follow-up of her hypertension. At her last visit 4
weeks ago you started her on lisinopril (Prinivil, Zestril), 10 mg daily, because of a blood
pressure of 158/92 mm Hg and confirmed hypertension on ambulatory blood pressure
monitoring. She is tolerating the medication well and has no side effects. She does not
take any other medications. Today her blood pressure is 149/90 mm Hg, which you
confirm on repeat measurement. This is also consistent with her home measurements.
At her last visit a basic metabolic panel was normal.
You repeat a basic metabolic panel today and the results are normal except for a BUN of
25 mg/dL (N 8–23) and a creatinine level of 1.5 mg/dL (N 0.6–1.1). At her last visit her
BUN was 12 mg/dL and her creatinine level was 0.7 mg/dL.

Which one of the following would be most appropriate at this time?


A) Continue her current treatment regimen
B) Increase lisinopril to 20 mg daily
C) Continue lisinopril at the current dosage and add amlodipine (Norvasc), 5 mg daily
D) Discontinue lisinopril and begin amlodipine, 5 mg daily
E) Discontinue lisinopril and begin losartan (Cozaar), 25 mg daily

ANSWER: D

This patient has essential hypertension and her goal blood pressure is <140/90 mm Hg
based on JNC 8 guidelines, or 130/80 mm Hg based on the more recent
recommendations of the American College of Cardiology/American Heart Association
Task Force on Clinical Practice Guidelines. Until recently, it was recommended that
physicians should tolerate a rise of <30% in serum creatinine after ACE inhibitor or
angiotensin receptor blocker (ARB) initiation. Rises in serum creatinine of >30% from
baseline increase the risk of renal failure, adverse cardiac outcomes, and death. A
recent study suggests that rises in serum creatinine of <30% also put patients at risk for
these outcomes, with a dose-response relationship between the magnitude of creatinine
change and the risk of adverse outcomes. This patient has more than a 30% rise in
creatinine and has no other factors, such as diabetes mellitus, heart failure, or chronic
kidney disease, that would indicate a need for ACE or ARB therapy for her hypertension.
Discontinuing her ACE inhibitor and starting a medication from a different class is the
most appropriate treatment at this time. Based on JNC 8 guidelines, additional options
for blood pressure medications include thiazide diuretics and calcium channel blockers.
Ref: James PA, Oparil S, Carter BL, et al: 2014 evidence-based guideline for the management of high blood
pressure in adults: Report from the panel members appointed to the Eighth Joint National Committee (JNC
8). JAMA 2014;311(5):507-520. 2) Schmidt M, Mansfield KE, Bhaskaran K, et al: Serum creatinine elevation
after renin-angiotensin system blockade and long term cardiorenal risks: Cohort study. BMJ 2017;356:j791.
3) Whelton PK, Carey RM, Aronow WS, et al: 2017
ACC/AHA/AAPA/ABC/ACPM/AGS/APhA/ASH/ASPC/NMA/PCNA guideline for the prevention, detection,
evaluation, and management of high blood pressure in adults. J Am Coll Cardiol 2018;71(19):e127-e248.

218. A 50-year-old female presents with pain in her right forefoot. She recently ran her
first full marathon after several years of inactivity and says the pain started gradually
over the last few weeks of her training and has slowly gotten worse. You order
radiographs of the foot, which show a stress fracture of the second metatarsal. You
would recommend
A) resumption of regular activity if the pain does not recur with activity after 1 week
of rest
B) no weight bearing on the right foot for 6 weeks
C) no weight bearing for a few days, followed by a walking boot, then a rigid-soled
shoe in 4–6 weeks
D) a walking boot for 12 weeks
E) referral to an orthopedic surgeon for further evaluation

ANSWER: C

The recommended treatment for metatarsal stress fractures is no weight bearing for a
few days, possibly using a posterior splint, transitioning to a walking boot or short leg
cast, and then a rigid-soled shoe in 4–6 weeks. Callus formation on a radiograph and a
lack of point tenderness signify adequate healing, and immobilization can be
discontinued. Other recommended conservative therapy includes modified rest for 6–8
weeks with continuation of activities of daily living and limited walking. Normal activity
can be resumed after 2–3 weeks with no pain. Additionally, the use of NSAIDs, ice, and
stretching, as well as cross-training is recommended. Resuming regular activity after
only 1 week of pain-free rest would not be recommended. Fractures of the fifth
metatarsal should be carefully investigated to rule out a Jones fracture that may require
orthopedic treatment. Treatment of the more common second and third metatarsal
stress fractures is relatively straightforward.

Ref: Bica D, Sprouse RA, Armen J: Diagnosis and management of common foot fractures. Am Fam
Physician 2016;93(3):183-191. 2) Brukner P, Clarsen B, Cook J, et al: Brukner & Khan’s Clinical Sports
Medicine , ed 5. McGraw Hill, 2017, pp 959-961.

219. A healthy 55-year-old white male with a family history of coronary artery disease
sees you for a routine health maintenance visit. He asks you what he could do to
decrease his risk of cardiovascular disease. He is a nonsmoker, does not drink alcohol,
and has no history of substance abuse. His BMI is normal and the physical examination
is otherwise unremarkable. His vital signs include a heart rate of 80 beats/min, a blood
pressure of 119/70 mm Hg, a respiratory rate of 15/min, and a temperature of 37.0°C
(98.6°F).
Laboratory Findings
Fasting glucose 92 mg/dL
Total cholesterol 190 mg/dL
LDL-cholesterol 98 mg/dL
HDL-cholesterol 50 mg/dL
Triglycerides 145 mg/dL
His calculated 10-year risk for cardiovascular disease is 5.4%. Which one of the
following has the best evidence to prevent cardiovascular disease in a patient such as
this?
A) Moderate-intensity exercise, 150 minutes weekly
B) A low-dose statin
C) Aspirin, 81 mg daily
D) Fish oil supplements
E) Niacin supplements

ANSWER: A

A systematic evidence review released by the U.S. Preventive Services Task Force
(USPSTF) noted that the most active people had median cardiovascular risk reductions
of about 30%–35% when compared with the least active. Statins are beneficial for both
primary and secondary prevention of cardiovascular disease, but the benefit is greater
when the baseline risk is greater. Current guidelines would not support statin therapy for
a patient with a 10-year risk of atherosclerotic cardiovascular disease (ASCVD) <5%.
Fish oil supplements have not proven to be useful for primary prevention of ASCVD.
Aspirin is recommended for the prevention of cardiovascular disease in adults 50–59
years of age with a >10% 10-year ASCVD risk who are not at increased risk of bleeding,
are expected to live at least 10 years, and are willing to take low-dose daily aspirin for 10
years (USPSTF B recommendation). Niacin is no longer recommended for
cardiovascular risk reduction due to a lack of evidence for benefit.

Ref: Shiroma EJ, Lee IM: Physical activity and cardiovascular health: Lessons learned from epidemiological
studies across age, gender, and race/ethnicity. Circulation 2010;122(7):743-752. 2) Final
Recommendation Statement: Aspirin Use to Prevent Cardiovascular Disease and Colorectal Cancer:
Preventive Medication . US Preventive Services Task Force, 2017. 3) Last AR, Ference JD, Menzel ER:
Hyperlipidemia: Drugs for cardiovascular risk reduction in adults. Am Fam Physician 2017;95(2):78-87.

220. An asymptomatic 42-year-old female sees you for a routine evaluation. On


examination her uterus is irregularly enlarged to the size seen at approximately 8 weeks
gestation. Pelvic ultrasonography shows several uterine fibroid tumors measuring <5 cm.
The patient does not desire future fertility.

Which one of the following would be the most appropriate management option?
A) Observation only
B) An oral contraceptive
C) A gonadotropin-releasing hormone (GnRH) agonist
D) Laparoscopic myomectomy
E) Hysterectomy

ANSWER: A
Uterine fibroid tumors (leiomyomas) are the most common tumors of the female
reproductive tract, with some evidence suggesting that the cumulative incidence in
women age 25–45 years is approximately 30%. Symptoms related to fibroids can
include menorrhagia, pelvic pain, obstructive symptoms, infertility, or pregnancy loss.
However, many fibroids are asymptomatic and are discovered incidentally, with
observation being the preferred management in this situation (SOR B). The risk of
malignant leiomyosarcoma is exceedingly small (0.23% in one study) and there is a risk
of side effects or complications from other treatment modalities.
For women who are symptomatic, the data is insufficient regarding the most appropriate
therapy. Surgical options include myomectomy, hysterectomy, uterine artery
embolization, and myolysis, but data to allow direct comparison is lacking. With the
exception of trials of GnRH-agonist therapy as an adjunct to surgery, there is not enough
randomized trial data to support the use of medical therapies such as oral
contraceptives, NSAIDs, or progestins in the treatment of symptomatic fibroids.

Ref: Hartmann KF, Fonnesbeck C, Surawicz T, et al: Management of uterine fibroids. Agency for Healthcare
Research and Quality, 2017. 2) De La Cruz MS, Buchanan EM: Uterine fibroids: Diagnosis and treatment.
Am Fam Physician 2017;95(2):100-107.

221. A 34-year-old female sees you because of cramping diarrhea for the past several
months following resection of her terminal ileum as treatment for Crohn’s disease. She is
not aware of any exposure to individuals with similar symptoms. She has not had any
fever, chills, nausea, vomiting, or myalgias, and she has not noticed any blood in her
stool. She is passing several loose stools daily, mostly after meals. She has not been
able to identify any clear relationship to the type of food she eats.

Which one of the following would be the best initial treatment option for this patient?
A) A dairy-free diet
B) Cholestyramine (Questran) daily
C) A Lactobacillus probiotic supplement (Lactinex) 4 times daily
D) Loperamide (Imodium) as needed
E) Psyllium fiber (Metamucil) twice daily

ANSWER: B

Diarrhea that develops in patients with ileal Crohn’s disease or following ileal resection is
usually due to increased amounts of bile acid remaining in the stool. This affects colonic
secretion and motility and various protein factors in the gut, resulting in the development
of bile acid diarrhea (BAD). Although various tests can be performed to evaluate the
stool, gut flora, and bowel function, a therapeutic trial with a bile acid sequestrant such
as cholestyramine is most often used for both the diagnosis and treatment of BAD.
Reducing fat intake may also be beneficial. Loperamide can lessen the diarrhea in some
patients but should not be the primary treatment because chronic use can result in
constipation. Fiber supplementation may help to produce a more formed stool and could
be used as an adjunct treatment when appropriate.
Ref: Kasper DL, Fauci AS, Hauser SL, et al (eds): Harrison’s Principles of Internal Medicine , ed 19.
McGraw-Hill, 2015, pp 1934-1935. 2) Goldman L, Schafer AI (eds): Goldman’s Cecil Medicine, ed 25.
Elsevier Saunders, 2016, pp 928-929.

222. A 48-year-old male comes to your office for follow-up of recently diagnosed panic
attacks. As part of his treatment plan he is taking sertraline (Zoloft), 50 mg daily, and
working with a mental health provider who has diagnosed posttraumatic stress disorder
associated with a traumatic childhood. He reports that his sleep continues to be
interrupted by nightmares.

Which one of the following additional medications may provide long-term control of his
symptoms?
A) Atenolol (Tenormin)
B) Lorazepam (Ativan)
C) Prazosin (Minipress)
D) Risperidone (Risperdal)
E) Zolpidem (Ambien)

ANSWER: C

An SSRI or SNRI should be used first as monotherapy for posttraumatic stress disorder
(PTSD), and should be optimized before an additional agent is added. Prazosin is an
effective augmenting therapy for patients with PTSD and sleep disturbance (SOR B).
Other -blockers and -blockers have been shown to be ineffective in the treatment of
PTSD. Benzodiazepines such as lorazepam can treat symptoms of hyperarousal but
have been associated with adverse effects and should be avoided in the treatment of
PTSD (SOR B). Hypnotics such as zolpidem are generally reserved for short-term use.
There is no evidence to support the use of atypical antipsychotics for PTSD and their
use should be avoided (SOR C).

Ref: Warner CH, Warner CM, Appenzeller GN, Hoge CW: Identifying and managing posttraumatic stress
disorder. Am Fam Physician 2013;88(12):827-834.

223. A 55-year-old female presents with a 3-month history of right shoulder pain. The
pain has begun to limit some daily activities such as brushing her hair and reaching high
shelves, and it is waking her up at night, especially when she lies on her right side.
On examination the shoulder appears normal. There is moderate subacromial
tenderness, a positive painful arc at 90°, normal range of motion and abduction strength,
and a positive Hawkins impingement sign. You diagnose rotator cuff impingement
syndrome.

You consider performing a subacromial corticosteroid injection. Which one of the


following is the most likely result?
A) More pain relief and functional improvement compared to NSAIDs
B) More effective pain relief compared to physical therapy
C) More durable relief if an image-guided intra-articular injection is used
D) A possibility of temporary pain relief, but no change in the long-term outcome

ANSWER: D
This patient has a typical presentation of impingement syndrome of the shoulder.
Subacromial injection of a corticosteroid may provide pain relief for up to several weeks
but does not alter long-term outcomes. According to some studies, these injections are
associated with greater health care utilization. Injection of a corticosteroid has not been
shown to provide superior pain relief compared to oral NSAIDs. Physical therapy is
superior for long-term pain relief.

Ref: Foster ZJ, Voss TT, Hatch J, Frimodig A: Corticosteroid injections for common musculoskeletal
conditions. Am Fam Physician 2015;92(8):694-699.

224. A patient is being discharged from the hospital following an acute non–ST-elevation
myocardial infarction. He is currently being treated with aspirin, lisinopril (Prinivil, Zestril),
and metoprolol. An echocardiogram performed in the hospital was normal and a lipid
panel included a total cholesterol level of 200 mg/dL and a triglyceride level of 225
mg/dL. On examination he has a pulse rate of 68 beats/min and a blood pressure of
130/80 mm Hg.

Which one of the following additional medications has been shown to improve survival in
patients like this?
A) Amlodipine (Norvasc)
B) Atorvastatin (Lipitor)
C) Gemfibrozil (Lopid)
D) Isosorbide mononitrate
E) Spironolactone (Aldactone)

ANSWER: B

Statins such as atorvastatin, antiplatelet drugs such as aspirin, angiotensin inhibitors


such as lisinopril, and -blockers such as metoprolol have all been shown to increase
survival after an acute coronary artery event. Calcium channel blockers such as
amlodipine, fibrates such as gemfibrozil, and nitroglycerins such as isosorbide
mononitrate have not been shown to increase survival. Spironolactone has been shown
to increase survival in patients with heart failure and reduced ejection fractions but not in
those with ischemic heart disease with preserved ejection fractions.

Ref: O’Gara PT, Kushner FG, Ascheim DD, et al: 2013 ACCF/AHA guideline for the management of ST-
elevation myocardial infarction: A report of the American College of Cardiology Foundation/American Heart
Association Task Force on Practice Guidelines. Circulation 2013;127(4):e362-e425.

225. Acne appearing at which one of the following ages should prompt detailed
endocrine laboratory testing for possible underlying systemic disease?
A) 3 weeks
B) 7 months
C) 5 years
D) 9 years
E) 13 years

ANSWER: C
As many as 20% of newborns will be affected by neonatal acne, usually in the form of
pustules confined to the cheeks, chin, eyelids, and forehead. This is typically mild, self-
limited, and best managed by reassuring the parents. Acne can also appear in infants,
typically males 6–12 months of age, and is also usually self-limited and not associated
with underlying endocrine pathology in the absence of any other findings suggesting
hormonal abnormalities such as clitoromegaly, breast or testicular development, pubic
hair growth, hirsutism, or a growth abnormality consistent with increased muscle
development. Acne appearing during mid-childhood is rare and, if present, warrants
referral for extensive laboratory testing to identify an underlying endocrine abnormality.
Preadolescents and adolescents are very likely to develop acne as a result of normal
ovarian/testicular development. In the absence of other findings to suggest an endocrine
problem, developing an effective treatment regimen is most appropriate for these
individuals.

Ref: Eichenfield LF, Krakowski AC, Piggott C, et al: Evidence-based recommendations for the diagnosis and
treatment of pediatric acne. Pediatrics 2013;131(Suppl 3):S163-S186.

226. A 30-year-old white male presents to the emergency department with a 4-day
history of fever to 101°F, a sore throat, rhinorrhea, and cough. An examination reveals
rhinorrhea and a boggy nasal mucosa, but is otherwise unremarkable. A chest
radiograph shows a questionable infiltrate.

Which one of the following would help determine if antibiotic treatment would be
appropriate?
A) A C-reactive protein level
B) A procalcitonin level
C) A WBC count with differential
D) An erythrocyte sedimentation rate
E) CT of the chest

ANSWER: B

Using a procalcitonin-guided therapy algorithm reduces antibiotic use by 3.47 days


without increasing either morbidity or mortality in adults with acute respiratory infections.
If the procalcitonin level is <0.10 mg/dL, a bacterial infection is highly unlikely and it is
strongly recommended that antibiotics not be prescribed. If the procalcitonin level is
0.10–0.24 mg/dL a bacterial infection is still unlikely and it is recommended that
antibiotics not be used. If the level is 0.25–0.50 mg/dL a bacterial infection is likely and
antibiotics are recommended. It is strongly recommended that antibiotics be given if the
level is >0.50 mg/dL, because a bacterial infection is very likely.

Ref: Morris C, Paul K, Safranek S: Procalcitonin-guided antibiotic therapy for acute respiratory infections.
Am Fam Physician2016;94(1):53-58.

227. A 5-year-old male is brought to your office after passing an intestinal worm. He lives
on a farm with cattle, pigs, and dogs. He has never traveled very far from home. He
does not have any respiratory symptoms or diarrhea, but has experienced some
abdominal bloating. A picture of the worm taken by his mother is shown below.
Which one of the following is the infecting organism?
A) Ascaris lumbricoides (roundworm)
B) Enterobius vermicularis (pinworm)
C) Giardia lamblia
D) Necator americanus (hookworm)
E) Taenia solium (tapeworm)

ANSWER: A

This case and image are consistent with Ascaris lumbricoides infestation. A.
lumbricoides is a large roundworm that typically infects the ileum. Symptoms are
variable but large infections can lead to intestinal obstruction. Pinworms are much
smaller and typically present with anal pruritus. Tapeworms can be large, but are flat and
segmental in appearance, and are typically found in the stool as segments called
proglottids. Hookworms are also round, but are typically 6–12 mm in length. They are a
significant cause of anemia in children globally. Giardia lamblia is a microscopic
protozoan parasite that is not visible on gross examination.

Ref: Kucik CJ, Martin GL, Sortor BV: Common intestinal parasites. Am Fam Physician 2004;69(5):1161-
1168. 2) Weatherhead JE, Hotez PJ: Worm infections in children. Pediatr Rev 2015;36(8):341-352.

228. A 55-year-old male veteran sees you for a routine health maintenance visit. He is
up to date on recommended immunizations. His father was diagnosed with colon cancer
at age 70 and his family history is otherwise negative. The patient underwent a
colonoscopy at age 50 and has a copy of his pathology results, which describe a single
hyperplastic polyp taken from the rectum.

Which one of the following would be the most appropriate screening for colorectal
cancer in this patient?

A) Annual fecal immunochemical testing (FIT) starting now


B) Repeat colonoscopy now
C) Repeat colonoscopy at age 60
D) Referral to a gastroenterologist for further management

ANSWER: C

Screening for colorectal cancer (CRC) is recommended for average-risk individuals


beginning at age 50 (SOR A). Individuals at higher risk include those with a personal
history of adenomatous polyps, CRC, inflammatory bowel disease, genetic cancer
syndromes, or a family history of either adenomatous polyps or CRC. Patients with a first
degree relative with CRC or adenomatous polyps discovered before age 60, or two or
more first degree relatives at any age with CRC or advanced adenoma, should undergo
colonoscopy screening starting at age 40 or 10 years before the youngest age a family
member was diagnosed, whichever comes first. The maximum surveillance interval for
these patients is 5 years (SOR C). Patients with a single first degree relative diagnosed
at age 60 or older, and patients with two affected second degree relatives, should
undergo screening starting at age 40 by any recommended method, and
at the same intervals for average-risk individuals (SOR C). Patients with small, distal
hyperplastic polyps are considered to have a normal colonoscopy (SOR C). There is no
need for referral to a gastroenterologist or interval fecal immunochemical testing (FIT)
following an adequate colonoscopy.

Ref: Short MW, Layton MC, Teer BN, Domagalski JE: Colorectal cancer screening and surveillance. Am
Fam Physician 2015;91(2):93-100. 2) Rex DK, Boland CR, Dominitz JA, et al: Colorectal cancer screening:
Recommendations for physicians and patients from the US Multi-Society Task Force on Colorectal Cancer.
Am J Gastroenterol 2017;112(7):1016-1030.

229. Additional workup or referral to an endocrinologist for evaluation of precocious


puberty would be indicated in which one of the following patients?

A) A 7-year-old female with some pubic hair


B) An 8-year-old female with breast buds
C) An 8-year-old male with some pubic hair and axillary odor
D) An 8-year-old male with penile enlargement
E) A 10-year-old female who has recently begun having menses

ANSWER: D

Penile enlargement in an 8-year-old male is a sign of precocious puberty. Isolated


sparse pubic and axillary hair growth and axillary odor is referred to as premature
adrenarche, and represents high levels of dehydroepiandrosterone rather than activation
of the hypothalamic-pituitary-gonadal axis that leads to puberty. The isolated findings of
premature adrenarche are generally considered benign. An 8-year-old with breast buds
and a 10-year-old with menarche are within the normal range of expected pubertal
development. Penile enlargement typically represents full activation of the hypothalamic-
pituitary-gonadal axis and warrants endocrinologic evaluation in boys younger than 9
years of age.

Ref: Kaplowitz P, Bloch C: Evaluation and referral of children with signs of early puberty. Pediatrics
2016;137(1). 2) Klein DA, Emerick JE, Sylvester JE, Vogt KS: Disorders of puberty: An approach to
diagnosis and management. Am Fam Physician 2017;96(9):590-599.

230. An obese 37-year-old white female sees you because of fatigue. She is otherwise
asymptomatic and has normal vital signs. A complete physical examination is
unremarkable with the exception of obesity. A CBC shows no anemia, but her WBC
count is 12,500/mm3 (N 4500–11,000). A TSH level and a comprehensive metabolic
panel are normal. She does not take any medications and has not had any recent
illnesses.

Which one of the following would be most appropriate at this point?


A) Reassurance that her leukocytosis is likely caused by her obesity and counseling
about weight loss
B) A repeat CBC with differential and a review of the peripheral smear
C) A blood culture
D) Flow cytometric testing
E) Referral to a hematologist for further workup

ANSWER: B
Leukocytosis is a relatively common finding with many possible etiologies. For most
cases without a clear cause, a repeat CBC with differential and a peripheral smear
review are indicated to confirm leukocytosis, determine subtypes, and look for
concerning abnormalities on the smear. Given this patient’s fatigue, a hematologic
referral may be indicated if leukocytosis is confirmed on repeat testing. Similar
recommendations would apply to flow cytometry testing. Blood cultures are not
necessary in cases of leukocytosis without evidence of infection. Obesity can cause
leukocytosis, but because of the patient’s fatigue it would not be appropriate at this time
to attribute the leukocytosis to obesity alone.

Ref: Herishanu Y, Rogowski O, Polliack A, Marilus R: Leukocytosis in obese individuals: Possible link in
patients with unexplained persistent neutrophilia. Eur J Haematol 2006;76(6):516-520. 2) Riley LK, Rupert
J: Evaluation of patients with leukocytosis. Am Fam Physician 2015;92(11):1004-1011.

231. The drug class of choice for the management of breathlessness in end-of-life care
is
A) anticholinergics
B) antipsychotics
C) benzodiazepines
D) corticosteroids
E) opiates

ANSWER: E

When administered at appropriate doses, opiates do not reduce or compromise


respiratory status and do not hasten dying. Opiates help to reduce the sense of air
hunger in patients with dyspnea. The use of opiates for palliative therapy in advanced
pulmonary disease is supported by clinical guidelines from the American Thoracic
Society.

Ref: Albert RH: End-of-life care: Managing common symptoms. Am Fam Physician 2017;95(6):356-361.

232. A 34-year-old male presents with low back pain and stiffness that has been slowly
worsening over the past 6 months. It is especially bothersome at night and in the
morning when he gets out of bed. It improves with physical activity. He has taken
ibuprofen, 400 mg several times a day, which provides moderate pain relief but is not
working as well as it used to. He does not have any other joint pain, there is no history of
trauma, and he is otherwise well. His BMI is 24.8 kg/m2. Radiographs of the lumbar
spine show mild degenerative changes of the lumbar vertebrae without other
abnormalities.

Which one of the following additional tests would most likely lead to a specific diagnosis?
A) An erythrocyte sedimentation rate
B) C-reactive protein
C) Antinuclear antibody
D) HLA-B27
E) Rheumatoid factor
ANSWER: D

This patient’s back pain is most consistent with an inflammatory cause rather than a
mechanical cause. Morning stiffness and improvement with physical activity are key
features of inflammatory back pain. Ankylosing spondylitis (AS), one subset of the
broader diagnostic category of axial spondyloarthritis, is the likely diagnosis in this
patient. Delays in diagnosis are common due to the widespread presence of mechanical
low back pain. The identification of patients with inflammatory back pain is important,
because early intervention with disease-modifying agents can preserve long-term joint
function. HLA-B27 is found in 74%–89% of patients with AS and it can be diagnostic in a
patient with typical inflammatory back pain symptoms. Inflammatory markers such as the
erythrocyte sedimentation rate and C-reactive protein are often elevated in patients with
AS but are not specific to this diagnosis. Rheumatoid arthritis is not a likely cause of
back pain in this patient without any other joint findings. Antinuclear antibody testing can
assist in the diagnosis of systemic lupus erythematosus, which can cause an
inflammatory arthritis, but it is similarly nonspecific and lupus typically has other findings
in addition to back pain.

Ref: Taurog JD, Chhabra A, Colbert RA: Ankylosing spondylitis and axial spondyloarthritis. N Engl J Med
2016;374(26):2563-2574. 2) Strand V, Singh JA: Evaluation and management of the patient with suspected
inflammatory spine disease. Mayo Clin Proc 2017;92(4):555-564.

233. An 80-year-old male sees you for the first time. He is asymptomatic except for
some fatigue. His pulse rate is 50 beats/min. An EKG shows a prolonged PR interval.
Which one of the following medications in his current regimen is the most likely
explanation for these findings?
A) Donepezil (Aricept)
B) Escitalopram (Lexapro)
C) Lisinopril (Prinivil, Zestril)
D) Memantine (Namenda)
E) Zolpidem (Ambien)

ANSWER: A

The 2015 American Geriatrics Society Beers Criteria for potentially inappropriate
medication use in older adults more than or equal 65 years of age states that donepezil
use should be avoided in patients with syncope, due to an increased risk of bradycardia
(Moderate Evidence Level; Strong Strength of Recommendation). Donepezil is a
cholinesterase inhibitor. Due to their cholinergic effect, these medications have a
vagotonic effect on the sinoatrial and atrioventricular nodes. This can cause bradycardia
or heart block in patients with or without underlying cardiac conduction abnormalities.
Syncope has been reported with these medications.
Memantine is an N-methyl-D-aspartate receptor antagonist and is not associated with
bradycardia. Escitalopram, lisinopril, and zolpidem are also not associated with
bradycardia.

Ref: Aricept (donepezil hydrochloride) label. US Food and Drug Administration, 2012. 2) American Geriatrics
Society 2015 Beers Criteria Update Expert Panel: American Geriatrics Society 2015 updated Beers Criteria
for potentially inappropriate medication use in older adults. J Am Geriatr Soc 2015;63(11):2227-2246.
234. A 14-year-old female is brought to your office for an annual well child check and
sports preparticipation physical examination. She says she does a lot of running during
basketball practices and games but has trouble controlling her weight. Most of her family
is overweight. She does not have any difficulty participating in sports, and has no
symptoms such as chest pain, shortness of breath, or headaches. She has no significant
past medical history.
On examination the patient’s height is 154 cm (61 in) and she weighs 63 kg (139 lb). Her
BMI is 26.4 kg/m2, which places her in the 90th percentile for her age. Her blood
pressure is 130/85 mm Hg, which places her between the 95th and 99th percentile for
her age, height, and sex. Her chart reveals that her blood pressure was at this level at
the last two visits. The physical examination is otherwise normal.

In addition to counseling and support for weight loss, which one of the following would
be most appropriate at this point?
A) Informing the patient and her parents that she is prehypertensive and having her
return for a blood pressure check in 3 months
B) Plasma renin and catecholamine levels
C) An imaging study of the renal arteries
D) A fasting basic metabolic panel, a lipid profile, and a urinalysis
E) Antihypertensive drug therapy

ANSWER: D

In a pediatric patient, blood pressure should be evaluated using comparisons based on


age, sex, and height. Although this adolescent’s blood pressure is prehypertensive for
an adult according to JNC 8 guidelines, it is stage 1 hypertension (between 95% and
99%) for her age, sex, and height. All pediatric patients with confirmed hypertension
should have further evaluation to check for renal dysfunction as well as other cardiac risk
factors. Additionally, renal ultrasonography is recommended to evaluate for renal
disease and echocardiography to evaluate for end-organ damage that would affect
treatment goals. Additional studies, such as plasma renin and catecholamine levels or
renovascular imaging, may be indicated in children with abnormalities on initial
evaluation that suggest secondary causes of hypertension.
Pharmacologic therapy is usually recommended for pediatric patients with symptomatic
hypertension, secondary hypertension, target organ damage, diabetes mellitus, or
persistent hypertension in spite of nonpharmacologic treatment. A low-sodium diet may
be helpful for decreasing blood pressure, and given this patient’s obesity, intensive
counseling about lifestyle changes is appropriate.

Ref: Flynn JT, Kaelber DC, Baker-Smith CM, et al: Clinical practice guideline for screening and management
of high blood pressure in children and adolescents. Pediatrics 2017;140(3):e20171904.

235. A 55-year-old male is hospitalized because of altered mental status. His group
home caregiver reports a 1-week history of the patient being confused and unable to
perform his activities of daily living. He has a history of hypertension, COPD, and bipolar
disorder, and his medications include losartan (Cozaar), inhaled tiotropium (Spiriva), and
valproic acid (Depakene).
On examination the patient’s vital signs are normal and he is oriented to person, but not
to time or place. The remainder of the physical examination is within normal limits.
Laboratory findings, including liver enzymes and renal function tests, are normal except
for an elevated ammonia level. Ultrasonography of the abdomen does not show any liver
abnormalities. Lactulose therapy is started.

Which one of the following is the most likely cause of this patient’s elevated ammonia
level?

A) Valproic acid
B) Occult upper gastrointestinal bleeding
C) Portal vein thrombosis
D) Gilbert syndrome

ANSWER: A

Patients treated with valproic acid (VPA) have a higher incidence of hyperammonemia.
Although the incidence of VPA-induced hyperammonemia varies, it should be
considered in patients taking VPA who present with altered mental status or
encephalopathy. In addition to discontinuing VPA, the encephalopathy should be
managed with ammonia-lowering drugs such as lactulose. Ammonia levels can also be
elevated in patients taking VPA who do not have encephalopathy, and close monitoring
of these patients for the development of encephalopathy is recommended. Gilbert
syndrome causes an asymptomatic elevation of bilirubin in the absence of underlying
hepatic disease. Occult gastrointestinal bleeding should be suspected in patients with an
elevated BUN level in the absence of underlying renal disease or volume depletion.
Patients with portal vein thrombosis present with abdominal pain and other symptoms of
an underlying predisposing disease such as cirrhosis (SOR C).

Ref: Kasper DL, Fauci AS, Hauser SL, et al (eds): Harrison’s Principles of Internal Medicine, ed 19.
McGraw-Hill, 2015, pp 2026, 2028, 2056.

236. A 75-year-old male with a history of hypertension, TIA, and atrial fibrillation sees
you for follow-up. Ten days ago he was on vacation in another state when he developed
chest pain. He went to a local hospital where he was diagnosed with an ST-elevation
myocardial infarction (STEMI) and was taken immediately for cardiac catheterization. He
had a drug-eluting stent placed in his left anterior descending artery. He brings some
discharge paperwork with him, including a medication list, but has not yet seen a local
cardiologist. He is concerned that he is taking too many blood thinners. He feels well and
does not have any chest pain, shortness of breath, or excessive bleeding or bruising.
Prior to his STEMI the patient was taking lisinopril (Prinivil, Zestril), 10 mg daily; warfarin
(Coumadin), 2.5 mg daily; and metoprolol succinate (Toprol-XL), 25 mg daily. Upon
discharge he was instructed to continue all of those medications and to add clopidogrel
(Plavix), 75 mg daily, and aspirin, 81 mg daily.
The patient’s vital signs and physical examination are normal except for an irregularly
irregular rhythm on the cardiovascular examination. His INR is 2.5.

Which one of the following would be most appropriate at this time?


A) Continue the current regimen
B) Discontinue aspirin
C) Discontinue clopidogrel
D) Discontinue warfarin
E) Decrease warfarin with a goal INR of 1.5–2.0
ANSWER: A

Current guidelines recommend that patients with an ST-elevation myocardial infarction


(STEMI) who also have atrial fibrillation take dual antiplatelet therapy such as aspirin
plus clopidogrel and a vitamin K antagonist, with a goal INR of 2.0–3.0. If a patient was
already taking a direct-acting oral anticoagulant (DOAC) instead of warfarin for atrial
fibrillation, the patient should continue with the DOAC in addition to dual antiplatelet
therapy. The duration of triple therapy should be as short as possible, and aspirin can
often be discontinued after 1–3 months. However, this patient’s STEMI occurred less
than 2 weeks ago and he should continue triple therapy.

Ref: Anderson JL, Morrow DA: Acute myocardial infarction. N Engl J Med 2017;376(21):2053-2064.

237. A 38-year-old white female presents with abdominal pain and insists that she be
referred for surgical evaluation. She has a history of multiple unexplained physical
symptoms that began
in her late teenage years. She is vague about past medical evaluations, but a review of
her extensive medical record reveals multiple normal blood and imaging tests, several
surgical procedures that have failed to alleviate her symptoms, and frequent requests for
refills of narcotic analgesics.

This patient’s history is most compatible with


A) illness anxiety disorder
B) malingering
C) panic disorder
D) generalized anxiety disorder
E) somatic symptom disorder

ANSWER: E

Somatic symptom disorder (formerly called somatization disorder) usually begins in the
teens or twenties and is characterized by multiple unexplained physical symptoms,
insistence on surgical procedures, and an imprecise or inaccurate medical history.
These patients also commonly abuse alcohol, narcotics, or other drugs.
Patients with illness anxiety disorder are overly concerned with bodily functions, and can
often provide accurate, extensive, and detailed medical histories. Malingering is an
intentional pretense of illness to obtain personal gain. Patients with panic disorder have
episodes of intense, short-lived attacks of cardiovascular, neurologic, or gastrointestinal
symptoms. Generalized anxiety disorder is characterized by unrealistic worry about life
circumstances accompanied by symptoms of motor tension, autonomic hyperactivity, or
vigilance and scanning.

Ref: Kurlansik SL, Maffei MS: Somatic symptom disorder. Am Fam Physician 2016;93(1):49-54.

238. A 20-year-old college student comes to the urgent care clinic with right knee pain
and swelling after injuring her knee in a recreational basketball game. Her feet were
planted when another player collided with her, causing her upper torso to rotate. She felt
immediate pain in the knee and was unable to complete the game.

Which one of the following is the most accurate and appropriate maneuver to detect an
anterior cruciate ligament tear?

A) The anterior drawer test


B) The lever sign test
C) The Lachman test
D) The McMurray test
E) The pivot shift test

ANSWER: C

The Lachman test is the most accurate test for an anterior cruciate ligament (ACL) tear
(SOR A). Accurate testing can lead to appropriate referral and treatment for ACL tears,
and early detection can lead to better outcomes. The Lachman test has higher validity
based on a sensitivity of 68% for partial ruptures and 96% for complete ruptures. The
other two commonly used tests are the anterior drawer test, which has a sensitivity of
38% and a specificity of 81%, and the pivot shift test, which is more technically difficult
than the other two tests and has a sensitivity ranging from 24% to 85%. The pivot shift
test is effective if done correctly but should not be used alone to diagnose an ACL tear
(SOR A). The lever sign test is a newer test that holds promise for detecting ACL tears
and is easily performed in the office. However, sensitivity and specificity reports vary
(SOR B). The McMurray test is used to detect meniscal tears.

Ref: Grover M: Evaluating acutely injured patients for internal derangement of the knee. Am Fam Physician
2012;85(3):247-252. 2) Koster Ch, Harmsen AM, Lichtenberg MC, Bloemers FW: ACL injury: How do the
physical examination tests compare? J Fam Pract 2018;67(3):130-134.

239. A 54-year-old male is concerned about testosterone deficiency. He has erectile


dysfunction with impaired erections and decreased libido. He has also noted hair loss on
his legs, breast tenderness, and fatigue. He has chronic renal disease and compensated
heart failure, and he takes opioids for chronic pain. Five years ago he had a non–ST-
elevation myocardial infarction and has done well with medical management.
The patient’s morning testosterone level is low on two separate readings and you want
to initiate testosterone replacement.
Laboratory Findings
Estimated glomerular filtration rate 58 mL/min/1.73 m2
Creatinine 2.0 mg/dL (N 0.7–1.3)
Hematocrit 55% (N 42–52)
Prostate-specific antigen 3.9 ng/dL (N 0.0–4.0)

Which one of the following is an ABSOLUTE contraindication to starting treatment with


testosterone in this patient?
A) The history of coronary artery disease
B) Benign prostatic hyperplasia
C) Chronic renal disease
D) Compensated heart failure
E) Polycythemia
ANSWER: E

Testosterone replacement has significant risks and contraindications. Absolute


contraindications include breast cancer, prostate cancer, a prostate-specific antigen
(PSA) level >4 ng/dL, an abnormal rectal examination with nodules, and polycythemia
with a hematocrit >54%. Relative contraindications include a baseline hematocrit >50%,
a desire for fertility, uncontrolled heart failure, untreated sleep apnea, and severe lower
tract symptoms. This patient has polycythemia with a hematocrit >54% and should not
be started on testosterone. Testosterone stimulates erythropoiesis and increases the
risk of thrombosis. Although there may be an association between testosterone
deficiency and coronary artery disease, a history of coronary artery disease is not a
contraindication to testosterone replacement. Patients with chronic renal disease who
are on chronic opioid therapy are at higher risk of developing secondary testosterone
deficiency. Testosterone replacement may increase PSA levels and should not be used
in patients with known or suspected prostate cancer.

Ref: Petering RC, Brooks NA: Testosterone therapy: Review of clinical applications. Am Fam Physician
2017;96(7):441-449.

240. A 62-year-old female comes to your office for evaluation of pain in her right thumb
and wrist associated with sewing. She does not have any injury, numbness, tingling, or
weakness. An examination reveals an otherwise healthy-appearing female with normal
vital signs and no deformity or swelling in her wrists or hands. She has tenderness to
palpation at the first dorsal compartment over the radial styloid and has pain with active
and passive stretching of the thumb tendons over the radial styloid. She is very worried
that she will have to stop sewing and asks if there is anything she could try to alleviate
her symptoms.

Which one of the following would be most appropriate at this point?


A) Reassurance that it will likely improve on its own within about a year
B) A corticosteroid injection into the first extensor compartment
C) Immobilization in a thumb spica splint and an NSAID for 1–4 weeks
D) Radiographs of the thumb and wrist
E) Referral to an orthopedic surgeon

ANSWER: C

De Quervain’s tenosynovitis usually occurs with repeated use of the thumb and is
characterized by pain in the radial wrist. The course is typically self-limited but can last
for up to a year, so waiting would not be a good option for this patient who wants to
continue her usual activities as soon as possible. Conservative therapy with
immobilization and NSAIDs is recommended if there are no contraindications to NSAIDs.
A corticosteroid injection is helpful but is typically reserved for severe cases or if
conservative therapy fails. Surgery may be beneficial but is generally not recommended
unless the course is severe, given the natural history of resolution.

Ref: Tallia AF, Cardone DA: Diagnostic and therapeutic injection of the wrist and hand region. Am Fam
Physician 2003;67(4):745-750. 2) Huisstede BM, Coert JH, Fridén J, Hoogvliet P; European HANDGUIDE
Group: Consensus on a multidisciplinary treatment guideline for de Quervain disease: Results from the
European HANDGUIDE study. Phys Ther 2014;94(8):1095-1110.

You might also like